Sei sulla pagina 1di 269

CONTROL SYSTEMS

For
ELECTRICAL ENGINEERING
INSTRUMENTATION ENGINEERING
ELECTRONICS & COMMUNICATION ENGINEERING
CONTROL SYSTEMS
SYLLABUS
ELECTRONICS & COMMUNICATION ENGINEERING

Basic control system components; block diagrammatic description, reduction of block


diagrams. Open loop and closed loop (feedback) systems and stability analysis of these
systems. Signal flow graphs and their use in determining transfer functions of systems;
transient and steady state analysis of LTI control systems and frequency response.
Tools and techniques for LTI control system analysis: root loci, Routh-Hurwitz criterion,
Bode and Nyquist plots. Control system compensators: elements of lead and lag
compensation, elements of Proportional-Integral-Derivative (PID) control. State
variable representation and solution of state equation of LTI control systems.

ELECTRICAL ENGINEERING

Principles of feedback; transfer function; block diagrams; steady-state errors; Routh


and Nyquist techniques; Bode plots; root loci; lag, lead and lead-lag compensation; state
space model; state transition matrix, controllability and observability.

INSTRUMENTATION ENGINEERING

Feedback principles. Signal flow graphs. Transient Response, steady-state-errors. Routh


and Nyquist criteria. Bode plot, root loci. Time delay systems. Phase and gain margin.
State space representation of systems. Mechanical, hydraulic and pneumatic system
components. Synchro pair, servo and step motors. On-off, cascade, P, P-I, P-I-D, feed
forward and derivative controller, Fuzzy controllers.

© Copyright Reserved by Gateflix.in No part of this material should be copied or reproduced without permission
ANALYSIS OF GATE PAPERS

ELECTRONICS ELECTRICAL INSTRUMENTATION


1 Mark 2 Mark 1 Mark 2 Mark 1 Mark 2 Mark
Exam Year Ques. Ques. Total Ques. Ques. Total Ques. Ques. Total
2003 5 8 21 3 7 17 4 10 24
2004 4 7 18 2 9 20 4 8 20
2005 3 10 23 3 6 15 - 10 20
2006 2 7 16 1 3 7 1 8 17
2007 2 8 18 1 7 15 2 6 14
2008 1 8 17 1 8 17 1 9 19
2009 - 6 12 4 5 14 1 6 13
2010 3 3 9 2 3 8 - 4 8
2011 2 4 10 3 3 9 1 6 13
2012 2 3 8 1 4 9 2 6 14
2013 2 6 14 2 4 10 1 4 9
2014 Set-1 5 1 7 2 2 6 2 4 10
2014 Set-2 3 3 9 2 3 8 - - -
2014 Set-3 3 2 7 2 3 8 - - -
2014 Set-4 3 2 7 - - - - - -
2015 Set-1 2 5 12 2 4 10 1 3 7
2015 Set-2 3 4 11 2 3 8 - - -
2015 Set-3 3 4 11 - - - - - -
2016 Set-1 3 2 7 1 4 9 3 3 9
2016 Set-2 2 4 10 1 4 9 - - -
2016 Set-3 3 3 9 - - - - - -
2017 Set-1 5 3 11 3 4 11 2 4 10
2017 Set-2 3 4 9 4 3 10 - - -
2018 2 3 8 4 2 8 2 3 8

© Copyright Reserved by Gateflix.in No part of this material should be copied or reproduced without permission
CONTENTS
Topics Page No
1. BLOCK DIAGRAMS

1.1 Open Loop System 1


1.2 Closed Loop System 1
1.3 Laplace Transform 2
1.4 Transfer Function 3
1.5 Block Diagrams 4
1.6 Signal Flow Graphs 8
1.7 Mathematical Modeling 13
1.8 Sensitivity 15
Gate Questions 17

2. TIME DOMAIN ANALYSIS

2.1 Introduction 45
2.2 Time Response of 2nd Order System 45
2.3 Time Response Specifications 48
2.4 Steady State Error 49
Gate Questions 53

3. TIME DOMAIN STABILITY

3.1 Introduction 75
3.2 Routh’s Stability Criterion 76
3.3 Root Locus 81
3.4 Minimum Phase System 89
3.5 Non-Minimum Phase System 89
3.6 All Pass System 89
Gate Questions 90

4. FREQUENCY DOMAIN ANALYSIS

4.1 Frequency Response 124


4.2 Correlation between Time & Frequency Response 125
4.3 Bode Plots 126

© Copyright Reserved by Gateflix.in No part of this material should be copied or reproduced without permission
5. POLAR & NYQUIST PLOTS

5.1 Polar plots 133


5.2 Nyquist Plots 135
5.3 Nichols Chart 137
Gate Questions 141

6. STATE VARIABLE ANALYSIS

6.1 Introduction 174


6.2 State of a System 174
6.3 State Model from Transfer Function 176
6.4 Transfer function from State Model 176
6.5 Solution of State Equation 177
6.6 Controllability 177
6.7 Observability 178
6.8 Stability of a System 178
Gate Questions 180

7. CONTROLLERS

7.1 Introduction 204


7.2 ON-OFF Controller 204
7.3 Proportional Controller 204
7.4 Integral Controller 205
7.5 Derivative Controller 205
7.6 PD Controller 206
7.7 PI Controller 206
7.8 PID Controller 206

8. COMPENSATORS

8.1 Introduction 207


8.2 Lead Compensator 207
8.3 Lag Compensator 208
8.4 Lead-Lag Compensator 208
Gate Questions 210

9. ASSIGNMENT QUESTIONS 224

© Copyright Reserved by Gateflix.in No part of this material should be copied or reproduced without permission
1 BLOCK DIAGRAMS

1.1 OPEN LOOP SYSTEM • Except oscillators, in positive feedback,


we have always unstable systems.
The open loop control system is a non-
feedback system in which the control input 1.2 CLOSED LOOP SYSTEM
to the system is determined using only the
current state of the system and a model of The closed loop control system is a system
the system. Control characteristic of such where the actual behavior of the system is
systems are independent of o/p of the sensed and then fed back to the controller
system. and mixed with the reference or desired
state of the system to adjust the system to
its desired state. Control characteristic of
the system depends upon the o/p of the
C
=G system.
R

Examples:
1) Automatic coffee server
2) Bottling m/c of cold drink
3) Traffic Signal C G
= (Negative feedback)
4) Electric lift, R 1 + GH
5) Automatic washing m/c
Examples:
1.1.1 ADVANTAGE 1) Electric iron
2) DC motor speed control
1) No stability problem. 3) A missile launching system (direction of
2) The open-loop system is simple to missile
construct and is cheap. 4) Changes with the location of target)
3) Open loop systems are generally stable 5) Radar Tracking system
6) Human Respiratory system
1.1.2 DISADVANTAGE 7) A man driving a vehicle (eye-sensor,
Brain-controller)
1) The open loop system is inaccurate & 8) Auto pilot system
unreliable 9) Economic inflation
2) The effect of parameter variation and
external noise is more. 1.2.1 ADVANTAGES
3) The operation of open-loop system is
affected due to the presence of non- 1) As the error between the reference
linearity in its elements. input and the output is continuously
measured through feedback, the closed-
Note: loop system works more accurately.
• An open loop stable system may 2) Reduced effect of parameter variation
become unstable when negative 3) BW of system can be increased
feedback is applied. 4) Reduced effect of non-linearity

© Copyright Reserved by Gateflix.in No part of this material should be copied or reproduced without permission
1
7) eat u ( t ) =
1.2.2 DISADVANTAGES s−a
1
1) The system is complex and costly 8) e − at u ( t ) =
s+a
2) Gain of system reduces with negative 1
feedback 9) teat u ( t ) =
3) The closed loop systems can become (s − a) 2
unstable under certain conditions. 1
10) te − at u ( t ) =
(s + a) 2
n!
1.3 LAPLACE TRANSFORM 11) t n e − at u ( t ) =
(s + a) n +1
ω
The ability to obtain linear approximations 12) sinωt u ( t ) = 2
of physical systems allows the analyst to s + ω2
consider the use of the Laplace s
13) cosωtu ( t ) = 2
transformation. The Laplace transform s + ω2
methods substitutes relatively easily solved ω
14) e − at sinωt u ( t ) =
algebraic equations for the more difficult (s + a) 2 + ω2
differential equations. s+a
The Laplace transform exists for linear 15) e − at cosωt u ( t ) =
differential equations for which the (s + a) 2 + ω2
transformation integral converges. ω
16) sinh ωt u ( t ) = 2
Therefore, for f (t) to be transformable, it is s − ω2
sufficient that s

17) cosh ωt u ( t ) = 2
s − ω2
∫ f (t) e
− st
dt < ∞
0 1.3.2 PROPERTIES OF LAPLACE
Signals that are physically realizable TRANSFORM
always have a Laplace transform. The
Laplace transformation for a function of 1) If the Laplace transform of f (t) is F(s),
time, f (t), is then
 df ( t ) 

L{f = (s)
(t)} F= ∫f (t)e
− st
dt • L =  sF ( s ) − f (0 )
+

0  dt 
1.3.1 IMPORTANT RESULTS  d 2f ( t )  2
 = s F ( s ) − sf ( 0 ) − f ' ( 0 )
+ +
• L 2
F(t) = F(s)  dt 
1) δ(t) = 1 2) Initial value theorem
1 • lim f (t) = lim sF(s)
2) u(t) = t →0 s →∞
s 3) Final value theorem
1 • lim f (t) = lim sF(s)
3) u(t − T) =e − sT t →∞ s →0
s
1 The final value theorem gives the final
4) t u (t) = 2 value (t→∞) of a time functions using
s
2
its Laplace transform and as such very
t 1
5) u (t) = 3 useful in the analysis of control systems.
2 s However, if the denominator of sF(s)
6)
n!
t n u ( t ) = n +1 has any root having real part as Zero or
s positive, then the final value theorem is
not valid.

© Copyright Reserved by Gateflix.in No part of this material should be copied or reproduced without permission
1.4 TRANSFER FUNCTION function of a system is the Laplace
The transfer function of a linear, time- transform of its impulse response.
invariant, differential equation system is • Step response of the system is the
defined as the ratio of the Laplace system output when input is unit step
transform of the output (response signal. The impulse response of the
function) to the Laplace transform of the system can obtained differentiating the
input (driving function) under the step response.
assumption that all initial conditions are e.g. If step response is cs ( t ) = e −2t then
zero. the impulse response will be ci ( t ) = −2e−2t
In control system the output (or response)
is related to the input by a transfer function 1.4.1 PROPERTIES OF TRANSFER
C (s) FUNCTION
as defined earlier, i.e. = G(s) or
R (s )
1) The transfer function is defined only for
C(s) = R(s)G(s) a linear time-invariant system. It is not
The output time response can be defined for nonlinear systems.
determined by taking inverse Laplace 2) The transfer function between an input
transform of relation of C(s). variable and an output variable of a
Note: system is defined as the Laplace
• The open loop poles at origin in the transform of the impulse response.
transfer function determine the type of Alternatively, the transfer function
the system. between a pair of input and output
1 variables is the ratio of the Laplace
e.g. If G(s) = 3
s ( s + 2 )( s + 3) transform of the output to the Laplace
transform of the input.
the number of open loop poles at origin
3) All initial conditions of the system are
is 3 hence the type of system is 3.
set to zero.
• The highest power of s in the
4) The transfer function of a continuous –
denominator of the transfer function
data system is expressed only as a
determines the order of the system.
function of the complex variables. It is
1
e.g. If G(s) = 3 not a function of the real variable, time,
s ( s + 2 )( s + 3) or any other variable that is used as the
the power of s in the denominator is 5. independent variable. For discrete –
Hence, the order of the system is 5. data systems modeled by difference
• Impulse response of the system is the equations, the transfer function is a
system output when input is impulse. If function of z when the z-transform is
the input is specified as unit impulse at used.
t = 0, then R(s) = 1 and the
1.4.2 PROCEDURE TO DETERMINE
transformed expression for the system
TRANSFER FUNCTION
output, is,
C(s) = G(s) 1) Formulate the equations for the system.
Thus the output time response is, 2) Take the Laplace transform of the
L−1C(s) = L−1G(s) system equations assuming initial
or c(t) = g(t) conditions as zero.
3) Specify the system output and the input.
The inverse Laplace transform of G(s)
4) Take the ratio of the Laplace transform
is, therefore, called the impulse
of the output and the Laplace transform
response of a system or the transfer

© Copyright Reserved by Gateflix.in No part of this material should be copied or reproduced without permission
of the input which is nothing but the means that no power is being withdrawn at
transfer function of system. the output.
1.4.3 TRANSFER FUNCTIONS OF 1.4.4 TRANSFER FUNCTIONS OF
CASCADED ELEMENTS NONLOADING CASCADED ELEMENTS
Many feedback systems have components The two simple RC circuits, isolated by an
that load each other. Consider the system amplifier as shown in Figure, have
shown in Figure. Assume that ei is the input negligible loading effects, and the transfer
and eo is the output. The equations for this function of the entire circuit equals the
system are product of the individual transfer functions.
Thus, in this case,

1
( i1 − i 2 ) dt + R1i1 =
C1 ∫
ei

1 1
∫ ( i 2 − i1 ) dt + R 2i 2 + ∫i 2dt = 0
C1 C1
1
C1 ∫
i 2 dt = e0
E0 (s ) 1
= (K)
Taking the Laplace transforms of Equations E i ( s ) ( R1C1s + 1)
1
C1s
{I1 ( s ) − I2 ( s )} + R1I1 ( s ) = E i (s) 1 E0 (s )
=
K
( R 2C2s+1) Ei ( s ) ( R1C1s+1)( R 2C2s+1)
1
C1s
{ I 2 ( s ) − I1 ( s )} + R 2 I 2 ( s ) +
1
C1S
I2 ( s ) = 0 Example:
Find the transfer function of the network
1 given below
I 2 (s) = E 0 (s)
C 2S
Eliminating I1(s) from above Equations and
writing Ei(s) in terms of I2(s), we find the
transfer function between Eo(s) and Ei(s) to
be
E0 (s ) 1 Solution:
= Transfer function of the network is
E i ( s ) ( R1C1s + 1)( R 2 C2s + 1) + R1C2s
E0 (s ) 1
1 =
= E1 ( s ) RCs + 1
( R1C1R 2C2s ) + ( R1C1 + R 2C2 + R1C2 ) s + 1
2

The overall transfer function is not the 1.5 BLOCK DIAGRAMS


product of 1/ ( R1C1s + 1) and 1/ ( R 2C2 s + 1) . The
A block diagram of a system is a pictorial
reason for this is that, when we derive the representation of the functions performed
transfer function for an isolated circuit, we by each component and the flow of signals.
implicitly assume that the output is Such a diagram depicts the
unloaded. In other words, the load interrelationships that exist among the
impedance is assumed to be infinite, which various components. In a block diagram all

© Copyright Reserved by Gateflix.in No part of this material should be copied or reproduced without permission
system variables are linked to each other C(s) G(s)
through functional blocks. Or =
R(s) 1 + G(s)H(s)
The functional block or simply block is a The transfer function relating C(s) to R(s) is
symbol for the mathematical operation on called the closed-loop transfer function.
the input signal to the block that produces This transfer function relates the closed-
the output. The transfer functions of the loop system dynamics to the dynamics of
components are usually entered in the the feed forward elements and feedback
corresponding blocks, which are connected elements.
by arrows to indicate the direction of the From Equation C(s) is given by
flow of signals.
G(s)
Note: C(s) = R(s)
• In block diagrams the comparison of 1 + G(s)H(s)
signals is indicated by summing Thus the output of the closed-loop system
points. clearly depends on both the closed-loop
• The point from where signal is taken for transfer function and the nature of the
feedback is called take-off point. input.
• The signal can travel only along the
direction of the arrow. 1.5.3 BLOCK DIAGRAM REDUCTION
TECHNIQUES
1.5.1 BLOCK DIAGRAM FOR OPEN LOOP To draw a block diagram for a system, first
SYSTEM write the equations that describe the
dynamic behavior of each component. Then
take the Laplace transforms of these
equations, assuming zero initial conditions,
and represent each Laplace- transformed
equation individually in block form. Finally,
Open-loop transfer function =
assemble the elements into a complete
C(s)
= G(s) block diagram.
R(s)
Example
1.5.2 BLOCK DIAGRAM FOR CLOSED Draw the block diagram for the RC circuit
LOOP SYSTEM shown in the figure.

Feed forward transfer function Solution


C(s) The equation for current in the circuit is
= = G(s)
E(s) e −e0
i= 1 … (I)
For the system shown in above figure, the R
output C(s) and input R(s) are related as Its Laplace transform will be
follows: E (s) − E 0 (s)
I(s) = i
Since C(s) = G(s) E(s) R
E(s) = R(s) – B(s)
= R(s) – H(s) C(s)
Eliminating E(s) from these equations gives
C(s) = G(s)[R(s) – H(s)C(s)]

© Copyright Reserved by Gateflix.in No part of this material should be copied or reproduced without permission
Now the equation for output voltage is 3) Merging the blocks in parallel: The
transfer function of the blocks in
e 0=
∫ idt … (II) parallel gets added algebraically.
C
Its Laplace transform will be
I(s)
E 0 (s) =
Cs

Now, assembling these two elements the


overall block diagram is

1.5.4 BLOCK DIAGRAM 4) Shifting the summing points: The


TRANSFORMATIONS summing point can be shifted before
the block or after the block using some
1) Interchanging the summing points: If additional blocks.
there is no block or take-off point
between 2 summing points, the
summing points can be interchanged
without affecting output.

7


2) Merging the blocks in series: If there
is no take-off or summing points in
between blocks they can be merged into
single block. The transfer function of
resultant block will be product of
individual transfer functions.
6) Shifting the take-off point before the
block:


© Copyright Reserved by Gateflix.in No part of this material should be copied or reproduced without permission
Example
Consider the system shown in Figure.
Simplify this diagram.

7) Shifting the take-off point after the


block:

Solution:
By moving the summing point of the
≡ negative loop containing H2 outside the
positive feedback loop containing H1, we
obtain Figure.

8) Shifting take-off point after summing


point:

Eliminating the positive feedback loop


Example:
9) Shifting take-off point before Find the transfer function from each input
summing point: to the output C.

Solution:
Considering X(s)=0, the block diagram
reduces to

© Copyright Reserved by Gateflix.in No part of this material should be copied or reproduced without permission
Solution:

C (s) G1G 2 G 3G 5
∴ =
R ( s ) 1 + G 2 + G 5 H 5 + G 2 G 5 H 5 + G 2 G 3G 5 C G1G 2 (G 4 − G 3 )
=
Now, considering R(s) =0 R 1 + H 5G 2 (G 4 − G 3 ) + G1G 2 H1 + G1G 2 (G 4 − G 3 )

Example:
Find closed loop transfer function of
system shown in Fig.

Solution:

C (s) G 4 G 5 (1 + G 2 )
∴ =
X ( s ) 1 + G 2 + G 5 H 5 + G 2 G 5 H 5 + G 2 G 3G 5

Example: C G1 (G 2 + G 3 )(1 + G 4 H 2 )
Find the transfer function for the system =
R 1 + G1H1H 2 (G 2 + G 3 )
whose block diagram representation is
shown in Fig.
1.6 SIGNAL FLOW GRAPHS

A SFG may be defined as a graphical means


of portraying the input – output
relationships between the variables of a set

© Copyright Reserved by Gateflix.in No part of this material should be copied or reproduced without permission
of linear algebraic equations. A linear 6) Forward-Path Gain: The forward-path
system is described by a set of N algebraic gain is the path gain of a forward path.
equations: 7) Non touching Loops: Two parts of a
N
SFG are non touching if they do not
yi = ∑a kj y k share a common node.
k =1
j=1, 2,………, N
These N equations are written in the form 1.6.2 SFG FROM BLOCK DIAGRAM
of cause – and – effect relations:
N An SFG can be drawn from given block
=j effectth
∑ ( gain from k to j) × (k
k =1
th
cause) or diagram of a system following the steps:

simply 1) Name all the summing points & take-off


=
output ∑gain × input points.
Laplace transform equation 2) Each summing point & take-off will
N point will be a node of SFG.
yi (s) = ∑G kj ( s ) Yk (s) j=1,2,………,N 3) Connect all the nodes with branches
k =1
instead of blocks & indicate the block
transfer functions as the gains of the
Note: branches.
• In a SFG signals can transmit through a
branch only in the direction of the Example
arrow. Draw signal flow graph from the given
• When constructing a SFG, junction block diagram.
points, or nodes, are used to represent
variables.
• The modes are connected by line
segments called branches, according to
the cause – and – effect equations. The
branches have associated branch gains
and directions
Solution
1.6.1 DEFINITIONS OF SFG TERMS 1) Represent the take off points &
summing points with nodes
1) Input Node (Source): An input node is
a node that has only outgoing branches.
2) Output Node (Sink): An output node is 2) Now connect the nodes with the
node that has only incoming branches. transfer functions of the blocks as the
3) Forward Path: A forward path is a path gains
that starts at an input node and ends at
an output node, and along which no
node is traversed more than once.
4) Loop: A loop is a path that originates
and terminates on the same node and
along which no other node is
Example:
encountered more than once.
As an exp. on the construction of SFG,
5) Path Gain: The product of the branch
consider the following set of algebraic
gains encountered in traversing a path
equations:
is called the path gain.
=
y 2 a12 y1 + a 32 y3

© Copyright Reserved by Gateflix.in No part of this material should be copied or reproduced without permission
=
y3 a 23 y 2 + a 43 y 4 combinations of two non-touching loops) –
(sum of gain products of all possible
y 4 = a 24 y 2 + a 34 y3 + a 44 y 4
combinations of three non touching loops)
=
y5 a 25 y 2 + a 45 y 4 +…
Solution: = 1 − ∑ La + ∑ L b Lc − ∑ L d L e L f + ....
a b,c d,e,f

∑ L = sum of all individual loop gains


a
a

∑ L L = sum of gain products of


b,c
b C all

possible combinations of two non touching


loops.
∑ Ld Le Lf = sum of gain products of all
d,e,f

possible combinations of three non


touching loops.
∆k = Non –touching determinant to kth
forward path cofactor of the kth forward
path determinant of the graph with the
loops touching the kth forward path
removed, that is, the cofactor ∆k is obtained
from ∆ by removing the loops that touch
path Pk
1.6.4 TRANSFER FUNCTION OF
INTERACTING SYSTEM

A two-path signal-flow graph is shown in


Fig. An example of a control system with
multiple signal paths is a multi legged
robot.
1.6.3 MASON’S GAIN FORMULA FOR
SIGNAL FLOW GRAPHS

In many practical cases, we wish to


determine the relationship between an
input variable and an output variable of the
signal flow graph. The transmittance
between an input node and an output node
is the overall gain, or overall transmittance,
between these two nodes.
Mason’s gain formula, which is applicable The paths connecting the input R(s) and
to the overall gain, is given by output Y(s) are
1 Path 1: P1 = G1G2G3G4 and
= P ∑ Pk ∆ k
∆ k Path 2:P2 = G5G6G7G8.
Where, Pk = path gain or transmittance of There are four self-loops:
kth forward path L1 = G2H2, L2 = H3G3,
∆ = determinant of graph L3 = G6H6, L4 = G7H7.
= 1 – (sum of all individual loop gains) + Loops L1 and L2 do not touch L3 and L4.
(sum of gain products of all possible Therefore the determinant is

© Copyright Reserved by Gateflix.in No part of this material should be copied or reproduced without permission
Δ = 1 – (L1 + L2 + L3 + L4) + (L1L3 + L1L4 + Δ = 1 – (L1 + L2 + L3 + L4 + L5 + L6 + L7 + L8)
L2L3 + L2L4). + (L5L7 + L5L4 + L3L4)
The cofactor of the determinant along path The cofactors are
1 is evaluated by removing the loops that Δ1 = Δ3 = 1 and Δ2 = 1 – L5 = 1 + G4H4.
touch path 1 from Δ. Therefore we have Finally, the transfer function is
L1 = L2 = 0 and Δ1 = 1 – (L3 + L4) Y(s) P + P ∆ + P
Similarly, the cofactor for path 2 is = = 1 2 2 3
T(s)
R(s) ∆
Δ2 = 1 – (L1 + L2). Signal-flow graphs and Mason’s signal-flow
Therefore the transfer function of the gain formula may be used profitably for the
system is analysis of feedback control systems,
Y(s) P∆ +P ∆ electronic amplifier circuits, statistical
= T(s) = 1 1 2 2
R(s) ∆ systems, and mechanical systems, among
G1G 2 G 3G 4 (1 − L3 − L 4 ) + G 5G 6 G 7 G 8 (1 − L1 − L 2 ) many other examples.
1 − L1 − L 2 − L3 − L 4 + L1L3 + L1L 4 + L 2 L3 + L 2 L 4
1.6.6 OUTPUT NODES AND NON INPUT
1.6.5 TRANSFER FUNCTION OF NODES
COMPLEX SYSTEM
The gain formula can be applied only
Finally, we shall consider a reasonably between a pair of input and output nodes.
complex system that would be difficult to Often, it is of interest to find the relation
reduce by block diagram techniques. A between an output-node variable and a non
system with several feedback loops and input-node variable. For example, to find
feed forward paths is shown in Fig. the relation y7/y2, which represents the
dependence of y7 on y2; the latter is not an
input.
Let 𝑦𝑦𝑖𝑖𝑖𝑖 be an input and 𝑦𝑦𝑜𝑜𝑜𝑜𝑜𝑜 be an output
node of a SFG. The gain 𝑦𝑦𝑜𝑜𝑜𝑜𝑜𝑜 /𝑦𝑦2 , where 𝑦𝑦2 is
not an input, may be written as
∑Pk ∆ k | from yin to yout
y out y out / yin
= = ∆
y2 y 2 / y in ∑Pk ∆ k | from yin to y2
The forward paths are ∆
P1 = G1G2G3G4G5G6, Since ∆ is independent of the inputs and
P2 = G1G2G7G6, the outputs,
P3 = G1G2G3G4G8,
y out ∑Pk ∆ k |from yin to yout
The feedback loops are =
L1 = -G2G3G4G5H2, y2 ∑Pk ∆ k |from yin to y2
L2 = -G5G6H1, Notice that ∆ does not appear in the last
L3 = -G8H1, equation
L4 = -G7H2G2,
L5 = -G4H4, Example:
L6 = -G1G2G3G4G5G6H3,
L7 = -G1G2G7G6H3,
L8 = -G1G2G3G4G8H3,
Loop L5 does not touch loop L4 or loop L7;
loop L3 does not touch loop L4; and all other
loops touch. Therefore the determinant is

© Copyright Reserved by Gateflix.in No part of this material should be copied or reproduced without permission
From the SFG in Fig. The gain between y2 In this system there is only one forward
and y7 is written path between the input R(s) and the output
1 5( 3 2)
y 7 y 7 / y1 G G G G + G G 1 + G H C(s). The forward path gain is P1 = G1G2G3
= = 1 2 3 4
= P12 1 From figure, we see that there are three
y 2 y 2 / y1 1.[1 + G 3 H 2 + H 4 + G 3 H 2 H 4 ]
individual loops. The gains of these loops
are
Example:
L1 = G1G2H1
Consider the SFG in Fig. above, the
L2 = G2G3H2
following input – output relations are
L3 = G1G2G3
obtained by use of the gain formula:
Note that since all three loops have a
y2 1 + G 3H 2 + H 4 + G 3H 2 H 4
= common branch, there are no non touching
y1 ∆ loops. Hence, the determinant ∆ is given by
y 4 G1G 2 (1 + H 4 ) ∆ = 1 – (L1 + L2 + L3)
= = 1 – G1G2H1 + G2G3H2 + G1G2G3
y1 ∆
The cofactor ∆1 of the determinant along
y 6 y 7 G1G 2 G 3G 4 + G1G 5 (1 + G 3 H 2 ) the forward path connecting the input node
= =
y1 y1 ∆ and output node is obtained from ∆ by
Where removing the loops that touch this path.
∆ = 1 + G1H1 + G 3H 2 + G1G 2 G 3H 3 + H 4 + G1G 3H1H 2 Since path P1 touches all three loops, we
+G1H1H 4 + G 3H 2 H 4 + G1G 2 G 3H 3H 4 + G1G 3H1H 2 H 4 obtain
∆1 = 1
Example Therefore, the overall gain between the
Find the no. of forward paths, individual input R(s) and the output C(s), or the
loops and non-touching pair in following closed loop transfer function, is given by
SFGs
C(s) P∆
= P= 1 1
R(s) ∆
G1G 2 G 3
=
1 + G1G 2 G1 + G 2 G 3G 2 + G1G 2 G 3
Which is the same as the closed loop
transfer function obtained by block
diagram reduction. Mason’s gain formula
thus gives the overall gain C(s)/R(s)
without a reduction of the graph.
Example
Consider the system shown in Figure. Example:
Draw signal flow graphs for
 dx 
(a) x 2 = a1  1 
 dt 
d 2 x 3 dx1
(b) x 3 = + − x1
dt 2 dt
(c) x 4 = ∫ x 3dt
Solution:
a)
A signal flow graph for this system is
shown in Figure. Let us obtain the closed
loop transfer function C(s)/R(s) by use of
Mason’s gain formula.

© Copyright Reserved by Gateflix.in No part of this material should be copied or reproduced without permission
b)
Example
Find C/R for the following system using
Mason’s gain rule

c)

Solution
Example Forward Paths
Find C/R for the control system given in P1 = G1G2
Fig. P2 = G4
P3 = G7G8
P4 = G1G5G8
P5 = G7G6G2
Loops
L1 = G9
L2 = G3
L3 = G5G6
Solution: ∆ ( G 3 + G 9 + G 5G 6 ) + G 9G 3
=
∆1 = 1 − G 9
∆ 2 = 1 − ( G 9 + G 3 + G 5G 6 ) + G 9 G 3
∆3 = 1 − G 3
∆4 =1
P1∆1 + P2 ∆ 2 + P3 ∆ 3 + P4 ∆ 4 + P5 ∆ 5
T≈
The signal flow graph is given in Fig. ∆
The two forward path gains are G1G 2 (1 − G 9 ) + G 4 (1 − G 9 − G 3 − G 5G 6 + G 9 G 3 ) + G 7 G 8 (1 − G 3 ) G1G 5G 8
=
1 − G 3 − G 9 − G 5G 6 + G 9G 3
P1 = G1G2G3 and P2 = G1G4.
The five feedback loop gains are
P11 = G1G2H1, 1.7 MATHEMATICAL MODELLING
P21 = G2G3H2,
P31 = -G1G2G3, A set of mathematical equations, describing
P41 = G4H2, the dynamic characteristics of a system is
and P51 = -G1G4. called mathematical model of the system.
Hence
∆ =−1 ( P11 + P21 + P31 + P41 + P51 ) 1) Series RLC circuit

=
1 + G1G 2 G 3 + G1G 2 H1 + G 2 G 3 H 2 − G 4 H 2 + G1G 4
and ∆1 =∆ 2 =1
Finally,
C P1∆1 + P2 ∆ 2 G1G 2 G 3 + G1G 4
= =
R ∆ 1 + G1G 2 G 3 − G1G 2 H1 − G 2 G 3 H 2 − G 4 H 2 + G1G 4
Applying KVL
di 1
iR + L + ∫ idt =
vm
dt c

© Copyright Reserved by Gateflix.in No part of this material should be copied or reproduced without permission
dq
But i =
dt
d 2q dq q
∴L 2
+R + =vm
dt dt v

2) Parallel RLC circuit At Balance


dx d2x
F − Kx − f =
M 2
dt dt
d2x dx
∴ M 2
+f + Kx =
F
dt dt
Applying KCL
v 1 dv 4) Rotational system
+ ∫ vdt + c =i a. Inertia
R L dt
dφ d 2θ
v= T=J 2
dt dt
Where, φ = magnetic flux b. Damper element

d 2 φ 1 dφ φ T=f
c + + =i dt
dt 2 R dt L c. Spring twisted
T=Kθ
Mathematical model
dθ d 2θ
3) Translation system T−f − kθ = J 2 :
a. Mass dt dt
d 2θ dθ
J 2 +f + kθ = T
dt dt

d2x
F=M
dt 2
1.7.1 ANALOGY

b. Damper element

dx
F=f
dt
c. Spring

Example
Consider the mechanical system shown in
F = Kx Fig. (a) and its electrical circuit analog
shown in Fig.(b). The electrical circuit

© Copyright Reserved by Gateflix.in No part of this material should be copied or reproduced without permission
analogy is a force-current analogy as ∆Y(s) = ∆G(s)R(s)
outlined in Table. The velocities, v1(t) and In the closed-loop system, we have
v2(t), of the mechanical system are directly G(s) + ∆G(s)
analogous to the node voltages v1(t) and Y(s) + ∆Y(s) = R(s)
1 + ( G(s) + ∆G(s) ) H(s)
v2(t) of the electrical circuit. The
simultaneous equations, assuming the Then the change in the output is
initial conditions are zero, are ∆G(s)
∆Y(s) = R(s)
(1 + GH(s) + ∆GH(s) )(1 + GH(s) )
When GH(s) >> ΔGH(s), as is often the case,
we have
∆G(s)
∆Y(s) = R(s)
[1 + GH(s)]
The change in the output of the closed-loop
system is reduced by the factor [1 + GH(s)],
which is usually much greater than one
over the range of complex frequencies of
interest.
These are the greatest system complexity,
need much larger forward path gain and
possibility of system instability (it means
undesired/ persistent oscillations of the
output variable).

Figure: (a) Two-mass mechanical system 1.8 SENSITIVITY


(b) Two mode electric circuit analog C1 =
M1, C2 = M2, L = 1/K, R1 = 1/b1, R2=1/b2 The sensitivity STG is the ratio of percentage
M1sV1 (s) + ( b1 + b 2 ) V1 (s) − b1V2 (s) =
R(s)
change in T to the percentage change in G.
V2 (s) Where, T – Transfer function
M 2sV2 (s) + b1 ( V2 (s) − V1 (s) ) + k =
0
s G- Forward path gain
These equations are obtained using the ∂T / T ∂T G
i.e. S=
T
= ×
force equations for the mechanical system G
∂G / G ∂G T
of Fig(a). Rearranging Eqs., we obtain
( M1s + ( b1 + b2 ) ) V1 (s) + (−b1 )V2 (s) =R(s) a) Open loop system
 k
(−b1 )V1 (s) +  M 2s + b1 +  V2 (s) =
0
 s

1.7.2 SENSITIVITY OF CONTROL =


T
C
= G
SYSTEMS TO PARAMETER VARIATIONS R
∂T
=1
The first advantage of a feedback system is ∂G
that the effect of the variation of the ∂T G G
parameters of the process, G(s), is reduced. STG = × =× 1 = 1
∂G T G
This illustrates the effect of parameter Hence open loop sensitivity is unity.
variations; let us consider a change in the
process so that the new process is G(s) +
ΔG(s). Then in the open-loop case, the
change in transform of the output is

© Copyright Reserved by Gateflix.in No part of this material should be copied or reproduced without permission
b) Closed loop system

G
=T C=
/R
1 + GH
∂T (1 + GH )1 − GH 1
= =
∂G (1 + GH ) (1 + GH )
2 2

∂T G 1 G
⇒ STG = × = (1 + GH )
∂G T (1 + GH )
2
G
1
∴ STG =
1 + GH
means ( STG ) < ( SGT )
closedloop openloop

Hence closed loop system is lesser


sensitive to parameter variation hence
closed loop system is better.

© Copyright Reserved by Gateflix.in No part of this material should be copied or reproduced without permission
GATE QUESTIONS(EC)(Basics of Control Systems)
d2 y dy
Q.1 An electrical system and its signal- 2
x(t) is initially at
+ 3 + 2y =
dt dt
flow graph representations are
rest. For input ( t ) = 2u(t) , the output
shown in the figure (a) and (b)
respectively. The values of G2 and H, y(t) is
respectively are a) (1 − 2e − t + e −2t ) u(t)
b) (1 + 2e − t − 2e −2t ) u(t)
c) ( 0.5 + e − t + 1.5e −2t ) u(t)
d) ( 0.5 + 2e − t + 2e −2t ) u(t)
[GATE -2004]

Q.4 Despite the presence of negative


feedback, control systems still have
problems of instability because the
a) components used have
nonlinearities
b) dynamic equations of the
systems are not known exactly
Z3 (s) − Z3 (s) c) mathematical analysis involves
a) , approximations.
Z2 ( s ) + Z3 ( s ) + Z4 (s) Z1 ( s ) + Z3 (s)
d) system has large negative phase
− Z3 (s) − Z3 (s) angle at high frequencies
b) ,
Z2 ( s ) − Z3 ( s ) + Z4 (s) Z1 ( s ) + Z3 (s) [GATE -2005]
Z3 (s) Z3 (s)
c) , Q.5 In the system shown below,
Z2 ( s ) + Z3 ( s ) + Z4 (s) Z1 ( s ) + Z3 (s)
–Z3 ( s ) Z3 ( s )
x ( t ) = ( sin t ) u(t) . In steady –state,
d) , the response y(t) will be
Z2 ( s ) − Z3 ( s ) + Z4 ( s ) Z1 ( s ) + Z3 ( s )
[GATE -2001]

Q.2 The open loop DC gain of a unity


negative feedback system with
1  π 1  π
closed –loop transfer function a) sin  t-  b) sin  t+ 
s+4 2  4 2  4
is 1 −t
s + 7s + 13
2
c) e sin t d) sin t − cos t
4 4 2
a) b)
13 9 [GATE -2006]
c) 4 d)13
[GATE -2001] Q.6 The unit–step response of a system
starting from rest is given by
Q.3 A system described by the following c ( t ) = 1 − e −2t for t ≥ 0 . The transfer
differential equation function of the system is

© Copyright Reserved by Gateflix.in No part of this material should be copied or reproduced without permission
1 2  π
a) b) y ( t ) = cos  2t −  for the input
1 + 2s 2+s  3
1 2s  π
c) d) signal x ( t ) = pcos  2t −  Then, the
2+s 1 + 2s
 3
[GATE -2006]
system parameter ‘p’ is
2
Q.7 The unit impulse response of a a) 3 b)
system is h=( t ) e− t , t ≥ 0 . For this 3
system, the steady- state value of 3
c)1 d)
output for unit step input is equal to 2
a)-1 b)0 [GATE -2010]
c)1 d)∞
Q.11 A system with transfer function
[GATE -2006]
G (s) =
( s 2 +9 ) ( s+2 )
is excited sin
Q.8 The frequency response of a linear, ( s+1)( s+3)( s+4 )
time–invariant system is given by (ωt).
5
H (f ) = . The step response
1+j10πf The steady-state output of the
of the system is system is zero at
 -t  a)ω =1 rad/s b)ω =2 rad/s
a) 5 (1-e-5t ) u ( t ) b) 5 1-e 5  u ( t ) c)ω =3 rad/s d)ω =4 rad/s
  [GATE-2012]
c) (1-e-5t ) u(t) )
1 1
d)
5 ( s+5) (s+1) Q.12 Negative feedback in a closed-loop
[GATE -2007] control system DOES NOT
a) reduce the overall gain
Q.9 A linear, time–invariant, causal b) reduce bandwidth
continuous time system has a c) improve disturbance rejection
rational transfer function with d) reduce sensitivity to parameter
simple poles at s = −2 and s = −4 , variation
and one simple zero at s = −1. A unit [GATE-2015-01]
step u(t) is applied at the input of
the system. At steady state, the
output has constant value of 1 .The Q.13 For the system shown in the figure,
impulse response of this system is Y(s)/X(s)=_________
a) exp ( -2t ) +exp ( -4t )  u(t)
b)  −4 exp ( −2t ) + 12 exp ( −4t ) − exp(− t)  u(t)
c) -4exp ( -2t ) +12exp ( -4t )  u(t)
d) -0.5exp ( -2t ) +1.5exp ( -4t )  u ( t )
[GATE-2017-02]
[GATE -2008]
Q.10 A system with the transfer function
Y (s) s
= has an output
X (s) s + p

© Copyright Reserved by Gateflix.in No part of this material should be copied or reproduced without permission
ANSWER KEY:

1 2 3 4 5 6 7 8 9 10 11 12 13
(c) (b) (a) (d) (a) (b) (c) (b) (c) (b) (c) (b) 1

© Copyright Reserved by Gateflix.in No part of this material should be copied or reproduced without permission
EXPLANATIONS

Q.1 (c) s+4


∴ G(s) =
From KVL in both loops In first loop. S + 6s + 9
2

Vi (=
s ) I1 ( s ) Z1 ( s ) +  I1 ( s ) − I 2 ( s )  Z3 (s) For D.C. s = 0
( s ) I1 ( s )  Z1 ( s ) + Z3 ( s )  − I2 ( s ) Z3 (s)
Vi = ∴ G(s) = open loop gain =
4
Vi ( s ) I ( s ) Z3 (s)
9
= I1 ( s ) − 2 Q.3 (a)
Z1 ( s ) + Z3 ( s ) Z1 ( s ) + Z3 ( s )
d2 y dy
...(i) 2
+ 3 + 2y = x(t)
dt dt
In second loop
s 2 Y ( s ) + 3sY ( s ) + 2Y ( s ) =
X(s)
 I 2 ( s ) − I1 ( s )  Z3 ( s ) + I 2 ( s ) Z2 ( s )
x ( t ) = 2u(t)
+ I 2 ( s ) Z4 ( s ) =
0
2
I 2 ( s ) ( Z 2 ( s ) + Z3 ( s ) + Z 4 ( s ) ) X (s) =
S
= I1 ( s ) .Z3 (s) ∴ ( s 2 + 3s + 2 ) Y ( s ) =
2
I 2 (s) Z3 (s) S
=
G2 =
I1 (s) Z2 ( s ) + Z3 + ( s ) + Z4 ( s )
2
Y (s) =
s ( s + 2 ) (s + 1)
From SFG, I1 ( s )
2 A B C
= Vi G1 (s) + I 2 ( s ) H(s) =+ +
s ( s + 2 ) (s + 1) S s + 2 s + 1
1 Z3
I1 ( s ) Vi
= + I2 2= A ( s + 2 )( s + 1) + Bs ( s + 1)
Z1 + Z3 Z1 + Z3
+C ( s + 2) s
From (i)
Z s = 0, 2 = 2A ⇒ A = 1
∴H = 3 s =−1, 2 =− C ⇒ C =−2
Z1 + Z3
s = 2, 2 = 2B ⇒ B = 1
(comparing above two equations )
1 1 2
Y ( s ) =+ −
Q.2 (b) S S+ 2 S+1
G(s) y ( t ) 1 + e −2t − 2e − t  u(t)
CLTF =
1 + G ( S) H(S)
s+4 Q.4 (d)
=
S + 7s + 13
2

1 + G ( s ) H(s) S2 + 7s + 13 Q.5 (a)


= y ( t ) = x ( t ) *h(t)
G(s) s+4
H(s)=1 for unity feedback Y ( s ) = X ( s ) *H(s)
1 S2 + 7s + 13 H (= jω ) =
1 1
∠ − 45°
= −1
G(s) s+4 s +1 2
S2 + 6s + 9 1 π
1
= = ∠−
G(s) s+4 2 4

© Copyright Reserved by Gateflix.in No part of this material should be copied or reproduced without permission
x ( t ) = ( sint ) u(t) When s =
−1
, B = −5
1  π 5
∴ y (=
t) sin  t −  5 5
2  4 Y ( s )= −
S s+ 1
Q.6 (b) 5
 − 
t
1
C ( s ) =−
1
=
2 ⇒ y ( t ) =5 1 − e 5  u(t)
S S + 2 s(s + 2)  
1 C(s) 2
R=( s ) H= (s) = Q.9 (c)
S R(s) s + 2
Transfer functions,
K(s + 1)
Q.7 (c) H (s) =
h ( t ) = e− t ( s + 2 ) (s + 4)
1
H (s) =
1 Input, R ( s ) =
S+1 s
1 Output C ( s ) = R ( s ) H ( s )
R (s) =
S Given: lim s C ( s ) = 1
s→0
1 1
=
Output H= ( s ) .R ( s ) .
Or
s.K(s + 1)
=1
(S + 1) S lim
s→0 s ( s + 2 ) (s + 4)
1 A B
= + K
S(S + 1) S S + 1 Or =1
8
1= A ( s + 1) + Bs ⇒K= 8
=
s 0,1= A 8(s + 1)
H (s) =
s= −1,1 = −B ( s + 2 ) (s + 4)
∴ Output =−
1
S S+1
1
=− (1 e − t ) u(t) =
4
+
12
S+ 2 S+ 4
When t=∞at steady state. Output =1
( −4e−2t + 12e−4t ) u(t)
h (t) =
Q.8 (b) Which is also the required impulse
5 response of the system.
H (f ) =
1 + j10πf
Q.10 (b)
5 5 1
=
H (s) = = Phase difference between input and
1 + 5S  1 1
output,
5 s +  s +
 5 5 π  π π
1 1 Φ =− −  −  = =30°
Step response = 3  2 6
S 1 And ω = 2rad / sec
s+ 
 5 From the transfer function,
1 1 A B ω
* = + Φ 90° − tan −1
=
S  1 S
s+
1 p
s+ 
 5 5 2
90° − tan −1 = 30°
 1 p
⇒ A  s +  + Bs = 1
 5
When =s 0,= A 5

© Copyright Reserved by Gateflix.in No part of this material should be copied or reproduced without permission
Q.11 (c)
For sinusoidal excitation
s = jω
∴ G ( jω )

=
( −ω 2
+ 9 ) ( jω + 2 )
( jω + 1)( jω + 3)( jω + 4 )
For zero steady –state output
G ( jω ) = 0

=
( −w 2
+ 9) w 2 + 4

( w2 + 1 )( w2 + 9 )( w 2 + 16 )
For zero steady-state output
⇒ ωn = 9
⇒ ω = 3rad / sec

© Copyright Reserved by Gateflix.in No part of this material should be copied or reproduced without permission
GATE QUESTIONS(EC)(Block Diagram & SFG)

Q.1 The equivalent of the block diagram s (s + 2) s ( s + 27 )


in the figure is given as c) d)
S + 29s + 6
2
S2 + 29s + 6
[GATE -2003]

Q.3 Consider the signal flow graph


shown in the figure. The gain x5 /x1 is
a)

b) 1 − (be + cf + dg)
a)
abc
bedg
b)
1 − (be + cf + dg)
abcd
c)
c) 1 − ( be + cf + dg ) + bedg
1 − ( be + cf + dg ) + bedg
d)
abcd
[GATE -2004]

d) Q.4 The input –output transfer function


100
of a plant H(s) = s(s+10)2 .The plant
is placed in a unity negative
feedback configuration as shown in
the figure below.
[GATE -2001]
Q.2 The signal flow graph of a system is
shown in the figure. The transfer
C(s) The signal flow graph that DOES
function of the system
R(s) NOT model the plant transfer
function H(s) is

a)

6 6s
a) b)
S + 29s + 6
2
S + 29s + 6
2

© Copyright Reserved by Gateflix.in No part of this material should be copied or reproduced without permission
b)
Q.7 Consider the following block
diagram in the figure.

c)

C(S)
The transfer function is
R(S)
d) G1G 2
a) b) G1G 2 +G1 + 1
1 + G1G 2
G1
c) G1G 2 + G 2 + 1 d)
[GATE -2011] 1 + G1G 2
[GATE-2014]
Q.5 The signal flow graph for a system is
given below. The transfer function Q.8 The block diagram of a feedback
Y(s) control system is shown in the
for this system is given as
U(s) figure. The overall closed-loop
gain G of the system is

s+1 s +1
a) 2
b) 2 a) G =
G1G 2
5s + 6s + 2 s + 6s + 2 1 + G1H1
s+1 1
c) 2 d) 2 G1G 2
5s + 4s + 2 5s + 6 s + 2 b) G =
1 + G1G 2 + G1H1
[GATE-2013]
G1G 2
c) G =
Q.6 For the following system, 1 + G1G 2 H1
G1G 2
d) G =
1 + G1G 2 + G1G 2 H1
[GATE-2016]

When X1 ( s ) = 0 , the transfer


y(s)
function is
x 2 (S)
s +1 1
a) b)
s2 s +1
s+2 s +1
c) d)
s(s + 1) s(s + 2)
[GATE-2014]

© Copyright Reserved by Gateflix.in No part of this material should be copied or reproduced without permission
ANSWER KEY:
1 2 3 4 5 6 7 8
(d) (d) (c) (d) (a) (d) (c) (b)

© Copyright Reserved by Gateflix.in No part of this material should be copied or reproduced without permission
EXPLANATIONS

Q.1 (d) Q.5 (a)


Take off point is moved after G 2 so No of forward path =2
/G 2 .
1 1
=2
, P2 =P1
Q.2 (d) S S2
P1 = 1 ∆1 = 1; ∇ 2 = 1
3 24 S + 27 −4 −2
∆1 =1 + + = =L 1 = , L 2
S S S 5 S2
P1∆1 −2
G (s) = L3 = −4, L 4 =
1 − ( loopgain ) + pairofnon − touchingloops S
−3 −24 −2 No non touching loops
=L1 = , L2 = , L3 ∆k= 1[L1 + L 2 + L3 + L 4 ]
S S S
L1and L3 are non –touching . 4 2 5
=1 + + 2 + 4 +
( s + 27 ) S S S
S + 4s + 2 + 4S + 4s
2 2

∴ G (s) = S =
 −3 24 2  −2 −3 S2
1−  − − + × 5s 2 + 6s + 2
 S S S S S =
( s + 27 ) S2
s(s + 27) 1 1
= = S C(s) + 2
29 6 R(s) S + 29s + 6 S 2
S s +1
1+ × 2
2
= =
S S 5s + 6s + 2 5s + 6s + 2
2 2

S2
Q.3 (c)
=
P1 abcd,= ∆1 1 Q.6 (d)
=L1 be, = L 2 cf= , L3 dg
Non-touching loops are
L1 &L3 = bedg
x5 abcd
∴ =
x1 1 − ( be + cf + dg ) + bedg If X 1 (s) = 0
Y(s)
; The block diagram becomes
Q.4 (d) X 2 (s)
For option (d),
1 1
s ⇒ ( s + 1)
Y(s) 100 / S3
= Y(s)
= s =
U(s) 1 + 100 X 2 (s) 1 + 1 . s s+2 s(s + 2)
S2 s (s + 1) s + 1
Which is not transfer function of
H(s).

© Copyright Reserved by Gateflix.in No part of this material should be copied or reproduced without permission
Q.7 (c)
By drawing the signal flow graph for
the given block diagram

Number of parallel paths are three


Gains P 1 = G 1 G 2 , P 2 = G 2 , P 3 =1
By mason's gain formula,
C(S)
= P1 + P2 + P3
R(S)
⇒ G1G 2 + G 2 + 1

Q. 8 (b)

Y G1G 2
=
X 1+G1G 2 +G1H1

© Copyright Reserved by Gateflix.in No part of this material should be copied or reproduced without permission
GATE QUESTIONS(EE)(Basics of Control Systems)

Q.1 The transfer function of the system


d 2 y dy du
described by + = + 2u
dt 2 dt dt
with u is input and y as output is
(s + 2)
a) 2 b) 2
( s + 1)
(s + s ) (s + s )
2 2s 1
c) d) a)
( s2 + s ) (s2 + s )  s  s 
18 1 +  1 + 
[GATE-2002]  12   3 
1
b)
Q.2 A control system is defined by the  s  s 
following mathematical relationship 27 1 +  1 + 
 6  9 
d2x
2
dx
(
+ 6 + 5x = 12 1 − e −2t ) c)
1
dt dt  s  s 
The response of the system as 27 1 +  1 + 
 12   9 
t → ∞ is
1
a) x = 6 b) x = 2 d)
c) x = 2.4 d) x = −2  s  s 
27 1 +  1 + 
[GATE-2003]  9  3 
[GATE-2003]
Q.3 A control system with certain
excitation is governed by the Q.5 For a tachometer, if θ(t) is the rotor
following mathematical equation displacement in radians, e(t) is the
d 2 x 1 dx 1 output voltage and K t is the
+ + x= 10 + 5e −4t + 2e −5t
dt 2
2 dt 18 tachometer constant in V/rad/sec,
The natural time constants of the E(s)
then the transfer function, will
response of the system are Q(s)
a) 2s and 5s b)3s and 6s be
c) 4s and 5s d)1/3s and 1/6s K
[GATE-2003] a) K t s 2 b) t
s
Q.4 The block diagram of a control c) K t s d) K t
system is shown in figure. The [GATE-2004]
transfer function G(s) =Y(s)/U(s) of
the system is Q.6 For the block diagram shown in
C(s)
figure, the transfer function is
R(s)
equal to

© Copyright Reserved by Gateflix.in No part of this material should be copied or reproduced without permission
With
a) X = c0s + c1 , Y = 1  ( s 2 + a 0s + a1 ) Z = b 0s + b1
s2 + 1 s2 + s + 1 b) =
X 1,=
c s + c1
,=
Z b0s + b1
a) b)
(s + a 0s + a1 )
Y 0 2
s2 s2
c)
s2 + s + 1
d) 2
1 c) X = (
c1s + c0 , Y = 2
b1s + b 0 )
,Z =
1
s2 s + s +1 (s + a1s + a 0 )
[GATE-2004] d) X =c1s + c0 , Y =1 ,Z =b1s + b 0
(s 2
+ a1s + a 0 )
Q.7 The unit impulse response of a [GATE-2007]
second order under damped system Q.10 A function y(t) satisfies the
starting from rest is given by following differential equation:
c(t)=12.5 e −6t sin 8t, t ≥ 0 The steady- dy(t)
+ y(t) =δ (t)
state value of the unit step response dt
of the system is equal to Where δ ( t ) is the delta function.
a) 0 b) 0.25
c) 0.5 d) 1.0 Assuming zero initial condition, and
[GATE-2004] denoting the unit step function by
u(t), y(t) can be of the form
Q.8 When subjected to a unit step input, a) e t b) e-t
the closed loop control system c) e t u(t) d) e-t u(t)
shown in the figure [GATE-2008]

Q.11 The measurement system shown in


the figure uses three sub-systems in
cascade whose gains are specified as
1
G1 , G 2 and . The relative small
will have a steady state error of G3
a) -1.0 b) -0.5 errors associated with each
c) 0 d) 0.5 respective subsystem G1 ,G 2 and G3
[GATE-2005] are ε1 ,ε 2 and ε3 .The error
associated with the output is:
Q.9 The system shown in figure below

1 ε1.ε2
a) ε1 + ε2 + b)
ε3 ε3
c) ε1 + ε2 − ε3 d) ε1 + ε2 − ε3
[GATE-2009]

Can be reduced to the form Q.12 The response h(t) of a linear time
invariant system to an impulse δ(t),

© Copyright Reserved by Gateflix.in No part of this material should be copied or reproduced without permission
under initially relaxed condition is
h ( t ) =e-t +e-2t . The response of this
system for a unit step input u (t) is
a) u ( t ) +e-t +e-2t
b) (e-t +e-2t )u(t)
c) (1.5-e-t -0.5e-2t )u(t)
Assuming, h1 = b1 and h= b 0 − b1a1 ,
d) e-t δ(t)+e-2t u(t) 0

the input-output transfer function,


[GATE-2011] C(s)
G (s) = of the system is given
U(s)
V2 (s)
Q.13 The transfer function of the by
V1 (s) b s+b
circuit shown below is a) G ( s ) 2 0 1
s +a 0s+a1
a s + a0
b) G ( s ) 2 1
a + b1s + b0
b s + b0
c) G ( s ) 2 1
a + a1s + a 0
0.5s + 1 3s + 6 a s + a1
a) b) d) G ( s ) 2 0
s +1 s+2 a + b0s + b1
s+2 s +1 [GATE-2014]
c) d)
s +1 s+2
[GATE-2013] Q.16 For the signal flow graph shown in
the figure, which one of the
Q.14 The signal flow graph for a system is following expressions is equal to the
given below. The transfer function Y(s)
transfer function ?
Y(s) X 2 (s) x (s )=0
for this system is given as 1
U(s)

s+1 s+1 G1
a) 2
b) 2 a) b)
5s +6s+2 s +6s+2 1 + G 2 (1 + G1 )
s+1 1
c) 2 d) 2 G2
5s +4s+2 5s +6s+2
1 + G1 (1 + G 2 )
[GATE-2013]
G1 G2
c) d)
Q.15 The signal flow graph of a system is 1 + G1G 2 1 + G1G 2
shown below. U(s) is the input and [GATE-2015]
C(s) is the output

© Copyright Reserved by Gateflix.in No part of this material should be copied or reproduced without permission
Y(s) 25
Q.17 Find the transfer function of Q.
X(s) s + 10s + 25
2

the system given below: 35


R. 2
s + 18s + 35
Nature of system
I. Over damped
II. Critically damped
III. Critically damped
G1 G2
a) + a) P-I, Q-II, R-III
1 − HG1 1 − HG 2 b) P-II, Q-I, R-III
G1 G2 c) P-III, Q-II, R-I
b) + d) P-III, Q-I, R-II
1 + HG1 1 + HG 2
[GATE-2018]
G1 + G 2
c)
1 + H ( G1 + G 2 ) Q.21 The number of roots of the
G1 + G 2 polynomial
d)
1 − H ( G1 + G 2 ) s + s + 7s5 + 14s 4 + 31s3 + 73s 2 + 25s + 200 in
7 6

the open left half of the complex


[GATE-2015]
plane is
Q.18 For the system governed by the set
of equations:
a) 3 b) 4 c) 5 d) 6
dx1 / dt = 2x1 + x 2 + u
[GATE-2018]
dx 2 / dt =
−2x1 + u
y = 3x1
the transfer function Y(s)/U(s) is
given by
a) 3(s+1)/ ( s 2 -2s+2)
b) 3(2s+1)/ ( s 2 -2s+1)
c) (s+1)/ ( s 2 -2s+1)
d) 3(2s+1) / ( s 2 -2s+2)
[GATE-2015]

Q.19 For a system having transfer


−s+1
functionG(s) = s+1 , a unit step
input is applied at time t=0. The
value of the response of the system
at t=1.5 sec (round off to three
decimal places) is ______.
[GATE-2017-01]

Q.20 Match the transfer functions of the


second-order systems with the nature
of the systems given below.
15
P. 2
s + 5s + 15

© Copyright Reserved by Gateflix.in No part of this material should be copied or reproduced without permission
ANSWER KEY:
1 2 3 4 5 6 7 8 9 10 11 12 13 14
(a) (c) (b) (b) (c) (b) (d) (c) (d) (d) (c) (c) (d) (a)
15 16 17 18 19 20 21
(c) (b) (c) (a) 0.554 (c) (a)

© Copyright Reserved by Gateflix.in No part of this material should be copied or reproduced without permission
EXPLANATIONS

Q.1 (a)
d 2 y dy du
+ = + 2u
dt 2 dt dt
⇒ s 2 Y ( s ) + sY ( s ) = sU ( s ) + 2U(s)
Y(s) s+2
∴ = 2
U(s) ( s + s ) As per the block diagram, the
corresponding signal flow graph is
Q.2 (c) drawn
Taking (LT) on both sides
( s 2 + 6s + 5) × ( s=) 12  1s − s +1 2 
24
=
s ( s + 2)
24
X(s) = One forward path P1 = 2 / s 2
s ( s + 2 )( s + 1) (s + 5)
The individual loops are,
Response at t → ∞ 3 12 18
Using final value theorem L1 = − , L2 = − andL3 = − 2
s s s
Lt Lt
x (t) = sX(s) L1 and L1 are non-touching loops
t→∞ s→0
36
Lt s × 24 L1L 2 = 2
= = 2.4 s
s → 0 s ( s + 1)( s + 2 ) (s + 5) The loops touches the forward path
∆1 =1
Q.3 (b)
Natural time constant of the The graph determinant is
response depends only on poles of ∆ =− 1 (L1 + L 2 + L3 ) + L1L 2
the system. 3 12 18 36
=1 + + + 2 + 2
C(s) 1
T= (s) = 2 s s s s
R(s) s + s / 2 + 1/18 Applying mason’s gain formula
Y(s) P ∆
= =
18 1
G=(s) = 1 1
18s + 9 s + 1 ( 6 s + 1) (3s + 1)
2
U(s) ∆
2
1= = 2/s 2
This is in the form =
(1 + sT1 )(1 + sT2 ) 1 + + + 2 + 2 s + 15s + 54
2
3 12 18 36
T 0 , T 2 = 6sec, 3sec s s s s
2 1
Q.4 (b) = =
( s + 9 ) ( s + 6) 27  1 + s   1 + s 
Integrator are represented as 1/s in   
 9  6 
S-domain

© Copyright Reserved by Gateflix.in No part of this material should be copied or reproduced without permission
Q.5 (c) C (s)
Transfer function =
R (s)
=  12.5e −6t sin 8t 
θ ( t ) = rotor displacement in
8
radians = 12.5 ×
( s + 6) + 82
2

ω (=
t ) = angular speed in 100
dt =
( s + 6) + 82
2
rad/sec
Output voltage; e(t) = K t ω(t) 1
When input is unit step, R ( s ) =
dθ s
= Kt .
dt 1 100
C (s) = .
Taking Laplace transform on both s ( s + 6 ) 2 + 82
E(s)
sides E ( s ) = K t sθ ( s ) ⟹ = K ts
Steady-state value of response,
θ(s)
using final value theorem
Lt
Q.6 (b) Csteady − state = C(t)
t →∞
Method-1: Using block-diagram
reduction technique. Lt
= s(s)
So, transfer function s→0
C(s) s 2 + s + 1 Lt  1 100 
= = = =
s → 0  s (s + 6) + 8 

R(s) s 2 s . 2 2
1
Method-2: Using signal flow graph
Q.8 (c)
Using signal flow grap

Three forward paths.


11 1 1 1
=
P1 = 2 , = P2 1.= &P =3 1
ss s s s
The no. of individual loop=0 Forward path gains
So graph determinant = ∆ = 1 2 −2
P1 =( −1) × =
and ∆1 =∆ 2 =∆ 3 =1 s+2 s+2
Applying Mason’s gain formula 3 2 2
=
And P2 = .
C(s) P1∆1 + P2 ∆ 2 + P3 ∆ 3 s s + 2 s(s + 2)
G(s) = =
R(s) ∆ Individual loop
1 3 2 −6
1
s 2 .1 + .1 + 1.1 L1 =−1× × =
s s2 + s + 1 s s + 2 s(s + 2)
= =
1 s2 Loop touches forward paths,
therefore,
Q.7 (d) = =
V1 1and V2 1
Transfer function of a system is the D = 1 − L1
unit impulse response of the system.
6 s ( s + 2) + 6
=
1+ =
s(s + 2) s(s + 2)

© Copyright Reserved by Gateflix.in No part of this material should be copied or reproduced without permission
Using Mason’s gain formula, Single flow graph of the block-
Y ( s ) P1Δ1 + P2 Δ 2 diagram
=
R (s) Δ
2 6
− ×1 + ×1
s+2 s(s + 2)
=
s ( s + 2) + 6
s(s + 2)
Y (s) 6 − 2s
= 2
R ( s ) s + 2s + 6
For unit step input, R(s)=1/s
 6 − 2s  There are two forward paths.
Y ( s ) = R ( s ) . 2 
 s + 2s + 6  1 1 CP
Error =E(s)=R(s)-Y(s) P1 = 1× c0 × × × P = 02
s s s
 6 − 2s 
= R (s) − R (s)  2 
1 CP
P2 = 1× c1 × 1× × P = 1
 s + 2s + 6  s s
 6 − 2s  These are four individual loops
= R ( s ) 1 − 2
 s + 2s + 6 
1  s 2 + 4s 
E (s) =  2
s  s + 2s + 6 
Steady state value of error, using
final value theorem
ess = lim sE(s) 1 a
s→ 0 L1 =− a1 × =− 1
 1 s 2 + 4s  s s
= lim s  . 2  1 1 a
s→ 0
 s s + 2s + 6  L 2 = × × − a 0 = − 20
s s s
 s 2 + 4s 
= lim  2 =0 1 1 bP
L3 = × × P × b 0 = 02
s → 0 s + 2s + 6
  s s s
1 b1P
Q.9 (d) L 4 = b1 × P × = 2
s s
The block-diagram can be redrawn
All the loops touch forward paths
as
∆1 =∆ 2 =1
∆ = 1 − ( L1 + L 2 + L3 + L 4 )
a 0 a1 b 0 P b1P
=1 + + − 2 −
s2 s s s
Using Mason’s gain formula
C ( s ) P1Δ1 + P2 Δ 2
=
R (s) s
c0 P c1P
+
= s2 s
a1 a 0 b 0 P b1P
1+ + 2 − 2 −
s s s s

© Copyright Reserved by Gateflix.in No part of this material should be copied or reproduced without permission
C (s) c0 P + c1ps Q.12 (c)
= 2 Transfer function of system is
R ( s ) S + ( a 0 − b1 ) S + ( a 0 − b 0 P )
impulse response of the system with
C (s) P ( c0 + c1s ) / ( s 2 + a1s + a 0 ) zero initial conditions.
=
R ( s ) 1 − ( b 0 + b1s ) P/ ( s 2 + a1s + a 0 ) Transfer function
= H= ( s )  ( e−1 + e−2t )
P ( c0 + c1s ) / ( s 2 + a1s + a 0 )
= … (i)
1 − ( b 0 + b1s ) P/ ( s 2 + a1s + a 0 )
1 1
= +
s +1 s + 2
C (s)
( s ) = 
=
xyP
… (ii) C(s) 1 1 
H= + 
R ( s ) 1 − yzP R(s)  s + 1 s + 2 
Comparing eq.(i) and (ii), we get 1
R ( s )= = ( step input )
c +cs s
xy = 2 0 1
S + a1s + a 0 C ( s ) = R ( s ) .H ( s )
b 0 + b1s 1 1 1  1 1
yz = =  + = +
S + a1s + a 0 s  s + 1 s + 2  S ( S + 1) S ( S + 2 )
2

Hence option (d) is correct.


1 1  11 1 
Q.10 (d) C ( s ) = − +  − 
 s s +1 2  s s + 2 
Taking (L.T.) on both sides
1.5 1 0.5
Y ( s )( s + 1) = 1 = − −
S S+1 S+ 2
1
∴ Y (s) = Response= C= ( t ) −1  C ( s ) 
s +1
Taking inverse laplace transform 1.5 1 0.5 
= −1  − −
Y ( t ) = e − t u(t)  s s + 1 s + 2 
C (=
t ) (1.5 − e − t − 0.5e −2t )u ( t )
Q.11 (c)
dG1 dG dG Q.13 (d)
=
∈1 , 2 =
∈2 & 3 ∈3
G1 G2 G3
G1G 3
Output (y)0 = x
G3
where X=input in
y = InG1 + InG 2 − InG 3 + Inx
Differentiating both sides 1
V2 ( s ) R+ 1 + RCs
dy dG1 dG 2 dG 3 dx Cs
= + − + = =
y G1 G2 G3 x V1 ( s ) 1
+R+
1 2 + RCs
No error is specified in input so Cs Cs
dx 1 + 10 × 10 × 100 × 10−6 s 1 + s
3
=0 = =
x 2 + 10 × 103 × 100 × 10−6 s 2 + s
dy
=∈1 + ∈2 − ∈3 . Q.14 (a)
y
No. of forward path =2
1 1
= P1 = 2
, P2
S S2

© Copyright Reserved by Gateflix.in No part of this material should be copied or reproduced without permission
∆1 = 1; ∇ 2 = 1 dx1
= 2x1 + x 2 + 4
−4 −2 dt
=L1 = , L2 dx 2
5 S2 = −2x1 + 4
−2 dt
L3 = −4, L 4 = y = 3x1
S
No non touching loops Considering the standard equation
∆k= 1[L1 + L 2 + L3 + L 4 ] =
x i AX + BU
4 2 5 =
y Cx + DU
=1 + + 2 + 4 +
S S S  x 1   2 1   x1  1
= x   −2 0   x  + 1 [4]
S + 4s + 2 + 4S + 4s
2 2
 2   2  
=
S2 x 
5s 2 + 6s + 2 y = [30]  1 
= x2 
S2
1 1 Transform function C(SI − A) −1 B
+ 2 −1
S 2
S s +1   s 0   2 1   1
= = =G ( s ) [3 0]  
5s + 6s + 2 5s + 6s + 2
2 2  −  −2 0   1
  0 s     
S2 −1
s − 2 −1 1
[3 0] 
Q.15 (c)  0 s  1
From the signal flow graph, s 1  1
C(s) [3 0]   
G (s) =  −2 s − 2  1
U(s)
s 2 − 2s + 2
By mason's gain relation,
1  s +1 
Transfer function = = 2 [3 0]  
P ∆ + P ∆ +… s −s+ 2  −2 + s − 2 
= 1 1 2 2
∆ 1  s + 1
= 2 [3 0]  
=P1 =
h1
; P2
h0 s − 2s + 2 s − 4 
S S2
 a  a a 3(s + 1)
∆1 = 1 + 1  ; ∆ 2 = 1 Δ=1+ 1 + 20 =
 s s s s − 2s + 2
2

Transfer function =
Q.19 0.554
h1  a 1  b 0
1+ +
s  s  s 2 b s+b Q.20 (c)
= 2 1 0
a a s +a1s+a 0
1+ 1 + 20
s s Option Characteristic Damping Damping
Equation Ratio ( ξ )
Q.16 (b)
P1 = G 2 P s 2 + 5s + 15 ξ =0.645 Under
∆ = 1 − [ −G1G 2 − G1 ] = 1 + G1 (1 + G 2 ) damping
Q s 2 + 10s + 25 ξ =1 Critical
P1∆1 G2
=
TF = damped
∆ 1 + G1 [1 + G 2 ] ξ =1.52
R s 2 + 18s + 35 Over
damped
Q.18 (a)

© Copyright Reserved by Gateflix.in No part of this material should be copied or reproduced without permission
Hence, the correct option is (C).

Q.21 (a)

Given: Characteristic equation,


s + s + 7s + 14s + 31s + 73s + 25s + 200
7 6 5 4 3 2

The R-H table is given by,


s7 1 7 31 25
s6 1 14 73 200
s 5
-7 -42 -175 0
s 4
8 48 200 0
s 3
32 96 0
s 2
24 200 0
s1
-170.67 0
s 0
200

From the above table number of sign


change in the first column is 4. So, the
number of left hand pole are
7 - 4 = 3.
8s 4 + 48s 2 + 200 =
0
s =x
2

8x 2 + 48x + 200 = 0
x + 6x + 25 =
2
0
x =−3 ± j4
s 2 =−3 ± j4

© Copyright Reserved by Gateflix.in No part of this material should be copied or reproduced without permission
GATE QUESTIONS(EE)(Block Diagram & SFG)

Q.1 As shown in the figure, a negative a) 1 b) 5


feedback system has an amplifier of c) 10 d) 100
gain 100 with ±10% tolerance in the [GATE-2013]
forward path, and an attenuator of
value 9/100 in the feedback path. Q.3 The closed-loop transfer function of
The overall system gain in 4
a system is (s) = 2 . The
approximately: ( s + 0.4s + 4 )
steady state error due to unit step
input is
[GATE-2014]

Q.4 The block diagram of a system is


a)10±1% b)10 ± 2% shown in the figure
c) 10 ±5% d) 10±10%
[GATE-2010]

Q.2 The open- loop transfer function of a


ω(s) 10
dc motor is given as = . If the desired transfer function of
Va (s) 1+10s C(s) s
When connected in feedback as
the system is = 2 then
R(s) s + s + 1
shown below, G(s) is
a)1 b)s
−s
c)1/s d) 3 2
s +s −s−2
[GATE-2014]
the approximate value of K a that
will reduce the time constant of
closed loop system by one hundred
times as compared to that of the
open- loop system is

ANSWER KEY:
1 2 3 4
(a) (c) (0) (b)

© Copyright Reserved by Gateflix.in No part of this material should be copied or reproduced without permission
EXPLANATIONS

(a)Q.1 Q.4 (b)


=
G 100 ± 10%
∆G
= 10%or0.1
G
H = 9 /100
Overall gain
G
T= …(i)
1 + GH If G(s) = S
100 C(s) S
=T = 10 = 2
9 R(s) s + s + 2
1 + 100 ×
100
dT (1 + GH ) − GH 1
= =
(1 + GH ) (1 + GH )
2 2
dG
dG
dT = …(ii)
(1 + GH )
2

∆T ∆G 1
= ×
T G (1 + GH )
∆T 1
= 10 × %
T 9
1 + 100 ×
100
So, overall system gain= 10 ± 1%

Q.2 (c)
10K
G ( s ) H ( S) = O.L.T.F
1 + 10s
τ = 10sec
10
=
C.L.T.F τ = 0.1
100
10K
C.L.T.F =
10K + 1 + 10s
10K
τ = 0.1 = ⇒ K ≅ 10
10K + 1

Q.3 (0)
Steady state error for Type-1 for
unit step input is 0.

© Copyright Reserved by Gateflix.in No part of this material should be copied or reproduced without permission
GATE QUESTIONS(IN)(Basics of Control Systems)

Q.1 The torque-speed curve of a Q.3 The transfer function of a Zero –


constant field armature controlled Order-Hold system with sampling
DC servomotor is shown in the interval T is
figure. The armature resistance in Ω 1
( ) 1
( )
2
a) 1 − e − Ts b) 1 − e − Ts
and torque constant in Nm/A of the s s
motor respectively are 1 − Ts 1
c) e d) 2 e − Ts
s s
[GATE-2012]

Q.4 Unit step response of a linear time


invariant (LT) system is given by
y(t) = (1 – e-at)u(t). Assuming zero
initial condition, the transfer
function of the system is
a) (1.76,0.68) b) (1.76,0.85) [GATE-2012]
c) (2.00,0.25) d) (0.01, 0.81) 1 2
[GATE-2004] a) b)
s +1 ( s + 1)( s + 2)
Q.2 The rotor of the control transformer
1 2
of a synchro pair gives a maximum c) d)
voltage of 1.0 V at a particular ( s + 2) ( s + 2)
position of the rotor of the control
transmitter. The transmitter rotor is
now rotated by 30o anticlockwise
keeping the transformer rotor
stationary. The transformer rotor
voltage for this position is
a) 1.0 V b) 0.566 V
c) 0.5 V d) 0 V
[GATE-2010]

ANSWER KEY:
1 2 3 4
(c) (b) (a) (d)

© Copyright Reserved by Gateflix.in No part of this material should be copied or reproduced without permission
EXPLANATIONS

Q.1 (c) y(t)= (1 − e −2t )u(t)


d
impulse response h(t) = y(t)
dt
δ(t) − e −2t δ(t) + 2e −2t u(t)
h(t) =
= 2e −2t u(t)
L{h(t)} = Transfer function
−2t 2
= =
T.F. L{2e 4(t)}
s+2
We know
m (t)
T= K m ( t ) φi a ( t )
Tm ( t ) = K ii a ( t ) (∵ ϕ = constant)
Among the options only the
armature resistance
=
R a 2.00 Ω will satisfy the torque
equation.
ea 4
ia ( t =
) = = 2.0Amp
Ra 2
T (t) 0.5
Ki = m = N − m/A = 0.25N-m/A
ia ( t ) 2.0
∴ K=i 0.25N − m/A

Q.2 (b)
=v r v m cos φ
= 1.0 cos 30
v r = 0.866v

Q.3 (a)
The transfer function of a zero-
order hold system having a
1
sampling interval T is 1 − e − Ts .
s
( )
Q.4 (d)

Given unit step response

© Copyright Reserved by Gateflix.in No part of this material should be copied or reproduced without permission
GATE QUESTIONS(IN)(Block Diagram & SFG)

Q.1 The signal flow graph


representation of a control system is
shown below. The transfer function
Y(s)
is computed as
R(s)

The transfer function (C/R) of the


system is
a)
( G1G 2 +G1G 3 )
(1+G1G 2 H 2 )
b)
( G1G 2 +G1G 3 )
a)
1
b)
S +1
2
(1-G1H1 +G1G 2 H 2 )
S S ( S2 + 2 ) ( G1G 2 +G1G 3 )
c)
S ( S2 + 1) 1 (1-G1H1 +G1G 2 H 2 +G1G 3H 2 )
c) d) 1−
S +2
2
S d)
( G1G 2 +G1G 3 )
[GATE-2006] (1-G1H1 +G1G 2 H 2 +G1G 3H 2 +G1G 2G 3H1 )
[GATE-2011]
Q.2 A feedback control system with high
K, is shown in the figure below: Q.4 The signal flow graph for a system is
given below. The transfer function
Y(s)
for this system is given as
U(s)

Then the closed loop transfer


function is.
a) Sensitive to perturbations in
G(s) and H(s)
b) Sensitive to perturbations in G(s) s +1 s +1
a) b)
and but not perturbations H(s) 5s + 6 s + 2
2
s + 6s + 2
2

c) Sensitive to perturbations in s +1 1
H(s) and but not to c) 2 d) 2
5s + 4 s + 2 5s + 6 s + 2
perturbations G(s) [GATE-2013]
d) Insensitive to perturbations in
G(s) and H(s)
[GATE-2007]

Q.3 The signal flow graph of a system is


given below.

© Copyright Reserved by Gateflix.in No part of this material should be copied or reproduced without permission
ANSWER KEY:
1 2 3 4
(a) (c) (c) (a)

EXPLANATIONS

Q.1 (a) −4 −2
= , L2 =L1
5 S2
−2
Q.2 (c) L3 = −4, L 4 =
S
1 No non touching loops
= SGT
→ 0 as k is
(1 + KG ( s ) .H(s) ) ∆k= 1[L1 + L 2 + L3 + L 4 ]
high 4 2 5
=1 + + 2 + 4 +
S S S
− kG ( S ) .H(s)
S + 4s + 2 + 4S + 4s
2 2
STH → −1
(1 + KG ( )
s .H(s) ) =
S2
5s 2 + 6s + 2
Q.3 (c) =
S2
1 1
= P1 G= 1G 2 ; P2 G1G 3 +
S2 S2 s +1
L1 = G1H1 ; L 2 = − G1G 2 H 2 ; L3 = =
5s + 6s + 2 5s + 6s + 2
2 2

= − G1G 3 H 2 S2
∆1 = 1; ∆ 2 = 1 .
∆ = 1 − ( L1 + L 2 + L3 )
=
1 − G1H1 + G1G 2 H 2 + G1G 3H 2
C ( P1∆1 + P2 ∆ 2 )
∴ =
R ∆

=
( G1G 2 + G1G 3 )
(1 − G1H1 + G1G 2 H 2 + G1G 3H 2 )
Q.4 (a)
No. of forward path =2
1 1
=P1 = 2
, P2
S S2
∆1 = 1; ∇ 2 = 1

© Copyright Reserved by Gateflix.in No part of this material should be copied or reproduced without permission
2 TIME DOMAIN ANALYSIS

2.1 INTRODUCTION 2.1.1 STANDARD TEST SIGNALS:


The various inputs affecting the
Time response of the output means performance of the system are
behavior of the response with respect to mathematically represented as standard
the time. In a practical system, output of Test signal.
the system takes some time to reach its I. Sudden input → Step Signal
final value. The final state achieved by the II. Velocity input → Ramp signal
system response (output) is called steady III. Acceleration input → Parabolic
state. signal
Following are the characteristics of the IV. Sudden shock → Impulse signal –
Steady State Response of a control system. Stability analysis
• The part of the time response that
remains even after the transients 2.2 TIME RESPONSE OF A SECOND
have died out, is said to be steady ORDER CONTROL SYSTEM FOR UNIT
state response. STEP INPUT:
• The steady state part of time
response reveals the accuracy of a A second order control system is one
control system. wherein the highest power of s in the
• Steady state error is observed if denominator of its transfer function equals
actual output does not exactly match 2. Transfer function of a second order
with the input. control system is given by
The state of output between application of C(s) ω2n
= 2 … (I)
input & steady state is called transient R(s) s + 2ζωn s + ω2n
state.
The parameter ξ and ωn will be explained
Following are the characteristics of the
later. The block diagram representation of
Transient State Response of a control
the transfer function given by above
system.
expression
• The part of the time response which
goes to zero after a large interval of
time, is known as transient
response.
• It reveals the nature of response.
• It gives an indication about the
speed of response. The response of the system is given by
e −ζωn t   1 − ζ 2  
Hence the total time response of the c(t) = 1 − sin  ωn 1 − ζ 2 + tan −1   
1 − ζ2  
 ζ   
system can be written as 
= Css + C t (t)
c(t) Where ωd = ωn 1 − ζ 2
 1− ζ2 
Note: and φ = tan −1  
The difference in the desired & actual  ζ 
 
output is called steady state error ess .

© Copyright Reserved by Gateflix.in No part of this material should be copied or reproduced without permission
α
ξ=
The error is given by ωn
e(t) = r(t) – c(t) And r(t) = 1 (unit step) actual damping factor
∴ e(t ) =
damping factor at critical damping(i.e.whenξ = 1)
 e −ζωn t   1 − ζ 2  
1 − 1 − sin  ωn 1 − ζ 2 + tan −1   
  ζ 
− ζ    
2
 1  
Or
e −ζωn t   1 − ζ 2 
e(t) = sin ωn 1 − ζ 2 + tan −1  
1− ζ2   ζ 
 
The steady state error is
e −ζω t n   1 − ζ 2 
ess = lim .sin ωn 1 − ζ 2 t + tan −1   The above figure shows loci of damping
t →∞
1− ζ2   ζ  ratio ξ. ξ is the cosine of the angle between
 
=0 the radial line to the roots and the negative
• The time response expression indicates axis when the roots and the negative axis
that for values of ξ < 1 the response when the roots are in the left-half s-plane,
presents exponentially decaying or ξ = cos θ.
oscillations having a frequency
2.2.2 NATURAL UNDAMPED FREQUENCY
ωn 1− ξ 2 and the time constant of
exponential decay is 1/ ξωn . When ξ = 0, the damping is zero, the roots
• The term ωn is called natural frequency of the characteristic equation are
of oscillations. imaginary, shows that the unit-step
• The term = ωd ωn 1 − ξ 2 is called
response is purely sinusoidal. Therefore,
ωn corresponds to the frequency of the
damped frequency of oscillations. undamped sinusoidal response.
2.2.1 DAMPING RATIO AND DAMPING
FACTOR

The two roots can be expressed as


s1 ,s 2 = ζωn ± jωn 1 − ζ 2
= −α ± jωd
Where
α = ζωn is called damping factor and
ω d = ωn 1 − ζ 2
The above figure shows the loci of ωn . ωn is
Note: the radial distance from the roots to the
α appears as the constant that is multiplied origin of the s-plane
to t in the exponential term, therefore α Note:
controls the rate of rise or decay of the 1) The left-half s-plane corresponds to
unit-step response. positive damping (i.e., the damping
Now, ξ is called as damping ratio & it is factor or damping ratio is positive).
given by Positive damping causes the unit-step
response to settle to a constant final

© Copyright Reserved by Gateflix.in No part of this material should be copied or reproduced without permission
value in steady state due to the negative
exponent of exp (- ξ ωnt). The system is
stable.
2) The right-half s-plane corresponds to
negative damping. Negative damping
gives a response that grows in
magnitude without bound with time,
and the system is unstable.
3) The imaginary axis corresponds to zero
damping (α = 0 or ξ = 0). Zero damping
results in a sustained oscillation
response, and the system is marginally
stable or marginally unstable.
4) 0 < ζ < 1: s1 ,s 2 =−ζωn ± jωn 1 − ζ 2 (-ζωn < 0)
Under damped
5) ζ = 1: s1, s2 = -ωn
Critically damped
6) ζ > 1:s1, s2 = - ζ ±ωn ζ 2 − 1 Over damped
7) ζ = 0:s1, s2 = ± jωn undamped 2.2.4 POLE LOCATIONS FOR DIFFERENT
8) ζ <0:s1, s2 = - ζ jωn + jωn 1 − ζ 2 VALUES OF 𝛏𝛏
negatively damped

2.2.3 TIME RESPONSE FOR DIFFERENT


VALUES OF 𝛏𝛏

© Copyright Reserved by Gateflix.in No part of this material should be copied or reproduced without permission
2.3 TIME RESPONSE SPECIFICATION =
Where n 1, 2,3, 4,5, 6 ………
Note:
In specifying the transient response • n = 1 for 1st over shoot (+ve peak)
characteristics of a control system to a unit- • n = 2 for 1st undershoot (-ve peak)
step input, it is common to specify the • n = 3 for 2nd over shoot
following: • n = 4 for 2nd undershoot
1. Delay time, td
2. Rise time, tr 1) Peak Overshoot 𝐌𝐌𝐏𝐏 : The maximum
3. Peak time, tp overshoot is the maximum peak value
4. Maximum overshoot, Mp of the response curve measured form
5. Setting time, ts unity. If the final steady-state value of
These specifications are defined in what the response differs from unity, then it
follows and are shown graphically in figure. is common to use the maximum percent
y(t) Maximum
Unit-step input overshoot overshoot. It is defined as
c ( tp ) − c (∞)
1.05
1.00
0.95
= %M P ×100%
c (∞)
0.90

= e − πξ / 1−ξ ×100%
2

0.50 The amount of the maximum (percent)


overshoot directly indicates the relative
Delay
time

stability of the system.


td

0.10
0 2) Settling time 𝐭𝐭 𝐬𝐬 : The setting time is the
~~

t
Rise time
tr Setting
time required for the response curve to
time ts reach and stay within a range about the
Figure: Typical unit-step response of a control system
final value of size specified in
1) Delay time 𝐭𝐭 𝐝𝐝 : The delay time is the
percentage of the final value (usually
time required for the response to reach
2% or 5%).
half the final value in the first attempt.
a) Settling time for 2% transition
1 + 0.7ξ
td = band: It is the time taken by the
ωn oscillations to decrease and stay
2) Rise time 𝐭𝐭 𝐫𝐫 : The rise time is the time within a limit of 2 % of the final
required for the response to rise from value & it is given by
10% to 90% of its final value for over 1
damped system. For under damped Ts = 4T where T =
ξωn
second-order systems, the 0% to 100%
4
rise time is normally used. ∴ Ts =
π −θ ξωn
tr = b) Settling time for 5% transition
ωd
band: It is the time taken by the
1− ξ2 oscillations to decrease and stay
Where, θ = tan −1
ξ within a limit of 5% of the final
3) Peak time 𝐭𝐭 𝐩𝐩 : The peak time is the time value & it is given by
required for the response to reach the 1
Ts = 3T where T =
first peak of the overshoot. ξωn
nπ nπ 3
=tp = ∴ Ts =
ωd ω n 1 − ξ 2 ξωn

© Copyright Reserved by Gateflix.in No part of this material should be copied or reproduced without permission
sR(s)
ess = lim
2.4 STEADY STATE ERROR s →0 1 + G (s) H (s)
1

It is the difference between the actual = lim = s
s →0 1 + G (s) H (s)
output and the desired output. The steady
state performance of a control= system is =
1 1
assessed by the magnitude of the steady 1 + lim G ( s ) H ( s ) 1 + K P
s →0
state error possessed by the system and the
Where,
system input specified as either step or
ramp or parabolic. K P = lim G ( s ) H ( s ) is called positional
s →0

error constant.
Case ‘a’: For type ‘0’
K P = constant
∴ ess = constant
Case ‘b’: For type ‘1’:
KP → ∞
The error signal generated after comparing
input & feedback signal is given by 1
∴ ess= = 0
E(s) = R(s) – B(s) 1+ ∞
Where, B(s) is the feedback signal & it is Case ‘c’: For type ‘2’:
given by KP → ∞
B(s) = H(s)C(s) 1
∴ ess= = 0
Now, the output C(s) = G(s)E(s) 1+ ∞
∴ E(s) = R(s) – H(s)G(s)E(s) Note:
1 For the same type of input, as the system
⇒ E(s) = R(S) type increases the steady state error
1 + G (s) H (s ) decreases.
This E(s) is the error in Laplace domain &
the corresponding error in time domain is 2.4.2 STEADY STATE ERROR FOR RAMP
e(t). Now the steady state error is the error INPUT
when t → ∞. 1
i.e. ess = lim e(t) For ramp input, R(s) = 2
t →∞ s
from the final value theorem We know that,
lim e(t) = lim sE(s) s× 2
1
t →∞ s →0
sR(s) s
=
∴ ess lim = = ess lim = lim
s →0
sE(s) lim
s →0 s → 0 1 + G (s) H (s ) s → 0 1 + G (s ) H (s)
sR(s) 1 1
1 + G (s) H (s) = =
lim s + lim sG ( s ) H ( s ) K v
s →0 s →0

2.4.1 STEADY STATE ERROR FOR UNIT Where, K V = lim sG ( s ) H ( s ) is called


s →0
STEP INPUT velocity error constant.
1 Case ‘a’: For type ‘0’
For unit step input, R(s) =
s = K v lim= sG ( s ) H ( s ) 0
We know that, s →0

1
∴ ess == ∞
Kv

© Copyright Reserved by Gateflix.in No part of this material should be copied or reproduced without permission
Case ‘b’: For type ‘1’: R(s) + + 8 C(s)
s(s+2)
K v = constant
1
∴ ess = =constant as
Kv
Case ‘c’: For type ‘2’: a) In the absence of derivative feedback (a
Kv → ∞ = 0), determine the damping factor and
1 natural frequency. Also determine the
∴ ess = = 0 steady-state error resulting from a unit-

Note: ramp input.
As the unit input changes from unit step to b) Determine the derivative feedback
ramp and ramp to parabola the steady state constant which will increase the
error increases for the same type. damping factor the system to 0.7. What
is the steady-state error to unit-ramp
2.4.3 STEADY STATE ERROR FOR input with this setting of the derivative
PARABOLIC INPUT feedback constant?
1 Solution
For parabolic input, R(s) = a) With a = 0, the characteristic equation
s3
when derivative feedback is zero, the
We know that,
system will be a unity feedback system
1
s× 3 with transfer function
sR(s)
ess lim= lim s C (s) 8
s →0 1 + G ( s ) H ( s ) s →0 1 + G (s) H (s) = 2
R ( s ) s + 2s + 8
1 1
= d 2θ dθ
lim s + lim s G ( s ) H ( s ) K a
2 2
= + 10 150E, Where(r − θ)is the
s →0 s →0 2
dt dt
Where, K a = lim s 2 G ( s ) H ( s ) is called The characteristics equation is
s →0

acceleration error constant. s 2 + 2s + 8 = 0


Equating with the standard form
Case ‘a’: For type ‘0’ s 2 + 2ξωn s + ω2n =0
=
K a lim s2G ( s ) H ( s ) 0 We get ω= =8 2 2rad / sec
s →0 n

1 And 2ξωn = 2
∴ ess == ∞
Ka 1
∴ ξ= = 0.353
Case ‘b’: For type ‘1’: 2 2
Ka = 0 Now for ramp input the velocity error
1 constant is
∴ ess == ∞ 8 8
Ka KV = lim sG ( s ) H ( s ) =
s× = =
4
Case ‘c’: For type ‘2’:
s →0 s (s + 2) 2
K v = constant 1
ess ( to unit − ramp ) = =0.25
∴ ess = constant 4
b) With derivative feedback, the transfer
function is
Example: C (s) 8
= 2
The system illustrated in Fig. is a unity R ( s ) s + (2 + 8a)s + 8
feedback control system with a minor The characteristics equation is
feedback loop (output derivative feedback). s 2 + (2 + 8a)s + 8 =0

© Copyright Reserved by Gateflix.in No part of this material should be copied or reproduced without permission
Equating with the standard form e − ξπ/ 1− ξ 2
= 0.25 or
s 2 + 2ξωn s + ωn2 =
0 ξ = 0.404
We get The peak time t p is specified as 2 sec. And
ωn= 8= 2 2rad / sec And so
2ξωn= 2 + 8a π
=t p = 2 or ωd = 1.57
2 × 0.7 × 2 2 =2 + 8a ωd
⇒ a= 0.245 Then the undamped natural frequency ωn
Now for ramp input the velocity is
error constant is ωd 1.57
=ωn = = 1.72
K V = lim sG ( s ) H ( s ) 1− ζ2 1 − 0.4042
s →0

8 8 There, we obtain
=
s× = K =ω2n =1.722 =2.95 N − m
s ( s + 2 + 8a ) 2 + 8a
2 + 8a 2ζωn 2 × 0.404 ×1.72
∴ ess ( to unit − ramp ) = = k = = 0.471sec
8 K 2.95
= 0.495
Example:
Example: A servomechanism is represented by the
Determine the values of K and k of the equation actuating signal. Calculate the
closed-loop shown in fig. so that the value of damping ratio, undamped and
maximum overshoot in unit-step response damped frequency of oscillations
is 25% and the peak time is 2 sec. Assume Solution:
that J = 1 kg-m2. d 2θ dθ
2
+ 10 =150 E Or
dt dt
C(s)
R(s) + + K 1 d 2θ dθ
- - Js s
2
+ 10= 150(r − θ)
dt dt
The equation in Laplace domain is
k s 2 θ(s) +=10sθ(s) 150 ( R(s) − θ(s) )
θ(s) 150
Solution: = 2
R(s) s + 10s + 150
The closed-loop transfer function is
Comparing this with
C(S) K
= 2
ω
2
R(S) Js + Kks + K n

+ 2ζ ω s + ω
2
By substituting J = 1 kg-m2 into this last s2 n n
equation, we have ω2n =150
C(S) K
= 2 10
R(S) s + Kks + K ∴ ζ= =0.41
2 ×12.25
Now equating with the standard transfer
ω2n =150
function
ωn = K ∴ ωn =1225rad / sec
The maximum overshoot Mp is 10
2ζωn = 10 ∴ ζ= =0.41
− ξπ/ 1− ξ 2 2 ×12.25
=M p e= 0.25
ωd = ωn 1 − ζ 2
From which
= 12.25 1 − 0.412
= 11.17rad / sec

© Copyright Reserved by Gateflix.in No part of this material should be copied or reproduced without permission
Example Solution
The open loop transfer function of a unity C(s) K/T
=
feedback system is given by R(s) s 2 + 1 s + K
K
G (s) = T T
s(1 + sT) Comparing with
Where, T and K are constants having C(s) ω2n
positive values. By what factor amplifier =
R(s) s 2 + 2ζωn s + ω2n
gain be reduced so that
a) The peak overshoot of unit step = K 1
ω2n and =
2ζωn
response of the system is reduced from T T
75% to 25% Therefore ζ =
1
b) The damping ratio increases from 0.1 to 2 KT
0.6 Let
ζ1 =0.1 when gain is K1 and
ζ 2 =0.6 when gain is K 2
Solution
1 1
G (s) =
K ∴ 0.1= and 0.6 =
s(1 + sT) 2 K1T 2 K 2T
Let the value of damping ration is ζ1 when K1
or K 2 =
the peak overshoot is 75% and ζ 2 when 36
peak overshoot in 25% Therefore the gain should be reduced by a
−πζ factor 36
Mp = e Let ζ 1 be the damping ratio when the
1− ζ2
percentage overshoot is 60%
For M p = 75% −ζ × 3.14
∴ 0.60 = e 1 or ζ1 =0.1604
ξ = ξ1 = 0.091 1 − ζ 21
and for M P = 25% ζ = ζ 2 = 0.4037 Similarly
Transfer function −ζ × 3.14
G(s) K C(s) = 0.15 e 2 = or ζ 2 0.52 or
= = = 1 − ζ 2

1 + G(s)H(s) Ts 2 + s + K R(s) 2

ζ1 K1 0.1604 K2
or
C(s)
=
K/T = = or
R(s) s 2 + 1 s + K ζ2 K 2 0.52 K1
T T K
K2 = 1
Therefore 10.51
1 Therefore gain should be reduced by a
= ωn K / T=and 2ζωn
T factor 10.51
Example
The open loop transfer function of a unity
K
feedback system is G(s) = . Find by
s(1 + Ts)
what factor the gain K be reduced so that
the overshoot is reduced from 60% to 15%
(b) Find by what factor the gain K should
be reduced so that the damping ratio is
increased from 0.1 to 0.6

© Copyright Reserved by Gateflix.in No part of this material should be copied or reproduced without permission
GATE QUESTIONS(EC)

Q.1 If the characteristic equation of a a)


closed –loop system is S2 + 2s + 2 =0
, then the system is
a)overdamped
b)critically damped
c)underdamped
d)undamped
[GATE -2001]

Q.2 Considers a system with the transfer


s+6
function ( s ) = 2 . Its b)
Ks + s + 6
damping ratio will be 0.5 when the
value of K is
a)2/6 b)3
c)1/6 d)6
[GATE -2002]

Q.3 The transfer function of system is


G (s) =
100
.For a unit- c)
( s + 1) (s + 100)
step input to the system the
approximate settling time for 2%
criterion is
a)100 sec b)4 sec
c)1 sec d)0.01 sec
[GATE -2002]

Q.4 A second-order system has the d)


C(s) 4
transfer function = 2 .
R(s) s + 4s + 4
With r(t) as the unit-step function ,
the response c(t) of the system is
represented by

[GATE -2003]

© Copyright Reserved by Gateflix.in No part of this material should be copied or reproduced without permission
Q.5 A causal system having the transfer 5 1
c) d)
function H ( s ) =
1
is excited with s + s +1
2
s + s +1
2

s+2 [GATE -2007]


10u ( t ) . The time at which the
Q.9 Step response of a set of three
output reaches 99% of its steady second-order under damped systems
state value is all have the same percentage
a)2.7sec b)2.5 sec overshoot. Which of the following
c)2.3 sec d)2.1 sec diagrams represents the poles of
[GATE -2004] three systems?
Q.6 In the derivation of expression for a)
peak percent overshoot, M p = exp
 −πζ 
  ×100 Which one of the
 1− ζ2 
 
following conditions is NOT
required? b)
a) System is linear and time
invariant
b) The system transfer function has
a pair of complex conjugate
poles and no zeros
c) There is no transportation delay
in the system c)
d) The system has zero initial
conditions.
[GATE -2005]

Q.7 A ramp input applied to an unity


feedback system results in 5%
steady state error. The type number
and zero frequency gain of the d)
system are respectively
a)1 and 20 b) 0 and 20
c)0 and 1/20 d)1 and 1/20
[GATE -2005]

Q.8 The transfer function of a plant is


5
(s) = . The second- [GATE -2008]
( s + 5) (s 2 + s + 1)
order approximation of T ( s ) using Q.10 Group I lists a set of four transfer
dominant pole concept is functions. Group II gives a list of
1 5 possible step responses (t). Match
a) b) the step responses with the
( s + 5) (s + 1) ( s + 5) (s + 1) corresponding transfer functions.
Group I

© Copyright Reserved by Gateflix.in No part of this material should be copied or reproduced without permission
25 −2.24
P= c)
S + 25
2
s − 2.59 s + 1.12
2

36 −3.82
Q= 2 d) 2
S + 20s + 36 s − 1.91s + 1.91
36 [GATE -2009]
R= 2
S + 12s + 36
36 Q.12 The differential equation
S= 2 d2 y
S + 7s + 49 dy
100 2 − 20 + y = x(t) describes a
Group II dt dt
1) system with an input x(t) and an
output y(t) .The system, which is
initially relaxed, is excited by a unit
step input. The output y(t) can be
represented by the waveform
a)
2)

b)

3)

c)

4)

d)

a)P-3,Q-1,R-4,S-2 b)P-3,Q-2,R-4,S-1
c)P-2,Q-1,R-4,S-3 d)P-3, Q-4,R-1,S-2
[GATE -2008]
[GATE -2011]
Q.11 The unit step response of an under-
damped second order system has Q.13 The open- loop transfer function of a
steady state value of -2 .Which one ω(s) 10
dc motor is given as = .
of the following transfer functions Va (s) 1+10s
has these properties? When connected in feedback as
−2.24 shown below,
a) 2
s + 2.59 s + 1.12
−3.82
b) 2
s + 1.91s + 1.91

© Copyright Reserved by Gateflix.in No part of this material should be copied or reproduced without permission
[GATE-2014]

Q.17 For the second order closed-loop


system shown in the figure, the
natural frequency (in rad/s) is

the approximate value of K a that


will reduce the time constant of
closed loop system by one hundred
times as compared to that of the
a) 16 b) 4
open- loop system is
a)1 b)5 c) 2 d) 1
c)10 d)100 [GATE-2014]
Q.18 The characteristic equation of a
[GATE-2013]
unity negative feedback system 1 +
Q.14 For the following feedback system KG(s) = 0. The open loop transfer
function G(s) has one pole at 0 and
1
G (s) = . The 2% two poles at -1. The root locus of the
(S + 1) + (S + 2 ) system for varying K is shown in the
settling time of the step response is figure.
required to be less than 2 seconds.

Which one of the following


compensators C(s) achieves this?
 1   0.03 
a) 3   b) 5  + 1
s+5  s 
 s+8 The constant damping ratio line, for
c) 2 ( s + 4 ) d) 4   ζ = 0.5, intersects the root locus at
s+3
[GATE-2014] point A. The distance from the origin
to point A is given as 0.5. The value
Q.15 The natural frequency of an of K at point A is____________.
undamped second-order system is [GATE-2014]
40 rad/s. If the system is damped
with a damping ratio 0.3, the Q.19 The output of a standard second-
damped natural frequency in order system for a unit step input is
rad/s is ________. given as
[GATE-2014] 2 −t  π
y ( t ) = 1− e cos  3t −  . The
3  6
Q.16 The steady state error of the system transfer function of the system is
shown in the figure for a unit step a) 2 b) 2 1
input is _________. ( s + 2 ) (s + 3) s + 2s + 1
3 4
c) d)
s + 2s + 3
2
s + 2s + 4
2

[GATE-2015]

© Copyright Reserved by Gateflix.in No part of this material should be copied or reproduced without permission
Q.20 The open-loop transfer function of a Q.23 The open loop transfer function
unity-feedback control system is (𝑠𝑠+1)
G(s)=𝑠𝑠𝑝𝑝 (𝑠𝑠+2)(𝑠𝑠+3)
K
given by G ( s ) =  . Where p is an integer, is connected
s(s + 2)
in unity feedback configuration as
For the peak overshoot of the shown in the figure.
closed-loop system to a unit step
input to be 10%, the value of K is
__________.
[GATE-2016]

Q.21 For the unity feedback control Given that the steady state error is
system shown in the figure, the zero for unit step input and is 6 for
open-loop transfer function G(s) is unit ramp input, the value of the
2 parameter p is __________
given as G ( s ) =
s(s + 1) [GATE-2017-01]

The steady state error e ss due to a


unit step input is
a) 0 b) 0.5
c) 1.0 d) ∞
[GATE-2016]

Q.22 In the feedback system shown


1
below G ( s ) = 2 .
(s + 2s)
The step response of the closed-loop
system should have minimum
setting time and have no overshoot.

The required value of gain k to


achieve this is
[GATE-2016]

ANSWER KEY:
1 2 3 4 5 6 7 8 9 10 11 12 13 14
(c) (c) (b) (b) (c) (c) (a) (d) (c) (d) (b) (a) (c) (c)
15 16 17 18 19 20 21 22 23
38.15 0.5 (c) 0.375 (d) 2.86 (a) 1 1

© Copyright Reserved by Gateflix.in No part of this material should be copied or reproduced without permission
EXPLANATIONS

Q.1 (c) =
2ξω n 4,=
ωn 2
S2 + 2ξωn + ω2n =0 ∴ξ = 1(Critical damping)
1 4 4
2ξω= 2,=ξ =
ts = = 2
ωn
n
ξωn 1× 2
ωn = 2
1 Q.5 (c)
= ξ < 1(underdamped) 1
2 H (s) =
s+2
Q.2 (c) r ( t ) = 10u ( t ) .
s+6 10
G (s) = R (s) =
 s 6 s
K  s2 + + 
 K K C ( s ) H=
= ( s ) .R ( s )
1 10
.
Comparing with S2 + 2ξωn + ω2n s+2 s
10 A B
6 ⇒ = + 10 =
ωn = s ( s + 2) s s + 2
K
1 A ( s + 2 ) + Bs
2ξωn = s=0,10=2A
K
⇒A= 5
6 1
2 × 0.5 × = s= −2,10 = −2B
K K ⇒B= −5
6 1
⇒ =2 ∴ C ( s ) =−
5 5
K K s s+2
1
K= c (=
t ) 5 1 − e −2t 
6
Steady state value when t=0 is
Q.3 (b) 5.99% of steady state value reaches
100 at
G (s) = 5 1 − e −2t  =0.99 × 5
( s + 1) (s + 100)
Taking dominant pole ⇒ 1 − e −2t =0.99
consideration, e −2t = 0.1
s = −100 pole is not taken. ⇒ −2t =ln0.1
100 ⇒t= 2.3sec
∴ G (s) =
s +1
Now it is 1st order system Q.6 (c)
∴ t s = 4T = 4 × 1 = 4s
Q.7 (a)
1
Q.4 (b) = =
eSS lim sE ( s ) , R ( s )
C(s) 4
s→0 S2
= 2
R(s) s + 4s + 4

© Copyright Reserved by Gateflix.in No part of this material should be copied or reproduced without permission
R(s) 36
= lim s R=
s → 0 1 + G(s) s + 12s + 36
2

1 12
= lim
= finite(given) ⇒=ξ = 1
s → 0 S + sG(s) 2× 6
⇒ Ris critically damped
K v = lim sG ( s )
s→0 49
1 1 S= 2
=
eSS lim = finite = 5% = s + 7s + 49
s → 0 sG ( s ) 20 7
⇒=ξ = 0.5
∴K = 20 2× 7
K v is finite for type 1 system having ⇒ S is under damped
ramp input.
Q.11 (b)
Q.8 (d) Steady state Value = -2
In dominant pole concept, the factor Denominator:
that has to be eliminated should be =2ξωn 1.91,
= ωn2 1.91
in time constant form. ⇒ ωn ≅ 1.4
5 5
= 1.91 1.91
( s + 5) (S + s + 1) 5  1 + s  (S2 + s + 1)
2 =ξ = < 1 …under damped
  2ωn 2.8
 5
1 Q.12 (a)
= 2
S + s +1 d2 y dy
100 2
− 20 + y = x(t)
dt dt
Q.9 (c)
Taking Laplace transform of both
Peak overshoot depends on ξas
sides
− ξπ/ 1− ξ 2 100s 2 Y ( s ) − 20sY ( s ) + Y ( s ) =
X (s)
Mp = e Y(s) 1 1
⇒= =
Where ξ = cos −1 θ X(s) 100s − 20s + 1 (10s − 1) 2
2

Where θ is the angle made by pole 1 1


from negative real axis .To make M p Poles are at s = ,
10 10
same, θ should be the same. As poles are on the right –hand side
of s-plane so given system is
Q.10 (d) unstable system. Only option (a)
Comparing the given transfer represents unstable system.
function with
ω2n Q.13 (c)
s 2 + 2ξωn S + ω2n G ( s ) H ( S) =
10K
O.L.T.F
25 1 + 10s
=
In P = ξ 0 τ = 10sec
s + 25
2

Therefore P is undamped 10
=
C.L.T.F τ = 0.1
36 100
Q= 2 10K
s + 20s + 36 C.L.T.F =
20 10K + 1 + 10s
⇒= ξ = 1.67 10K
2× 6 τ = 0.1 = ⇒ K ≅ 10
⇒ Qis over damped 10K + 1

© Copyright Reserved by Gateflix.in No part of this material should be copied or reproduced without permission
Q.14 (c) If we compare with standard 2nd
By observing the options, if we place order system transfer function
other options, characteristic equation wn2
will have 3rd order one, where we i.e., 2
s + 2ξw n s + w n 2
cannot describe the settling time.
If C(s) = 2(s+4) is considered wn2 =4 ⇒ wn =2rad / sec
The characteristic equation, is
s2 +3s+2+2s+8= 0 Q.18 (0.375)
s2 +5s+10=0 We know that the co-ordinate of
Standard character equation point A of the given root locus i.e.
s 2 + 2ξωn s + ω2n =
0 magnitude condition
=ω2n 10;ξωn 2.5
= G ( s ) H(s) = 1
Given, 2% settling time, Here, the damping factor ξ = 0.5 and
4 the length of 0A = 5 ξ = 0.5
⇒ < 2 ⇒ ξωn > 2
ξωn

Q.15 (38.15 r/ sec)


Given ωn = 40 r / sec
ξ = 0.3
ωd =ωn 1 − ξ 2

ωd = 40 1 − ( 0.3)
2

ωd = 38.15r / sec Then in the right angle triangle


OX OX 1
cos θ = ⇒ cos 60 = ⇒ OX =
Q.16 (0.5) OA 0.5 4
4 2 AX AX 3
=
Given G (s) = H (s)   ⇒ sin θ =⇒ sin 60 = ⇒ AX =
s+2 s+4 OA 0.5 4
For unit step input, So, the co-ordinate of point A is
 k p = lim G(s)H ( s ) -1 + j 3
s →0
4 4
 4  2  Substituting the above value of A in
k p = lim   
s →0 s + 2
  s + 4  the transfer function and equating
kp = 1 to 1
A i.e. by magnitude condition,
Steady state error = ess =
1+ kp k
=1
ess =
1 s(s+1) 2 s= -1 + j 3
1+1 4 4
2
1
ess = ⇒ 0.50 1 3  9 3 
k= + . + 
16 16  16 16 
2

Q.17 (c) k = 0375


Y(s) 4
Transfer function = 2
U(S) S + 4s + 4 Q.19 (d)
Here ξωn =1

© Copyright Reserved by Gateflix.in No part of this material should be copied or reproduced without permission
3
1-ξ 2 = Q.22 (1)
2
1 Minimum setting time and no
ξ= overshoot implies case of critical
2 damping.
ωn = 2 At critical damping ζ = 1.
k
Q.20 (2.86) H (s ) = 2
s + 2s + k
K = 2.86
Peak over shoot 10% ωn = k
−πξ
2ζωk = 2 ⇒ 2.1× k = 2 ⇒ k = 1
1−ξ2
⇒e =
0.1
2
 −πξ 
⇒  =
[1n0.1]2
 1 − ξ 
2

1 − ξ2  π 
2

⇒ =1n0.1 
ξ2
 π 
2
1
⇒ 2= 1 + 
ξ 1n0.1 
1
⇒ 2 = 2.86
ξ
1
⇒= ξ 2 = 0.34
2.86
⇒ ξ =0.59
→ The characteristic equation of
above transfer function is
s + 2s + k =
2
0
Comparing with standard equation
s 2 + 2ξωn s + ωn 2 = 0
⇒ 2ξ k =
2
2
⇒ k=

2
2 1
⇒ k  = = 2.86
 2ξ  ξ2
 k = 2.86

Q.21 (a)
1
For unit step input ess =
1+ kp
2
k p = lim G(s) = lim = ∞
s →0 s → 0 s(s + 1)

1
So, ess = =0
1+ ∞

© Copyright Reserved by Gateflix.in No part of this material should be copied or reproduced without permission
GATE QUESTIONS(EE)

Q.1 The block diagram shown in figure


gives a unity feedback closed loop
control system. The steady state
error in the response of the above
system to unit step input is

a) 1 b) 0.25
c) 0.1 d) 0
[GATE-2007]

a) 25% b) 0.75% Statement for common data Questions


c) 6% d) 33% Q.4 and Q.5:
[GATE-2003] R-L-C circuit shown in figure

Q.2 The block diagram of a closed loop


control system is given by figure.
The values of K and P such that the
system has a damping ratio of 0.7
and an undamped natural frequency
ωn of 5rad/sec, are respectively
equal to
Q.4 For a step-input ei , the overshoot in
the output e0 will be
a) 0, since the system is not under
damped
b) 5%
c) 16%
a) 20 and 0.3 b) 20 and 0.2 d) 48%
c) 25 and 0.3 d) 25 and 0.2 [GATE-2007]
[GATE-2004]
Q.5 If the above step response is to be
Q.3 Consider the feedback system observed on a non-storage CRO,
shown below which is subjected to a then it would be best have the ei as a
unit step input. The system is stable a) step function
and has following parameters b) square wave of 50Hz
k p = 4, k i = 10, ω =500 and c) square wave of 300Hz
ξ =0.7 . The steady state d) square wave of 2.0KHz
value of
[GATE-2007]
Z is

© Copyright Reserved by Gateflix.in No part of this material should be copied or reproduced without permission
Q.6 The transfer function of a linear input r(t) having a magnitude of 10
time invariant system is given as and a duration of one second, as
1 shown in the figure is
G (s) = 2
s + 3s + 2
The steady state value of the output
of the system for a unit impulse
input applied at time instant t=1 will
be
a) 0 b) 0.5
c) 1 d) 2 a) 0 b) 0.1
[GATE-2008] c) 1 d) 10
[GATE-2011]
Q.7 The transfer function of a system is Q.11 A two-loop position control system
100 in above shown fig
given as 2 . The system is
s +20s+100
a) an overdamped system
b) an underdamped system
c) a critically damped system
d) an unstable system
[GATE-2008]

Q.8 The unit-step response of a unity The gain k of the Tacho-generator


feedback system with open loop influences mainly the
transfer function a) peak overshoot
G(s)=K/((s+1)(s+2)) is shown in the b) natural frequency of oscillation
figure. The value of K is c) phase shift of the closed loop
transfer function at very low
frequencies (ω→0)
d) phase shift of the closed loop
transfer function at very high
frequencies(ω→∞)
[GATE-2011]
a) 0.5 b) 2
c) 4 d) 6 Q.12 An open loop control system results
[GATE-2009] in a response of e-2t (sin5t + cos5t)
for a unit impulse input. The DC gain
2 of the control system is __________.
Q.9 For the system , the [GATE-2015]
(s + 1)
approximate time taken for a step
Q.13 The unit step response of a system
response to reach 98% of its final
with the transfer function
value is
1 − 2s
a) 1s b) 2s G (s) = is given by which one
c) 4s d) 8s 1+ s
[GATE-2010] of the following waveforms?
a)
Q.10 The steady state error of a unity
feedback linear system for a unit
step input is 0.1. The steady state
error of the same system, for a pulse

© Copyright Reserved by Gateflix.in No part of this material should be copied or reproduced without permission
100
c)
s + 10s + 100
2

102
d) 2
s + 5s + 100
[GATE-2016]

Q.15 Consider a unity feedback system


with forward transfer function is
b) given by,
1
G (s) =
( s + 1)( s + 2 )
The steady state error in the output
of the system for a unit-step input is
_____ (up to 2 decimal places).
[GATE-2018]
c)
Q.16 The unit step response y(t) of a
unity feedback system with open
loop transfer function
K
G (s) H (s) = is shown
( s + 1) ( s + 2 )
2

in the figure. The value of K is


_________ (up to 2 decimal places).
d)

[GATE-2018]
[GATE-2015]

Q.14 A second-order real system has the


following properties:
a) the damping ratio ζ = 0.5 and
undamped natural frequency
ωn = 10rad / s
b) the steady state value of the
output, to a unit step input, is
1.02.
The transfer function of the system is
102
a) 2
s + 5s + 100
102
b) 2
s + 10s + 100

© Copyright Reserved by Gateflix.in No part of this material should be copied or reproduced without permission
ANSWER KEY:
1 2 3 4 5 6 7 8 9 10 11 12 13 14
(a) (d) (a) (c) (c) (a) (c) (d) (c) (a) (a) 0.241 (a) (b)
15 16
0.67 8

© Copyright Reserved by Gateflix.in No part of this material should be copied or reproduced without permission
EXPLANATIONS

Q.1 (a) Comparing with standard equation


 3   15  = s 2 + 2ξωn s + ω2 n
G (s) 
= × 
 s + 15   s + 1  k ω2=
= n 5=
2
25
=
45
& H (s) 1 (where ωn = undamped natural
( s + 1) (s + 15) frequency)
Open loop transfer function 2ξωn= 2 + kP
45 ⇒ 2 × 0.7 × 5 = 2 + 25P
= G=( s ) .H ( s )
( s + 1) (s + 15) (where ξ = damping ratio)
The system is type-0 P=0.2
Steady state error to unit-step input
where Q.3 (a)
k p = Position error constant Step input ⇒ R ( s ) =
1/ s
LT
= G ( s ) H(s)
s→0
LT
⇒ = G ( s ) H(s)
s→0
LT 45
=kp = 3
s → 0 ( s + 1) (s + 15)
1 1  k  ω2 
=
ess = =  kp + i   2 2 
1+ kp 1+ 3 G(s)
 s   s + 2ξωs + ω 
= 0.25or25% and H(s)=1
C(s) G(s) G(s)
Q.2 (d) = =
R(s) 1 + G ( s ) H(s) 1 + G(s)
k
G (s)= andH ( s ) = 1 + sP ( s ) R ( s ) − C(s)
E=
s ( s + 2)
Closed-loop transfer function  C (s) 
= R ( s ) 1 − 
T (s) =
G (s)  R (s) 
1 + G ( s ) H(s)  G(s)  R(s)
=R ( s ) 1 −  =
k / s(s + 2)  1 + G(s)  1 + G(s)
=
k ki
1+ .(1 + sP) .R(s)
s ( s + 2) Z (s) =
=
ki
.E ( s ) s
k s 1 + G(s)
= Steady state value of Z
s ( s + 2 ) + k(1 + sP)
LT
k Zss = s Z(s)
⇒ T (s) = s→0
s + ( 2 + kP ) s + k
2

So, characteristic equation


= s 2 + ( 2 + kP ) s + P

© Copyright Reserved by Gateflix.in No part of this material should be copied or reproduced without permission
ki 1 R
LT . s. 2ξωn =
= s s L
s→0  ki   ω2  R 1 R LC R C
1+  kp +   2 2  ξ= × =× =
 s   s + 2ξωs + ω  L 2ωn L 2 2 L
10 10 × 10−6
ki =ξ = 0.5
= = 1 2 1× 10−3
ω2
ki . 2
ω Overshoot =Pp = e − πξ/ 1− ξ 2

= e − π×0.5 1− 0.52

= 0.163 or 16.3%M
Q.4 (c)
Q.5 (c)
1
ωn =
Lc
1
=
1× 10 × 10 × 10−6
−3

di 1
dt c ∫
ei =R i + L + idt
= 104 rad/ sec
Taking Laplace transform
 1 
E i (s) =  R + Ls +  I(s)
 Cs 
E i (s)
I(s) =
1
R + Ls +
Cs
1
e0 = ∫idt 4
c Settling time ( t s ) =
  ξωn
1 1  Ei ( s )  4
⇒ E0 ( s ) = I (s) =   = = 0.8msec
Cs Cs  R + Ls + 1  10 × 0.5
4

 Cs  For a square wave T/2 should be


E i (s) E0 ( s ) greater than t s
t0 (s) =
RCs + LSC + 1 E i ( s )
2
For f1 = 50Hz
1 T1 1
= ⇒= = 10ms >> t s
 R 1  2 2 × 50
LC  s 2 + s + 
 L LC  For f 2 = 300Hz
Characteristic eq. T2 1
R 1 ⇒= = 1.67ms > t s
⇒ s2 + s + = 0 2 2 × 300
L LC For f 3 = 2kHz
Comparing with s 2 + 2ξωn s + ωn 2 T3 1
1 ⇒= = 0.25ms > t s
ωn = 2 2 × 2 × 103
Lc Therefore, it would be best to have
ei as a square wave of 300Hz.

© Copyright Reserved by Gateflix.in No part of this material should be copied or reproduced without permission
LT sR(s)
(a) Q.6 =
s → 01 + G ( s ) H(s)
r(t) unit impulse applied at t
1
=1= δ(t − 1)R ( s ) =1  r ( t )  =
e− s LT s.
1
= = s
C (s) 1 s→0 k 1+ k / 2
G= (s) = 2 1+
R ( s ) s + 3s + 2 ( s + 1)( s + 2 )
−s 1
e
= C ( s ) R= (s) G (s) 2 ⇒ 0.25 =
s + 3s + 2 1+ k / 2
Steady state value of output, using ⇒ 1 + =4
k
final value theorem 2
LT ⇒k= 6
css = s C(s)
s→0
LT se − s
= = 0 Q.9 (c)
s → 0 s 2 + 3s + 2 C (s) 2
=
R (s) s + 1
Q.7 (c)
1
M (s) = 2
100 R ( s ) = ( step input )
s + 20s + 100 s
Comparing with standard form,  2  2
= C ( s ) R= ( s)  
ωn 2  s + 1  s(s + 1)
M (s) = 2
s + 2ξωn s + ωn 2 1 1 
= 2 −
∴ 2ξωn = 20  s s + 1 
ω = 100 C (=t ) −1 [ C(s) = ] 2 1 − e− t 
n

ξ =1 Final value of C (=
t ) C=
ss 2
∴ ωn = 10 98% of Css= 0.98 × 2= 1.96
∴ The system is critically damped. Let t=T, the response reaches
98% of its final values.
Q.8 (d) = 2 1 − e − t 
1.96
Steady state value of response=0.75
Input is unit-step so steady state T ≈ 4sec.
error
Q.10 (a)
ess =1 − 0.75 = 0.25
Let the system is represented as
R(s)
= E=
Error (s)
1 + G ( s ) H(s)
k
Where G ( s ) =
( s + 1)( s + 2 )
1
H ( s ) 1andR
= = (s) Y (s) G (s)
s =
Steady state error using find value X ( s ) 1 + G ( s ) H(s)
theorem H(s) =1(unity feedback)
LT X (s)
ess = sE(s) = E=
Error (s)
s→0 1+ G (s)
Steady-state error for

© Copyright Reserved by Gateflix.in No part of this material should be copied or reproduced without permission
1 Characteristic equation
(s)
x= =, ess 0.1
1+G(s)H(s)=0
s
LT 1
ess = sE(s) ⇒ 1+ =0
s→0 s(s + 1 + k)
LT sX ( s ) ⇒ s ( s + 1 + k ) + 1 =0
=
s → 01 + G ( s ) ⇒ s 2 ( R + 1) s + 1 =0
⇒ 0.1 Comparing with
1 s 2 + 2ξωn s + ωn2
LT s × s natural frequency ωn = 1
=
s → 01 + G(s) remains constant and
LT 1 does not depend on k
⇒ 0.1
s → 01 + G(s) 2ξωn= k + 1
When input k +1
ξ=
x ( t ) 10  u ( t ) − u(t-1) 
= 2
damping ratio depends on k
 1 e− s 
x=( s ) 10  −  = m
peak overshoot = e − ξπ/ 1− ξ 2

s s  p

10 Since m p depends on ξ which


= 1 − e − s 
s  depends on k. Hence peak overshoot
LT is influenced by gain (k) of the
ess = sE(s) techo-generator.
s→0
LT s × (s) Q.12 (0.241)
=
s → 01 + G(s) = g ( t ) e −2 [sin 5t + cos 5t]
10 s+2
LT s × s 1 − e 
−s 5
=G (s) +
= (s + 2) + 5 {s + 2} + 52
2 2

s ⇒ 0 1 + G(s)
DC gain means G ( s ) s = 0
LT 10(1 − e − s )
=⇒ ess = 0 5 2 7
s → 0 1 + G(s) G ( 0) = + 2 2 =
2 +5 2 +5
2 2
29
Q.11 (a)
Q.13 (a)

=G (s) =
1
and H ( s ) 1 ( s ) G(s) × U(s)
Y=
s(s + 1 + k)

© Copyright Reserved by Gateflix.in No part of this material should be copied or reproduced without permission
(1 − 2s) 1 Hence, steady state error in the output
Y (s) = . of the system for a unit-step input is
(1 + s) s
0.67.
A B
Y (=s) +
(s) (s + 1) Q.16 8
A = 1, B = −3 K
Given G ( s ) =
( t ) u ( t ) − 3e− t u(t)
y= ( s + 1) ( s + 2 )
2

y ( t )= (1 − 3e ) u(t)
−t
CLTF is given by,
Y (s) G (s) H (s ) G (s )
= =
X (s) 1 + G (s) H (s) 1 + G (s)
Q.14 (b) [Unit feedback system]
The standard 2nd order T/F is
 ω2n 
 2
K 2 
 s + 2ξωn + ωn 
2

It is given that ξ =0.5 & ωn =10 K


Y (s) ( s + 1) ( s + 2 )
2
100
G(s) = K 2 =
s + 10s + 100 X (s) 1 + K
Now to satisfy the steady state O/P ( s + 1) ( s + 2 )
2

1.02
1  Y (s) K
y ( ∞=
)
100
= 1.02 ⇒ K
= 1. =
 K X ( s ) ( s + 1) ( s + 2 ) + K
lt 2
s → 0 s s 2 + 10 s + 100
 
1.02 ×100 102
=G(s) = Y ( s )=
1
×
K
s + 10s + 100 s + 10s + 100
2 2
s ( s + 1) ( s + 2 ) + K
2

Q.15 0.67 From final value theorem,


y ( ∞ ) lim=
= sY ( s ) 0.8
s →0
Given: The open loop transfer [From time response shown in the
function for given unity feedback figure steady state value in time
system is
domain is 0.8]
1
G (s) H (s) = K
= 0.8
( s + 1)( s + 2 ) 2+K
 H ( s ) = 1 = 1.6 + 0.8K
K
Steady state error for unit step input K =8
1 Hence, the value of K is 8.
( ess ) =
1+ Kp
Where, K p (Position error coefficient)
= lim G ( s ) H ( s )
s =0

1 1
=K p lim =
s = 0 ( s + 1)( s + 2 ) 2
Steady state error
1 1
( e=
ss ) = = 0.67
1 1.5
1+
2

© Copyright Reserved by Gateflix.in No part of this material should be copied or reproduced without permission
GATE QUESTIONS(IN)

Q.1 If a first order system and its time Q.5 A unit ramp input is applied to the
response to a unit step input are as system shown in the shown in the
shown below. The gain K is adjoining figure. The steady state
error in its output is

a) 0.25 b) 0.8
c) 1 d) 4
[GATE-2008] a)0 b)0.5
c)1 d)2
Statement for linked Answer Questions [GATE-2010]
Q.2 & Q.3:
A unity feedback system has open loop Q.6 A unity feedback system has an
100 open loop transfer function
transfer function G ( s ) = . The time k
s(s+p) G(s) .The value of k that
s(s + 3)
at which the response to a unit step input
π yields a damping ratio of 0.5 for the
reaches its peak is seconds. closed loop system is
8 a)1 b)3
c)5 d)9
Q.2 The damping coefficient for the closed
[GATE-2010]
loop system is
a) 0.4 b) 0.6 Q.7 The unit-step response of a negative
c) 0.8 d) 1 unity feedback system with the
[GATE-2008] open- loop transfer function is
6
Q.3 The value of p is G (s) =
a) 6 b) 12 s+5
c) 14 d) 16 a) 1-e -5t
b) 6-6e-5t
[GATE-2008] 6 6
c) - e-5t
6 6
d) - e-11t
5 5 11 11
Q.4 A unity feedback system has the [GATE-2011]
K(s + b)
transfer function 2 . The
S (s + 20) Q.8 The open- loop transfer function of a
value of b for which the loci of all the ω(s) 10
dc motor is given as = .
three roots of the closed loop Va (s) 1+10s
characteristic equation meet at a When connected in feedback as
single point is shown below, the approximate
a) 10/9 b)20/9 value of K a that will reduce the
c) 30/9 d)40/9 time constant of closed loop
[GATE-2009]

© Copyright Reserved by Gateflix.in No part of this material should be copied or reproduced without permission
system by one hundred times as 0.1 2
a) b)
compared to that of the open- loop s + 0.1 s+2
system is 100 20
c) d)
s + 100 s + 0.1
[GATE-2014]

Q.10 A system with transfer function


1
a) 1 b) 5 G (s) = 2 has zero initial
c) 10 d) 100 s +1
[GATE-2013] conditions. The percentage
overshoot in its step response is
Q.9 A plant has an open-loop transfer ___________ %
function, [GATE-2015]
20
G P (s)
(S + 0.1)(S + 2 ) (S + 100)
The approximate model obtained by
retaining only one of the above
poles, which is closest to the
frequency response of the original
transfer function at low frequency is

ANSWER KEY:
1 2 3 4 5 6 7 8 9 10
(d) (b) (b) (b) (b) (d) (d) (c) (a) 100

© Copyright Reserved by Gateflix.in No part of this material should be copied or reproduced without permission
EXPLANATIONS

Q.1 (d) ( s + b ) ( 3s 2 + 40s ) − ( s3 + 20s 2 ) =


0
k
T (s) = and ⇒ 3s 2 + 40s 2 + 3bs 2 + 40bs − s3
( ST + 1 + K )
+20s 2 = 0
k
C (s) = ⇒ 2s + ( 3b + 20 ) s 2 + 40bs =
3
0
S ( ST + k + 1)
Now, s = 0 is not the breakaway
k
= =
Css limS.C (s) point
S→ 0 k +1 So, 2s3 + ( 3b + 20 ) s + 40b =0
4
= 0.8= ⇒ 5k For all the three root loci to meet at
5 a single point, we need that this
= 4k + 4 ⇒ k = 4 equation has equal roots.
So, (3b + 20) 2 = 4 × 2 × 40b
Q.2 (b)
100 ⇒ 9b 2 + 120b + 400 = 320b
T (s) = 2 ⇒ 9b − 200b + 400 =
( s + Ps + 100 )
2
0
⇒ 9b ( b − 20 ) − 10 ( b − 20 ) =
0
π π
ωn= 10 and t =
p = ⇒ ( 9b − 20 )( b − 20 ) =
0
8 ωn 1 − δ 2
20
So, 𝑏𝑏 = 20 𝑜𝑜𝑜𝑜 9
⇒ξ =3
5 But b=20is not the required value of
b because it will cancel out an open
Q.3 (b) 20
P –loop pole so, b = is the required
0.6 = ⇒ P = 12 9
20 value

Q.4 (b) Q.5 (b)


k(s + b)
G (s) = 2 2
G(s)
s (s + 20) s(s + 1)
For unity feedback, characteristic For unit Ramp input
equation is 1 + G ( s ) =
0 =r ( t ) R.t.u
= (t) R 1
⇒ s3 + 20s 2 + ks + kb =
0 Velocity error constant,

⇒k=
( s + 20s )
3 2
=K v Lt = S.G ( s ) 2
s→0
(s + b) R 1
∴ ess = = = 0.5
We need to find the breakaway Kv 2
point.
So,
dk
=0 Q.6 (d)
ds K
T (s) =
( s + 3s + K )
2

3 3
Or, −
dk
=
0
ωn Kand
= ξ = or
2ωn 2 K
ds

© Copyright Reserved by Gateflix.in No part of this material should be copied or reproduced without permission
-rξ
9
K= For ξ = 0.5 ⇒ K = 9 . 1-ξ 2
4ξ 2 %overshoot=e ×100=100

Q.7 (d)
C(s) 6
=
CLTF =
R(s) s + 11
The unit step response is given by
6 6 /11 6 /11
C=(s) = −
s(s + 11) s s + 11
6 6 
C (=
t )  − e −11t  u(t)
 11 11 

Q.8 (c)
10K
G ( s ) H ( S) = O.L.T.F
1 + 10s
τ = 10sec
10
=
C.L.T.F τ = 0.1
100
10K
C.L.T.F =
10K + 1 + 10s
10K
τ = 0.1 = ⇒ K ≅ 10
10K + 1

Q.9 (a)
Given
20
G (s ) =
( s + 0.1)( s + 2 ) (s + 100)
20
=
( 0.1)( 2 ) (100) 1 +  1 +  1 + 
s s s
 0.1  2  100 
1
G (s) =
 s  s  s 
1 +  1 +  1 + 
 0.1   2   100 
Approximate model at low
frequency
1
then G ( s ) =
 s 
1 + 
 0.1 
0.1
⇒ G (s) =
(s+0.1)

Q.10 (100)
Comparing the denominator, ξ=0

© Copyright Reserved by Gateflix.in No part of this material should be copied or reproduced without permission
3 TIME DOMAIN STABILITY

3.1 INTRODUCTION 3.1.1 ASYMPTOTICALLY STABLE SYSTEM

The stability of a system relates to its The stationary impulse response, h(t), is
response to inputs or disturbances. A zero
system which remains in a constant state i.e. lim h(t) = 0
t →∞
unless affected by an external action and For a system to be asymptotically or
which returns to a constant state when the absolutely stable, each of the poles of the
external action is removed can be transfer function lies strictly in the left half
considered to be stable. plane (has strictly negative real part).
In other words, a system is said to be stable if
a) Bounded input gives bounded o/p 3.1.2 MARGINALLY STABLE SYSTEM
b) O/P should reduce to zero when input
is removed. The stationary impulse response is
The stability of the system can be different from zero, but limited
determined with the knowledge i.e. 0 < lim h(t) < ∞
t →∞
location of poles of the system.
For a system to be marginally stable, one or
1) If all the poles of the system lie in
more poles lies on the imaginary axis (have
the left half of s plane, then the
real part equal to zero), and all these poles
system is stable.
are distinct. Besides, no poles lie in the
2) If there are non-repeated poles on
right half plane. A marginally stable system
the jω axis, system is marginally
has its output oscillates with constant
stable.
frequency & amplitude.
3) If there are repeated poles of the
system on jω axis, system is
3.1.3 UNSTABLE SYSTEM
unstable.
4) If there is one or more than one pole The stationary impulse response is
in R.H. of imaginary axis, system is unlimited
unstable. i.e. lim h(t) = ∞
t →∞

For a system to be unstable, at least one


pole lies in the right half plane (has real
part greater than zero). Or: There are
multiple poles on the imaginary axis.
Note:
A stable system is either asymptotically
stable or marginally stable.

© Copyright Reserved by Gateflix.in No part of this material should be copied or reproduced without permission
The Routh stability criterion provides a
convenient method of determining control
systems stability. It determines
1) The number of characteristic roots
within the unstable right half of the s-
plane, and the number of characteristic
roots in the stable left half.
2) The number of roots on the imaginary
3.1.4 STEP RESPONSE FOR axis. It does not locate the roots. The
DIFFERENT POLE LOCATION Routh-Hurwitz test is performed on the
denominator of the transfer function, it
is the characteristic equation. For
instance, in a closed-loop transfer
function with G(s) in the forward path,
and H(s) in the feedback loop, we have:
G(s)
T(s) =
1 + G(s)H(s)
If we simplify this equation, we will
have an equation with a numerator N
(s), and a denominator D(s):
N(s)
T(s) =
D(s)
The Routh-Hurwitz criteria will focus
on the denominator polynomial D(s).
Here are the three tests of the Routh-
Hurwitz Criteria. For convenience, we
will use n as the order of the polynomial
(the value of the highest exponent of s
in D(s)). The equation D(s) can be
represented generally as follows:
D ( s ) =a 0s n + a1s n −1 +…+ a n −1s1 + a n
3.2 ROUTH’S STABILITY CRITERION
3.2.1 ALGORITHM FOR APPLYING
There are certain other methods to verify ROUTH’S STABILITY CRITERION
the stability of the control systems.
The algorithm described below, like the
stability criterion, requires the order of
D(s) to be finite.
1) Remove the roots at origin to obtain the
polynomial, and multiply by −1 if
necessary, to obtain

© Copyright Reserved by Gateflix.in No part of this material should be copied or reproduced without permission
a 0s n + a1s n −1 +…+ a n −1s1 + a n =
0 Until the nth row of the array has been
completed. Missing coefficient are
Where, a 0 ≠ 0 & a n > 0 .
replaced by zeros. The resulting array is
2) If the order of the resulting polynomial called the Routh array. The powers of s
is at least two and any coefficient ai is are not considered to be part of the
zero or negative, the polynomial has at array. We can think of them as labels.
least one root with nonnegative real The column beginning with a0 is
part. To obtain the precise number of considered to be the first column of the
roots with nonnegative real part, array.
proceed as follows. Arrange the The Routh array is seen to be triangular.
coefficients of the polynomial, and It can be shown that multiplying a row
values subsequently calculated from by a positive number to simplify the
them as shown below: calculation of the next row does not
sn a 0 a 2 a 4 a 6 … affect the outcome of the application of
s n a1 a 3 a 5 a 7 … the Routh criterion.
s n b1 b 2 b3 b 4 … 3) Count the number of sign changes in the
first column of the array. It can be
sn c1 c 2 c3 c4 … shown that a necessary and sufficient
s n
d1 d 2 d 3 d4 … condition for all roots of
M M MM M … a 0s n + a1s n −1 +…+ a n −1s1 + a n =0 to be
s n e1 e 2 located in the left-half plane is that all
s n f1 the ai are positive and all of the
coefficients in the first column be
s n g1 positive.
Where the coefficients bi are generated Note:
until all subsequent coefficient are zero. Let us apply Routh’s stability criterion to
a a −a a the following third-order polynomial:
b1 = 1 2 0 3
a1 a 0s3 + a1s 2 + a 2s + a 3 =
0
a a −a a Where, all the coefficients are positive
b2 = 1 4 0 5
a1 numbers. The array of coefficients becomes
a a −a a s3 a0 a2
b3 = 1 6 0 7
a1 s2 a1 a3
Similarly, cross multiply the coefficient s1 a1a 2 − a 0 a 3
of the two previous rows to obtain the a1
ci , di , etc. s0 a3
b a −a b
c1 = 1 3 1 2
b1 The condition that all roots have negative
b a −a b real parts is given by
c2 = 1 5 1 3
b1 a1a 2 > a 0 a 3
b a −a b
c3 = 1 7 1 4
b1 Example:
c b −b c Consider the following polynomial
d1 = 1 2 1 2 s 4 + 2s 2 + 3s 2 + 4s + 5 =0
c1
Comment on stability.
c b −b c
d2 = 1 3 1 3
c1

© Copyright Reserved by Gateflix.in No part of this material should be copied or reproduced without permission
Solution: While solving for coefficient of 3rd row we
Let us follow the procedure just presented get zero while the other element is -2 i.e.
and construct the array of coefficients. (The non zero so it is special case-1 of routh’s
first two rows can be obtained directly array. For further calculations take this
from the given polynomial. The remaining zero as small +ve number ∈.
terms are obtained from these. If any s5 1 2 3
coefficients are missing, they may be s 4
1 2 5
replaced by zeros in the array.) 3
∈ -2
s
s4 1 3 5
s 2
2 ∈ +2 5
s 3 2 4 0
s 2 1 5 ∈
s1 -6 s1 −4 ∈ −4 − 5 ∈2
s0 5 2 ∈ +2
s 0
5
The number of changes in sign of the
coefficients in the first column is two (1 to Now, first element of 3rd row is ∈= 0 (will
−6&−6 to 5). This means that there are be considered as positive)
two roots with positive real parts. So the First element of 4th row is
system is unstable. 2 ∈ +2
lim = +∞
∈→ 0 ∈
Note:
First element of 5th row is
The result is unchanged when the
coefficients of any row are multiplied or −4 ∈ −4 − 5 ∈2
lim = −2
divided by a positive number in order to ∈→ 0 2 ∈ +2
simplify the computation. There are 2 sign changes
+∞ to − 2 & − 2 to + 5 Hence there are 2
3.2.2 SPECIAL CASE 1 poles on RHS of s-plane. Therefore the
system is unstable.
If a first-column term in any row is zero,
but the remaining terms are not zero or 3.2.3 SPECIAL CASE 2
there is no remaining term, then the zero
term is replaced by a very small positive When all the elements of any row are zero,
number ∈ and the rest of the array is it is special case-2. The Routh’s array can
evaluated. be solved by following the procedure
• Form an auxiliary equation using the
Example: elements of the row just above the row
Consider a characteristics equation with all zero elements.
s5 + s 4 + 2s3 + 2s 2 + 3s + 5 • Take derivative of the auxiliary
Determine the stability. equation & the coefficients of the
Solution: resultant equation will replace the row
The array of coefficients is with all zeros.
s5 1 2 3
Example:
s4 1 2 5 s5 + 2s 4 + 24s3 + 48s 2 − 25s − 50 =
0
s3 0 Solution:
s2 The array of coefficients is
s5 1 24 -25
s1 s4 2 48 -50⟵ Auxiliary polynomial
s0 P(s)

© Copyright Reserved by Gateflix.in No part of this material should be copied or reproduced without permission
s3 0 0
Example:
Here all the coefficients of s3 row are zero. Consider a system with closed loop transfer
Now form the auxiliary equation using the function
coefficients of s4 row. The auxiliary R(s)
K
C(s)
+
equation P(s) is
2
s(s +s+1)(s+4)

P(s) = 2s4 + 48s2 - 50


Now differentiate the equation we get
dp(s) C(s) K
= 8s3 + 96s = 2
ds R(s) s(s + s + 1)(s + 4) + K
The terms in the s3 row are replaced by the Therefore, the characteristic equation is
coefficients of the last equation, that is, 8 s(s 2 + s + 1)(s + 4) + K =0
and 96. The array of coefficients then
s 4 + 5s3 + 5s 2 + 4s + K = 0
becomes
Now, the Routh’s array for this equation is
s5 1 24 -25
s4 1 5 K
s4 2 48 -50
s 3 5 4 12
s 3 8 96 dp(s)
← coefficient of s2 21/5 K
ds s1  84  21
s 2 24 -50  − 5K  /
 5  5
s1 112.7 0
s0 K
s0 -50
For the system to be stable all the elements
There is 1 sign hence the system is
of the 1st column should be +ve
unstable.
i.e. K > 0 & ( 84 / 5 – 5K ) > 0 ⇒ K < 84 / 25
Note: Therefore for stability, K should lie in the
Whenever there is all zero elements in any range
row, there will be 84
0<K<
1) Either pair of real roots with opposite 25
sign (1 +ve & 1 –ve) If K becomes greater than 84/25, the
2) Or complex conjugate roots system becomes unstable.
3) Or a pair of roots on imaginary axis
This can be found out by solving the Note:
auxiliary equation. In the above example K cannot be greater
Now, solving the auxiliary equation of than 84/25, this is the last value of K for
the above example 2s 4 + 48s 2 − 50 = 0. which the system will be stable & this is
We get s2 = 1, s2 = -25 called as marginal value of K i.e. for K =
or s = ±1, s = ±j5 84/25 the system will be marginally stable
These two pairs of roots are a part of (on the verge of instability)
the roots of the original equation.
Clearly, the original equation has one 2) To determine the frequency of
root with a positive real part (s = +1) oscillations
also there are 2 roots on imaginary axis.
In the above example the auxiliary
3.2.4 APPLICATION OF ROUTH’S polynomial for K m arginal = 84 / 25 is
STABILITY CRITERIA
( 21/ 5) s 2 + 84 / 25 =
0
1) To determine the range of gain K for ⇒s = −4 / 5
2

stable system. ⇒ s =j 4 / 5

© Copyright Reserved by Gateflix.in No part of this material should be copied or reproduced without permission
Put s = jω  39 
K > 0 &  − 4K  > 0
∴ ω=4 / 5rad / sec  4 
Therefore for stability, K should lie in the
Example: range
The characteristic equation of a feedback 39
When K = here will be a zero at the first
control system is 16
s3 + 3Ks2 + (K + 2) s + 4 = 0. entry in the fourth row. This will indicate
Determine the range of K for which system presence of symmetrical roots, which as
is stable shown below, will be pure imaginary.
Solution: 39
Routh’s array is K m arginal = will cause sustained
16
s3 1 K+2
oscillations.
s2 3K 4
The subsidiary equation of third row for
s 1
3K + 6K − 4
2
39 13 39
3K K= , is s 2 + =0
16 4 16
s 0 4 s = ± j 0.75
For stability, 3K > 0 i.e. K > 0 Thus the frequency of sustained
And 3K2 + 6K – 4 > 0 i.e. K> -1 ± 1.53 oscillations is 0.75 rad/sec
i.e. K > 0.53
The range of K is thus ∞ > K > 0.53 Example:
Example: s6 + 3s5 + 4s4 + 6s3 + 5s2 + 3s + 2 = 0
The open loop transfer function of a unity The Routh’s array is
feedback system is given by s6 1 4 5 2 2s4 + 4s2 + 2 = 0
K s5 3 6 3 s4 + 2s2 + 1 = 0
G(s) =
s(s + 3)(s 2 + s + 1) s4 2 4 2 4s3 + 4s = 0
Determine the values of K that will cause s 3 0 0 0 s3 + s = 0
sustained oscillations in the closed loop 1 1 2s2 + 2 = 0
system. Also find the oscillation frequency. s2 + 1 = 0
Solution: s 2 2 2 2s = 0
The characteristic equation is s1 0 0
K 2
1 + G(s) = 1+ =
0 s0 1
s(s + 3)(s 2 + s + 1)
s (s + 3) (s2 + s + 1) + K = 0 There are two rows which become zero
s4 + 4s3 + 4s2 + 3s + K = 0 and there is no sign change in the first
The Routh’s array is column of the Routh’s array.
s4 1 4 K Note:
s3 4 3 0 We cannot comment on stability until the
s 2 13 K roots of auxiliary equation are not known.
Now the auxiliary equations is
4 s4 + 2s2 + 1= 0
s 1
 39  ⇒ s= ± j& s = ±j
 − 4K 
 4  As there are repeated roots on imaginary
13 axis, the system is unstable.
4
s 0 K Example:
The condition for system stability is A feedback control system has an open loop
transfer function of

© Copyright Reserved by Gateflix.in No part of this material should be copied or reproduced without permission
Ke − s The closed-loop transfer function is:
G(s)H(s) =
s(s 2 + 2s + 1) C (s) K
= 2
Determine the maximum value of K for the R ( s ) s + 2s + K
close loop stability The characteristic equation is:
Solution: s 2 + 2s + K = 0
For low frequencies The roots of the characteristics equation
e-s = (1-s) (i.e. closed loop poles) are:
K(1 − s)
G(s)H(s) = 2 22 − 4K 4 − 4K
s(s + 2s + 1)
2
S =− ± =−1 ±
2 2 2
K(1 − s) From the above equations, it is clear that
1 + G(s)H(s) = =0
s(s 2 + 2s + 1) the roots depend on gain K of the system. If
∴ s(s + 2s + 1) + K(1 − s) =
2
0 the gain is varied from 0 to ∞, the closed
s + 2s + s + K − Ks =0
3 2 loop poles will also change. Following table
shows the variation of closed loop poles
s3 + 2s 2 + s(1 − K) + K = 0
with gain K:
The Routh’s array is
s3 1 (1-K)
s2 2 K
s 1 (2(1-K)-
K)/2
s0 K
For stability K > 0 and
2(1 – K) – K > 0 or 2 – 3K > 0
3
K<
2
Hence the restriction on K is
0< K<
3 When all the values of closed loop poles are
2 plotted on a graph we get:

3.3 ROOT LOCUS

Root locus analysis is a graphical method


for examining how the roots of a system
change with variation of a certain system
parameter, commonly gain with in The locus of the roots of the closed loop
a feedback system. This is a technique used system (closed loop poles) as a function of
as a stability criterion in the field of control a gain of K, as it is varied from 0 to infinity
systems developed by Walter R. results in Root-Locus.
Evans which can determine stability of the
system. 3.3.1 ANGLE CONDITION
Consider the closed loop system:
Consider a characteristics equation
1 + G (s) H (s) = 0
⇒ G ( s ) H ( s ) =−1 + j0
Now, ∠G ( s ) H ( s ) =
± ( 2q + 1)180o
The open loop poles of the system are:
s = 0 & S = −2

© Copyright Reserved by Gateflix.in No part of this material should be copied or reproduced without permission
Note: Now,
−1 + j0 is a point on –ve real axis which can K
G ( s ) H ( s ) s = −0.75
= = 1
be traced at the angles ±180 , ±540 , ±900 o o o
0.75 ×1.25 × 3.25
……… with respect to the +ve real axis. ∴ K = 3.04
Angle condition is used to whether a point
in s-plane lies on root locus or not. 3.3.3 RULES FOR CONSTRUCTING ROOT
LOCUS

Example: Rule 1: Symmetry


Check whether the point s = −0.75 lies on The root locus is always symmetrical about
the root locus of real axis.
K
G(s)H(s) = or not. Rule 2: Number of branches
s ( s + 2 )( s + 4 )
The number of branches is equal to the
Solution: number of poles of the open-loop transfer
Put s = −0.75 in the given transfer function function.
K 1
G(s)H(s) = e.g. G(s) = the poles are at
−0.75 ( −0.75 + 2 )( −0.75 + 4 ) s ( s + 2 ) (s + 4)
0o 0, −2, −4 As the total number of open loop
Now, ∠G(s)H(s) = =−180o
poles is 3, the number of branches of root
o o o
180 .0 .0
Therefore s = −0.75 lies on root locus. locus is 3.

3.3.2 MAGNITUDE CONDITION Rule 3: Real-axis root locus


If the total number of open loop poles and
Consider a characteristics equation zeros to the right of any point on the real
1 + G(s)H(s) = 0 axis is odd, then this point lies on the root
⇒ G ( s ) H ( s ) =−1 + j0 locus.
s
Now, G ( s ) H ( s ) = 1 e.g. G(s) =
(s + 3)
Once it is confirmed that a point s lies on poles ⇒ s =−3 & zeroes ⇒ s =0
the root locus using angle condition, we can
find the corresponding gain K of the system
at that point.

Example:
Find the gain of the system with transfer a) To the point s = −1 , on the right side
K there is 1 open loop zero & no pole i.e.
Function G(s)H(s) = at a
s ( s + 2 )( s + 4 ) total = 1 + 0 = 1 which is odd. Hence the
point s = −0.75 . point s = −1 lies on the root locus.
Solution: b) To the point 𝑠𝑠 = −5, on the right side
In the last example we confirmed that there is 1 open loop zero & 1 open loop
s = −0.75 lies on the root locus & now the pole i.e. total = 1+ 1 = 2 which is even.
corresponding gain can be found out using Hence the point 𝑠𝑠 = −5 does not lie on
magnitude condition. the root locus.
Put s = −0.75 in the given transfer function
K Rule 4: Root locus end-points
G(s)H(s) = The locus starting point (K=0) are at the
−0.75 ( −0.75 + 2 )( −0.75 + 4 )
open-loop poles and the locus ending

© Copyright Reserved by Gateflix.in No part of this material should be copied or reproduced without permission
points (K=∞) are at the open loop zeros. If
there P number of open loop poles & Z Rule 6: Intersection of asymptotes
number of open loop zeros, then m number The asymptotes for the root locus
of branches will end at zeros and 𝑃𝑃 − 𝑍𝑍 intersects at a point on the real axis called
branches terminate at infinity. centroid & it is given by
s+3 ∑real part of poles of G ( s ) H(s) − ∑real part of zeros of G ( s ) H(s)
e.g. G ( s ) = σ=
(P − Z)
s ( s + 2 )( s + 4 )
s+3
poles ⇒ s = 0, −2, −4 ; total = 3 e.g. G(s) =
s ( s + 2 ) (s + 4)
zeroes ⇒ s =−3 ; total =1
There will be 3 (no. of open loop poles) = σ
( −4 − 2 − 0 ) −= (−3) −6 + 3 −3
=
root locus branches, out of which 1 (no. of (3 − 1) 2 2
open loop zero) branch will terminate at a
zero &3 − 1 = 2 number of branches will Rule 7: Real Axis Breakaway and Break-
terminate at infinity. in points:
A breakaway point is a point on root locus
Rule 5: Slope of asymptotes where multiple roots of the characteristics
The branches of root locus which equation occur. Numerous root loci appear
terminates at infinity follows the path to break away from the real axis as the
along the straight line called asymptotes. system poles move from the real axis to the
The number of a symptotes= P − Z. complex plane. At other times the loci
The angles of asymptotes with +ve real axis appear to return to the real axis as a pair of
180(2q + 1) complex poles becomes real.
are: ∅ =
(P − Z)
Where, q = 0, 1, 2, 3... (P-Z-1)
s+3
e.g. G (s) = poles
s ( s + 2 )( s + 4 )
⇒ s = 0, −2, −4
Total = 3 & zeroes ⇒ s =−3total =1
The number of asymptotes is 3 − 1 =2.
The angles of asymptotes with real axis are:
180(2 × 0 + 1)
=∅1 = 90o
(3 − 1)
180(2 ×1 + 1)
=∅2 = 270o The figure shows a root locus leaving the
(3 − 1)
real axis between -1 and −2 and returning
to the real axis between +3 and +5. The
point where the locus leaves the real axis,
−σ1 , is called the breakaway point, and the
point where the locus returns to the real
axis, σ 2 , is called break-in point.
Note:
At the breakaway or break-in point, the
branches of the root locus form an angle of
180/n with the real axis, where n is the
number of closed loop poles arriving or
departing from the single breakaway or

© Copyright Reserved by Gateflix.in No part of this material should be copied or reproduced without permission
break-in point on the real axis. Thus for the K= +0.88 for s =
−0.46
poles shown in the figure the branches at K= −6.06 for s =
−2.86
the breakaway point form 90°angles with As K is +ve for s = −0.46, it is a valid
the real axis. breakaway point & s = −2.86 is not a valid
breakaway point.
Procedure to find breakaway or break-
in point: Rule 8: Intersection with imaginary axis
K The jω axis crossing is a point on the root
1) Let G(s) =
s(s + a) locus that separates the stable operation of
Write the characteristics equation for the system from the unstable operation.
the above transfer function: The value of ω at the axis crossing yields
s 2 + as + K = 0 the frequency of oscillation. The
2) Find the expression for K intersection of root-locus with imaginary
K= −(s 2 + as) axis can be obtained by solving the
auxiliary equation from the Routh’s array.
dK
3) Find & equate it to zero. e.g. If the auxiliary equation is s 2 + 4 =0,
ds
solving for s we get s = ± j2 .
dK
= − ( 2s + a ) =0 Therefore the root locus intersects at
ds + j2 & − j2 .
dK
4) The roots of the = 0 gives the Example:
ds For the system of the following figure,
breakaway or break-in point. find the frequency and gain, K, for which
Root is s = −a / 2 , hence breakaway the root locus crosses the imaginary axis.
point is −a / 2 . For what range of K is the system stable?
5) For the breakaway or break-in point if
G(s) satisfies angle condition, it is a
valid break away point.
Note:
For a point on root locus between two
adjacently placed poles, there exists a Solution:
breakaway point if the number of poles & The closed loop transfer function for the
zeros to the right hand side is even. system is
K(s + 3)
T(s) = 4
Example s + 7s + 14s 2 + ( 8 + K ) s + 3K
3

Determine the breakaway points for the


The routh’s array will be
K
root locus of G(s)H(s) =
s ( s + 1)( s + 4 ) 1 14 3K
s4
Solution: 7 8+K
s3
The characteristics equation is
s2 90 -K 21K
s3 + 5s 2 + 4s + K =0
Now, K =−s − 5s − 4s
3 2
1 −K 2 − 65K + 720
s
dK 90 − K
∴ =−3s 2 − 10s − 4 =0
ds s0 21 K
Roots of this equation are
s= −0.46 & s = −2.86 For system to be stable all the elements of
Now, substituting these values in the 1st column should be positive.
expression of K i.e. 21K > 0

© Copyright Reserved by Gateflix.in No part of this material should be copied or reproduced without permission
⇒ K > 0 and −K 2 − 65K + 720 > 0
⇒ K < 9.65
For marginal stability, K mar = 9.65
Auxiliary equation is:
( 90 − K ) s 2 + 21K =
0
∴ 80.35s 2 + 202.7 =
0
Solving equation we get, s = ± j1.59
Therefore the root locus touch the
imaginary axis at ± j1.59 for K mar = 9.65 .

Rule 9: Angles of Departure and Arrival


In order to sketch the root locus more
accurately, we want to calculate the root
locus departure angle from the complex
poles and the arrival angle to the complex
zeros.
The angle of departure & arrival are given Here
by =
∅ d 180o − ∅ and = ∅ a 180o + ∅ θ 2 = 90o ,
Where ∅= ∑∅ − ∑∅
P Z
−1  1 
= θ 4 tan=   26.56 &
o

∑ P∅ is the angle made by the other poles 2


with the pole at which angle of departure is −1  1 
to be calculated or with the zero at which= θ3 tan=   45
o

1
angle of arrival is to be calculated.
Now, ∅= ∑∅ P − ∑∅ Z= θ 2 + θ 4 − (θ3 )
∑∅ Z is the angle made by the other zeros
with the pole at which angle of departure is = 90o + 26.56o − 45o = 71.56o ( )
to be calculated or with the zero at which ∴∅= d 180o −=
∅ 180o − 71.56o
angle of arrival is to be calculated. = 108.4°
Example:
Example
Given the unity feedback system of
Determine number asymptotes for
following figure, find the angle of departure
(s + 1)(s + 4)
from the complex poles and sketch the root G(s)H(s) = .
locus. (s + 3)(s + 5)
Solution
No. of open loop poles n = 2
No. of open loop zeros m = 2
No. of root loci ending on ∞ = 2 − 2 = 0

Solution:
Using the poles and zeros of
G ( s ) = K(s + 2) / [(s + 3)(s 2 + 2s + 2)] No. of asymptotes = 0
as plotted in the figure, we calculate the
sum of angles drawn to a point ε close to
the complex pole, −1 + j1 , in the second
quadrant.

© Copyright Reserved by Gateflix.in No part of this material should be copied or reproduced without permission
Example ∴ s = j2 2
K
Given that KG ( s ) = Sketch 8) Breakaway point:
s ( s + 2 )( s + 4 ) The breakaway point is the solution of
the root locus of 1 + KG(s) = 0 and compute dK
=0
the value of K that will yield a “dominant” ds
second order behavior with a damping K= −s ( s + 2 )( s + 4 ) =
−(s3 + 6s 2 + 8s)
ratio, ξ = 0.7 .
dK
Solution = −(3s 2 + 12s + 8)
ds
We have, n = 3 and m = 0.
Solving we get s = −0.845
Open loop zero: none
Open loop poles: s = 0, −2, −4
1) As= 3, the number of root locus
branches=3.
2) n – m = 3 – 0 = 3, therefore 3 branches
will terminate at infinity.
3) Number of asymptotes = 3.
4) Angle of asyptotes are
(2 × 0 + 1) ×1800
= φ1 = 600
3
(2 ×1 + 1) ×1800
= φ2 = 1800
3
(2 × 2 + 1) ×1800
= φ3 = 3000
3 Example
5) Centroid σ =
( −4 − 2 − 0 ) − 0 = −2 K(s + 1)(s + 3)
G(s)H(s) = find Break away
3 (s + 4)(s + 5)
6) Identifying the root locus branches point/Break in point.
Solution:
The characteristics equation is
1 + G(s)H(s) = 0
K(s + 1)(s + 3)
∴ 1+ = 0
(s + 4)(s + 5)
s 2 + 9s + 20
K=
s 2 + 4s + 3

=
dk {=
( s + 4s + 3) ( 2s + 9 ) − ( s + 9s + 20 ) ( 2s + 4 )}
2 2

0
(s + 9s + 20 )
2
ds 2
7) Crossing the imaginary axis
The characteristics equation is ( s1 =
−2.43, −4.3)
s3 + 6s 2 + 8s + K =
0 Both the points satisfy angle condition
s3 1 8 0 hence they are valid break away
s 2 6 K point/break in point. s = −4.3 lies between 2
s1 48 − K poles hence it is a breakaway point while
6 s = −2.43 lies between 2 zeros hence it is
s0 K break in point.
When K = 48, the s1 row will have all
the elements zero. Auxiliary equation Example:
will be 6s 2 + 48 =0

© Copyright Reserved by Gateflix.in No part of this material should be copied or reproduced without permission
Sketch the root loci for the system shown in Point s = 0 corresponds to the actual
Figure breakaway point. But points
s= −1.65 + 0.9367 j & s =
−1.65 − 0.9367 j are
neither breakaway nor break-in points,
because they do not satisfy angle
condition.
Solution:
1) No of open loop poles= P= 3 , s = 0,
0, −3.6
2) No of open loop zeros= Z= 1 , s = −1
3) No of root locus branches= P= 2
4) No of branches terminating at infinity
= P − Z = 3 −1 = 2
5) Number of asymptotes = 2
σ=
( 0 + 0 − 3.6 ) − (−1) = −1.3
2
Angle of asymptotes are +90 & -90
6) Intersection with jω axis characteristics Example
equation is s + 3.6s + Ks + K =
3 2
0 For a unity feedback system the open Loop
Routh’s array will be transfer function is given below
s 1
3
K K
G(s) = 2 Draw the root locus
s 3.6
2
K s(s + 6s + 25)
for 0 ≤ K ≤ ∞ .
2.6K
s1
Solution:
3.6 1) No of poles = P = 3
s K
0
s = 0, −3 + 4 j, −3 − 4 j
For stability K > 0 & 2.6K/3.6 > 0 2) No of zeros= Z= 0
⇒ K mar = 0 3) No of root locus branches= P= 3
Auxiliary equation from s is 2 4) No of branches terminating at infinity
3.6s 2 + K = 0 = P − Z =3−0 =3
5) Number of asymptotes
3.6s 2 + 0 = 0 ⇒ s 2 = 0 ⇒ s = 0, 0
= P − Z =3−0 =3
Therefore root locus will touch
imaginary axis at origin. Centroid σ =
( 0 − 3 − 3) − 0 = −2
7) Break away point 3
The characteristics equation is Angle of asyptotes are
s + 3.6s + Ks + K =
3 2
0 (2 × 0 + 1) ×1800
= φ1 = 600
s (s + 3.6)
2 3
K= − (2 ×1 + 1) ×1800
(s + 1) = φ2 = 1800
dK
= −
( 3s + 7.2s ) ( s + 1) − (s + 3.6s )
2 3 2

=
3
0 (2 × 2 + 1) ×1800
ds (s + 1) 2 = φ3 = 3000
3
s3 + 3.3s 2 + 3.6s = 0 6) Intersection with jω axis
Solving we get, s = 0 Characteristics equation is
s= −1.65 + 0.9367 j s3 + 6s 2 + 25s + K = 0
s= −1.65 − 0.9367 j Routh’s array will be

© Copyright Reserved by Gateflix.in No part of this material should be copied or reproduced without permission
s3 1 25 K
s2 6 K sτ1 + 1
150 − K
s1
6
s K
0

For stability K > 0 & 150 − K > 0


⇒ K < 150 ⇒ K mar = 150
Auxiliary equation from s2 row will be
6s 2 + 150 = 0 K
solving we get s = ± j5
( sτ1 + 1) (sτ 2 + 1)
therefore the root locus will touch j
imaginary axis at ±j5.
7) Breakaway point r1
From the charcteristics equation
K= −(s3 + 6s 2 + 25s)
-1 -1
− ( 3s 2 + 12s + 25 ) =
dK
= 0 1 r2 2

ds
solving we get, s =−2 + j2.0817, −2 − j2.0817
K
Both the points does not satisfy the
angle condition hence they are not valid s ( sτ1 + 1) (sτ 2 + 1)
breakaway points.
8) Angle of departure
The angle of departure from the
complex pole in the upper half s plane is
∅ d =180o − (126.87 o + 90o ) =−36.87 o

2) Addition of a zero pulls root locus to the


left & stability of the system increases
K ( sτ a + 1)
s ( sτ1 + 1) (sτ 2 + 1)

3.4 MINIMUM PHASE SYSTEM


Note:
1) Addition of a pole pulls root locus
towards right hand side & the stability
of the system decreases.

© Copyright Reserved by Gateflix.in No part of this material should be copied or reproduced without permission
A system whose transfer function has all 1 − j ωT
G( jω) =
the poles and zeros in the left half of the s- 1 + j ωT
plane is called as minimum-phase system. Pole-zero configurations is shown in figure:
1 + sT1
e.g. G(s) =
1 + sT2

It has a magnitude of unity at all


frequencies and a phase angle (-2tan-1ωT)
which varies from 0o to -180o as ω is
increased from 0 to ∞.

3.5 NON MINIMUM PHASE SYSTEM

A system whose transfer function has one


or more zeros in the right half s-plane is
known as non minimum phase system.
1 − sT1
e.g. G(s) =
1 + sT2

Note:
A common example of a minimum phase
element is transportation lag which has the
transfer function
G(s) = e − sT
Put s = jω
∴ G( jω) = e − jωT
Now, G ( jω ) = 1
and ∠G ( jω ) =
−57.3ωT

3.6 ALL PASS SYSTEM

An all pass system has a transfer function


having a pole-zero patterns which is anti
symmetric about the imaginary axis, i.e. for
every pole in the left half plane; there is a
zero in the mirror image position. A
common example of such a transfer
function is

© Copyright Reserved by Gateflix.in No part of this material should be copied or reproduced without permission
GATE QUESTIONS(EC)(Stability Analysis)

Q.1 The feedback control system in the c)1 d) -∞


figure is stable. [GATE-2004]

Q.6 The open-loop transfer function of a


unity feedback system is
K
G (s) =
a)for all K ≥ 0 b)only is K ≥ 0 s ( s + s + 2 ) (s + 3)
2

c)onlyif 0 ≤ K < 1 d)only if 0 ≤ K ≤ 1 The range of K for which the system


[GATE-2001] is stable is
21
a) >K >0 b) 13 > K > 0
Q.2 The phase margin of a system with 4
the open-loop transfer function 21
(1 − s) c) <K<∞ d) −6 < K < ∞
G (s) H (s) = 4
(1 + s)(2 + s) [GATE-2004]
a) 0° b) 63.4°
c) 90° d) ∞ Q.7 For the polynomial
[GATE-2002] P ( s ) = s5 + s 4 + 2s3 + 2s 2 + 3s + 15 , the
number of roots which lie in the
Q.3 The system shown in the figure
right half of the s-plane is
remains stable when
a)4 b)2
c)3 d)1
[GATE-2004]

Q.8 The positive values of “K” and “a” so


a) K < −1 b) −1 < K < 1 that the system shown in the figure
c) 1 < K < 3 d) K < −3 below oscillates at a frequency of 2
[GATE-2002] rad/ sec respectively are

Q.4 The characteristic polynomial of a


system is
q ( s ) = 2s + s + 4s + 2s + 2s + 1 . The
5 4 3 2
a)1, 0.75 b)2, 0.75
system is c)1, 1 d)2, 2
a)stable b)marginally stable [GATE-2006]
c)unstable d)oscillatory
[GATE-2002] Common Data for Questions Q.9 & Q.10:
Consider a unity-gain feedback control
Q.5 The gain margin for the system with system whose open-loop transfer function
open loop transfer function as + 1
2(1 + s) is G ( s ) = 2
G (s) H (s) = is S
s2
a) ∞ b) 0

© Copyright Reserved by Gateflix.in No part of this material should be copied or reproduced without permission
Q.9 The value of “a” so that the system Q.13 The number of open right half plane
has a phase–margin equal to π / 4 is poles of
approximately equal to 10
G (s) = 5 is
a)2.40 b)1.40 s +2s +3s3 +6s 2 +5s+3
4

c)0.84 d)0.74 a) 0 b) 1
[GATE -2006] c) 2 d) 3
Q.10 With the value of “a” set for phase– [GATE -2008]
margin of π / 4 , the value of unit–
impulse response of the open-loop Q.14 The feedback system shown below
system at t = 1 second is equal to oscillates at 2rad/s when
a)3.40 b)2.40
c)1.84 d)1.74
[GATE -2006]

Q.11 If the closed-loop transfer function


of a control system given as a)K=2 and a=0.75 b)K=3 and a=0.75
s−5 c)K=4 and a=0.5 d)K=2 and a=0.5
T (s) = , then it is [GATE -2012]
( s + 2 ) (s + 3)
a) an unstable system Q.15 The forward path transfer function
b) an uncontrollable system of a unity negative feedback system
c) a minimum phase system K
d) a non-minimum phase system is given by G ( s ) =
[GATE -2007]
( s + 2 ) (s − 1)
The value of K which will place both
Q.12 A certain system has transfer the poles of the closed-loop system
s+8 at the same location, is ________
function G ( s ) = 2 , where α [GATE-2014]
s + αs - 4
is a parameter. Consider the Q.16 Consider a transfer function
standard negative unity feedback
ps 2 + 3ps − 2
configuration as shown below. Gp (s) = 2 with
s + ( 3 + p ) s + (2 − p)
pa positive real parameter. The
maximum value of p until which G p
Which of the following statements is remain stable is _______.
true? [GATE-2014]
a) The closed loop system is never
stable for any value of α Q.17 Match the inferences X, Y, and Z,
b) For some positive values of α the about a system, to the
closed loop system is stable, but corresponding properties of the
not for all positive values elements of first column in Routh's
c) For all positive values of, the Table of the system characteristic
closed loop system is stable equation.
d) The closed loop system is stable X: The system is stable...
for all values of, both positive P: ... when all elements are positive
and negative Y: The system is unstable...
[GATE -2008] Q: ... when any one element is zero
Z: The test breaks down...

© Copyright Reserved by Gateflix.in No part of this material should be copied or reproduced without permission
R: ... when there is a change in sign Q.20 Which one of the following options
of coefficients correctly describes the locations of
the roots of the equation s4+s2+1=0
a) X → P, Y → Q, Z → R on the complex plane?
b) X → Q, Y → P, Z → R a) Four left half plane (LHP) roots.
c) X → R, Y → Q, Z → P b) One right half plane (RHP) root,
d) X → P, Y → R, Z → Q one LHP root and two roots on the
[GATE-2016] imaginary axis.
c) Two RHP roots and two LHP
Q.18 The transfer function of a linear roots.
time invariant system is given by d) All four roots are on the
H ( s ) = 2s 4 − 5s3 − 5s − 2 . imaginary axis.
[GATE-2017-01]
The number of zeros in the right half
of the s-plane is __________.
[GATE-2016] Q.21 Consider p ( s ) =s3 + a 2s 2 + a1s + a 0
with all real coefficients. It is known
Q.19 The first two rows in the Routh that its derivative p ' ( s ) has no real
table for the characteristic equation roots. The number of real roots of
of a certain closed-loop control
p ( s ) is
system are given as
S3 1 (2K + 3) a) 0 b) 1 c) 2 d) 3
S2 2K 4 [GATE-2018]
The range of K for which the system
is stable is
a) —2.0 <K<0.5 b) 0<K<0.5
c) 0< K <∞ d) 0.5 < K <∞
[GATE-2016]

ANSWER KEY:

1 2 3 4 5 6 7 8 9 10 11 12 13 14 15
(c) (d) (d) (c) (a) (a) (b) (b) (c) (c) (d) (c) (c) (a) 2.25
16 17 18 19 20 21
2 (d) 3 (d) (c) (b)

© Copyright Reserved by Gateflix.in No part of this material should be copied or reproduced without permission
EXPLANATIONS

Q.1 (c) Q.3 (d)


G1G 2 K
T.F. =
1 + G1G 2 H =
Y(s)
= s K
K(s − 2) R(s)  3 K  s − (3 + K)
1−  + 
( s + 2)
2 s s 
=
K ( s − 2 ) (s − 2)
1+ For system to be stable,
( s + 2)
2
3+ K < 0
K(s − 2) ⇒ K < −3
=
( s + 2)
+ K ( s − 2)
2 2

Q.4 (c)
∴ char. equation
Routh table is
= s 2 + 4 + 4s + Ks 2 − 4Ks + 4K s 5 2(1) 4(2) 2(1)
(1 + K ) s 2 + K ( 4 − 4K ) s + 4K + 4 =0
s4 1 2 1
Routh table is
1) 1 + K4K + 4 s3 0 0 0
2) 4 − 4K s2
⇓ 0 s1
4(1 − K)
s0
3) 4(K + 1)
For system to be stable ∴
d 4
ds
( s + 2s 2 + 1) =
0
1− K > 0
1 > K & K ≥ 0 (Given in question) 4s3 + 4s = 0
From 3rd row s= ± j,s = ±j
K > −1∴ 0 ≤ K < 1 d 2
ds
( s + 1) = 0
Q.2 (d) 2s = 0 ⇒ s = 0
ωg where G ( s ) H ( s ) = 1 is Double roots on imaginary axis so
system is unstable.
1− s 1+ ω2
= = 1
(1 + s ) (2 + s) 1+ ω2 4 + ω2 Q.5 (a)
4 + ω2 =
1
∠ G (s) H (s) =
−180° + tan −1 ω
⇒ 4 + ω2 = 1 For ωΦ =
−180° + tan −1 ω =
−180°
ω = −3 (imaginary)
2
2 1 + ω2
So no gain crossover frequency ω = 0 G (=
s) H (s) = ∞
ω2
∴ PM = ∞ 1
=
G.M = 0

In db G.M = ∞

© Copyright Reserved by Gateflix.in No part of this material should be copied or reproduced without permission
Q.6 (a) −12t − 15
K
G (s) = t
s ( s + s + 2 ) (s + 3)
2
15
s ⇒ −12 −
H (s) = 1 t
s° 15
1+ G (s) H (s)
∴Two sign change from
K s 2 to s and s to s°
= 1+
s ( s3 + 3s 2 + s 2 + 3s + 2s + 6 ) ∴2 roots on RHS of s plane.
Q.8 (b)
s + 4s + 5s + 6s + K
4 3 2 1+ G (s) H (s)
= K(s + 1)
s(s3 + 4s 2 + 5s + 6) = 1+ 3 = 0
1+ G (s) H (s) = 0 s + as 2 + 2s + 1
s4 1 5K s3 + as 2 + 2s + 1 + Ks + K
⇒ = 0
s3 4 6 s3 + as 2 + 2s + 1
s 2 7  14  K s3 + as 2 + ( 2 + K ) s + K + 1 =0
 
2 4  s3 1 2 + K
s 1 7 0 s2 a K + 1
× 6 − 4K
2 a ( 2 + K ) − (K + 1)
7 s
a
2
For oscillation,
s 0 K
a ( 2 + K ) − (K + 1)
For system to be stable, K>0 =0
2 a
( 21 − 4K ) > 0 K +1
7 a=
21 21 K+2
>K⇒K< as + K + 1 =0
2
4 4
21 s= jω,s 2 = − ω2 = −4
∴ >K>0
4 ⇒ −4a + K + 1 =0
K +1 K +1 K +1
=
a ⇒ = ⇒K
= 2
Q.7 (b) 4 4 K+2
P ( s ) = s5 + s 4 + 2s3 + 2s 2 + 3s + 15 a = 0.75
s5 1 2 3
s4 1 2 15 Q.9 (c)
s3 0(∈) -12 0 π
s2 2 ∈ +12 15 0 PM =
4
∈ π
⇒ 180 + tan −1 aω − 180° =
s0 −12 ( 2 ∈ +12 ) 4
− 15 ∈
∈ π
tan = aω ⇒ aω = 1
 2 ∈ +12  4
 
 ∈  Now for gain crossover frequency
1 + a 2 ω2
2 + 12 G(s) = 1 ⇒ =
1
=t ω2
 1+ 1 =ω2 (as aω = 1 )
Let  be a small positive no.
ω2 = 2

© Copyright Reserved by Gateflix.in No part of this material should be copied or reproduced without permission
ω = ( 2)
1/4

1
=a = 0.84
21/4

Q.10 (c)
0.84s + 1
G (s) =
s2
H ( s ) 1,=
= R (s) 1
∴ C (s) = G ( s ) .R(s) Since sign changes twice in Routh–
0.84s + 1 array therefore, there are two poles
C (s) = on right half plane.
s2
1 + 0.84s 
c ( t ) = L−1   Q.14 (a)
 s
2
Characteristic equation is
 1 0.84 
= L−1  2 + 1+ G (s) H (s) = 0
S S  k(s + 1)
1+ 3 =
c ( t )= [ t + 0.84] U(t)
0
s + as 2 + 2s + 1
1, c ( t ) =
At t = 1 + 0.84 =
1.84 s3 + as 2 + ( 2 + k ) s + (1 + k )
=0
Q.11 (d) Routh array for this is
As there is a right half zero, the s3
system is a non-minimum phase 1 (2 + k)
s2
system. a (1 + k)
s1
a ( 2 + k ) − (1 + k)
Q.12 (c) s0
Closed loop gain is a
G(s) s+8 (1 + k)
= 2 a ( 2 + k ) − (1 + k)
1 + G(s) s + αs − 4 + s + 8 For oscillation =0
Characteristic equation a
q ( s ) = s 2 + ( α + 1) s + 4 ⇒a=
 1+ k 
 
Closed loop system is stable only for  2+ k 
α > −1 .Therefore, for all positive of, Now
the closed loop system is stable. as 2 + (1 + k ) = 0
2
-aω + (1 + k) = 0
Q.13 (c) Given ω = 2rad / sec
Characteristic equation −4a + (1 + k ) = 0
q ( s ) = s5 + 2s 4 + 3s3 + 6s 2 + 5s + 3
(1 + k)
1 −4 + (1 + k ) = 0
Putting s = (2 + k)
z
'( z )
−4 (1 + k ) + ( 2 + k )(1 + k ) = 0
q = 3z + 5z 4 + 6z 3 + 3z 2 + 2z + 1
5

Routh array is (1 + k ) ( 2 + k ) − 4 =


0
k = −1, 2
But k = −1 is not possible as system
will not oscillate for this as
a = 0 so k = 2

© Copyright Reserved by Gateflix.in No part of this material should be copied or reproduced without permission
1+ k 3 Q.17 (d)
a= = = 0.75
2+ k 4
Q.18 (3)
Q.15 (2.25) We can proceed here by taking this
K polynomial as characteristic equation
Given G ( s ) = and conclusion can be draw by using
( s + 2 ) (s − 1) RH criterion. As we are interested to
H(s)=1 know how many roots are lying on
Characteristic equation: right half of s plane.
1 + G (s) H (s) =0
K
1+ =0
( s + 2 ) (s − 1)
9
The poles are S1,2 =−1 ± − 4K
4
9
If   −K = 0 , then both poles of the
4
closed loop system at the same
location.
9
So, K = ⇒ 2.25
4
 t
1
φ (t)=  
0 1

Q.16 (2) → The number of roots i.e. the


ps + 3ps − 2
2 number of zeros in this case in right
Given G p ( s ) = half of plane is number of sign
s + ( 3 + p ) s + (2 − p)
2
changes
By R - H criteria → Number of sign changes = 3
The characteristic equation is
s2 + ( 3 + p ) s + ( 2 − p ) =
0 Q.19 (d)
i.e. s + ( 3 + p ) s + ( 2 − p ) =
2
0
By forming R-H array,
From the table we can find
characteristic equation
S3 + 2ks 2 + ( 2k + 3) s + 4 =
0
For stability ( 2k )( 2k + 3) > 4
For stability, first column elements 4k + 6k − 4 > 0 4
2

must be positive and non-zero 1


i.e. (1)(3 + p) > 0 ⟹ p > - 3 (k- ) ( k + 2 ) > 0
2
and (2)(2-p) > 0 ⟹ p < 2
1
i.e. – 3< p< 2 > and
So the conditions are k  
The maximum value of p unit which 2
G p remains stable is 2 k > −2 combining k > −2

Q.20 (c)

© Copyright Reserved by Gateflix.in No part of this material should be copied or reproduced without permission
Q.21 (b)
Given
p ( s ) =s3 + a 2s 2 + a1s + a 0
p ' ( s ) =3s 2 + 2a 2s + a1
Given p ' ( s ) has no real roots.
We know that, if p ( s ) has 'n ' real
roots, then p ' ( s ) will have at least 'n
−1' real roots.
Hence, from the given condition,
n − 1 =0
n =1

© Copyright Reserved by Gateflix.in No part of this material should be copied or reproduced without permission
GATE QUESTIONS(EC)(Root Locus)

Q.1 The root-locus diagram for a closed-


loop feedback system is shown in a)
the figure. The system is over damped.

a) only if 0 ≤ K ≤ 1 b)
b) only if 1 < K < 5
c) only if K > 5
d) if 0 ≤ K < 1 or K > 5
[GATE -2001]
Q.2 Which of the following points is NOT c)
on the root locus of a system with
the open–loop transfer function
K
G (s) H (s) = ?
s(s + 1)(s + 3)
a) s = − j 3 b) s = −1.5
c) s = −3 d) s = −∞
d)
[GATE -2002]
Q.3 The root locus of the system
K
G ( s ) H(s) = has the
s(s + 2)(s + 3)
break-away point located at
a)(-0.5,0) b)(-2.5478,0)
c)(-4,0) d)(-0.784,0) [GATE -2005]
[GATE -2003] Q.6 A unity feedback control system has
K an open-loop transfer function
Q.4 Given G ( s ) H(s) = , the K
s(s + 1)(s + 3) G (s ) = . The gain K for
point of intersection of the s ( s +7s+12 )
2

asymptotes of the root loci with the which s =−1 + j1 will lie on the root
real axis is locus of this system is
a)-4 b)1.33 a)4 b)5.5
c)-1.33 d)4 c)6.5 d)10
[GATE -2004] [GATE-2007]
Q.5 A unity feedback system is given as Q.7 The feedback configuration and the
K(1 − s) pole–zero locations of
G (s) = . Indicate the correct
s(s + 3)
root locus diagram

© Copyright Reserved by Gateflix.in No part of this material should be copied or reproduced without permission
s 2 -2s+2
G (s) = are shown below.
s 2 +2s+2
The root locus for Negative values of
K, i.e. for −∞ < K < 0, has
breakaway/break-in points and
angle of departure at pole P (with
respect to the positive real axis)
equal to s +1
a)
( s + 2 )( s + 4 )( s + 7 )
s+4
b)
( s + 1)( s + 2 )( s + 7 )
s+7
c)
a) ± 2and0° b) ± 2and45° ( s + 1)( s + 2 )( s + 4 )
c) ± 3and0° d) ± 3and45° ( s + 1)( s + 2 )
[GATE-2009] d)
( s + 7 )( s + 4 )
Q.8 The root locus plot for a system is [GATE-2014]
given below. The open loop transfer
function corresponding to this plot Q.10 The open-loop transfer function of a
is given by unity-feedback control system is
K
G (s) = 2
s + 5s + 5
The value of K at the breakaway
point of the feedback control
system's root-locus plot is
[GATE-2016]

Q.11 The forward-path transfer function


s(s + 1)
a) G ( s ) H ( s ) = K and the feedback-path transfer
( s + 2 ) (s + 3) function of a single loop negative
(s + 1) feedback control system are given
b) G ( s ) H ( s ) = K K(s + 2)
s ( s + 2 ) (s + 3) 2 as G ( s ) = 2 and H(s)=1,
1 s + 2s + 2
c) G ( s ) H ( s ) = K respectively, If the variable
s ( s − 1) (s + 2)(s + 3) parameter K is real positive, then
(s + 1) the location of the breakaway point
d) G ( s ) H ( s ) = K
s(s + 2)(s + 3) on the root locus diagram of the
[GATE-2011] system is ___.
[GATE-2016]
Q.9 In the root locus plot shown in the Q.12 A linear time invariant system with
figure, the pole/zero marks and the the transfer function
arrows have been removed. Which 𝐾𝐾(𝑠𝑠2 +2𝑠𝑠+2)
one of the following transfer G(s)= 2
(𝑠𝑠 −3𝑠𝑠+2)
functions has this root locus? Is connected in unity feedback
configuration as shown in the figure.

© Copyright Reserved by Gateflix.in No part of this material should be copied or reproduced without permission
For the closed loop system shown,
the root locus for 0<K<∞ intersects
the imaginary axis for K=1.5. The
closed loop system is stable for
a) K>1.5
b) 1<K<1.5
c) 0<K<1.5
d) No positive value of K

ANSWER KEY:

1 2 3 4 5 6 7 8 9 10 11
(d) (b) (d) (c) (c) (d) (b) (b) (b) 1.25 -3.414

© Copyright Reserved by Gateflix.in No part of this material should be copied or reproduced without permission
EXPLANATIONS

Q.1 (d) Angle of asymptotes


For over damping roots of
characteristic equation should lie on π
= (2K + 1)
negative axis and be unequal. P−Z
π π 5π
Q.2 (b) = (2K + 1) , π,
3 3 3
∴ Break away point is (-0.784,0)as -
2.55 does not lie on RL.

Q.4 (c)
ΣP − ΣZ
Centroid =
RL lies where no. of poles and zeros P−Z
to the right of the pole is odd. −1 − 3
= = −1.33
∴ s =−1.5 doesn’t lie on RL 3

Q.3 (d) Q.5 (c)


1+ G (s) H (s) = 0 1+ G (s) H (s) =
0
K s 2 + 3s
⇒ 1+ =0 K=
s ( s + 2 ) (s + 3) 1− s
For breakaway & break in point
K
⇒ 1+ =0 dK
s(s + 5s + 6)
2 = (1 − s )( 2s + 3) + s 2 + 3s = 0
ds
⇒K=−1. ( s3 + 5s 2 + 6s ) =−s 2 + 2s + 3 =0
⇒ s 2 − 2s − 3 =
− ( 3s 2 + 10s + 6s )
dK 0
⇒ =
ds ⇒ ( s − 3)( s + 1) = 0
dK
=0 s= 3, −1
ds -1is the breakaway point and 3 is
⇒ 3s 2 + 10 s + 6 =0 the break in point.
−10 ± 100 − 72
s=
6
Q.6 (d)
−10 ± 5.3
= = −0.784, −2.55 G(s)
6 T.F. =
1 + G(s)
As H ( s ) = 1
For the point s =−1 + j1 to lie on
root locus
1+ G (s) = 0
K
−2 − 3 −5 ⇒ 1+ 2 =0
Centroid = = = −1.66 s(s + 7s + 12)
−3 3

© Copyright Reserved by Gateflix.in No part of this material should be copied or reproduced without permission
(s 2
+ 7s + 12 ) + K =
0 (s + 4)
Putting ( s + 1)( s + 2 )( s + 3)
s =−1 + j ( −1 + j) (1 − 2j − 1 − 7 + 7 j + 12) + K =0 From pole zero plot

⇒K=+10

Q.7 (b)

Q.10 (1.25)
In this first we need to find the
break point by finding the root of
dk
1+ G (s) H (s) =
0 = 0 and then by using magnitude
ds
K(s 2 − 2s + 2) condition value of k can be obtained.
1+ = 0 K
s 2 + 2s + 2 G (s) = 2
s 2 + 2s + 2 s + 5s + 5
K= − 2 K = −(s 2 + 5s + 5)
s − 2s + 2
∂K dk
=0
Put = 0 we have
∂s ds
(s 2 − 2s + 2) ( s + 1) ⇒ 2s + 5 =0 ⇒ s =− 2.5
→ Applying magnitude condition G ( s ) = 1
− ( s 2 + 2s + 2 ) ( s − 1) =
0
K
2s 2 − 4s 2 + 4 =0 =1
s + 5s + 5 s = −2.5
2
2s = +4
2

 
s= ± 2 ⇒
k
 =1
Angle of departure is  (−2.5) + 5x ( −2.5 )  + 5 
2

Φ D 180° + Φ
=
 k 
⇒ =
=
Where Φ ∑Φ − ∑Φz p  6.25 − 12.5 + 5 
1

Φ = 135° k
Φ D =180 − 135° =45° ⇒ =1 ⇒ k =1.25
−1.25
So (b) options is correct.
Q.11 (-3.414)
Q.8 (b) To find break point, from
From plot we can observe that one characteristic equation we need to
pole terminates at one zero at arrange k as function of s, then the
position -1 and three poles root of
dk
= 0 gives break point.
terminates to ∞. It means there is ds
four poles and 1 zero .Pole at -3 goes s 2 + 2s + 2 + k + 2k = 0
on both sides. It means there are ⇒ k ( s + 2 ) = -s + 2 s + 2
2

two poles at -3.


 s 2 + 2s + 2 
⇒ k = − 
Q.9 (b)  s+2 
For transfer function

© Copyright Reserved by Gateflix.in No part of this material should be copied or reproduced without permission
 
 ( s + 2 ) ds ( s + 2s + 2 ) − ( s + 2s + 2 ) ds (s + 2) 
d 2 2 d
dk
⇒ = − 
ds  (s + 2)2 
 
dk  ( s + 2 )( 2s + 2 ) − ( s 2 + 2s + 2 ) 
⇒ = − 
ds  (s + 2) 2 
dk
⇒ = 0
ds
⇒ 2s 2 + 2s + 4s + 4 − s 2 − 2s − 2 = 0
⇒ s + 4s + 2 =
2
0
⇒ s =−0.58 and − 3.414

• To find the valid break point we


need to find that lies on root
locus
• -3.414 lies on root locus
• So break point -3.414.

© Copyright Reserved by Gateflix.in No part of this material should be copied or reproduced without permission
GATE QUESTIONS(EE)(Stability Analysis)
Q.1 The roots of the closed loop a) Stable
characteristic equation of the b) Unstable
system are c) Conditionally stable
d) stable for input u1 , but unstable
for input u 2
[GATE-2007]

a) -1 and -15 b) 6 and 10 Q.6 If the loop gain k of a negative


c) -4 and -15 d) -6 and -10 feedback system having a loop
[GATE-2003] transfer function k(s + 3) / ( s + 8 ) is
2

Q.2 The loop gain GH of a closed loop to be adjusted to induce a sustained


system is given by the following oscillation then
K a) The frequency of this oscillation
expression . The value must be 4 / 3 rad/s
s ( s + 2 ) (s + 4)
b) The frequency of this oscillation
of K for which the system just
must be 4 rad/s
becomes unstable is
c) The frequency of this oscillation
a) K=6 b) K=8
c) K=48 d) K=96 must be 4 or 4 / 3 rad/s
[GATE-2003] d) Such a k does not exist
[GATE-2007]
Q.3 For the equation, s3 − 4s 2 + s + 6 =0
the number of roots in the left half Q.7 Figures shows a feedback system
of s-plane will be where k > 0
a) zero b) one
c) two d) three
[GATE-2004]
Q.4 A unity feedback system, having an
k (1 − s )
open loop gain G ( s ) H ( s ) = ,
(1 + s ) The range of k for which is stable
will be given by
becomes stable when
a) 0 < k < 30 b) 0 < k < 39
a) lkl > 1 b) k > 1
c) lkl < 1 d) k < −1 c) 0 < k < 390 d) k > 390
[GATE-2005] [GATE-2008]

Q.5 The system shown in the figure is Q.8 The first two rows of Routh’s
tabulation of a third order equation
are as follows.
S3 2 2
S 2
4 4
This means there are

© Copyright Reserved by Gateflix.in No part of this material should be copied or reproduced without permission
a) two roots at s = ±j and one root Q.11 A single-input single-output feedback
in right half s-plane system has forward transfer
b) two roots at s = ±j2 and one root function G(s) and feedback transfer
in left half s-plane function H(s) It is given that I
c) two roots at s = ±j2 and one root G(s).H(s)I<1. Which of the following
is true about the stability of the
in right half s-plane
system?
d) two roots at s = ±j and one root
a) The system is always stable
in left half s-plane b) The system is stable if all zeros
[GATE-2009] of G(s).H(s) are in left half of the
s-plane
Q.9 An open loop system represented by c) The system is stable if all poles
the transfer function of G(s).H(s) are in left half of the
(s-1)
G (s) = is s-plane
( s+2 ) (s+3) d) It is not possible to say whether
a) stable and of the minimum phase or not the system is stable from
type the information given
b) stable and of the non-minimum [GATE-2014]
phase type Q.12 Given the following polynomial
c) unstable and of the minimum
equation s3 + 5.5s 2 + 8.5s + 3 =0 , the
phase type
number of roots of the polynomial,
d) unstable and of the non-
which have real parts strictly less
minimum phase
than -1, is
[GATE-2011]
[GATE-2016]
Q.10 The feedback system shown below Q.13 The open loop transfer function of a
oscillates at 2rad/s when unity feedback control system is
K(s + 1)
given by G ( s ) = ,
s (1 + Ts ) (1 + 2s)
K > 0, T > 0 .The closed loop system
will be stable if
4(K + 1) 4(T + 2)
a) 0 < T < b) 0 < K <
K −1 T−2
(T + 2) 8(K + 1)
a)K=2 and a=0.75 b)K=3 and a=0.75 c) 0 < K < d) 0 < T <
c)K=4 and a=0.5 d)K=2 and a=0.5 T −1 K −1
[GATE -2012] [GATE-2016]

ANSWER KEY:
1 2 3 4 5 6 7 8 9 10 11 12 13
(d) (c) (b) (c) (d) (b) (c) (d) (b) (a) (a) 2 (c)

© Copyright Reserved by Gateflix.in No part of this material should be copied or reproduced without permission
EXPLANATIONS

Q.1 (d)
1 + GH = 0 (Characteristic equation)
45
1+ =
0
( s + 15) (s + 1)
s 2 + 16s + 60 = 0
∴ ( s + 10 )( s + 6 ) =
0
∴ roots are (-6, -10). No. of sign changes in first column of
Routh-array=2.
Q.2 (c) According to Routh-Hurwitz
Characteristic equation criterion, the number of changes of
=1 + GH = 0 sign in the first column gives the
K number of positive real part roots of
⇒ 1+ =0
s(s + 2)(s + 4) the polynomial.
Characteristic eq. So, no. of roots in RHS of s-plane=2.
⇒ s3 + 6s 2 + 8s + k =0 Total no. of roots=3
The Routh-array is formed as Hence, no. of roots in LHS of s-plane
follows =3-2=1

Q.4 (c)
Characteristic equation
1+ G (s) H (s) = 0
k(1 − s)
⇒ 1+ = 0
According to Routh-Hurwitz 1+ s
criterion, for a stable system there ⇒ s + 1 + k (1 − s ) =0
should be no change of sign in the
⇒ s (1 − k ) + 1 + k =0
first column of Routh array.
48 − K k +1
So, > 0 and K > 0 s=
6 k −1
0 < K < 48 For a stable system pole lies in left
Hence for K=48, the system just hand side of s=plane, it means and
becomes unstable. must be negative for stable system.
k +1
s<0 <0
Q.3 (b) k −1
s 2 − 4s 2 + s + 6 =0 Case-I
The Routh-array is formed as k + 1 < 0 and k − 1 > 0
follows k < −1 and k > 1
Which is not possible.
Case-II
k + 1 > 0 and k − 1 < 0
k > −1andk < 1
−1 < k < 1

© Copyright Reserved by Gateflix.in No part of this material should be copied or reproduced without permission
k <1 ⇒ s = ± j4
So, frequency of oscillation 4 rad/
Q.5 (d) sec.

Q.7 (c)
k
G (s) = and H(s)=1
s ( s + 3) (s + 10)
Characteristic equation
⇒ 1+ G (s) H (s) = 0
Pole is in LHS of s-plane, hence k
stable. 1+ = 0
s ( s + 3) (s + 10)
(s − 1)
(s + 2) (s − 1) ⇒ s ( s + 3)( s + 10 ) + k =0
=( T / F )1 = (s − 1) s3 + 13s 2 + 30s + k =
1+ ×
1 (s + 3) 0
(s + 2) (s − 1) Routh-Array
1
(s − 1)
( T / F)2 = 1 (s − 1)
1+ ×
(s − 1) (s + 2)
s+2
=
( s − 1) (s + 3) According to Routh-Hurwitz criterion.
Hence unstable as it has pole at right For a stable system, signs of first
hand side of S-plane. column do no change
13 × 30 − k
k > 0 and >0
Q.6
(b) 13
Loop transfer function Therefore system to be stable
(s + 3) 0 < k < 390
= G= ( s ) H(s) k
(s + 8) 2
∴ characteristic equation Q.8 (d)
= 1+ G (s) H (s) = 0
Routh-Array
(s + 3)
⇒ 1+ k = 0
(s + 8) 2
⇒ s 2 + 16s + 64 + (s + 3) =
0
⇒ s + (16 + k ) s + 64 + 3k =
2
0 The third row vanishes. An auxiliary
Routh- Array equation is formed using elements
of 2nd row.
Auxiliary equation
A ( s )= 4s 2 + 4= 0
⇒ s =± j .
For sustained oscillation The derivative of this auxiliary
16+k=0 equation is taken wrt s and the
⇒ k= −16s 2 + 64 + 3k coefficients of the differentiated
= s 2 + 64 + 3x ( −16 ) = 0 equation are taken as the elements
s 2 + 16 =
0 of 3rd row.

© Copyright Reserved by Gateflix.in No part of this material should be copied or reproduced without permission
dA(s) (1 + k)
= 8s −4 + (1 + k ) = 0
ds (2 + k)
Routh-Array −4 (1 + k ) + ( 2 + k )(1 + k ) =
0
(1 + k ) ( 2 + k ) − 4 =
0
k = −1, 2
But k = −1 is not possible as system
will not oscillate for this as
There is no root in RHS of s-plane. a = 0 so k = 2
Two roots of s = ± j , so one root is in 1+ k 3
LHS of s-plane. a= = = 0.75
2+ k 4
Q.9 (b) Q.11 (a)
s −1
G (s) =
( s + 2 ) (s + 3) Q.12 (2)
one zero at s=1 The polynomial is
two poles at s=-2 & -3 S3 + 5.5S2 + 8.5S + 3 = 0 , since we are
Since zero lies in RHS of s-plane. interested to see the roots wrt S. -1
It is non-minimum phase type so in the above equation replace S
system. by z-1 then the equation is
Since both poles lie in LHS of s- ( Z − 1)3 + 5.5( Z − 1) 2 + 8.5( Z − 1) + 3 =0
plane, system is stable.
⇒ Z3 − 3Z2 + 3Z − 1 + 5.5
Q.10 (a) (Z2 + 1 − 2Z) + 8.5Z − 8.5 + 3 =0
Characteristic equation is ⇒ Z 3 + 2.5Z 2 + 0.52 − 1 =0
1+ G (s) H (s) = 0 ⇒ Z 3 + Z 2 (−3 + 5.5)
k(s + 1) + Z (3 + 8.5 − 11) + (−1 + 5.5 − 8.5 + 3) =0
1+ 3 = 0
s + as 2 + 2s + 1 Using RH table
s3 + as 2 + ( 2 + k ) s + (1 + k ) = 0
Routh array for this is
s3
1 (2 + k)
s2 The single sign change in 1st column
a (1 + k)
s1 indicate that out of 3 roots 1 root lie
a ( 2 + k ) − (1 + k)
s0 on the right half of S=-1 plane if
a memory remaining 2 lies on left half
(1 + k) of S =-1 plane.
a ( 2 + k ) − (1 + k)
For oscillation =0 Q.13 (c)
a
To comment closed 100b system
 1+ k 
⇒a=   stability we need the characteristic
 2+ k  equation. Here it is given that it is a
Now unity feedback system.
as 2 + (1 + k ) = 0 Unity feedback system
−aω2 + (1 + k ) =
0 So the characteristic equation is
S(1 + TS)(1 + 2s) + K(S + 1) =
0
Given ω = 2rad / sec
⇒ (S + TS2 )(1 + 2S) + KS + K =
0
−4a + (1 + k ) =
0
⇒ S + 2S2 + TS2 + 2T S3 + KS + K =0

© Copyright Reserved by Gateflix.in No part of this material should be copied or reproduced without permission
⇒ S3 (2T) + S2 (2 + T) + S(1 + k) + k = 0
 2 + T  2  k +1  k
⇒ S3  S +  S + = 0
 2T   2T  2T
→ for stability using criterion R(t)
 2 + T   K +1 K
  >
 2T   2T  2T
K +1 1
⇒ (T + 2)(K + 1) > K ⇒ >
K T+2
1 1 1  T +1  T + 2
⇒ > −1 ⇒ > −  ⇒ K < −
K T+2 K T+2  T + 1 

© Copyright Reserved by Gateflix.in No part of this material should be copied or reproduced without permission
GATE QUESTIONS(EE)(Root Locus)

Q.1 A unity feedback system has an K K


c) d)
open loop transfer function, s(s + 1) s ( s 2 − 1)
2

K
G ( s ) = 2 . Its root locus plot will be [GATE-2005]
s
a) Q.3 A closed-loop system has the
characteristic function
(S2 − 4 ) ( s + 1) + K ( s − 1) =
0.
Its root locus plot against K is
b) a)

c) b)

d)
c)

[GATE-2002]

Q.2 Figure shows the root locus plot


(location of poles not given) of a d)
third order system whose open loop
transfer function is

[GATE-2006]

K K Q.4 The characteristic equation of a


a) b) closed-loop system is
s3 s ( s + 1)
2

s ( s + 1)( s + 3) + k ( s + 2 ) = 0, k > 0 .

© Copyright Reserved by Gateflix.in No part of this material should be copied or reproduced without permission
Which of the following statements is a) It corresponds to a frequency
true? greater than K
a) Its roots are always real b) It corresponds to a frequency
b) It cannot have a breakaway less than K
point in the range -1 < Re [s ] < 0 c) It corresponds to a frequency K
c) Two of its roots tend to infinity d) Root locus of modified system
along the asymptotes Re [s ] = -1 never transits to unstable region
[GATE-2015]
d) It may have complex roots in the
right half plane Q.7 An open loop transfer function G(s)
[GATE-2010] k
of system is G ( s ) = .
s ( s + 1) (s + 2)
Q.5 The open loop transfer function G(s)
of a unity feedback control system is For a unity feedback system, the
 2 breakaway point of the root loci on
k  s+  the real axis occurs at,
given as, G ( s ) = 2
3
a)-0.42
s ( s+2 ) b) -1.58
From the root locus, it can be c) -0.42 and -1.58
inferred that when k tends to d) none of the above.
positive infinity [GATE-2015]
a) three roots with nearly equal
real parts exist on the left half of Q.8 The gain at the breakaway point of
the s-plane the root locus of a unity feedback
b) one real root is found on the system with open loop transfer
right half of the s-plane Ks
function G(s) is
c) the root loci cross the jω axis for ( s − 1) (s − 4)
a finite value of k:k≠0
a) 1 b) 2
d) three real roots are found on the
c) 5 d) 9
right half of the s-plane
[GATE-2016]
[GATE-2011]

Q.6 The open loop poles of a third order


unity feedback system are at 0,-1, -2.
Let the frequency corresponding to
the point where the root locus of the
system transits to unstable region
be K. Now suppose we introduce a
zero in the open loop transfer
function at -3, while keeping all the
earlier open loop poles intact. Which
one of the following is TRUE about
the point where the root locus of the
modified system transits to unstable
region?

© Copyright Reserved by Gateflix.in No part of this material should be copied or reproduced without permission
ANSWER KEY:
1 2 3 4 5 6 7 8
(b) (a) (b) (c) (a) (d) (a) (a)

© Copyright Reserved by Gateflix.in No part of this material should be copied or reproduced without permission
EXPLANATIONS

Q.1 (b) ⇒ ( S2 − 4 ) ( s + 1) + K ( s − 1) =
0
K
G ( s ) H(s) = K ( s − 1)
s2 ⇒ 1+ ≡ 1 + G(s)H(s)
∴ P−Z= 2 (S 2
− 4 ) ( s + 1)
0 Open loop transfer function
Centroid = = 0
2 K ( s − 1)
= G(s)H(s)
=
Angle of asymptotes = 90° , 270° ( S2 − 4 ) ( s + 1)
∴ Option (b) is correct. Zero of OLTF s=1; z=1
Poles of OLTF s=-1, -2, +2, P=3
Q.2 (a) The root locus starts from open-loop
poles and terminates either on
open-loop zero or infinity.
Root locus exist on a section of real
axis it the sum of the open-loop
poles and zeros to the right of the
section is odd.
Number of branches terminating on
infinity.= P-Z=3-1=2
These are three asymptotes with
Angles of asymptotes
angle 60° ,180° and 300°
( 2k + 1) × 180o ( 2k + 1) × 180o
Angle of asymptotes = =
P−Z 2
( 2k + 1) × 180o = 90 and270
=
o o

P−Z Intersection of asymptotes on real


Where k=0, 1, 2 upto (P-Z) -1 as axis (centroid)
angles are 60° ,180° and 300° it means Σpoles − Σzero ( −1 − 2 + 2 ) − (1)
P–Z=3 = = = −1
P−Z 2
Intersection of asymptotes on real Option (d) is correct on the basic of
axis above analysis.
Σpoles − Σzero
X=
P−Z Q.4 (c)
Since, system does not have zeros Characteristic equation
Σpoles s ( s + 1)( s + 3) + k ( s + 2 ) =
X= 0
P k ( s + 2)
As asymptotes intersect at origin, it 1+ =
0
means all the three poles are at s ( s + 1)( s + 3)
origin. Comparing with 1+G(s)H(s)=0
Hence, option (a) is correct. G(s)H(s) = Open-loop transfer
function (OLTF)
Q.3 (b) k ( s + 2)
=
Characteristic function s ( s + 1)( s + 2 )
no. of zero=Z=1 zero at -2

© Copyright Reserved by Gateflix.in No part of this material should be copied or reproduced without permission
no. of poles=P=3 poles at 0,-1 &-3 Angle of asymptotes
No. of branches terminating at ( 2k + 1) × 180o
infinity =
P−Z
=P-Z=3-1=2
Angle of asymptotes ( 2k + 1) × 180o
=
( 2k + 1) × 180o 2
= = ( 2k + 1) × 90o
P−Z
( 2k + 1) × 180o = 90oand270o
= Σpoles − Σzero
2 Centroid =
= ( 2k + 1) × 90o P−Z
 2
= 90oand270o 0+0−2−− 
Σpoles − Σzero =  3 = − 2
Centroid = 2 3
P−Z
0 − 1 − 3 − ( −2 )
= = −1
2
Breakaway point lies in the range -1
< Re[s] < 0 and two branches
terminates at infinity along the
asymptotes Re(s) = -1.
Since, all the
three branches
Q.5 (a) 2
 2 terminates at Re ( s ) = − .
ks+  3
G ( s ) = 2
3 So all the three roots have nearly
and H(s) =1
s ( s + 2) equal real part.
Characteristic equation
 2
ks+  Q.8 (a)
1+G(s)H(s)=0 ⇒ 1 + 2
 3
=0 Ks
s ( s + 2) G(s)
(s − 1)(s − 4)
 2 To find Break away point
⇒ s3 + 2s 2 + k  s +  = 0
 3
2k
⇒ s3 + 2s 2 + ks + = 0
3
Routh Array
As k > 0, there is no sign change in
the 1st column of routh array. So the dk
We need to find the root of =0
system is stable and all the three ds
roots lie on LHS of s-plane. where
For k > 0 ( k ≠ 0 ) , none of the row of (s − 1)(s − 4)  s 2 − 5s + 4 
K= − = 
routh array becomes zero. So root s  s 
loci does not cross the jω axis. d d
no. of zero=Z=1 s (s 2 − 5s + 4) − (s 2 − 5s + 4) (s)
dk
no. of poles=P=3 = ds 2
ds
ds s
No. of branches terminating at
⇒ S(2S − 5)(s − 5s + 4) =
2
0
infinity
=P-Z=3-1=2 ⇒ 2S2 − 5S − S2 + 5S − 4 =0

© Copyright Reserved by Gateflix.in No part of this material should be copied or reproduced without permission
⇒ S2 − 4 =0 ⇒ S =±2
From the pole zero plot it is clean
that Break away point must be as it
is in between 2 poles
Now to find gain at this point use
magnitude condition
KS KS
⇒ =1 ⇒ =1 ⇒ K =1
(s − 1)(s − 4) s = 2 (1)(−2)

© Copyright Reserved by Gateflix.in No part of this material should be copied or reproduced without permission
GATE QUESTIONS(IN)(Stability Analysis)

Q.1 The range of the controller gains a) 1 < k < 3 b) 0 < k < 10
( K p , K i ) that makes the closed loop c) 5 < k < ∞ d) 0 < k < ∞
[GATE-2010]
control system (shown in the
following figure) stable is given as
Q.4 The first two rows of Routh’s table
of a third-order characteristic
equation are
s3 3 3
s 2
3 4
It can be inferred that the system
Ki has
a) K i < 0 and K p < − 20 a) one real pole in the right- half of
12
K s-plane
b) K i < 0 and K p > i − 20 b) a pair of complex conjugate
12
poles in the right-half of s-plane
c) K i < 0 and K p > 0
c) a pair of real poles symmetrically
Ki placed around s=0
d) K i < 0 and K p > − 20
12 d) a pair of complex conjugate
[GATE-2006] poles on the imaginary axis of
the s-plane
Q.2 A closed loop control system is [GATE-2011]
shown below. The range of the
controller gain K c which will make Q.5 The value of a 0 which will ensure
the real parts of all the closed loop that the polynomial s3+2s+a 0 has
poles more negative than -1 is roots on the left half of the s-plane is
a) 11 b) 9
c) 7 d) 5
[GATE-2016]

Q.6 For the feedback system given


below, the transfer function
1
a) K c >-4 b) K c >0 G (s) = . The system
( s + 1)
2

c) K c >2 d) K c <2
CANNOT be stabilized with
[GATE-2008]

Q.3 The open loop transfer function of a


unity gain feedback system is given
k(s + 3)
by : G(s) . The range of
( s + 1) (s + 2)
3 7
positive values of k for which the a) C ( s ) = 1 + b) C ( s )= 3 +
closed loop system will remain s s
stable is:

© Copyright Reserved by Gateflix.in No part of this material should be copied or reproduced without permission
9 1
c) C ( s )= 3 + d) C ( s ) =
s s
[GATE-2016]

Q.7 Consider a standard negative


feedback configuration with
1 s +α
G (s) = and H ( s ) =
( s + 1)( s + 2) s
. For the closed loop system to have
a poles on the imaginary axis, the
value of α should be equal to (up to
one decimal place) ________________.
[GATE-2018]

© Copyright Reserved by Gateflix.in No part of this material should be copied or reproduced without permission
ANSWER KEY:
1 2 3 4 5 6 7
(d) (c) (d) (d) (d) (c) 9

EXPLANATIONS

Q.1 (d)
Characteristic equation is System is stable, for all positive K.
(
s ( s + 2 )( s + 10 ) + k pS + k i =
0 ) (from Routh Hurwitz criterion)
(or) 0 < K< ∞
s3 + 12s 2 + 20s + k pS + k i =
0
1 20 + k p
s3
12 ki
s2 Q.4 (d)
s1
( 240 + 12k p − ki )
12 s3 3 3
1
ki s2 4 4
∴ K i > 0and240 + 12k p − k i > 0 s 0 (∈> 0)
Ki 1 4
⇒ Kp > − 20
12 ∴ C.E is 3S3 + 4S2 + 3S + 4
Q.2 (c) =0 ⇒ ( 3S + 4 ) ( S2 + 1) =0
s ( s + 3) + K c =
0 4
S=− ±J
Gs 2 + 3s + K c =
0 3,
−3 ± 9 − 4K c 9 Q.5 (d)
S1,2 = =
−1.5 ± − Kc
2 4

9 9
−1.5 + − K c < −1or − K c < 0.5
4 4
9
− K c < 0.25, K c > 2 OR 6 − a0
4 For Stability > 0 ⇒ a0 < 6
Put S=Z-1 and apply RH criterion for 3
the polynomial in z
Q.6 (c)
Q.3 (d) The characteristic equation of
1+ G (s) H (s) =
0 system is 1+G(s) = 0
C(S)
⇒ ( s + 1)( s + 2 ) + k ( s + 3) =
0 ⇒ 1+ 2 =0
S + 2S + 1
⇒ S2 + 2S + 1 + C(S) =0
9
if we take C(S)= 3 + then
5

© Copyright Reserved by Gateflix.in No part of this material should be copied or reproduced without permission
S3 1 4
2
S 2 9
S −1/ 2
1

S0 9
9
S2 + 2S + 1 + 3 + =0
S
⇒ S3 + 2S2 + 4S + 9 = 0
So system is unstable, remaining
options gives stable.

Q.7 9
1
G (s) =
( s + 1)( s + 2)
s +α
H (s) =
s
C.E = 1 + G (s) H (s) = 0;
s ( s + 1)( s + 2) + ( s + α ) =0
s 3 + 3s 2 + 2 s + s + α =0
s 3 + 3s 2 + 3s + α =
0
If system is m arg inal stable
3× 3 =α
α =9

© Copyright Reserved by Gateflix.in No part of this material should be copied or reproduced without permission
GATE QUESTIONS(IN)(Root Locus)

Q.1 The roots locus of a plant is given in Q.3 If the root locus plot of the closed
the following figure. The rot locus loop system passes through the
crosses imaginary at ω = 4 2rad / s points ± j 11 , the maximum value of
with gain K= 384. It is observed that K for stability of the unity feedback
the point s = −1.5 + j 1.5 lies in the closed loop system is
root locus. The gain K at a) 11 b) 6
s= −1.5 + j1.5 is computed as c) 10 d) 6 11
[GATE-2007]

Q.4 The open loop transfer function of a


unity feedback system is
k(s+2)
G (s) =
( s+1+j1) (s+1-j1)
The root locus plot of the system has
a) Two breakaway points located at
a) 11.3 b) 21.2
s = -0.59 and s = -3.41
c) 41.25 d) 61.2
b) One breakaway point located at
[GATE-2006]
s = -0.59
c) One breakaway point located at
Statement for linked Answer
s = -3.41
Questions Q.2 & Q.3:
d) One breakaway point located at
A transfer function with unity DC
s = -1.41
gain has three poles at -1, -2 and -3
and no finite zeros. A plant with this [GATE-2008]
transfer function is connected with
this transfer function is connected Q.5 Consider the second- order system
with a proportional controller of with the characteristic equation
gain K in the forward path, in a unity s(s+3)+K(s+5)=0 Based on the
feedback configuration. properties of the root loci, it can be
shown that the complex portion of the
root loci of the given system for 0<k
Q.2 The transfer function is
<∞ is described by a circle, and the
S
a) two breakaway points on the real
( s − 1)( s − 2 ) (s − 3) axis are .
6 5
b) a) −5 ± b) −5 ± 5
( s + 1)( s + 2 ) (s + 3) 2
c)
S c) −5 ± 10 d) −5 ± 2 5
( s + 1)( s + 2 )( s + 3) [GATE-2011]
6
d) Q.6 The open loop transfer function of a
( s − 1)( s − 2 )( s − 3) unity gain negative feedback control
[GATE-2007] system is given by

© Copyright Reserved by Gateflix.in No part of this material should be copied or reproduced without permission
s 2 +4s+8 Q.8 A loop transfer function is given by:
G (s ) = .The angle θ, at K(S + 2)
s ( s+2 ) (s+8) G ( S) H ( S) = 2
S (S + 10)
which the root locus approaches the
zeros of the system satisfies The point of intersection of the
asymptotes of G(s)H(s)on the real
1 3π 1
a) θ =π-tan -1   b) =
θ -tan -1   axis in the s-plane is at __________.
4 4 3 [GATE-2014]
π 1 π 1
c) θ= -tan -1   d) θ= -tan -1  
2 4 4 3 Q.9 The open loop transfer function of a
[GATE-2012] s 2 + 6s + 10
system G ( s ) = 2 . The angle
s + 2s + 2
Q.7 The Bode plot of transfer function of arrival of its root loci are
G(s) is shown in the figure below. π π
a) ± b) ±
4 3
π 5π
c) ± d) ±
2 6
[GATE-2015]

The gain (20log |G(s))| is dB and -8


at 1 rad/s and 10rad/s respectively.
The phase is negative for all ω then
G(s) is

39.8 39.8
a) b)
s s2
32 32
c) d) 2
s s
[GATE-2013]

ANSWER KEY:
1 2 3 4 5 6 7 8 9
(c) (b) (c) (c) (c) (d) (b) -4 (a)

© Copyright Reserved by Gateflix.in No part of this material should be copied or reproduced without permission
EXPLANATIONS

Q.1 (c) S =−2 ± 2 =−0.59 / −3.41


G (s) =
K RL exists at −3.41
s ( s + 4 ) (s + 8)
Q.5 (c)
1+ G (s) = 0, G ( s ) =
− G (s)
−s(s + 3)
K K=
= 1,= 1 (s + 5)
s s+ 4 |s+8|
At s =−1.5 + j1.5 =1.5 =1.5(−1 + j1) Dk
= 0⇒
( )
− ( s + 5 ) . ( 2s + 3) − ( s 2 + 35 ) .1
( S + 5)
2
=s 1.5 2, s + 4 ds
=0 ⇒ s 2 + 10s + 15 =0
= 2.52 + 1.52 = 8.5,
−10 ± 100 − 60
s +=
8 6.52 + 1.5=
2
44.5 s= =−5 ± 10
2
∴ K − 1.5 2 8.5 44.5 = 41.25
Note that the data given at the Q.7 (b)
intersecting point with imaginary
axis is not necessary

Q.2 (b)
With unity DC gain, poles at s=1,-2
and -3and no finite zeros
6
Plant T.F =
( s + 1)( s + 2 ) (s + 3)
= ω 1= to ω 10
Q.3 (c)
Is 1 dec are change & change is (G)
At s = ± J 11, G ( s ) .H ( s ) is 40 dB
= 1where G ( s ) .H ( s ) ∴ S lope is 40dB / dec
6k ∴ There are 2poles is orign
=
( s + 1) (s + 2(s + 3) K
So, G ( s ) = 2
S
6k
∴ =1 ⇒ k =10 G atω=1 = 32dB ( given )
2.3.5.2
k
⇒ 2 log =
32dB
Q.4 (c) ω2 ω=1
k(s + 2) − ( s 2 + 2s + 2 ) ⇒ 20log=k 32dB ⇒ =k 39.8
=(s) = ⇒ k
( s2 + 2s + 2 ) (s + 2) 39.8
∴G = 2
dk
= 0⇒
( s 2 + 2s + 2 ) .1 − ( s + 2 ) .2 ( s + 1)
S

( s + 2) Q.8 (4)
2
ds
= 0 ⇒ s + 4s + 2 = 0
2

© Copyright Reserved by Gateflix.in No part of this material should be copied or reproduced without permission
The point of intersection of the
asymptotes is nothing but a
centroid.
Centroid.

σ=
∑real part of poles − ∑real part of zeros
(P − Z )
K(s + 2)
Given , G ( s ) H ( s ) =
s 2 (s + 10)
P=3; Z=1⇒ (P − Z) = 2

σ=
[ −10 + 0 + 0] − [−2] = −8
= −4
2 2

Q.9 (a)
Angle of arrival is calculated on a
complex zero and it is given by,
φa= 180 − GH (at a +ve imaginary
zero )
G (s) =
( s + 3 + i ) (s + 3 − i)
( s + 1 + i ) (s + 1 − i)
G ( −3 + i ) =
[ −3 + i + 3 + i][ −3 + i + 3 − i ]
[ −3 − i + 1 + i ][ −3 + i + 1 − i]
=
[ 2i]
[ −2 + 2i][ −2]
G ( −3 + i )
 2
= 90° − 180° − tan −1  − [180°]
 2
= 90° − 180° + 45° − 180°= 135°
φa= 180° − 135°= 45°= π
4
Other angle will be same with
opposite sign ± π
4

© Copyright Reserved by Gateflix.in No part of this material should be copied or reproduced without permission
4 FREQUENCY DOMAIN ANALYSIS

4.1 FREQUENCY RESPONSE extracting the information regarding


stability as well as relative stability of a
It is a measure of magnitude and phase of
the output as a function of frequency, in system without the need to evaluate roots
comparison to the input. In simplest terms, of the characteristic equation.
if a sine wave is injected into a system at a
given frequency, a linear system will 4.1.2 RELATION WITH TRANSFER
respond at that same frequency with a FUNCTION
certain magnitude and a certain phase
angle relative to the input. Consider the transfer of a system as:
C (s)
T(s) =
4.1.1 ADVANTAGE OF FREQUENCY R (s)
RESPONSE The frequency response function can be
obtained simply by replacing s by 𝑗𝑗𝑗𝑗
1) The design and parameter adjustment C ( jω )
of the open-loop transfer function of a Then T( jω) =
R ( jω )
system for specified closed-loop
performance is carried out somewhat 1
e.g. if T ( s ) = then T ( jω) =
1
more easily in frequency domain than 1+ s 1 + jω
in time domain. From this frequency response function we
2) Further the effects of noise disturbance can calculate
and parameter variations are relatively
easy to visualize and assess through 1) Magnitude: The magnitude of the
frequency response. If necessary the frequency response function is also
transient response of a system can be called as the gain of system.
obtained from its frequency response 1
through the Fourier integral. An e.g. if T ( jω) = then
1 + jω
interesting and revealing comparison of
1
frequency and time domain approaches T ( jω ) =
is based on the relative stability studies 1 + ω2
2

of feedback systems. 2) Phase: The phase angle of the frequency


3) The Routh criterion is a time domain response function is nothing but the
approach which establishes with phase shift provided by the system to
relative stability of a system, but its the input.
adoption to determine the relative 1
e.g. if T ( jω ) = then
stability is involved and requires 1 + jω
repeated application of the criterion. 0
The root locus method is a very ∠T ( jω) = =− tan −1 ( ω)
 ω
powerful time domain approach as it tan −1  
reveals not only stability but also the 1
actual time response of the system. On
the other hand, the Nyquist criterion is
a powerful frequency domain method of

© Copyright Reserved by Gateflix.in No part of this material should be copied or reproduced without permission
4.2 CORRELATION BETWEEN TIME resonant Mr , the resonant frequency ωr , &
AND FREQUENCY RESPONSE the bandwidth are all uniquely related to
the damping ratio ξ & the natural
Consider a second order system with open undamped frequency ωn of the system.
loop transfer function The resonant peak is given by
ω2n 1
G (s) = Mr =
s(s + 2ξωn ) 2ξ 1 − ξ 2
ω2n The resonant frequency is
Then G ( jω) =
jω( jω + 2ξωn ) ωr ωn 1 − 2ξ 2
=
Where ξ is the damping factor ωn is the Bandwidth is
undamped natural frequency of B.W.= ωn 1 − 2ξ 2 + 2 − 4ξ 2 + 4ξ 4
oscillations.
Note:
Now the closed loop transfer function with
1
unity feedback will be • From the equation M r = , Mr
C ( jω ) ωn 2 2ξ 1 − ξ 2
= T ( jω=)
R ( jω ) ( jω) + 2ξωn ( jω) + ωn2
2 gets vanished when
0
1 − 2ξ 2 =
4.2.1 RESONANT PEAK AND RESONANT 2ξ 2 = 1
FREQUENCY & BANDWIDTH ξ 2 = 1/ 2
∴ ξ= 0.707
The frequency response magnitude & time
response of a 2nd order system are as i.e. at ξ 0.707,
= = Mr 1
shown in the figure.
• For a second order system, the resonant
peak Mr of its frequency response is
indicative of its damping factor ξ for
0 < ξ ≤ 1/ 2 .
• The resonant frequency ωr of the
frequency response is indicative of its
natural frequency for a given ξ and
hence indicative of its speed of
 4 
response  as t s =  .
 ( ξω )
 n 

• The frequency at which M has a value of


1/ 2 is of special significance and is
called the cut-off frequency ωc . The
signal frequencies above cut-off are
greatly attenuated in passing through a
system.
Example
In the frequency response a order 2nd For a unity feedback system
system shows a peak called resonant peak K
G (s ) = determine the values of
Mr and the corresponding frequency is s (1 + sτ )
called resonant frequency ωr . For a second-
order feedback control system, the peak

© Copyright Reserved by Gateflix.in No part of this material should be copied or reproduced without permission
K and τ, so that M r = 1.06 and 4.3 BODE PLOTS
ωn = 12rad / sec .
A Bode plot is a graph of the transfer
Solution function of a linear, time-invariant system
K
=G (s) = , H (s) 1 versus frequency, plotted with a log-
s (1 + sτ ) frequency axis, to show the
K system's frequency response. It is usually a
K
C (s) s (1 + sτ ) combination of a Bode magnitude
= = =
K τ plot, expressing the magnitude of the
R (s) 1 + K τs + s + K s 2 + s + K
2
1
frequency response gain, and a Bode phase
s (1 + sτ ) τ τ plot, expressing the frequency
Comparing denominator with response phase shift i.e. a Bode plot
s 2 + 2ξωn s + ω2n consists of two graphs:
K 1) A plot of the magnitude in dB of a
∴ ω2n = sinusoidal transfer functions against the
τ
frequency in logarithmic scale.
K 2) A plot of the phase angle against the
i.e. ωn = … (1)
τ frequency in logarithmic scale.
1
and 2ξωn = Note:
τ
1 The magnitude & the phase angle can be
i.e. ξ = … (2) calculated as shown in the example
2 Kτ
e.g.
1
=
Now, M r = 1.06 if T ( jω ) =
1
then the magnitude is
2ξ 1 − ξ 2 1 + jω
∴ ξ 1− ξ2 =
0.4716 T ( jω ) =
1
& the phase angle is
i.e. ξ 2 (1 − ξ 2 ) = 1 + ω2
2
0.2225
0
∴ ξ 4 − ξ 2 + 0.2225 = 0 ∠T ( jω ) = =− tan −1 ( ω )
 ω 
i.e. ξ 2 = 0.6658, 0.3341 tan −1  
1
∴ ξ= 0.8159, 0.578 but ξ cannot be more
that 0.707 4.3.1 ADVANTAGES OF USING
∴ ξ= 0.578 LOGARITHMIC SCALE
Usingequation (1)
• The main advantage of using the
K logarithmic plot is that multiplication of
ωn =
τ magnitude can be converted into
K addition.
i.e. 12 =
τ • The logarithmic representation is useful
i.e. K=144 τ in that it shows both the low-and high
Using equations (2), frequency characteristics of the transfer
1 function in one diagram.
0.578 = • Expanding the low frequency range by
2 144τ × τ
use of a logarithmic scale for the
1 frequency is very advantageous since
i.e. 0.578 =
2 ×12 × τ characteristics at low frequencies are

= τ 0.072,
= K 10.3806 most important in practical systems.

© Copyright Reserved by Gateflix.in No part of this material should be copied or reproduced without permission
Note: 4.3.2.2POLES OR ZEROS AT ORIGIN
It is not possible to plot the curves right G ( jω ) H ( jω ) = ( jω) ± p
down to zero frequency because of the
∴ G ( jω ) H ( jω ) =
ω± p
logarithmic frequency ( log 0 = −∞ ) ; this
does not create a serious problem. In dB 20 log10 log10 ω± =
p
20 × ± p log10 ω
and ∠G ( jω ) H ( jω ) =
( 90 )
±p

4.3.2 BASIC FACTORS OF G (jω) H (jω) Note:


For poles and zeros at origin, the
Consider the open loop transfer function of magnitude plot will be a straight line with
a system as slope of ± p × 20dB / decade passing
K 's z (s + z1 )(s + z 2 ) through ω =1.
G(s)H(s) = p
s (s + p1 )(s + p 2 ) 1
In the above equation either 𝑠𝑠 𝑧𝑧 or 𝑠𝑠 𝑝𝑝 will be e.g. If G ( jω ) H ( jω ) = then

there at a time. Converting the above 1
equation into time constant form we get G ( jω ) H ( jω ) = in dB
ω
Ks z (1 + Ta s )(1 + Tbs ) …
G(s)H(s) = p 1
20 log10 = −20 log10 ω
s (1 + T1s )(1 + T2s ) … ω
K ' z1 × z 2 … which is an equation of straight line
Where K =
p1 × p 2 … =
(y mx + c) with a slope of −20dB / decade .
T1 , T2 , Ta , Tb are the time constants of Also ∠G ( jω ) H ( jω ) = −90o
different poles & zeros. Therefore the phase plot will be a straight
Each term involved in the transfer function line
contributes some magnitude & phase to the
system. Putting 𝑠𝑠 = 𝑗𝑗𝑗𝑗 in the above
expression we get the factors in the
transfer function.
1) Gain K
2) Poles or zeros at origin ( jω) ± p
3) Simple poles & simple zeros (1 + jωT ) ±1 Note:
4) Quadratic poles and zeros The slope is sometime written as
 2

±1 dB/octave −20db / decade =
−6db / octave
 ω   jω 
1 + 2ξj  +  
  ωn   ωn   4.3.2.3SIMPLE POLES AND ZEROS
 

4.3.2.1THE GAIN K G ( jω ) H ( jω )= (1 + jωT) ±1

( )
±1
∴ G ( jω ) H ( jω=
) 1 + ( ωT )
2
G(s)H(s) = K
G ( jω ) H ( jω=
) K + j0
( 1 + (ωT ) )
±1
2
In dB 20 log10
∴ G ( jω ) H ( jω ) =
K and
∠G ( jω ) H ( jω ) =
0 = 20 × ±1log10 1 + ( ωT )
2

Note: and ∠G ( jω ) H ( jω ) =
 tan −1 ( ωT ) 
±1

As gain K is constant, its magnitude plot


20 log10 K will be constant.

© Copyright Reserved by Gateflix.in No part of this material should be copied or reproduced without permission
Note:
• For simple poles and zeros the
magnitude plot will be a straight line
with slope +20 dB/decade and
−20 dB/decade respectively.
• For low frequencies (ω<<1/T)
±20 log10 1 + ( ωT ) ≈ ±20 log10 1 =0dB
2

1
Therefore for ω < the magnitude plot
T
will be a straight line coinciding with 0
dB line.
• For high frequencies (ω>>1/T)
±20 log10 1 + ( ωT ) ≈ ±20 log10 ωTdB
2 4.3.2.4QUADRATIC POLES & ZEROS
±1
  ω   jω  
2

G ( jω) H ( jω) = 1 + 2ξj   +   


1
Therefore for ω > the magnitude plot
T   ωn   ωn  

will be a straight line with slope of ±1
±20 dB/decade.  2 2
  ω     ω  2 

• The magnitude plot of simple pole & ∴ G ( jω) H ( jω=)  1 −    +  2ξ    


zero has some approximation & the    ωn     ωn   
 
error due to this approximation is for ±1
  2 2
 2 
zero ± n × 3dB ( + ve & − ve for pole).   ω   ω  
Where n is the order of simple poles or = In dB 20 log10  1 −    +  2ξ    
zeros.    ωn     ωn   
 
1
e.g. If G ( jω ) H ( jω ) = then 2
  ω 2    ω  2
1 + j ωT = ±20 log10 1 −    +  2ξ   
1   ωn     ωn  
G ( jω ) H ( jω ) =  
(1 + ( ωT )
2

1 and ∠G ( jω) H ( jω)


In dB 20 log10 log10 ±1
(1 + ( ωT )   ω 
2

 2ξ   
=−20log10 (1 + (ωT)) dB 2

= tan 1   ωn  
2 
which is a straight line with slope -  1−  ω  
1  ω  
20dB/decade & will start from ωC =   n 
T Note:
• The magnitude plot for quadratic poles
& zeros is a straight line with slope
±40 dB/decade .
• The corner frequency for Quadratic
poles & zeros is ωc =ωn .

© Copyright Reserved by Gateflix.in No part of this material should be copied or reproduced without permission
4.3.3 PROCEDURE FOR PLOTTING  dB   dB 
BODE PLOT  −20  +  −20 
 decade   decade 
= −40dB / decade
1) Write the transfer function in time
• After drawing the slopes for simple pole
constant form & put s = jω.
& pole at origin we will have to shift the
2) Then identify the corner frequencies
graph up by 26𝑑𝑑𝑑𝑑.
associated with the basic factors.
3) Draw the asymptotic log-magnitude
1) The total phase angle of the transfer
curves with proper slopes between the
function is as shown in the table
corner frequencies.
4) Shift the curve up or down by 20 log10 K .
( upforK > 1 & down for K < 1)
5) The phase angle curve of G (jω) H (jω)
can be drawn by adding the phase angle
curves of individual factors.

Example
Sketch the Bode plot for Plotting the magnitude & phase plot
20 against frequency we get
G(s)H(s) =
s (1 + 0.1s )

Solution
The transfer function is already in the
time constant form. Identify the factors
from the transfer function
1) Gain K = 20
Its mgnitude = 20 log10 20 = +26dB .
Therefore the magnitude plot will be
shifted up by 26 𝑑𝑑𝑑𝑑.
1
2) 1 pole at origin Its magnitude plot is a
s
straight line with slope −20dB / decade
passing through point ω =1
1 4.3.4 GAIN CROSSOVER FREQUENCY
3) 1 simple pole T=0.1 & the corner
1 + 0.1s
1 1 It is the frequency at which the gain of the
frequency ωc = = = 10rad / sec system is unity or 0 dB i.e. at gain crossover
T 0.1
.The magnitude plot is a straight line frequency
with slope −20dB / decade starts from | G ( jω) H ( jω) ω=ω =
1
gc

ωc =10rad / sec . In 20 log10 1 = 0dB

Note: 4.3.5 PHASE CROSSOVER FREQUENCY


• As plot of pole at origin already has
slope of −20 dB/decade, the slope after It is the frequency at which the phase of the
simple pole will be system is −180𝑜𝑜 i.e. at phase crossover
frequency

© Copyright Reserved by Gateflix.in No part of this material should be copied or reproduced without permission
∠G ( jω) H ( jω) ω=ω =
−180o Example:
pc
For a unity feedback system
242(s + 5)
4.3.6 GAIN MARGIN G(s) = sketch Bode
s ( s + 1) (s 2 + 5s + 121)
As we seen earlier in root locus that, if gain plot & determine ωgc , ωpc Gain margin,
K of the system is increased, after some phase margin.
value of gain the system becomes unstable. Solution:
Gain margin of the system is defined as the First convert the transfer function into
allowable gain so that the system reaches time constant form
on the verge of instability. Also the gain s 
margin of system is defined as the 242 × 5  + 1
G (s) = 5 
reciprocal of the gain of system at phase
 s2 5s 
crossover frequency. 121× s ( s + 1)  + + 1
1  121 121 
i.e. G.M. =
| G ( jω) H ( jω) ω=ω  s
10 × 1 + 
pc

In dB =  5
 5s s2 
G.M. = 20 log10
1 s (1 + s ) 1 + + 
| G ( jω) H ( jω) ω=ω  121 121 
The factors in the transfer function in the
pc

−20 log10 | G ( jω) H ( jω) ω=ω


G.M. = increasin order of their corner frequencies
pc

Note: For stable systems the gain margin is are


+ve. 1) Gain K = 10
Its magnitude will be 20 log10 10 = +20dB
4.3.7 PHASE MARGIN Therefore the magnitude plot will be
shifted up by 20 dB.
Phase margin of a system is an additional 1
2) 1 Pole at origin
phase lag that can be introduced in the s
system at gain crossover frequency till it It magnitude plot will be a straight line
reaches on the verge of instability. with slope −20 dB/decade passing
= 180o + ∠G ( jω ) H ( jω ) ω=ω
P.M. through 𝛚𝛚 = 𝟏𝟏. The phase angle
contributed by this pole will be −90o .
gc

Note:
3) Simple pole (1 + s )
For stable systems the phase margin
should be positive. Its magnitude plot will be a straight line
with slope −40 dB/decade starting
4.3.8 CRITERION FOR STABILITY from the corner frequenc
1 1
ωc = = = 1rad / sec
Sr Stability Frequen G.M. P.M. T 1
. cy Conditi Conditio Note:
N Conditio on n
o. n
In −40 dB/decade slope the slope of
1 Stable 𝜔𝜔𝑝𝑝𝑝𝑝 Positive Positive  1
previous pole  i.e.  −20 dB/decade is
> 𝜔𝜔𝑔𝑔𝑔𝑔  s
2 Unstable 𝜔𝜔𝑝𝑝𝑝𝑝 Negative Negative also added.
< 𝜔𝜔𝑔𝑔𝑔𝑔
The phase angle contributed by this pole
3 Marginally 𝜔𝜔𝑝𝑝𝑝𝑝 Zero Zero
 ω
Stable = 𝜔𝜔𝑔𝑔𝑔𝑔 will be − tan −1   .
1

© Copyright Reserved by Gateflix.in No part of this material should be copied or reproduced without permission
 s
1) Simple zero 1 + 
 5
Its magnitude plot will be a straight line
with slope −20 dB/decade starting
from the corner frequenc
1 1
ωc = = = 5rad / sec .
T 1
5
Note:
For a simple zero the slope is always
+20 dB/decade but the slope of previous 2
poles are added hence the resultant slope is
dB  dB 
+20 +  −40  =−20dB / decade
decade  decade 
The phase angle contribute by this zero will
 ω
be − tan −1   .
5
 5s s2 
2) Quadratic poles 1 + + 
 121 121 
The magnitude plot will be a straight
line with slope −60 dB/decade starting
from the corner frequency
=
ωc = 11rad / sec .
121
Note:
The slope for quadratic poles is
−40 dB/decade but the slope of previous 3
factors will also be added hence the
resultant slope is Example:
dB  dB  Find the open-loop transfer function of a
−40 +  −20  =−60dB / decade system whose approximate plot is shown
decade  decade 
in Fig.
The phase angle contributed by these poles
 5ω  log
  db 2.5 10 25
will be − tan −1  1212  .
 1 − ω 
 121 

-12
-20 db/dec 20 db/dec

Solution:
1) The starting slope of the graph is
−20 dB/decade hence there is a pole of
order1 at origin.
2) First corner frequency is at 2.5 rad/
sec& at this frequency there is a change

© Copyright Reserved by Gateflix.in No part of this material should be copied or reproduced without permission
of +20 dB/decade in slope hence there 4) Thus the transfer function for the given
is a zero at 2.5 rad/sec. Bode plot is
3) Next corner frequency is at 10 rad/sec& K
G ( jω ) H ( jω ) =
at this frequency there is change  s  s 
+20 dB/decade in slope hence there is s 1 +  1 + 
 2.5   40 
a zero at 10 rad/sec.
5) To find gain K write the equation for the
4) Next corner frequency is at 25 rad/sec&
straight line at the starting of the graph
at this frequency there is change
y = −20 log ω + c
−20 dB/decade in slope hence there is
a pole at 25 rad/sec. At ω = 2.5, y = +40dB
5) Thus the transfer function is ∴ +40 = −20 log 2.5 + c
 s  s  c = +47.95dB
K 1 +  1 +  Now c is the shift in the graph
G ( jω ) H ( jω ) = 
2.5   10 
 s  c = 20 log10 K
s 1 +  ∴ K = 250
 25 
6) To find gain K write the equation for the
straight line at the starting of the graph
y= −20logω + c
At ω = 2.5, y = −12dB
∴ −12 = −20 log 2.5 + c
c = −4.04dB
Now c is the shift in the graph
c = 20 log10 K
∴ K = 0.63

Example
Determine the transfer function whose
approximate plot is shown in Fig.
-20 db/dec
db
-40 db/dec
40

40
2.5 10 log
-60 db/dec
=1
Solution:

1) The starting slope is −20 dB/decade


hence there is a pole of order 1 at
origin.
2) First corner frequency is at 2.5 rad/sec
& at this frequency there is a change of
−20 dB/decade in slope hence there is
a simple pole at2.5 rad/sec.
3) Next corner frequency is at 40rad/sec &
there is a slope change of −20 dB/
decade at this frequency hence there is
another simple pole at 40rad/sec.

© Copyright Reserved by Gateflix.in No part of this material should be copied or reproduced without permission
5 POLAR & NYQUIST PLOTS

5.1 POLAR PLOTS

In polar plots the magnitude of the


frequency response is plotted against the
phase angle for variations in frequency ω
i.e. G ( s ) H ( s ) is plotted against
∠G ( s ) H ( s ) .
To sketch the polar plot of G (jω) for the
entire range of frequency ω, i.e., from 0 to
infinity, there are four key points that
usually need to be known:
1) The start of plot where ω = 0 Plotting gain & phase on the graph we get
2) The end of plot where ω = ∞
3) The point where polar plot cuts the –ve
real axis gives the magnitude at phase
crossover frequency
I.e. | G ( jω ) H ( jω ) ω=ω .
pc

4) The angle with respect to +ve real axis


where the polar plot cuts the unit circle
gives phase angle at gain crossover
frequency i.e. ∠G ( jω ) H ( jω ) ω=ω .
gc

Example:
5
Draw the polar plot for G ( s ) H ( s ) = .
s 5.1.1 POLAR PLOTS OF SOME
Solution: STANDARD FUNCTIONS
For the given transfer function, the
magnitude & the phase angle are: 1) G(s)H(s) =
1

G ( jω ) H ( jω ) =
5 (1 + Ts )

5
∴ G ( jω ) H ( jω ) =
ω
0o
and ∠G ( jω ) H ( jω ) =
ω
tan −1   1
0 2) G(s)H(s) =
s (1 + T1s )(1 + T2s )
ω
= − tan −1   = − tan −1 ( ∞ ) = −90o
0
Now find the magnitude & phase at
different values of ω

© Copyright Reserved by Gateflix.in No part of this material should be copied or reproduced without permission
1
3) G(s)H(s) =
s (1 + T1s )(1 + T2s )(1 + T3s )
3

e.g. Order − 1 system will always have its


polar plot ending at origin in 4th
quadrant.

4)
1 5.1.2 POLAR PLOT FOR 𝐒𝐒𝟏𝟏
G(s)H(s) =
s (1 + T1s )(1 + T2s )(1 + T3s )(1 + T4s )
2

The curve S1 is the whole +ve imaginary


axis from ω = 0 to ω = ∞ . The polar plot of
S1 will be the polar plot of given transfer
function.
K
e.g. If G(s)H(s) = the
s ( s + 2 )( s + 10 )
Note: polar plot for S1 will be the polar plot of
1) The start of the polar plots (at ω = 0 )
given transfer function. As given transfer
depends on the type number of the
function is type − 1 and order − 3 , its polar
system.
plot will be

e.g. Type − 0 system will always have its 5.1.3 POLAR PLOT FOR S 2
polar plot starting from +ve real axis.
The curve S2 covers the whole RHS s-plane
2) The end of the polar plot (at ω = ∞ )
depends on the order of the system. from +90o to − 90o & its polar will be plotted
by putting s = lim Re jθ where θ varies from
R →∞

+90 to − 90 .
o o

K
e.g.If G(s)H(s) = put
s ( s + 2 )( s + 10 )
s = lim Re jθ
R →∞

© Copyright Reserved by Gateflix.in No part of this material should be copied or reproduced without permission
As R → ∞ , s>>2 & s>>10 therefore the by putting s = lim Re jθ where θ varies from
R →0
transfer function can be approximated as
K −90 to + 90 .
o o

G(s)H(s) = 3 ; Putting s = lim Re jθ K


s R →∞ e.g. If G(s)H(s) =
K s ( s + 2 )( s + 10 )
we get G ( s ) H ( s ) =
( ) put s = lim Re jθ
3
lim Re jθ R →0
R →∞
As R → 0, s<<2 & s<<10 therefore the
K transfer function can be approximated as
= = 3 j3θ
0e − j3θ
Re K
As θ varies from +90o to − 90o , G ( s ) H ( s ) G(s)H(s) = putting s = lim Re jθ
20s R →0

can be written as we get


G ( s ) H ( s ) = 0 from −270o to + 270o . G (s) H (s) =
K
= ∞e − jθ
(
20 × lim Re jθ
R →0
)
as θ varies from −90o to + 90o , G ( s ) H ( s )
can be written as
G ( s ) H ( s ) = ∞ from +90o to − 90o .

5.1.4 POLAR PLOT FOR S3

The curve S3 is the whole –ve imaginary


axis from ω = −∞ to ω = 0 . The polar plot
for S3 will be the inverse polar plot of given
transfer function.
K 5.2 PROCEDURE TO FIND G.M. & P.M.
e.g. If G(s)H(s) = the
s ( s + 2 )( s + 10 )
polar plot for S3 will be the inverse polar
plot of given transfer function. As given
transfer function is type − 1 and order − 3 ,
its inverse polar plot will be

• The frequency at which polar plot


intersects unity circle is the gain
crossover frequency ωgc .
5.1.5 POLAR PLOT FOR S 4
• The frequency at which polar plot cuts
–ve real axis is the phase crossover
The curve S4 is around origin from
frequency ωpc .
−90o to + 90o & its polar plot will be plotted

© Copyright Reserved by Gateflix.in No part of this material should be copied or reproduced without permission
• The gain margin is reciprocal of the Note:
distance from origin to the point at For a marginally stable system, the gain
which polar plot cuts the –ve real axis. margin & phase margin both are 0.
• Phase margin is the angle made by the
line joining origin & the intersection Example
point of polar plot with unit circle with Draw the polar plot for the transfer
the positive real axis +180o . 1
function G(s)H(s) =
s (1 + Ts )
5.2.1 STABLE SYSTEM Solution
For the given transfer function, the
magnitude & phase angle are
1
G( jω)H( jω) =
jω (1 + jωT )
∴ G ( jω ) H ( jω )
1
=
ω 1 + ( ωT )
2
• In this figure the intersection of polar
plot with –ve real axis is within unit and ∠G ( jω ) H ( jω )
circle i.e. x < 0
0o
1 = o −90o − tan −1 ( ωT )
=
G.M. = 20 log10 is positive +90 + tan ( ωT )
−1
x
• The angle ∅ lies between 0o &−180o Find the gain & phase angle at
ω= 0&ω= ∞
= 180o + ∅ is positive
P.M.
• As gain margin & phase margin both are
𝛚𝛚 |𝐆𝐆(𝐣𝐣𝐣𝐣)𝐇𝐇(𝐣𝐣𝐣𝐣)| ∠𝐆𝐆(𝐣𝐣𝐣𝐣)𝐇𝐇(𝐣𝐣𝐣𝐣)
positive therefore the system is stable.
0 ∞ −90o
∞ 0 −180o
5.2.2 UNSTABLE SYSTEM

Example
Draw the polar plot for
• In this figure the intersection of polar 1
plot with –ve real axis is within unit G(s)H(s) = .
circle i.e. x>0
(1 + T1s )(1 + T2s )
1 Solution
G.M. = 20 log10 is negative For the given transfer function, the
x
magnitude & phase angle are
• The angle ∅ > −180o 1
= 180o + ∅ is negative G( jω)H( jω) =
P.M. (1 + jωT1 )(1 + jωT2 )
• As gain margin & phase margin both are
negative therefore the system is ∴ G ( jω ) H ( jω )
unstable.

© Copyright Reserved by Gateflix.in No part of this material should be copied or reproduced without permission
1 G(s)
= and Closed loop T.F. ⇒
1 + ( ωT1 )
2
1 + ( ωT2 )
2 1 + G(s)H(s)

0o For this system if


∠G ( jω ) H ( jω ) =
+ tan −1 ( ωT1 ) + tan −1 ( ωT2 )
− tan −1 ( ωT1 ) − tan −1 ( ωT2 )
= 1) P is the number of open loop poles on
the right hand side of s plane.
Find the gain & phase angle at
1
ω = 0 &ω = ∞ e.g. if G ( s ) H ( s ) =
(1 + s )(1 − s )
ω G ( jω ) H ( jω ) ∠G ( jω ) H ( jω ) The poles are at s = −1 & s = 1 hence
0 1 0o P = 1 . ( s = 1 is the open loop pole on
∞ 0 −180o RHS of s-plane)
2) Z is the number of closed loop poles on
the right hand side of s plane.
e.g. if the closed loop transfer function is
G(s) 1
= 2
1 + G(s)H(s) s + s − 2
The poles are at s = −2 & s = 1 hence Z =
1. ( s = 1 is the closed loop pole on RHS
of s-plane)
3) N is the number of encirclement of
−1 + j0 point by the Nyquist plot in
5.3 NYQUIST PLOT anticlock wise direction.
Then the relation between N, P & Z is
Polar is the basis of Nyquist plot. For a N= P − Z . For a system to be stable the
closed loop system to be stable, the Nyquist number of encirclement should be
plot of G(s)H(s) must encircle the (−1, j0) equal to the number of open loop poles
point as many times as the number of poles on RHS of s-plane.
of G(s)H(s) that are in the right half of s- i.e. N = P
plane, and the encirclement, if any, must be ∴ Z = 0 (number of closed loop poles on
made in the counter clockwise direction. RHS of s-plane is 0)
Nyquist stability criterion is based on the
complex analysis result known as Cauchy’s 5.3.2 PROCEDURE TO DRAW NYQUIST
principle of argument. PLOT
5.3.1 NYQUIST STABILITY CRITERION The Nyquist plot is a polar plot of the
function D(s) = 1 + G(s)H(s) when travels
Consider a system with block diagram
around the contour given in figure

The open loop & closed loop transfer


function of the system are
Open loop T.F.⇒ G(s)H(s)

© Copyright Reserved by Gateflix.in No part of this material should be copied or reproduced without permission
• If the poles of the system lie on the RHS K K
=- =-
of s-plane, the system will be unstable. 12×20 240
• The poles on the imaginary axis & at
origin make the system marginally 1) If K>240 then the intersection point will
stable but not unstable they are not greater than −1 + j 0 i.e. −1 + j 0 lies
covered under the curve. inside the Nyquist plot, hence there are
The contour in this figure covers the 2 encirclements of −1 + j 0 in the
whole unstable half plane of the
clockwise direction
complex plane s. The Nyquist plot for
∴N = −2 and also from the transfer
the above contour can be drawn by
function
drawing the separate polar plot of the
P = 0 (i.e. No of open loop poles on RHS
curves S1 ,S2 ,S3 & S4 .
of s-plane)
As we know
5.3.3 TO DETERMINE THE STABILITY N= P − Z
OF THE SYSTEM i.e. −2 = 0 − Z
Solving we get, Z = 2 it means that
Consider a transfer function there are 2 closed loop poles on RHS of
G(s)H(s) =
K
, open loop poles s-plane hence the system is unstable
s ( s + 2 )( s + 10 ) for K>240.
on the RHS of s-plane i.e. P = 0. The polar 2) If K<240, then the intersection point
plot for the above transfer function will be will be smaller than −1 + j0 i.e. −1 + j0
lies outside the Nyquist plot, hence
there is no encirclement of −1 + j0 .
∴N = 0 and also from the transfer
function
P = 0 (i.e. No of open loop poles on RHS
of s-plane)
As we know
N= P − Z
i.e. 0= 0 − Z
Now to calculate intersection of polar plot Solving we get, Z = 0 it means that there
with –ve real axis, no closed loop poles on RHS of s-plane
K K hence the system is stable for K>240.
 G ( s ) H ( s ) = 3
=
s ( s + 2 )( s + 10 ) s + 12s 2 + 20s
Example
K
G ( jω ) H ( jω ) = For a system with transfer function
− jω − 12ω2 + j20ω
3
40
(s)H(s) = , determine its
=
K ( s + 4 ) ( s 2 + 2s + 2 )
j ( 20ω − ω3 ) − 12ω2 stability & find gain margin.
Equating imaginary part to zero we get, Solution
20ω − ω3 = 0 ⇒ ω2 = 20 ⇒ ω = 20 G (s) H (s) =
40
This frequency is nothing but the phase ( s + 4 ) ( s 2 + 2s + 2 )
crossover frequency 𝜔𝜔𝑝𝑝𝑝𝑝 40
=
Now substituting ω = ωpc = 20 we get, ( s + 4 ) ( s + (1 − i ) ) ( s + (1 + i ) )
K
G ( jω ) H ( jω ) =
j ( 0 ) − 12 20
2

© Copyright Reserved by Gateflix.in No part of this material should be copied or reproduced without permission
There are no open loop poles at origin or at on RHS of s-plane is 0 i.e. P=0. We know
imaginary axis; therefore we will draw that N=P-Z
polar plots only for S1 ,S2 , & S3 . i.e. 0=0-Z∴ Z = 0
Which means there are no closed loop
poles on RHS of s-plane. Hence the
system is stable.
6) The gain margin can calculated as
1
G.M. = 20 log10 = +2.28dB
0.769

Example:
Consider a system with transfer function
K
G (s) H (s) = & Nyquist plot
1) For S1 draw the polar plot for given ( T1s + 1)( T2s + 1)
transfer function.
2) For S2 draw the inverse polar plot for
given transfer function.
3) For S3 put s = lim Re jθ where θ varies
R →∞

from +90 to − 90o


o

Examine the stability of the system


Solution:
1) From the transfer function, the number
of open loop poles on the RHS of s-plane
is 0 i.e.P = 0.
2) The point -1+j0 lies outside the plot
hence N=0.
We know that,
N= P − Z
4) To find the intersection of plot with –ve i.e. 0 = 0 − Z ⇒ Z = 0 (no closed loop
real axis poles on RHS of s-plane). As Z=0 the
G (s) H (s) =
=
40 40 system is stable
( s + 4 ) ( s + 2s + 2 ) s + 6s + +10s + 8
2 3 2

put s = jω 5.4 THE NICHOLS PLOT


40 40
= The Nichols plot is similar to the Nyquist
− jω3 − 6ω2 + j10ω + 8 j (10ω − ω3 ) + ( 8 − 6ω2 )
plot in that it is a locus as a function ofω,
now equate imaginary part to zero the difference being the chosen axes. On a
10ω − ω3 =0 ⇒ ωpc = 10 Nichols plot these are the magnitude in dB
Now, the intersection with –ve real axis on the vertical axis & the phase in degrees
40 on the horizontal axis. The origin is chosen
is G ( s ) H ( s ) ω=ω = as 0 dB and −180o . For example, the
( )
2
j0 + 8 − 6 × 10
pc

Nichols plot for


= −0.769 G(s)H(s) =
10
is
5) The point -1+j0 lies outside the Nyquist s (1 + 0.1s )(1 + 0.05s )
plot, hence N=0. Also from the transfer
function the number of open loop poles

© Copyright Reserved by Gateflix.in No part of this material should be copied or reproduced without permission
The gain margin is the gain at the
frequency at which phase angle is −180o &
the phase margin is the phase angle at the
frequency at which gain is 0 dB.

© Copyright Reserved by Gateflix.in No part of this material should be copied or reproduced without permission
GATE QUESTIONS(EC)

Q.1 The Nyquist plot for the open-loop


transfer function G(s) of a unity
negative feedback system is shown
in the figure, if G(s) has no pole in
the right-half of s-plane, the number
of roots of the system characteristic
equation in the right-half of s-plane is

a) always stable
b) unstable with one closed-loop
right hand pole
c) unstable with two closed-loop
a) 0 b) 1 right hand poles
c) 2 d) 3 d) unstable with three closed-loop
[GATE-2001] right hand poles
[GATE-2003]
Q.2 The system with the open loop Q.5 The approximate Bode magnitude
transfer function plot of a minimum-phase system is
1 shown in the figure. The transfer
G (s) H (s) = 2 has a gain
s(s + s + 1) function of the system is
margin of
a) −6dB b) 0dB
c) 3.5dB d) 6dB
[GATE-2002]

Q.3 The gain margin and the phase


margin of a feedback system with
s
G (s) H (s) = are
( s + 0.1)
3
( s + 100 )
3
a) 10 8

( s + 10 ) (s + 100)
2
a) 0 dB, 0° b) ∞, ∞
c) ∞, 0° d) 88.5 dB, ∞ ( s + 0.1)
3

[GATE-2003] b) 10 7

(s + 10)(s + 100)
( s + 0.1)
2
Q.4 In the figure, the Nyquist plot of the c) 10 8
open-loop transfer function G(s) ( s + 10 ) ( s + 100 )
2

H(s) of a system is shown. If


( s + 0.1)
3

d) 10 9
G(s)H(s) has one right-hand pole. ( s + 10 )( s + 100 )
2

The closed –loop system is


[GATE-2003]

© Copyright Reserved by Gateflix.in No part of this material should be copied or reproduced without permission
Q.6 Consider the Bode magnitude plot c)
shown in the figure. The transfer
function H(s) is

d)

(s + 10) 10(s + 1)
a) b)
( s + 1) (s + 100) ( s + 10 ) (s + 100)
102 ( s + 1) 103 ( s + 100 )
c) d)
( s + 10 )( s + 100 ) ( s + 1)( s + 10 ) [GATE-2005]
[GATE-2004] Q.9 The polar diagram of a conditionally
stable system for open loop gain
Q.7 A system has poles at 0.01 Hz, 1 Hz K=1 is shown in the figure. The open
and 80Hz; zeros at 5 Hz, 100 Hz and loop transfer function of the system
200 Hz. The approximate phase of is known to be stable. The closed
the system response at 20 Hz is loop system is stable for
a) −90° b) 0°
c) 90° d) −180°
[GATE-2004]

Q.8 Which one of the following polar


diagrams corresponds to a lag
network?
a)
1 1
a) K < 5 and <K<
2 8
1 1
b) K < and < K < 5
8 2
1
c) K < and 5 < K
8
1
d) K < and K < 5
b) 8
[GATE-2005]

Common Data for Questions Q.10 &


Q.11:
The open loop transfer function of a unity
3e −2s
feedback system is given by G ( s ) =
s(s + 2)

© Copyright Reserved by Gateflix.in No part of this material should be copied or reproduced without permission
Q.10 The gain and phase crossover 1 1
a) b)
frequencies in rad/sec are, ( s + 1) ( s + 20) s ( s + 1) ( s + 20)
respectively
100 100
a) 0.632 and 1.26 c) d)
b) 0.632 and 0.485 s ( s + 1) ( s + 20) s ( s + 1) (1 + 0.05s )
c) 0.485 and 0.632 [GATE-2007]
d) 1.26 and 0.632
[GATE-2005] Q.15 The magnitude of frequency
response of an under damped
Q.11 Based on the above results, the gain second order system is 5 at
and phase margins of the system 10
0 rad/sec and peaks to at
will be 3
a)-7.09 and87.5° b)7.09 and87.5
5 2 rad/sec . The transfer function of
c)7.09 and-87.5° d)-7.09 and-87.5°
[GATE-2005] the system is
500 375
a) 2 b) 2
Q.12 The open-loop transfer function of a s +10s +100 s + 5s +75
unity-gain feedback control system c) 2
720
d) 2
1125
K
is given by G ( s ) = .The s +12s +144 s + 25s + 225
( s + 1) (s + 2) [GATE-2008]
gain margin of the system in dB is Common data for Questions Q.16 & Q.17:
given by The Nyquist plot of a stable transfer
a) 0 b) 1 function G(s) is shown in the figure. We are
c) 20 d) ∞ interested in the stability of the closed loop
[GATE -2006] system in the feedback configuration
shown
Q.13 The Nyquist plot of G ( jω) H( jω) for
a closed loop control system, passed
through (−1, j 0) point in the GH
plane. The gain margin of the system
in dB is equal to
a)infinite b)greater that zero
c)less than zero d)zero
[GATE-2006] Q.16 Which of the following statement is
true?
Q.14 The asymptotic Bode plot of a a) G(s) is an all-pass filter
transfer function is as shown in the b) G(s) has a zero in the right –half
figure. The transfer function G(s) plane
corresponding to this Bode plot is c) G(s) is the impedance of a
passive network
d) G(s) is marginally stable
[GATE -2009]

Q.17 The gain and phase margins of G(s)


for closed loop stability are
a) 6dB and 180° b) 3dB and 180°
c) 6dB and 90° d) 3dB and 90°
[GATE-2009]

© Copyright Reserved by Gateflix.in No part of this material should be copied or reproduced without permission
Q.18 For the asymptotic Bode magnitude
plot shown below, the system
transfer function can be

[GATE -2011]

Q.20 The gain margin of the system under


10s+1 100s+1 closed loop unity negative feedback
a) b) 100
0.1s+1 0.1s+1 is G ( s ) H ( s ) =
S ( S+10 )
2
100s 0.1s+1
c) d)
10s+1 10s+1 a)0 dB b)20 dB
[GATE-2010] c)26 dB d)46 dB
[GATE -2011]
Q.19 For the transfer function,
G ( jω) = 5 + jω , the corresponding Q.21 The Bode plot of transfer function
G(s) is shown in the figure below.
Nyquist plot for positive frequency
has the form
a)

The gain (20log |G(s))| is dB and -8


b) at 1 rad/s and 10rad/s respectively.
The phase is negative for all ω. Then
G(s) is
39.8 39.8
a) b) 2
s s
32 32
c) d) 2
s s
c) [GATE-2013]
Q.22 Consider the feedback system
shown in the figure. The Nyquist
plot of G(s) is also shown. Which
one of the following conclusions
is correct?
d)

a) G(s) is an all-pass filter


b) G(s) is a strictly proper transfer
function

© Copyright Reserved by Gateflix.in No part of this material should be copied or reproduced without permission
c) G(s) is a stable and minimum- a) encircles the s-plane point (-
phase transfer function 1+j0) in the counterclockwise
d) The closed-loop system is direction as many times as the
unstable for sufficiently large number of right-half s-plane poles.
and positive k b) encircles the s-plane point (0 - jl)
[GATE-2014] in the clockwise direction as
many times as the number of
Q.23 The phase margin in degrees of right-half s-plane poles.
10
G (s) = c) encircles the s-plane point (-1+
( s + 0.1)( s + 1) + ( s + 10 ) j0) in the counterclockwise
calculated using the asymptotic direction as many times as the
Bode plot is ________. number of left-half s-plane poles.
[GATE-2014] d) encircles the s-plane point (1+j0)
in the counterclockwise direction
Q.24 The Bode asymptotic magnitude as many times as the number of
plot of a minimum phase system is right-half s-plane zeros
shown in the figure [GATE-2016]

Q.28 The number and direction of


encirclements around the point –1 +
j0 in the complex plane by the
1− s
If the system is connected in a unity Nyquist plot of G ( s ) = is
4 + 2s
negative feedback configuration, the a) zero
steady state error of the closed loop b) one, anti-clockwise
system, to a unit ramp input, is ______ c) one, clockwise
[GATE-2014] d) two, clockwise.
Q.25 In a Bode magnitude plot, which one [GATE-2016]
of the following slopes would be
exhibited at high frequencies by a Q.29 In the feedback system shown
4th order all-pole system? below G ( s ) =
1
a) -80 dB/decade b) -40 dB/decade (s + 1)(s + 2)(s + 3)
c) +40 dB/decade d) +80 dB/decade
[GATE-2014]

Q.26 The polar plot of the transfer The positive value of k for which the
10 ( s + 1) gain margin of the loop is exactly 0
function G (s) = for dB and the phase margin of the loop
s + 10
is exactly zero degree is __________.
0 ≤ ω < ∞ will be in the
[GATE-2016]
a) first quadrant b) second quadrant
c) third quadrant d) fourth quadrant Q.30 The asymptotic Bode phase plot of
[GATE-2015] 1
G (s ) = , with k
(s + 0.1)(s + 10)(s + p1 )
Q.27 A closed-loop control system is
& p1 both positive, is shown below.
stable if the Nyquist plot of the
corresponding open-loop transfer
function

© Copyright Reserved by Gateflix.in No part of this material should be copied or reproduced without permission
for k < k 0 . The maximum value of k 0
is ________
[GATE-2018]

Q.33 For a unity feedback control system


with the forward path transfer
function
K
The value of p1 is _______. G (s) =
s (s + 2)
[GATE-2016]
The peak resonant magnitude M r of
the closed-loop frequency response
Q.31 The Nyquist stability criterion and the
is 2. The corresponding value of
Routh criterion both are powerful
the gain K (correct to two decimal
analysis tools for determining the
stability of feedback controllers. places) is _______.
Identify which of the following [GATE-2018]
statements is FALSE

a) Both the criteria provide


information relative to the stable gain
range of the system.
b) The general shape of the Nyquist
plot is readily obtained from the Bode
magnitude plot for all minimum phase
systems.
c) The Routh criterion is not
applicable in the condition of transport
lag, which can be readily handled by
the Nyquist criterion.
d) The closed-loop frequency
response for a unity feedback system
cannot be obtained from the Nyquist
plot.
[GATE-2018]

Q.32 The figure below shows the Bode


magnitude and phase plots of a stable
transfer function
n0
G (s) = 3
s + d 2s + d1s + d 0
2

Consider the negative unity feedback


configuration with gain k in the feed
forward path. The closed loop is stable

© Copyright Reserved by Gateflix.in No part of this material should be copied or reproduced without permission
ANSWER KEY:
1 2 3 4 5 6 7 8 9 10 11 12 13 14 15
(a) (b) (b) (a) (a) (c) (a) (d) (b) (d) (d) (d) (d) (d) (a)
16 17 18 19 20 21 22 23 24 25 26 27 28 29 30
(b) (c) (a) (a) (c) (b) (d) 45 0.5 (a) (a) (a) (a) 60 1
31 32 33
(d) 0.1 14.92

© Copyright Reserved by Gateflix.in No part of this material should be copied or reproduced without permission
EXPLANATIONS

Q.1 (a) Q.5 (a)


N = 0 (1encircle mention CW = ω 0.1to10, +120dB change
direction ∴ 3 zeros at 0.1
and other in CCW) = ω 10to100, −40dB
P=0 (no pole in right half)
So, N= P − Z
[ i.e. + 60to + 20] change
Z = P − N =0 So two poles at ω=10
∴ No roots on RH of s-plane. ω 100, −20dB change
=
∴ one pole at ω = 100
Q.2 (b) K ( s + 0.1)
3

∴ T (s) = 2
G (s) H (s) =
1
( s + 10 ) (s + 100)
s(s + s + 1)
2
20 log log ( K / ω ) |ω=10 = 140
ωφ when ∠of G ( s ) H ( s ) =
−180°
K
ω |ω=10 = 107
-180 = -90 - tan -1 -9 ω
1-ω2 ∴K = 108
ω
= -tan -1
1-ω2 Q.6 (c)
1- ω2 = 0 20 log K = −20dB
ωφ = 1 rad/sec ⇒ K= 10−1= 0.1
Value of gain at ωφ = 1 Zero at ω = 1 & poles at ω = 10,100
K(s + 1)
G (s) H (s) =
1 H (s) =
 s  s 
(1-ω )  + 1  + 1
2
+ jω
2 2
 10   100 
∴ G.M. =
−20 log1 =
0 0.1(s + 1) × 10 × 100
=
( s + 10 ) (s + 100)
Q.3 (b) 100(s + 1)
s =
G (s) H (s) = ( s + 10 ) (s + 100)
( s + 100 )
3

G.M. and P.M. of the system cannot Q.7 (a)


be determined. Pole at 0.01 and 1 Hz gives −180°
phase. Zero at 5 Hz gives +90°
Q.4 (a) phase
The encirclement of critical point (- ∴ at 20Hz −90° phase shift is
1, 0) in A.C.W direction is once. provided.
∴ N = 1, P = 1 (given)
Z = P − N =0 Q.8 (d)
No zero in RH of s-plane. So system 1
is stable. Let be a lag network
s +1
At ω = 0, Mag = ∞, ∠ = -tan -1 0 = 0

© Copyright Reserved by Gateflix.in No part of this material should be copied or reproduced without permission
ω= ∞, Mag = 0, ∠ = − tan −1 ∞ = −90° 1
⇒ 20 log =−7.09
If in the direction of ω increasing 2.26
phase shift is decreasing system is Since G.M is negative system is
lag network. unstable.
∴ P.M.is also negative
Q.9 (b) P.M. = −87.5°
Q.12 (d)
For 2nd order system GM=∞

System is stable in region -0.2 to -2


& on the left side of -8 as no. of Q.13 (d)
encirclement there is zero. 1
0.2K < 1 ⇒ K < 5 G.M = 20 log dB
2K > 1 ⇒ K > 0.5 a
∴ 0.5 < K < 51 > 8K a=1
1 ∴ G.M. =0
K< (negative sign only shows
8 Q.14 (d)
that it is on negative axis) K
G (s) =
Q.10 (d) s ( s + 1) (s + 20)
1. Gain cross over frequency where K × 20
=
gain is G(s) = 1  s 
s(1 + s)  1 + 
3  20 
⇒ =
1
ω ( ω2 + 4 )
1/2 Bode plot is in (1+sT) form
K
9 20 log = = 1000
|ω= 0.1 60dB
⇒ =
1 ω
ω ( ω2 + 4 )
2
⇒K= 5
⇒ ωg =
1.26 ∴ G (s) =
100
Phases cross over frequency s ( s + 1) (1 + 0.5s)
Where ∠ GH = 180°
 ωΦ 
⇒=ωΦ 0.632=  cot 2ωΦ 
 2 
Q.17 (c)
1
Q.11 (d) GainMargin = 20 log
3 X
G.M at ωΦ = 1
0.632(0.6322 + 4)1/2 = 20 log
a = 2.26 0.5
1 = 20 log 2
G.M = 20 log = 6dB
a
And phase margin = 90°

© Copyright Reserved by Gateflix.in No part of this material should be copied or reproduced without permission
Q.18 (a) Q.21 (b)
System transfer function
 s 
K 1 + 
G (s) H (s) = 
0.1 
 s 
1+ 
 10 
Here, 20logK = 0
⇒K= 1
10s + 1
Therefore, G ( s ) H ( s ) = =ω 1= to ω 10
0.1s + 1 Is 1 dec are change & change is (G)
is 40 dB
Q.19 (b) ∴ S lope is 40dB / dec
G ( jω )= 5 + jω ∴ There are 2poles is orign
G ( jω
= ) 25 + ω2 K
So, G ( s ) = 2
At ω=0 S
G ( 0 )= 25 + 0= 5 G atω=1 = 32dB ( given )
At ω= ∞ k
⇒ 2 log =
32dB
G ( ∞ ) =∞ ω2 ω=1
⇒ 20log= k 32dB ⇒ = k 39.8
Q.20 (c) ∴G = 2
39.8
100 S
G (S) H (S) =
s ( s + 10 )
2
Q.22 (d)
For larger values of K, it will encircle
Φ= −90° − 2 tan −1 ( ω /10 ) the critical point (-1+j0), which
For phase cross–over frequency, makes closed-loop system unstable.
Φ = −180°
∴ −180 = −90°
−2 tan −1 ( ω /10 ) Q.24 (0.50)
⇒ω= 10rad / sec
Put, s = jω
100
G ( jω) H ( jω) =
jω ( jω + 100 )
2

G ( jω ) H ( jω )
ω =10
100 100 1
= = = → Due to initial slope, it is a type-1
ω ( ω 2 + 100 ) 10 ( 200 ) 20
system, and it has non zero velocity
=
G.M.in =
dB 20 log 20 26dB error coefficient (K v )
→ The magnitude plot is giving 0dB
at 2r/sec.
Which gives K v
∴ kv = 2

© Copyright Reserved by Gateflix.in No part of this material should be copied or reproduced without permission
A phase margin concepts is find k
The steady state error ess =
kv value for marginal stability using
reflection.
given unit ramp input; A =1
1 C.E:- s3 +11s 2 + 6s + 6k = 0
ess =
2
ess = 0.50

Q.25 (a)
→In a BODE diagram, in plotting the For marginal stability odd order row
magnitude with respect to frequency, of S should be zero i.e.
a pole introduce a line 4 slope-20dB 60 − k
/ dc = 0 ⇒ k = 60
11
→If 4th order all-pole system means
gives a slope of (-20) * 4 dB / dec i.e.
Q.30 (1)
-80dB / dec
Since it is the phase plot given we
can't use the slope concept as these
Q.26 (a)
are non linear curves.
10(s + 1)
G (s) = So we can take any phase angle of at
s + 10 a given frequency as reference and
Put s= jω can obtain p1
10(jω+1)
G(jω) = → phase of transfer function
(jω+10)  ω  −1  ω  −1  ω 
ω= 0, M = 1 < 0 ∅ ( ω) =− tan −1   − tan   − tan  
 0.1   10   P1 
ω= ∞, M =< 10 0 → from the plot at ω = 0.1, ∅ = −450
 0.1 0.1 0.1 
-450 = -  tan -1 + tan -1 + tan -1 
 0.1 10 P1 
Solving for P, we get p1 = 1.

So, zero is nearer to imaginary axis. Q.31 (d)


Hence plot will move clockwise Let us consider O.L.T.F. of a unity
direction. feedback system as
K
It is first quadrant. G (s) =
s (s + a )
Which represents a type-1 and
Q.28 (a) order-2 system for which Nyquist
1 − jω plot will be as shown below:
G( jω) =
4 + 2 jω
=ω 0,=
Gjω  0.25, ∠G(
= jω) 0
ω=
∞,1G( jω) =
0.5, G( jω) –180
= o 
N=0

Q.29 (60)
The given condition implied
marginal stability. One alternative
way without going for gain margin,

© Copyright Reserved by Gateflix.in No part of this material should be copied or reproduced without permission
Closed loop transfer function for the G ' ( jω) ω=ω < 1 (or 0 dB)
system is given as, pc

G (s) 20 log K + G ( jω) ω=ω < 0dB


T (s ) = pc

1 + G ( s ) .1 20 log K + 20dB < 0dB


K / s (s + a ) 20 log K < −20dB
=
1 +  K / s ( s + a )  log K < −1
K Hence, Maximum value of K i.e. K 0 ,
T (s) = for which system is stable is given as
s + as + K
2

(10 )= 0.1
−1
We can draw its Nyquist plot by = K=
K max 0
simply substituting s = jω and
obtain polar co-ordinates of T ( jω) Q.33 14.92
for different values of ω as, Given: For a unity feedback system
K
T ( jω ) =
K G (s) = and resonant peak
s (s + 2)
( jω) + jaω + K
2

K Mr = 2
T ( 0)
= = 1
( 0) + 0 + K
T ( ∞ ) =0
Hence statement given in option (D) We can find its closed loop transfer
is FALSE. function as,
C (s) G (s)
= T= (s)
R (s ) 1 + G (s ) H (s )
K / s (s + 2) K
= = 2
.1 s + 2s + K
K
1+
s (s + 2)
Here, one should note that the value of
D.C. gain is 1, not ‘K’.
Q.32 0.1

Magnitude and phase angle plots for


unstable system with
n0 We have formula for resonant peak as,
G (s) = 3
s + d 2s + d1s + d 0
2
Mr =
DCgain
Let G ' ( s ) = K.G ( s ) 2ξ 1 − ξ 2
For the closed loop system to be Substituting the given value of M r and
stable obtained D.C. gain, we get
GM > 1 1
1 =2
>1 2ξ 1 − ξ 2
G ' ( jω) ω=ω
pc
4ξ 1 − ξ 2 =1

© Copyright Reserved by Gateflix.in No part of this material should be copied or reproduced without permission
On squaring both the sides,
16ξ 2 (1 − ξ 2 ) =1
16ξ4 − 16ξ2 + 1 =0
If we put ξ 2 =x then equation
becomes
16x 2 − 16x + 1 =0
x = 0.067
We have, ξ 2 = x = 0.067
=ξ =
0.067 0.258
Here, characteristic equation is
s 2 + 2s + K =
0
Comparing it with standard equation,
we get
ω2n = K ⇒ ωn = K
And 2ξωn = 2
1 1
=
ξ =
ωn K
1
= =
Hence, K 14.92
ξ2

© Copyright Reserved by Gateflix.in No part of this material should be copied or reproduced without permission
GATE QUESTIONS(EE)

Q.1 The polar plot of a type-1, 3-pole, angle and dB gain at a frequency of
open loop system is shown in figure ω = 0.5 a are respectively
below. The closed loop system is.

a) Always stable.
b) Marginally stable.
c) Unstable with one pole on the
right half s-plane. a) 4.9° , 0.97dB b) 5.7° , 3dB
d) Unstable with two poles on the
right half s-plane. c) 4.9° , 3dB d) 5.7° , 0.97dB
[GATE-2001] [GATE-2003]

Q.2 The asymptotic approximation of Q.4 The Nyquist plot of loop transfer
the log-magnitude versus frequency function G(s)H(s) of a closed loop
plot of a minimum phase system control system passes through the
with real poles and one zero is point (-1, j0) in the G(s)H(s) plane.
shown in figure. Its transfer function The phase margin of the system is
is a) 0° b) 45°
c) 90° d) 180°
[GATE-2004]

Q.5 In the system shown in figure, the


input x ( t ) = sin t . In the steady-
state, the response y ( t ) will be

20 ( s + 5 ) 10 ( s + 5 )
a) b)
s ( s + 2 )( s + 25 ) sin ( t - 45° ) b) sin ( t + 45° )
1 1
( s + 2 ) ( s + 25)
2
a)
2 2
20(s + 5) 50( s + 5)
c) d) 2 c) sin ( t − 45 )
°
d) sin ( t+45° )
s ( s + 2 ) (s + 25)
2
s ( s + 2 ) ( s + 25)
[GATE-2004]
[GATE-2001]
Q.6 The open loop transfer function of a
Q.3 The asymptotic Bode plot of the unity feedback control system is
transfer function K/[1+(s/a)] is given as
given in figure. The error in phase

© Copyright Reserved by Gateflix.in No part of this material should be copied or reproduced without permission
as + 1 [GATE-2005]
G (s) = . The value of ‘a’ to give
s2
a phase margin of 45° is equal to Q.11 The Bode magnitude plot of
a) 0.141 b) 0.441 104 (1 + jω)
H ( jω ) = is
c) 0.841 d) 1.141 (10 + jω)(100 + jω)
2

[GATE-2004]
Q.7 A system with zero initial conditions a)
has the closed loop transfer
function.

T (s) =
( s2 + 4)
( s + 1)( s + 4 )
The system output is zero at the
frequency
a) 0.5 rad / sec b) 1 rad / sec b)
c) 2 rad / sec d) 4 rad / sec
[GATE-2005]

Q.8 The gain margin of a unity feedback


control system with the open loop
( s + 1)
transfer function G ( s ) = 2 is
s c)
1
a) 0 b)
2
c) 2 d) ∞
[GATE-2005]

Q.9 In the GH(s) plane, the Nyquist plot


of the loop transfer function
d)
πe −0.25s
G (s) H (s) = passes through
s
the negative real axis at the point
a) (-0.25, j0) b) (-0.5, j0)
c) (-1, j0) d) (-0.2, j0)
[GATE-2005]

Q.10 If the compensated system shown in [GATE-2006]


the figure has a phase margin of 60° Q.12 Consider the following Nyquist plots
at the crossover frequency of 1 of loop transfer functions over ω =0
rad/sec, then value of the gain k is to ω = ∞ . Which of these plots
represents a stable closed loop
system?

a) 0.366 b) 0.732
c) 1.366 d) 2.738

© Copyright Reserved by Gateflix.in No part of this material should be copied or reproduced without permission
b) marginally stable
c) unstable with one pole on the
RHS s-plane
d) unstable with two poles on the
RHSs-plane
[GATE-2009]
a) (1) only b) all except (1) Q.16 The asymptotic approximation of
c) all except (3) d)(1) and (2) only the log-magnitude vs frequency plot
[GATE-2006] of a system containing only real
poles and zeros is shown. Its
If x Re G ( jω) , =
Q.13 = & y lmG( jω) then transfer function is
for ω → 0+ , the Nyquist plot for
G ( s ) = 1/ s ( s + 1)( s + 2 )
a) x = 0 b) x = −3 / 4
y
c) x= y − 1/ 6 d) x =
3
[GATE-2007]
Q.14 The asymptotic Bode magnitude
plot of a minimum phase transfer
function is shown in the figure. This 10 ( s+5 ) 1000 ( s+5 )
a) b)
transfer function has s ( s+2 )( s+25 ) s ( s+2 )( s+25 )
2

100 ( s+5 ) 80 ( s+5 )


c) d)
s ( s+2 )( s+25 ) s ( s+2 )( s+25 )
2

[GATE-2009]

Q.17 The open loop transfer function of a


unity feedback system is given by
G ( s ) =(e-0.1s )/s . The gain margin of
this system is
a) Three poles and one zero a) 11.95 dB b) 17.67dB
b) Two poles and one zero c) 21.33 dB d) 23.9dB
c) Two poles and two zeros [GATE-2009]
d) One pole and two zeros
[GATE-2008] Q.18 The frequency response G(s)
Q.15 The polar plot of an open loop stable =1/[s(s+1) (s+2)] plotted in the
system is shown below. The closed complex G(jω) plane (for 0 < ω<∞) is
loop system is a)

b)

a) always stable

© Copyright Reserved by Gateflix.in No part of this material should be copied or reproduced without permission
c)

The gain (20log |G(s))| is dB and -8


at 1 rad/s and 10rad/s respectively.
The phase is negative for all ωThen
G(s) is
39.8 39.8
d) a) b) 2
s s
32 32
c) d) 2
s s
[GATE-2013]

Q.22 For the transfer function


5(S + 4)
G (s) =
[GATE-2010] s ( s + 0.25 ) (s 2 + 4s + 25)
The values of the constant gain term
Q.19 The frequency response of a linear and the highest corner frequency of
system is provided in the tabular the Bode plot respectively are
form below a)3.2, 5.0 b)16.0, 4.0
c)3,2, 4.0 d)16.0, 5.0
[GATE-2014]

The gain margin and phase margin Q.23 The magnitude Bode plot of a
of the system are network is shown in the figure
a)6dB and 30° b)6dB and -30°
c)-6dB and 30° d)-6dBand-30°
[GATE-2011]

Q.20 A system with transfer function

G (s ) =
( s 2 +9 ) ( s+2 )
is excited sin The maximum phase angle .m and
( s+1)( s+3)( s+4 ) the corresponding gain Gm
(ωt). respectively, are
The steady-state output of the a) -30° and 1.73dB
system is zero at b) -30° and 4.77dB
a)ω =1 rad/s b)ω =2 rad/s c) +30° and 4.77dB
c)ω =3 rad/s d)ω =4 rad/s d) +30° and 1.73dB
[GATE-2012] [GATE-2014]

Q.21 The Bode plot of transfer function Q.24 Nyquist plots of two functions G 1 (s)
G(s) is shown in the figure below. and G 2 (s) are shown in figure.

© Copyright Reserved by Gateflix.in No part of this material should be copied or reproduced without permission
Q.26 Consider the following asymptotic
Bode magnitude plot (ω is in
rad/s).

Nyquist plot of the product of G 1 (s)


and nG 2 (s) is
a)

Which one of the following transfer


function is best represented by the
above Bode magnitude plot?
2s
a)
b) (1 + 0.5s )(1 + 0.25s ) 2
4(1 + 0.5s)
b)
s(1 + 0.25s
2s
c)
(1 + 2 s )(1 + 4 s )
4s
d)
c) (1 + 2 s )(1 + 4 s ) 2
[GATE-2016]

Q.27 Loop transfer function of a feedback


s+3
system is G(s)H(s) = 2 .Take
s (s − 3)
the Nyquist contour in the clockwise
d) direction. Then the Nyquist plot of
G(s) H(s) encircles -1 + j 0
a) Once in clockwise direction
b) Twice in clockwise direction
c) Once in anticlockwise direction
d) Twice in anti clockwise direction
[GATE-2016]

[GATE-2015]

Q.25 The phase cross-over frequency of


100
the transfer function G ( s ) =
(s + 1)3
in rad/s is
1
a) 3 b)
3
c) 3 d) 3 3
[GATE-2016]

© Copyright Reserved by Gateflix.in No part of this material should be copied or reproduced without permission
ANSWER KEY:
1 2 3 4 5 6 7 8 9 10 11 12 13 14 15
(d) (d) (d) (a) (b) (c) (c) (a) (b) (c) (a) (a) (b) (c) (d)
16 17 18 19 20 21 22 23 24 25 26 27
(b) (d) (a) (a) (c) (b) (a) (c) (b) (a) (a) (a)

© Copyright Reserved by Gateflix.in No part of this material should be copied or reproduced without permission
EXPLANATIONS

Q.1 (d)
Nyquist plot will be. k
= G ( jω )
ω2
1+
a2
Phase of G ( jω ) ∠G ( jω )
 ω
= − tan −1  
a
at ω = 0.5a
k k
G ( jω ) ω= 0.5a
= =
∴ number of encirclement of (-1, j0) ( 0.5a )
2
1.25
=-2 and number of right sided pole 1+
a2
in open loop system=0 In dB
N= P − Z
= 20 log k − 20 log(1.25)1/2
∴ −2 = 0 − 2
⇒Z=2 = 20 log k − 0.97dB
∴ Closed loop system is unstable From the plot
with two poles on the right half of s- Magnitude = 20 log k
plane. ∴ error in dB gain =0.97dB
 0.5a 
∠ G( jω) ω= 0.5a = − tan −1  
Q.2 (d)  a 
Type : 2  0.5a  φ − 0
Poles = 0, 0, 2, 25 log10  = °
Zero =5  0.1a  −45
Gain mlogω + 20logk
= ∠ G( jω) ω= 0.5a = φ = −31.45°
54 = −40 log(0.1) + 20logk Error in phase-angle
⇒k= 5 =−26.56° − (−31.45° )
5(1 + 0.2S) = 5.89° ≈ 5.7°
∴ T.F. =
s (1 + 0.5S)(1 + 0.04S)
2

50(S + 5) Q.4 (a)


= 2
s (S + 2)(S + 25)

Q.3 (d)
k
= G=
Transfer function (s) s
1+
a
Put s = jω
k
G ( jω ) = ωg is gain cross over frequency at
ω
G ( jω ) H ( jω )
1+j
a which the gain
Magnitude of G ( jω ) becomes unity.

© Copyright Reserved by Gateflix.in No part of this material should be copied or reproduced without permission
In this case, phase 1
ωg =
∠ G ( jω ) H ( jω ) is − 180° at a
ω = ωg . = =
Gain G jωg 1 ( )
φ = -180 °
1 + a 2 ωg2
So phase margin= 180 + φ= 0 ° °
⇒ =1
ωg2
Q.5 (b) 1
1+ a2 ×
x ( t )= sin sin t = 1 < 0 ⇒
a2 = 1
ω = 1rad / sec 1/ a 2
s 1
= G=
Transfer function (s) a2
= 2
s +1
For sinusoidal input,= s jω= j =a =
1
0.841
jω j 1 21/4
G ( jω )= = = ∠ 45°
ω =1
jω + 1 j + 1 2
Q.7 (c)
1 1
=y(t) = ∠ 45°.1∠ 0° ∠ 45° s2 + 4
2 2 T (s) =
( s + 1)( s + 4 )
sin ( t + 45° )
1
=y(t) For frequency response, put
2
s = jω
( jω ) + 4
2
Q.6 (c) T ( jω ) =
( jω + 1) ( jω + 4)
4 − ω2
=
(1 + jω )( 4 + jω )
Magnitude of
4 − ω2
T ( jω ) = T ( jω )
(1 + ω2 )(16 + ω) 2
4 − ω2
as + 1 T ( jω ) =0
G (s) = (1 + ω2 )(16 + ω) 2
s2
ω = 2rad / sec
ωg = gain-crossover frequency at
The system output is zero at 2 rad/
which open gain is loop sec.
= 180° + ∠ G ( jω )
Phase margin
45° 180° + ∠ G( jωg )
= Q.8 (a)
ωpc = Phase cross over frequency at
∠ G ( jωg ) = −130°
which phase angle of OLTF is −180°
jaωg + 1
∠ G ( jωg ) = or at which OLTF cuts real axis
( jω )
2
g

=−2 × 90° + tan −1 aωg


∠ G ( jωg ) =
−180° + tan −1 aωg =
−135°
tan −1 aωg = −45°
aωg = 1

© Copyright Reserved by Gateflix.in No part of this material should be copied or reproduced without permission
πe − j0.25×2π
( ) (
G jωpc H jωpc =
j2π
)
j π π
=
−  cos − jsin  = −0.5 + j0
2 2 2

Q.10 (c)
k + 0.366s
=G (s) = andH ( s ) 1
T ( s ) OLTF
= = G ( s )=
H ( s ) G=
( s ) .1 G(s) s ( s + 1)
s +1 k + 0.366s
T (s) = = ( s ) H(s)
OLTF G=
s2 s ( s + 1)
jωpc + 1 Phase margin = 60° at ωgc (gain
(
T jωpc ) =
( )
2
jωpc cross over frequency)

∠ T ( jωpc ) =
−180° + tan −1 ωpc ( ) ( )
⇒ 180° + ∠ G jωpc H jωpc =
60°

OLTF cuts real axis at ωpc , so at ωpc ∠ G ( jωgc ) H ( jω ) =


−120 − 90
gc
° °

phase of OLTF is −180°  0.366ωgc 


− tan −1 ωgc + tan −1  
∠ T ( jωpc ) =
−180° + tan −1 ωpc  k 
−180° + tan −1 ωpc
−180° = = −120 °

ωpc = 0 ωgc 1rad / sec− 90° − tan −1 (1)


=
 0.366 × 1 
1 + ω2pc + tan −1  = −120°
=M T=
jωpc ( ) =
ω2pc
∞  k 
 0.366 
1 1 = −90° − 45° + tan −1  
Gain margin= = = 0  k 
M ∞ = −120°
 0.366 
Q.9 (b) ⇒ tan −1   = 15
°

 k 
πe −0.25s
G (s) H (s) =  0.366 
s ⇒ k ° 
= 1.366
Putting s = jω , for frequency  tan15 
response
Q.11 (a)
πe − j0.25ω
= G ( jω ) H ( jω ) = 104 (1 + jω )
jω H ( jω ) =
(10 + jω )(100 + jω )
2
Phase angle ∠ G ( jω ) H ( jω )
= −90 − 0.25ω ωpc at nyquist plot (1 + jω )
=
jω 
2
cuts negative real axis and this  jω  
10 ×  1 +  ×  1 + 
frequency  10   100 
( ) ( )
∠ G jωpc H jωpc is − 180° H ( jω ) =
k (1 + jω )
jω 
2
ω  jω  
∠ G ( jωpc ) H ( jω ) =
−90 °
− 1 +  1 + 
pc
4  10   100 
ω 1
−180° =
−90° − ω=
360° or2π Where = k = 0.1
4 10
Corner frequencies

© Copyright Reserved by Gateflix.in No part of this material should be copied or reproduced without permission
ω1 = 1rad / sec No. of encirclements = N=0
ω2 = 10rad / sec N=P-Z=0
⇒ 0-Z=0
and ω3 = 100rad / sec ⇒ Z=0
For frequency less than ω1i.e.ω < ω1 Hence system is stable.
Gain of the system is constant as 2.
there is no pole at origin.
=
Gain 20 =log k 20= log 0.1 20dB
At
=ω ω1 = 1rad / sec or log =
ω1 log
= 0
There is Zero, so system gain
increases with slope +20 dB/decade
and system gain becomes 0 dB at
ω = 10rad / sec or
logω=ω =10
log
= 10 1
=
At ω2 10 or log ω =
ω2=10 log10
= 1
Two anti-clockwise encirclement
N=2
There is pole, so slope is -20 dB/ P-Z=N=2
decade. ⇒ 0-Z=2
Overall slope ω2 < ω < ω3 Z=2
=20 dB/decade-20 dB/decade Hence system is unstable.
=0 dB/decade 3.
So, gain remains constant between
ω 2 < ω < ω3
or 1 < logω < 2
At =ω ω= 3 100rad / sec or
log ω − ω3 = 100 = log100 = 2
The double poles are present.
So, system gain decrease with -40
dB/ decade.

Q.12 (a) Two clockwise encirclement of -1


Assuming no. of open loop poles in Hence N=-2
the RHS of s-plane=P=0 Complete P-Z=N=-2
nyquist plots. ⇒ 0-Z=-2
1. Z=2
Hence the system is unstable.
4.

© Copyright Reserved by Gateflix.in No part of this material should be copied or reproduced without permission
Two clockwise encirclement of -1 Q.15 (d)
Hence N=-2
⇒ P-Z=N=-2
Z=2
Hence the system is unstable.
So, option (a) is correct.
Q.13 (b)
1
G (s) =
s ( s + 1)( s + 2 )
Put s = jω
1 − j (1 − jω )( 2 − jω )
=G ( jω ) = Two clockwise encirclement of 1+j0
jω (1 + jω )( 2 + jω ) ω (1 + ω2 )( 4 + ω2 )
N = −2
− j ( 2 − 3jω − ω2 ) −3ω − j ( 2 − ω2 ) Open-loop system is stable
= =
ω (1 + ω2 )( 4 + ω2 ) ω (1 + ω2 )( 4 + ω2 ) ⇒P= 0
N = P− Z−2 = 0− Z
Z=No. of closed loop poles in RHS of
8-planes.
Hence the system is unstable.

Q.16 (b)
Initial slope= -40 dB/dec.
−3 j ( ω2 − 2 )
= + Hence the system is type-2. So the
(1 + ω2 )( 4 + ω2 ) ω (1 + ω2 )( 4 + ω2 ) corresponding term of the transfer
3 function is 1/ s 2 .
At ω → 0,x → − ,y → −∞ At ω=2rad/s, slope changes by -20
4
dB/dec. from -40 dB/dec to -60
Q.14 (c) dB/dec. Hence the corresponding
Initial slope is -40dB/ decade, it term of the transfer function is
means there are double pole at 1
. At ω=5 rad/s, slope
origin.  s
1 + 
Slope changes from -40dB/ decade  2
to -20dB/ decade. It means there is changes by 20 dB/dec from -60
a zero. dB/dec to -40 dB/dec. Hence, the
Slope changes from -20dB/ decade corresponding term of the transfer
to 0dB/ decade at some other  s
frequency. It means there is one function is  1 +  .
 5
more zero.
At ω=25the slope changes by -20
Therefore transfer function has two
dB/dec. from -40 dB/dec to -60
poles and two zeros.
dB/dec. Hence the corresponding
term of the transfer function is
 1 
 s .
 1+ 
 25 

© Copyright Reserved by Gateflix.in No part of this material should be copied or reproduced without permission
 s
k 1 +  Q.18 (a)
G (s) =  5
1
 s  s 
s2  1 +   1 +  G (s) =
 2   25  s ( s + 1)( s + 2 )
At ω 0.1( ω ≤ 2rad / sec )
= Put s = ( jω )
Gain in G ( jω ) =
1
=dB 20log k − 40 log ω jω (1 + jω )( 2 + jω )
=80 20log k − 40 log 0.1 1 (1 − jω )( 2 − jω )
20log k = 40 = ×
jω (1 + jω )( 2 + jω ) (1 − jω )( 2 − jω )
k = 100
 s =
( 2 − jω − 2 jω − ω ) 2

100  1 +  jω (1 − ω )( 4 + ω )
2 2

G (s) =  5
 s  s 
s2  1 +   1 +  =
( 2 − ω ) − 3jω
2

 2   25  jω (1 + ω )( 4 + ω )
2 2

1000 ( s + 5 )
= 2
s ( s + 2 )( s + 25 ) =
−3
+j
( ω2 − 2 )
(1 + ω2 )( 4 + ω2 ) ω (1 + ω2 )( 4 + ω2 )
Q.17 (d) −3
Re  G ( jω ) 
Open loop transfer function, (1 + ω )( 4 + ω2 )
2

e −0.1s and
G (s) =
s ω2 − 2
imj  G ( jω ) 
Put s = jω ω (1 + ω2 )( 4 + ω2 )
e − j0.1ω
G ( jω ) = As ω → 0, Re  G ( jω )  → −
3
jω 4
At phase crossover frequency (ωpc ) ,
and lmj  G ( jω )  → −∞
phase of OLTF is −180°
As Re  G ( jω )  → −0 ,
π
∠ G ( jω ) ω ω =−180 =− π − °
pc 2 And lmj  G ( jω )  → +0
−0.1ωpc = −π at ω = ωpc
π Phase across frequency, the plot
−0.1ωpc =−
2 crosses negative real axis and
ωpc = 5π imaginary part of G(jω) is zero.
ω2 pc − 2
e − j0.1ω
is always 1 for any value of 𝜔𝜔 = (
lmj  G jωpc  = ) 0
(
ωpc 1 + ω2pc 4 + ω2pc )( )
1
G ( jω ) = ω2 pc = 2 rad/ sec
ω
Gain at ωpc (phase-cross frequency) So, plot crosses negative real axis at
1 ω = 2 rad/sec.
G ( jω ) ω = Therefore option (a) is correct.
pc=5 5π
1
Gain margin 20 log = Q.19 (a)
(
G jωpc ) At gain across over frequency
= 20
= log 5π 23.9dB ( )
ωgc , magnitude of G ( jω ) is 1.

© Copyright Reserved by Gateflix.in No part of this material should be copied or reproduced without permission
(
G jωgc = 1 ) ∴ S lope is 40dB / dec
∴ There are 2poles is orign
Phase of G ( jω ) = ∠ G jωgc ( ) K
So, G ( s ) = 2
= −150° S
= 180° + ∠ G jωgc
Phase margin ( ) G atω=1 = 32dB ( given )

= 180 − 150 = 30
° ° ° k
⇒ 2 log =
32dB
ω2 ω=1
At phase cross frequency ωpc , ( ) ⇒ 20log=k 32dB ⇒ =k 39.8
Phase of G ( jω ) is
39.8
∴G = 2
−180° , ∠ G jωpc = (
−180° ) S
M = magnitude of G ( jω ) at
Q.22 (a)
ωpc
= G
=( )
jωpc 0.5 ( ) G (s) =
5(s + 4)
1 s ( s + 0.25 ) (s 2 + 4s + 25)
Gain margin 20 log
M If we convert it into time constants,
1  s
= 20
= log 6dB 5×4 1+ 
0.5
G (s) =  4
s   4 s  
2

Q.20 (c) s[0.25] 1+ 25 1+ .s+ 
 0.25   25  5  
For sinusoidal excitation
s = jω  s
3.2 1+ 
∴ G ( jω ) G (s) =  4
( −ω + 9 ) ( jω + 2 )  s  4 s 
2
2
s 1+ 1+ .s+ 
=
( jω + 1)( jω + 3)( jω + 4 )  0.25   25 5 

For zero steady –state output Constant gain term is 3.2


ωn = 5 →highest corner frequency
G ( jω ) = 0

=
( −w 2
+ 9) w 2 + 4 Q.23 (c)

( w2 + 1 )( w2 + 9 )( w 2 + 16 ) (1 + 3s)
G ( s ) = k.
(1 + s)
For zero steady-state output  1
⇒ ωn = 9 3k.  s + 
G (s) =  3
⇒ ω = 3rad / sec
(s + 1)
Q.21 (b) Here k=1
1 1 1
=⇒ = 1
T 3 αT
=ωm = 1 ;α 1
3 3
4
G(s) ω==
1 ⇒ 3
3 4
3
= ω 1= to ω 10 =
G m in dB 20=log 3 4.77dB
Is 1 dec are change & change is (G)
is 40 dB

© Copyright Reserved by Gateflix.in No part of this material should be copied or reproduced without permission
1 − α  log ω2 = log 8 −
12
⇒ ω2 = 4
Φ m = sin −1 
1 + α  40
1 − 1  So one of the corner frequency is
 sin −1  
1 ω2 =4s at this frequency 2 poles
α = sin 
= 1 −1 3=
3 1 + 
1 2
 3 should exist because the change in
Φ m = 30° slope is -40db
From this we can say option A
satisfies the condition
Q.24 (b)
(i) A zero at origin
(ii) one of corner frequency 4H term
 s
will be  1 +  having 2 poles 1
 4

G1 ( s )
=
1
=;G 2 ( s ) s Q.27 (a)
s S+3
1 GH =
G1 ⋅ G 2 ( s ) = s ⋅ =1 S(S − 3)
s
(ω + a )
2 1/2
1
=GH =
Q.25 (a) ω (ω + a ) ω2
2 2 1/2

Phase Crossover frequency


ωPC : GH ω=ωPC =
−180°  ω  ω
GH=  tan -1  - 180°+180°-tan -1 
 3  3
−3 tan −1 ω ⇒ 180° =
GH = −3 tan −1 WPC
ω
⇒ WPC= tan 60°= 3 = 2 tan −1
3
1 ¯ ω
Q.26 (a) → GH = 2 tan −1
ω 2
3
By looking to the plot we can say
At ω = 0, GH = ∞ 0
that since the initial slope is +20
there must be a zero on the origin ¯ 0
At ω = ∞, GH = 0 180

1 0
=
At ω 3,=
GH 90
9
So the plot start at 0° and goes to
180° through 90° . Since there are 2
poles on origin we will get 2 ∞
radius semicircle those will start
where the mirror image ends and
If we find ω2 we can get the answer will terminate where the actual plot
by eliminating options started in clockwise direction. So the
M 2 − M1 plot will be
Slope =
log ω2 − log ω1
0 − 12
⇒ −40 =
log 8 − log ω2
12
⇒ log 8 − log ω2 =
40

© Copyright Reserved by Gateflix.in No part of this material should be copied or reproduced without permission
So the Nyquist plot of G(s) H(s)
Encircles — 1 + j0
Once in clockwise direction

© Copyright Reserved by Gateflix.in No part of this material should be copied or reproduced without permission
GATE QUESTIONS(IN)

Q.1 A unity feedback system has the


following open loop frequency
response:
Ω|(rad 2 3 4 5 6 8 10
/sec)|
|G(jω)| 7.5 4.8 3.15 2.25 1.70 1.00 0.64
∠G(jω) 1180 1300 1400 1500 1570 1700 1800

The gain and phase margin of the


system are
a) 0dB, −180° b) 3.88dB, −170° a)3 pole and 1 zero
c) 0dB,10° d) 3.88dB,10° b)1 pole and 2 zeros
[GATE-2006] c)2 poles and 1 zero
d)2 poles and 2 zeros
Common Data for Question Q.2 & Q.3:
[GATE-2008]
The following figure represents a
proportional control scheme of a order
Q.5 A unity feedback control loop with
system with transportation lag.
an open transfer function of the
K
form has a gain crossover
s(s + a)
frequency of 1 rad/s and a phase
margin of 60° if an element having
Q.2 The angular frequency in radians/s s- 3
a transfer function is inserted
at which the loop phase lag s+ 3
becomes180° is into the loop, the phase margin will
a)0.408 b)0.818 become
c)1.56 d)2.03 a) 0° b) 30°
[GATE-2007] c) 45° d) 60°
Q.3 The steady state error for a unit step [GATE-2009]
input when the gain K c = 1 is
1 1 Q.6 The asymptotic Bode magnitude
a) b) plot of a lead network with its pole
4 2
and zero on the left half of the s-
c) 1 d) 2
plane is shown in the adjoining
[GATE-2007]
figure. The frequency at which the
Q.4 The Bode asymptotic plot of a phase angle of the network is
transfer function is given below, in maximum (in rad/ s) is
the frequency range shown, the
transfer function has

© Copyright Reserved by Gateflix.in No part of this material should be copied or reproduced without permission
3 1 Q.11 The approximate phase response of
a) b)
10 20 1002 e −0.01s
is
1 1 s 2 + 0.2 s + 1002
c) d)
20 30
[GATE-2010] a)

Common data for question Q.7 & Q.8:


The open-loop transfer function of a unity
negative feedback control system is given
k
by G ( s ) =
(s + 5)3
Q.7 The value of K for the phase margin
of the system to be 45° is
a) 250 5 b) 250 2 b)
c) 125 5 d) 125 2
[GATE-2011]

Q.8 The value of K for the damping ratio


ζ to be 0.5, corresponding to the
dominant closed loop complex
conjugate pole pair is
a) 250 b) 125
c) 75 d) 50
[GATE-2011] c)
Q.9 The open loop transfer function of a
unity negative feedback control
system is given by
150
(s) = . The gain margin
s ( s+9 ) (s+25)
of the system is
a)10.8dB b)22.3 dB
c)34.1 dB d)45. 6 dB
[GATE-2012] d)

Q.10 The loop transfer function of a


feedback control system is given by
1
G (s ) H ( s) = Its phase
s ( s + 1) (9s + 1)
crossover frequency (in rad/s),
approximated to two decimal places,
is_________.
[GATE-2014]
[GATE-2018]

© Copyright Reserved by Gateflix.in No part of this material should be copied or reproduced without permission
Q.12 An input p(t) = sin(t) is applied to
s −1
the system G ( s ) = . The
s +1
corresponding steady state output is
y(t) = sin(t + φ ) , where the phase φ
(in degrees), when restricted to
00 ≤ φ ≤ 3600 , is _______________.

[GATE-2018]

Q.13 Consider the transfer function


2
G (s) = . The phase
( s + 1)( s + 2)
margin of G(s) in degree is
____________.

[GATE-2018]

ANSWER KEY:
1 2 3 4 5 6 7 8 9 10 11 12 13
(d) (d) (b) (c) (a) (b) (b) (b) (c) 0.33 (a) 90 180

© Copyright Reserved by Gateflix.in No part of this material should be copied or reproduced without permission
EXPLANATIONS

Q.1 (d)   ω 
Ωg =
8 ⇒ PM ⇒ 180° − 170° =
10° = 180 +  −90° − tan −1   
  a 
10rad
Ωc
⇒ GM
= ⇒ a =3
sec
s− 3 k
 1  PM of T ' (s) = .
= 20
= log10   3.88dB s + 3 s(s + σ)
 0.4 
T (s) =
'
180° + ∠ T ' ( jω ) ω=1 =

Q.2 (d)
K ce− s K c e − jω Q.6 (b)
=GH ( s ) = , GH ( jω ) 1
s +1 1 + jω ωm = ω1ω2 = 0.1× 0.5 =
∠ GH ( jω ) =− ω − tan (ω)
−1 20
Phase lag = ω + tan −1 ( ω ) =
π rad
Q.7 (b)
= 3.142rad
 ω
Is satisfied only at PM of 45° = 180° − 3 tan −1   =
ω = 2.03rad / sec 5
 ω
45° ⇒ tan −1   =1 ⇒ ω =5rad / sec
Q.3 (b) 5
For K c = 1 and H ( s ) = 1
e− s
OLTF=G(s)= K
s +1 ∴ G ( Jω ) =
1⇒ 1⇒ K
=
(ω + 52 )
2 3/2
For r(t)=R.u(t),R=1
Positional error constant =
=K p Lt = Lt G ( s ) 1 = 250 2
s→0 s→0

R 1
=ess = Q.8 (b)
1+ Kp 2
or
E(s) 1 s +1 Q.9 (c)
= =
R(s) 1 + G(s) s + 1 + K c e − s G.M is evaluated using 𝜔𝜔𝑃𝑃𝑃𝑃
E (s) =
(s + 1)
s(s + 1 + e − s )
φ = −90 tan −1 ω 9 ( )
=ess Lt = Lt E ( s )
1
1 ω
tan −= (25 )=
180°[ Q ω ωpc ]
s→0 s→0 2 A B

Q.4 (c)
⇒ tan −1 ω
( )
9 + tan
−1 ω
25 ( )
Compare with Bode magnitude plot = 90° Taking tan on both sides
of standard transfer function. tanA + tanB
tan ( A + B ) =
1 − tanA tanB
Q.5 (a) tan 90° = ∞
PM of T ( s ) = 60° ⇒ 1 − tanA tan B =0

© Copyright Reserved by Gateflix.in No part of this material should be copied or reproduced without permission
⇒ ω2 =225 So, Phase decrease further. Option
⇒ tanA tanB = 0 (a) satisfy.
⇒ω= 15r / s
∆Q
G=
15 225 + 81 225 + 625 Q.12 90
1
⇒ G.M in dB = 20 log   = 34dB s −1
4 G (s) =
...(1)
s +1
Q.10 (0.33) =y (t ) G ( jω )ω =1 sin(t + φ )

Given G ( s ) H ( s ) =
1 jω − 1
where φ =
∠G ( jω ) =
s ( s + 1) (9s + 1) jω + 1
Phase cross over frequency ∠G ( jω ) =
180 − tan −1 (ω ) − tan −1 (ω )
WPC ⇒ G(jωpc )H G(jωpc )=-180°
= 180 − 2 tan −1 (ω )
WPC ⇒ G(jω)H(jω)=-180° φ=
∠G ( jω ) ω =1 =
1800 − 900 =
900
-90°-tan -1 ( ω ) -tan -1 ( 9ω ) =- 180° G ( jω ) ω =1 = 1
⇒ tan -1
( ω ) +tan ( 9ω ) =90°
-1
So, y (t ) =
1 ⋅ sin(t + 900 )
 ω+9ω 
⇒ tan -1  =90° φ = 900
 1-9ω 
2

1
⇒ 1-9ω2 =0 ⇒ ω=
3
ω=0.33r/s Q.13 180

Q.11 (a) =G ( jω ) = 1
2
ω +1 ω2 + 4
2

104 e −0.01s (ω 2 + 1)(ω 2 + 4)


= 4 ∴ω = ω gc
G ( s) =
s 2 + 0.2 s + 1002
ω gc2 ω gc2 + 5 =
0
Put s = jω
∴ω gc =
0 rad / sec
104 e −0.01 jω
G ( jω )=
−ω 2 + 0.2 jω + 104 ω 
P∠G ( jω ) = − tan −1 (ω ) − tan −1   =00
 0.2ω   
2
∠G ( jω ) =
−0.01ω − tan −1  4 2 
 10 − ω  P.M . = 180 0

ω= 0; ∠G ( jω ) =
0; ω =10; ∠G ( jω ) = −5.80
 20 
ω =100; ∠G ( jω ) =−1 − tan −2  
 0 
180
=
− − 900 =
−57.3 − 900 =
−147.30
π
ω = 200
−3600  40 
∠G ( jω ) = − tan −1  4 
π  −3 × 10 

© Copyright Reserved by Gateflix.in No part of this material should be copied or reproduced without permission
6 STATE VARIABLE ANALYSIS

6.1 INTRODUCTION • It can be applied to time invariant


systems.
State space analysis is an excellent method • It can be applied to multiple input
for the design and analysis of control multiple output systems.
systems. The conventional and old method • Its gives idea about the internal state of
for the design and analysis of control the system.
systems is the transfer function method.
The transfer function method for design 6.2 STATE OF A SYSTEM
and analysis had many drawbacks.
The state of a system is the minimum set of
6.1.1 DRAWBACKS OF TRANSFER variables (state variables) whose
FUNCTION ANALYSIS knowledge at time =
t t 0 , along with the

knowledge of the inputs at time t ≥ t 0


• Transfer function is defined under zero
initial conditions. completely describes the behavior of a
• Transfer function approach can be dynamic system for a time t ≥ t 0 . State
applied only to linear time invariant variable is a set of variables which fully
systems. describes a dynamic system at a given
• It does not give any idea about the instant of time.
internal state of the system. Consider a system having an inputs, b
• It cannot be applied to multiple input outputs and c state variables. Then,
multiple output systems. Output variables
• It is comparatively difficult= to perform   Y1 ( t ) , Y2 ( t ) , Y3 ( t ) ………… Yb (t)
transfer function analysis on computers. Input variables 
Any way state variable analysis = can be U1 (t), U 2 (t), U 3 (t) ………… U a (t)
performed on any type systems and it is State variables
=
very easy to perform state variable analysis X1 (t), X 2 (t), X 3 (t) ………… X c (t)
on computers. The most interesting feature Then the system can be represented as
of state space analysis is that the state shown below.
variable we choose for describing the
system need not be physical quantities
related to the system. Variables that are not
related to the physical quantities
associated with the system can be also
selected as the state variables. Even
variables that are immeasurable or
unobservable can be selected as state 6.2.1 STATE EQUATION BASED MODELING
variables. PROCEDURE

6.1.2 ADVANTAGES OF STATE VARIABLE The complete system model for a linear
ANALYSIS time-invariant system consists of
1) A set of n state equations, defined in
• It can be applied to non linear system. terms of the matrices A and B.

© Copyright Reserved by Gateflix.in No part of this material should be copied or reproduced without permission
2) A set of output equations that relate any also based on the linear graph system
output variables of interest to the state description.
variables and inputs, and expressed in 3) Determination of a suitable set of
terms of the C and D matrices. output equations and derivation of the
The task of modeling the system is to appropriate C and D matrices.
derive the elements of the matrices, and
to write the system model in the form: 6.2.2 BLOCK DIAGRAM REPRESENTATION
=x ' Ax + Bu OF LINEAR SYSTEMS DESCRIBED BY
    STATE EQUATIONS
 x   a11 a12 … a1n   x 
  1
D    a 21 a 22 … a 2n    The matrix-based state equations express
1

x2  = x2  the derivatives of the state-variables


dt    M M M  
explicitly in terms of the states themselves
 M  a  M
a n 2 … a nn   
 x n   n1
 x n  and the inputs. In this form, the state vector
is expressed as the direct result of vector
integration. The block dia. representation is
 b11 … b1r  shown in Fig. This general block diagram
b …   u1  shows the matrix operations from input to
+
21 b 2r   M
 M M M   output in terms of the A, B, C, D matrices,
   u r  but does not show the path of individual
 n1
b b n2 b nr  variables.
=
y Cx + Du • Step 1: Draw an integrator (s −1 ) blocks,
    and assigns a state variable to the
 y   c11 c12 … c1n   x  output of each block.
 1   c 21 c 22 … c 2n   1  • Step 2: At the input to each block
 y2  =   x 
   M M M  2 (which represents the derivative of its
  c
M
a m2 … a mn   
 M state variable) draw a summing
 y n   m1
 x n  element.
• Step 3: Use the state equations to
 d11 … d1r 
 d   u1  connect the state variables and inputs
… d   M to the summing elements through
+
21 2r

 M M M   scaling operator blocks.


   u r  • Step 4: Expand the output equations
d m1 d m2 d mr 
and sum the state variables and inputs
The matrices A and B are properties of the through a set of scaling operators to
system and are determined by the system form the components of the output.
structure and elements. The output
equation matrices C and D are determined
by the particular choice of output variables.
The overall modeling procedure developed
in this chapter is based on the following
steps:
1) Determination of the system order n
and selection of a set of state variables
from the linear graph system Example
representation. Find the transfer function relating the
2) Generation of a set of state equations output y(t) to the input u(t) for a system
and the system A and B matrices using a described by the first-order linear state and
well defined methodology. This step is output equations:

© Copyright Reserved by Gateflix.in No part of this material should be copied or reproduced without permission
dx d 2 x(t)
= ax ( t ) + bu ( t ) 2
= x=
3 x '2
dt dt
=
y(t) cx(t) + du(t) 3
d x(t)
= x=
4 x 3'
dt 3
Now, x1′ = x2
Solution: x '2 = x 3
The Laplace transform of the state equation
= aX(s) + bU(s)
is sX(s) and x 3' = −2x 3 − 3x 2 − x1 + u(t)
Which may be rewritten with the state  x1   0 1 0   x1   0 
variable X(s) on the left-hand side: =
d  x   0 0 1   x  + 0  u(t)
(s − a)X(s) = bU(s) dt  2   2  
 x 3   −1 −3 −2   x 3  1 
Then dividing by (s−a), solve for the state
variable: 0 1 0 0 
b = A =  0 0 1  B 0  
X (s) = U(s) and substitute into the
s−a  −1 −3 −2  1 
Laplace transform of the output equation
=Y(s) cX(s) + dU(s) : Y(s) s 2 +3s+3
3) =
X(s) 1
 bc 
=Y(s)  + d  U(s) ⇒ Y ( s ) = s 2 X ( s ) + 3sX ( s ) + 3X ( s )
s − a 
ds + ( bc − ad ) From the above equation we can write
= U(s) the differential equation as
s−a
d 2 x(t) dx(t)
The transfer function is: Y(t) = +3 + 3x ( t )
Y ( s ) ( ds + ( bc − ad ) )
2
dt dt
=
H (s) = Put x ( t ) = x1
U (s) (s − a)
dx(t)
= x2
6.3 STATE MODEL FROM TRANSFER dt
FUNCTION d 2 x(t)
= x3
dt 2
Consider a transfer  x1 
Y (s) s 2 + 3s + 3 = Y [3 3 1]  x 2  + [ 0] u(t)
=T(s) = 3
U ( S) s + 2s + 3s + 1
2
 x 3 
1) Write the transfer function in the form
C = [3 3 1]
=T (s)
()
Y s X s
=
() s + 3s + 3
2
1
D = [ 0]
. . 3
X (s) U (s) 1 s + 2s 2 + 3s + 1
X(s) 1
2) = 3 2 6.4 TRANSFER FUNCTION FROM STATE
U(S) s +2s +3s+1
MODEL
⇒ s3 X ( s ) + 2s 2 X ( s ) + 3sX ( s ) + X ( s ) =
U (s)
From the above equation we can write Consider a state model derived for linear
the differential equation as time invariant system as,
d 3 x(t) d 2 x(t) ( t ) Ax(t) + Bu(t) and
x=
'
dx(t)
3
+ 2 2
+3 + x (t) =
u(t)
dt dt dt =
y(t) Cx(t) + Du(t)
Put x ( t ) = x1 The transfer function of the above state
dx(t) model is,
= x=
2 x1'
dt

© Copyright Reserved by Gateflix.in No part of this material should be copied or reproduced without permission
Y (s)
T(s) = =C[sI − A]−1 B + D Properties of state transition matrix:
U (s )
1) ∅ ( t ) = e A×0 = I(identity matrix)
Where I is an Identity matrix
2) ∅ −1 ( t ) =∅ ( − t )
3) ∅ −1 ( t1 + t 2 ) = ∅ ( t1 ) × ∅ ( t 2 )
Example:
Consider a system having state model
 x1'   −2 −3  x1  3 Example:
= '   + u & Find state transition matrix for
 x 2   4 2   x 2  5   0 −1
A= 
x 
Y = [1 1]  1   2 −3
x2 
Solution: Solution:
We know that,  0 −1
A= 
T.F. =C[sI − A]−1 B + D  2 −3
 −2 −3  3 1 0   0 −1
A= , B =   , C = [1 1] [sI −=
A]
 s − 
4 2 5  0 1   2 −3
1 0   −2 −3 s +1 
[sI −=
A] s  −  = 
0 1   4 2   −2 s + 3
s+2 +3  s + 3 −1
= 
 -4 s-2   +2 s 
∴ [sI − A ] =
−1 
s-2 -3  ( s + 1)( s + 2 )
 +4 s+2 
[sI-A ] =  2   s+3 −1 
-1

s +8  ( s + 1)( s + 2 ) ( s + 1)( s + 2 ) 
s − 2 −3  =
 +4 s + 2  3  +2 s 

∴ T.F. = [1 1]  
5   ( s + 1)( s + 2 ) ( s + 1)( s + 2 ) 
s2 + 8  
∅(t) =
We know that, e At = −1[sI − A]−1
 3
[s-2+4 -3+s+2]    2e − t − e −2t −e − t + e −2t 
= 5 = 8s+1 ∴∅ ( t ) =  − t −2t 
2
s +8 s 2 +8  2e − 2e −e − t + 2e −2t 

6.5 SOLUTION OF STATE EQUATION 6.6 CONTROLLABILITY

Consider a constant matrix A & input A system is said to be controllable at time


control forces are zero, then the state t 0 if it is possible by means of an
equation will be x ( t ) = Ax(t)
'
unconstrained control vector to transfer
the system from any initial state to any
Such an equation is called as homogeneous
other state in a finite interval of time.
equation & its solution is given by
x ( t ) = e At x(0)
Where e At = ∅(t) is called as state
transition matrix &
∅(t) =
e At = −1[sI − A]−1

© Copyright Reserved by Gateflix.in No part of this material should be copied or reproduced without permission
Kalman’s test for Controllability: 1 1
Now, Qo =
= CT A T CT   
Consider a LTI system with state equation 0 1
Qo =1 − 0 =1
( t ) Ax(t) + Bu(t) . For this state
x=
'

≠ 0; hence the system is observable


equation a Qc is defined as
Qc  B AB A 2 B … A n −1B
n×n 6.8 STABILITY OF THE SYSTEM
The system will be completely controllable
if the rank of matrix Qc is ‘n’ i.e. | Qc |≠ 0 . The transfer function of a system can be
6.7 OBSERVABILITY obtained from the state equations as
Y (s)
T(s) = =C[sI − A]−1 B + D
A system is said to be observable at time t 0 U (s)
if, with the system in state x(t 0 ) , it is C × Adj[sI − A ] × B
possible to determine this state = from the +D
det [sI − A ]
observation of the output over a finite
interval of time. Adj[sI − A ]
Q [sI − A]−1 =
det [sI − A ]
Kalman’s test for Observability: C × Adj[sI − A ] × B + D × det [sI − A ]
=
det [sI − A ]
Consider a LTI system with state equation
x=( t ) Ax(t) + Bu(t) . For this state As the denominator of the transfer function
'

is det [sI − A ] , the characteristics equation


equation a Qo is defined as
Qo CT A T CT (A T ) 2 CT … (A T ) n −1 CT  will be sI − A = 0
n×n
The roots of this characteristics equation
The system will be completely observable if
will be the closed loop poles of the system
the rank of matrix Qo is ‘n’ i.e. | Qo |≠ 0 .
& if any root is positive (on RHS of s-plane)
system will be unstable.
Example:
Check the controllability & observabilty of Example:
the system Determine the stability of system
 x1'   1 1   x1  0  =x’ Ax + Bu
=  '     +   u(t) and
 x 2   −2 −1  x 2  1   −1 2  0 
A=  , B= 
x   0 2 1 
y(t) = [1 0]  1  Solution:
x2 
1 0   −1 2 
[sI − A ] = s — 
Solution: 0 1   0 2 
For the given system s + 1 −2 
 1 1   
0 =  0 s − 2
A=
− −  , B = 1  , C = [1 0 ]  
 2 1   The characteristics equation is
0 1  sI − A = ( s + 1)( s − 2 ) = 0
=Qc [=
B AB] 1 −1
  The roots of the characteristics equation
Qc =0 − 1 =−1 are s = 2 & s = −1
As on root (s = 2) lies on RHS of s-plane,
≠ 0 hence the system is control lable
system is unstable.

© Copyright Reserved by Gateflix.in No part of this material should be copied or reproduced without permission
Example:
Find out state model of the following
d3 y d2 y dy
system. 3 + 2 2 + 3 + y = u
dt dt dt
Solution:
The above equation can be written as
s3 Y ( s ) + 2s 2 Y ( s ) + 3sY ( s ) + Y ( s ) =
U(s)
The transfer function will be
Y (s) 1
=T(s) = 3
U ( S) s + 2s + 3s + 1
2

It can also be written as


Y (s) X (s) 1
=T (s) = . 1. 3
X (s) U (s) s + 2s + 3s + 1
2

X (s) 1
= 3
U ( s ) s + 2s + 3s + 1
2

∴ s3 X ( s ) + 2s 2 X ( s ) + 3sX ( s ) + X ( s ) =
U (s)
d3x d2x dx
i.e. 3 + 2 2 + 3 + x =u
dt dt dt
put x = x1
dx
= x= 2 x1'
dt
d2x
=
2
x= 3 x '2
dt
d3x
=3
x= 4 x 3'
dt
Now, x1' = x 2
x '2 = x 3
x 3' =x4 =− x1 − 3x 2 − 2x 3 + u
 x1'   0 1 0   x1   0 
   0 0 1   x  + 0 u
∴  x '2    2  
 x 3'   −1 −3 −2   x 3  1 
 
Y (s)
=
1 ⇒ y(t) =
x(t)
X (s)
 x1 
∴ y [1 0 0]  x2  + [ 0] u
=
 x3 

© Copyright Reserved by Gateflix.in No part of this material should be copied or reproduced without permission
GATE QUESTIONS(EC)

Q.1 The transfer function Y(s)/U(s) of a e− t 0  0 et 


system described by the state c)   d)  t 
0 e− t  e 0 
equations
x ( t ) =
−2x ( t ) + 2u ( t ) and y ( t ) =
0.5x(t) [GATE-2004]
is Q.5 A linear system is equivalently
a) 0.5 / (s − 2) b) 1/ (s − 2) represented by two sets of state
c)
0.5
d)
1 equations. X=AX+BU and W=CW+DU
s+2 s+2 The eigen values of the
[GATE-2002] representations are also computed
as [λ] and [μ] Which one of the
Q.2 The zero-input response of a system following statements is true?
given by the state –space equations a) [λ]=[μ] and X=W
 x 1  1 0   x1   x1 ( 0 )   1  b) [λ]=[μ] and X≠W
=
 x  1 1   x  and  =  c) [λ]≠[μ] and X=W
 2   2  x 2 (0)  0  d) [λ]≠[μ] and X≠W
is [GATE-2005]
 te t  e t 
a)   b)  
Q.6 A linear system is described by the
 t  t
following state equation
 et   t 
c)  t  d)  t   0 1
X ( t ) =AX ( t ) + BU ( t ) , A =
 −1 0 
 te   te   
[GATE-2003] The state –transition matrix of the
system is
Q.3 The state variable equations of a
 cos t sin t 
system are: a)  
1) x 2 =−3x1 − x 2 + u ,  − sin t cos t 
2) x = 2x1 y= x1 + u  − cos t sin t 
2 b)  
The system is  − sin t − cos t 
a) controllable but not observable  – cos t − sin t 
c)  
b) observable but not controllable  − sin t cos t 
c) neither controllable not
cos t − sin t 
observable d)  
d) controllable and observable cos t sin t 
[GATE-2004] [GATE-2006]

1 0  Q.7
The state space representation of a
Q.4 Given, A=  the state separately excited DC servo motor
0 1  dynamics is given as
transition matrix e At is given by  dω 
 dt   −1 1   ω   0 
0 e− t  e t 0 = di    + u
a)  − t  b)    a   −1 −10   i a  10 
e 0 0 et   dt 

© Copyright Reserved by Gateflix.in No part of this material should be copied or reproduced without permission
Where, ω is the speed of the motor, 1 1 0 1
ia is the armature current and u is c)   d)  
 −1 −2   −2 −3
the armature voltage. The transfer
[GATE-2007]
ω(s)
function of the motor is
U(s) Q.10 A signal flow graph of a system is
a) 2
10
b) 2
1 given below.
s + 11s + 11 s + 11s + 11
10s + 10 1
c) 2 d) 2
s + 11s + 11 s + s +1
[GATE-2007]

Statement for Linked Answer Questions


Q.8 & Q.9:
Consider a linear system whose state space
representation is x ( t ) = Ax ( t ) . . If the
The set of equations that
initial state vector of the system is correspond to this signal flow graph
1 is
( 0 ) =   , then the system response is
 −2   x1   β -γ 0   x1  1 0 
u 
a)  x 2  = γ α 0   x 2  + 0 0   1 
d  
 e −2t 
x (t) =  −2t 
. If the initial state vector of dt    
 
 
 u2 
 −2e   x 3  -α -β 0   x 3  0 1 
1  x1   0 α γ   x1   0 0
the system changes to x ( 0 ) =   then the d     u 
 −1 b)  x 2  = 0 -α -γ   x 2  + 0 1   1 
dt   
0   2 
u
 e− t   x 3  0 β -β   x 3  1
system response becomes x ( t ) =  − t 
 −e   x1  -α β 0   x1   1 0
d     u 
c)  x 2  = -β -γ 0   x 2  + 0 1   1 
dt   
Q.8 0   2 
u
The Eigen-value and Eigen-vector
 x 3   α γ 0   x 3  0
pairs ( λ i , vi ) for the system are
 x1   -γ 0 β   x1   0 1 
d     u 
d)  x 2  = γ 0 α   x 2  + 0 0   1 
 1   1   dt   
0 -α   x 3  1 0   2 
u
a)  −1,    and  −2,     x 3  -β
 1   2 
[GATE-2008]
  1    1 
b)  −2,    and  −1,   
  −1    −2   Q.11 Consider the system
  1    1  dx 1 0  p
c)  −1,    and  −2,    = Ax + BuwithA =   andB =q 
dt 0 1   
  −1    −2  
where p and q are arbitrary real
  1    1  numbers. Which of the following
d)  −2,    and 1,   
  −1    −2   statements about controllability of
[GATE-2007] the system is true?
a) The system is completely state
Q.9 The system matrix A is controllable for any nonzero
 0 1 1 1 values of p and q.
a)   b)  −1 −2  b) Only P=0 and q=0 result in
 −1 1   controllability.

© Copyright Reserved by Gateflix.in No part of this material should be copied or reproduced without permission
c) The system is uncontrollable for
all values of p and q.
d) We cannot conclude about
controllability from the given
data
[GATE-2009]

Common Data for Questions Q.12 & A state space model of the above
Q.13: system in terms of the state vector x
The signal flow graph of a system is shown
and the output vector y = [ y1 y2 ] is
T
below:
a) x = [ 2] x+ [1] u; y = [12] x

 [ -2] x+ [1] u; 1


b) x= y=   x
2
-2 0  1
 
c) x=  x+   u;y= [12] x
Q.12 The state variable representation of  0 -2  1
the system can be  2 0  1  1
 
d) x=  x+   u;y=   x
 1 1 0  0 2 2 2
a) x =   x + u
-1 0  2 [GATE-2011]
-1 1  0
b) x =   x + u Q.15 The state variable description of an
-1 0  2 LTI system is given by
y = [0 0.5] x  X 1   0 a1 0   X1   0 
y = [0 0.5] x       
=  X2   0 0 a 2   X2  +  0  u
1 1 0   X 3   a 3 0 0   X 3   1 
c) x =   
 x + u
-1 0 2  X1 
-1 1 0  
d) x =  y = (1 0 0 )  X 2 
 x + u
X 
-1 0 2  3
y = [ 0.5 0.5] x Where y is the output and u is the
y = [ 0.5 0.5] x input. The system is controllable for
[GATE-2010] a) a1 ≠ 0, a 2 ≠ 0, a 3 ≠ 0
b) a1 = 0, a 2 ≠ 0, a 3 ≠ 0
Q.13 The transfer function of the system is c) a1 = 0, a 2 ≠ 0, a 3 =
0
s+1 s −1
a) 2 b) 2 d) a1 ≠ 0, a 2 ≠ 0, a 3 =
0
s +1 s +1
[GATE-2012]
s+1 s-1
c) 2 d) 2
s +s+1 s +s+1 Statement for Linked Answer Questions
[GATE-2010] 16 & 17
The state diagram of a system is shown
Q.14 The block diagram of a system with below is described by the state –variable
one input u and two outputs y1 and equations:
y 2 is given below. X =AX + Bu; y =CX + Du

© Copyright Reserved by Gateflix.in No part of this material should be copied or reproduced without permission
The system is
a) controllable and observable
b) uncontrollable and observable
c) uncontrollable and unobservable
d) controllable and unobservable
[GATE-2014]

Q.16 The state-variable equations of the Q.19 An unforced liner time invariant
system in the figure above are (LTI) system is represented by
 −1 0   −1   
= a) X   X+ u  X1  =  −1 0   X1 
 1 −1 1     0 −2   X 2 
X2 
 −1 0   −1
= b) X   X+ u If the initial conditions are x 1 (0) = 1
 −1 −1 1 and x 2 (0) = -1, the solution of the
y=[1 − 1] X + u state equation is
y = [ −1 − 1] X + u a) x 1 (t) = ‒1, x 2 (t) = 2
b) x 1 (t) = ‒e-t, x 2 (t)=2e-t
 −1 0  −1
=c) X   X+ u c) x 1 (t)=e-t, x 2 (t)= ‒e-2t
 −1 −1 1 d) x 1 (t) = ‒e-t, x 2 (t) = —2e-t
 −1 −1  −1 [GATE-2014]
=d) X   X+ u
0 −1 1 Q.20 Consider the state space system
y = [ −1 − 1] X − u expressed by the signal flow diagram
y=[1 − 1] X − u shown in the figure.
[GATE-2013]

Q.17 The state transition matrix eAt of the


system shown in figure above is
 e-t 0   e-t 0
a)  -t -t 
b)  -t -t 
 te e  -te e 
e-t 0  e-t -te-t  The corresponding system is
c)  -t -t 
d)  -t 
e e  0 e  a) always controllable
[GATE-2013] b) always observable
c) always stable
Q.18 Consider the state space model d) always unstable
system, as given below [GATE-2014]
  
 x1   −1 1 0   x1   0  Q.21 The state equation of a second-order
        linear system is given by
x2  = 0 −1 0   x 2  +  4  u; = 
x(t) =
Ax ( t ) , x ( 0) x0
    0 0 −2   x   0 
 x3    3  
 
 x1 
y = [111]  x 2 
 x 3 

© Copyright Reserved by Gateflix.in No part of this material should be copied or reproduced without permission
1  e− t 
For x 0 =   , Χ ( t ) =  − t  and for Q.24 The state equation and the output
 −1   −e  equation of a control system are
0  e− t e −2t  given below:
x0 =   , x ( t ) =  −t  ,
1   −e +2e −2t   −4 −1.5   2
= x  x + u
3  4 0  0
When x 0 =   , x(t) is
5 y = [1.5 0.625] x
 −8e − t 11e −2t  Then transfer function
a)  − t  representation of the system is
 8e 22e −2t 
3s + 5
 11e − t −8e −2t  a) 2
b)   s + 4s + 6
−t
 −11e +16e −2t  3s − 1.875
b) 2
 3e − t −5e −2t  s + 4s + 6
c)  −t  4s + 1.5
 −3e +10e −2t  c) 2
s + 4s + 6
 5e − t −3e −2t  6s + 5
d)  −t  d) 2
 −5e +6e −2t  s + 4s + 6
[GATE-2014] [GATE-2018]

Q.22 The state transition matrix ɸ(t) of a


 x 1  0 1   x1 
system   =   
 x 2  0 0   x 2 

t 1 1 0 
a)  b) 
1 0  
 t 1
0 1 1 t 
c)  d) 
1 t  0 1

[GATE-2014]

Q.23 A second-order linear time-


invariant system is described by the
following state equations
d
- x1 ( t ) + 2 x1 ( t ) =3u ( t )
dt
d
- x2 ( t ) + x2 ( t ) = u (t)
dt
where x 1 (t)and x 2 (t) are the two
state variables and u(t) denotes the
input. If the output c(t) = x 1 (t), then
the system is
a) controllable but not observable
b) observable but not controllable
c) both controllable and observable
d) neither controllable nor observable
[GATE-2016]

© Copyright Reserved by Gateflix.in No part of this material should be copied or reproduced without permission
ANSWER KEY:
1 2 3 4 5 6 7 8 9 10 11 12 13 14 15
(d) (c) (d) (b) (b) (a) (a) (a) (d) (d) (c) (b) (c) (b) (d)
16 17 18 19 20 21 22 23 24
(a) (a) (b) (c) (a) (b) (d) (a) (a)

© Copyright Reserved by Gateflix.in No part of this material should be copied or reproduced without permission
EXPLANATIONS

Q.3
(d)
Q.1 (d)  x 1   −3 −1  x1   1 
x ( t ) =
−2x ( t ) + 2u ( t ) …(i) = x   2 0   x  +  0  u
 2   2  
y ( t ) = 0.5x(t) …(ii)
From (i), Taking Laplace transform  x1   1 
of (i) =y [10]  +  u
 x 2  0
sX ( s ) = −2X ( s ) + 2U(s)
Qc = [B AB]
X(s)[s+2]=2U(s)
2U(s)  − 3
⇒ X (s) = AB =  
(s + 2) 2
Taking Laplace transform of (ii)  1 − 3
Y ( s ) = 0.5X(s) ∴=Qc  ≠0
0 2 
0.5 × 2U(s) ∴ Controllable
Y (s) =
s+2 Q0 =  CT A T CT 
Y (s) 1
∴ =  1  T  −3 2 
U(s) (s + 2) =CT = ,A  −1 0 
0  
Q.2 (c)  − 3
A T CT =  
 s 0  1 0   −1
( sl − A=)  0 s  − 1 1 
     1 − 3
∴=Q0  ≠0
s − 1 0   0 −1
= 
 −1 s − 1 ∴ observable
s − 1 0 
 +1 s − 1 Q.4 (b)
 
( sl − A ) =
−1
 s 0 1 0
( s − 1)
2
[ sl − A=] 0 s  − 0 1
   
 1  s − 1 0 
 s −1 0 
[ sl − A ] =
 0 s − 1
=   
 +1 1 
 ( s − 1) 2 s − 1  =
e At [ sl − A ]−1

 1 
 et 0 s −1 0 
L −1
[ sl − A ] −1
=e = t
At
 = =
et
 
0

 te et  et 
 0 1  0
x ( t ) = e At  x ( t o )   s − 1 
 et 0  1  et 
= = t t   t
 te e   0   te  Q.5 (b)
Eigen values of A = [λ]
Eigen values of W = [ μ ]

© Copyright Reserved by Gateflix.in No part of this material should be copied or reproduced without permission
The eigen value of a system are ω (s) 10
⇒ =
always unique U(s) ( s + 11s + 11)
2

So, [ λ ] = [ μ ]
But a system can be represented by
Q.8 (a)
different state models having
Sum of the Eigen value =Trace of the
different set of variables.
principle diagonal matrix
X=W
Sum = −3. Only potion (a) satisfies
X≠W
both conditions.
Both are possible conditions.
Q.9 (d)
Multiplication of the eigen
Q.6 (a)
value=determinant of the matrix
Φ=( t ) L−1[sl − A]−1 Therefore from options it seems
−1
  s 0  0 1 
−1
determinant should be ± 2. Only
= L  −  option (d) satisfies as det=2
  0 s   −1 0  
−1
−1s −1 Q.11 (c)
=L  
1 s  1 0
A= 
 s 1  0 1
 s + 1 s + 1  D 
2 2
p
= L−1   B=  
 −1 s   =
s 2 + 1 q 
 s 2 + 1 s 2 + 1  For controllability condition is
 cos t sin t  Qc = [B, AB, ….A n −1B] ≠ 0
= 
 − sin t cos t  1 0  p 
AB = 
0 1   q 
Q.7
(a)
 p + 0 p
 dω  = =  q 
 dt   −1 1   ω   0  0 + q   
= di    +  u
 a   −1 −10   i a  10  p p
=
So, Qc =
q 
0
 dt  q
dω So, the system is uncontrollable for
⇒ =− ω + i a …(i)
dt all values of p and q.
di
⇒ a =− ω − 10i a + 10u …(ii) Q.12 (b)
dt
Taking Laplace transform (i) &(ii)
Q.13 (c)
⇒ sω ( s ) =
− ω ( s ) + Ia (s)
Forward path gain,
⇒ ( s + 1) ω ( s ) = Ia (s) …(iii)  1 1 1
⇒ sIa ( s ) = − ω ( s ) − 10Ia ( s ) + 10U(s)
= p1 2=     ( 0.5 )
 s  s  s2
⇒ ω ( s ) =( −10 − s ) Ia ( s ) + 10U(s) 1 1
= p 2 2=   (1) ( 0.5 )
=( −10 − s )( s + 1) ω ( s ) + 10U(s) s s
=− s 2 + 11s + 10  ω ( s ) + 10U(s) ∆1 = 1
( )
⇒ [s 2 + 11s + 11]ω s + 10U(s) ∆2 = 1
 1 1
∆ = 1 − − − 2 
 s s 

© Copyright Reserved by Gateflix.in No part of this material should be copied or reproduced without permission
1 1 Q.15 (d)
=1 + + 2
s s 0 a1 0
Transfer function of the system, A =  0 0 a 2  ,
Y(s) P1∆1 + P2 ∆ 2  a 3 0 
= 0
U(s) ∆
0
1 1
+
s +1 B =  0 
= = s2 s
 1 
1+ + 2 s + s + 1
2
1 1
C = [1 0 0]
T
s s
For system to be controllable, the
Q.14 (b)
metric Qc must be nonsingular.
Y1 (s) 1 Y2 (s) 2
= =
U(s) s + 2 U(s) s + 2 Qc =  B AB A 2 B
Y1 ( s ) X1 (s) 1 AB = [ 0 a 2 0]
T
=
X1 (s)U(s) s+2  0 0 a1a 2 
Y2 ( s ) X 2 (s) 2 A =  a 2 a 3
2
0 0 
=
X 2 (s)U(s) s + 2  0 a1a 3 0 
X1 (s) 1 Y (s)
= = A 2 B = [ a1a 2 0 0]
2
and 1 1
U(s) s + 2 X1 (s)
X 2 (s) 1 Y (s) 0 0 a1a 2 
=
U(s) s + 2
= and 2
U(s)
2 Qc =  0 a 2 0 
sX1 ( s ) + 2X1 ( s ) = U(s)  1 0 0 
And, Y1 ( s ) = X1 ( s ) Qc = − a1a 22
sX 2 ( s ) + 2X 2 ( s ) =
U(s) For Qc to be nonsingular
And, Y2 ( s ) = 2X 2 ( s ) Qc ≠ 0
x 1 ( t ) + 2x 2 ( t ) =
U(t) ∴ a1 ≠ 0 and a 2 ≠ 0 and a 3R
And, y1 ( t ) = x1 (t)
Q.16 (a)
x 2 ( t ) + 2x 2 ( t ) =
U(t)
And, y 2 ( t ) = 2x 2 (t)
From Questions
1
[ y1y=
2] [12 ]1
T
=y =  2 So, x 1 =− x1 − u
 
x 2 =− ( x 2 + x 1 ) =− (x 2 − x1 − u)
x 1 ( t ) =
−2x1 ( t ) = U(t)
x 2 = x1 − x 2 + u
x 2 ( t ) =
−2x 2 ( t ) + U(t)
y = x 2
 x 1   −2 0   x1  1
= x   −2 0   x  + 1 u(t) y = x1 − x 2 + u
 2   2  
 x 1   −1 0   x1   −1
Or x =[ −2] x + [1] u = x   1 −1  x  +  1  u
Only option (b) is satisfied  2   2  
 −1 0   −1
=x   X+  u
 1 −1 1

© Copyright Reserved by Gateflix.in No part of this material should be copied or reproduced without permission
Q.17 (a) Q.19 (c)
 −1 0  Solution of state equation of
A=  X (t) = L-1 [SI ‒A-1] X (0)
 1 −1
1  −1 0 
s + 1 0  X ( 0) =   A =  
sl − A =  −1 s + 1  −1  0 −2 
  −1
S + 1 0 
[ sl − 1] [SI − A ] =
−1 −1
 0 S + 2
1 s+1 0  
= (s+1)×(s+1) � �  + 0 
1 s+1 =
1 S 2
 1
0 
 (S + 1) (S + 2)  0 S + 1
 s +1
[ sl − 1]−1 =
   1 
 0 
 1 1  S +1
[SI − A ] =
−1
 ( s + 1) 2 s + 1   
  0 1 
Φ (=
t ) e=
At
L−1 {( s − l ) }
−1
 S + 2 
 −1  1  
 e− t 0 L  S + 1 0 
e =  −t
At
  
L [SI − A ] =  
−1 −1
 te e− t 
  1 
 0 L−1  
Q.18 (b)   S + 2  
From the given state model, e− t 0 
L ( SI − A ) =
−1  
−1

 −1 1 0  0    0 e −2t 
 
   4  c = 111
A =  0 −1 0  B =   [ ]  X1 (t)  e −t
0  1 
 0 0 −2   0   X (t)  =  −2t   
 2   0 e   −1
Controllable: Q c = c= [B AB A2B]
 X1 (t)   −e t 
if |Qc|≠0 controllable  X (t)  =  −2t 
 0 4 −8  2   −e 
= Qc = 4 −4 4  ⇒| Qc 0 X1 ( t ) = e − t

 0 0 0  X 2 ( t ) = −e −2t
∴ uncontrollable
Q.20 (a)
 C 
From the given signal flow graph,
Observable : Q0 =  CA 

the  state model is
CA 2 
 X1   0 1 0   X1   0 
If | Qc|   
≠ 0 → observable
=   1   X 2  +  0  u
X2   0 0
1 1 0  X 3   a 3 a 2 a1   X 3  1 
=Qc = 
−1 0 −2  ⇒   Qc 1  X1 
 1 −1 4  Y = [ C1 C2 C3 ]  X 2 
∴ Observable.  X 3 
The system is uncontrollable and
observable 0 1 0
A =  0 0 1 
a 3 a 2 a1 

© Copyright Reserved by Gateflix.in No part of this material should be copied or reproduced without permission
−1
0   s −1
−1 1  s 1
[SI − A] =  = ⇒ 2
=B =0  ;C [C1 C2 C3 ] 0 s 
 s 0 s 
1  1 1 2
−1  s s 
Controllability: φ (t) = L
0 1 
Qc =  B AB A 2 B  s
0 0 1  1 t 
φ (t) = 
Qc = 0 1 a1  0 1

1 a1 a 2 + a1 
2
Q.23 (a)
|Qc =| 1 ≠ 0 The set of equation of the system
are
Observability dx (t)
 C  → 1 + 2x1 (t ) =⇒ 3u ( t )
Q0   CA  ⇒
dt
x1 (t ) =
  −2x1 (t ) + 0 x2 ( t) + 3u ( t )
CA 2 
dx 2 (t)
 C1 C1 C3  + x 2=(t) u ( t ) ⇒
  dt
a 3c3 c1 + a 2c3 c2 + a1c3
  x 2 (t ) = 0 x1 (t ) − x 2 (t) + u ( t )
c2a 3 + c3 (a1a 3 ) a 2c2 + c3 (a1a 2 + a 3 ) c1 + a1c2 + c3 (a12 + a 2 ) 
C ( t ) = x1 (t) + 0x 2 (t)
| Q0 | ⇒
→ we can frame the n state of the
depends on a1 , a 2 , a 3 & c1 & c 2 &c3 system as
It is always controllable  x1  -2 0   x1  3
 x  =  0 -1  x  + 1 4
Q.21 (b)  2   2  
Apply linearity principle, x 
Y = [ 0 1 ]  1 
 3 1 0  x2 
=5 a  −1 + b 1  s=
a 3;=b 8
       −2 0 
 e− t   e− t → A matrix is  
−e −2t   0 −1
⇒ x ( t ) = 3  −t  +  −t 
 −e   −e +2e −2t   3
B matrix is  
 11e − t −8e −2t  1
⇒ x ( t ) =  −t 
 −11e +16e −2t  C matrix is [1 0]
Q.22 (d) → for control ability determinant of
Given state model, B AB ≠ 0
 X1 (t)  0 1   X1 (t)  3 −6 
 X (t)  = 0 0   X (t)  1 −1 =−3 + 6 ≠ 0 so controllable
 2    2   
0 1  → For observability determinant of
A =  C
0 0  C  ≠ 0
state transition matrix Ca 
φ (t) ⇒ L−1 ( SI − A ) 
−1  1 0
  → =0 so not observable
 −2 0 
→ Final controllable but not
observable

Q.24 (a)

© Copyright Reserved by Gateflix.in No part of this material should be copied or reproduced without permission
 3s 5 
=T (s)  2 + 2
From the given state space  s + 4s + 6 s + 4s + 6 
representation of the system, we can
 3s 
find matrices as, T ( s ) 1x1 =  2
 −4 −1.5   2  s + 4s + 6 1x1
(A) =   ( B) =  
 4 0  0
[C] = [1.5 0.625]
We can find the transfer function
using
T ( s ) C ( sI − A ) .B
−1
= (i)
 
 s 0   −4 −1.5 
[sI − A=]  − 
0 s   4 0 
 s + 4 1.5 
= 
 −4 s 
sI − A = s ( s + 4 ) − ( −4 ) ×1.5 = s 2 + 4s + 6
 s −1.5 
Adj[sI − A ] =  
4 s + 4
Obtained by interchanging principle
diagonal elements and changing signs
of other elements
Adj[sI − A ]
Hence, [sI − A ] =
−1

sI − A
 s −1.5 
 s 2 + 4s + 6 s 2 + 4s + 6 
= 
 4 s+4 
 2 
 s + 4s + 6 s 2 + 4s + 6 
 s −1.5 
 s + 4s + 6 s + 4s + 6   2 
2 2
[sI − A ] .B =
−1
  
 4 s + 4 0
 2 
 s + 4s + 6 s 2 + 4s + 6 
 2s 
 s + 4s + 6 
2
[sI − A ] .B =
−1
 
 8 
 2 
 s + 4s + 6 
Substituting values of [sI − A ] .B and
−1

C in equation (i),
 2s 
 s 2 + 4s + 6 
T ( s ) = [1.5 0.625]  
 8 
 2 
 s + 4s + 6 

© Copyright Reserved by Gateflix.in No part of this material should be copied or reproduced without permission
GATE QUESTIONS(EE)

Q.1 Given the homogeneous state-space b) The system is uncontrollable and


 −3 1  unstable.
equation X =   x the steady c) The system is controllable and
 0 −2 
stable.
state value of X ss = lim x(t) , given d) The system is uncontrollable and
x →∞

the initial state value of stable.


t ) [10 − 10] , is
x (=
T [GATE-2002]

0   −3 Q.5 A second order system starts with


a) X ss =   b) X ss =  
0   −2  2
an initial condition of   without
 −10  ∞  3
c) X ss =   d) X ss =  
 10  ∞  any external input. The state
[GATE-2001] transition matrix for the system is
e −2t 0 
given by  −t 
. The state of the
Q.2 The state transition matrix for the
 0 e 
system X = AX with initial state is system at the end of 1 second is
a) ( s − A )
−1
given by
b) e At X(O) 0.271 0.135
a)   b)  
1.100  0.368
c) Laplace inverse of ( s − A ) 
−1

   0.271 0.135
d) Laplace inverse of c)   d)  
0.736  1.100 
( s − A ) X(O) 
−1
[GATE-2003]
 
[GATE-2002]
Q.6 The following equation defines a
separately excited dc motor in the
2 0 1
Q.3 =
For the system X   X +   u; form of a differential equation
0 4 1 d 2 ω B dω K 2 K
+ + ω = Va
y = [ 4 0] X with u as unit impulse dt 2
J dt LJ LJ
and with zero initial state, the The above equation may be
output y becomes organized in the state-space form as
a) 2e 2t b) 4e 2t follows
c) 2e 4t d) 4e 4t  d 2ω 
 2  dω 
[GATE-2002] = dt  P  dt  + QVa
 dω   
 dt  ω
 2 3 1 
Q.4 =
For the system X   X+ u,
 0 5 0  where the P matrix is given by
which of the following statement is  B K2   K2 B
 – −   − − 
true? a) J LJ  b) LJ J
 
a) The system is controllable but  1 0   0 1 
unstable.

© Copyright Reserved by Gateflix.in No part of this material should be copied or reproduced without permission
 0 1   1 0   t − e −3t   t − e −3t 
c) X ( t ) =  −3t  d) X ( t ) =  − t 
c)  K 2 B
 
d) B K2 

 3e   e 
− –  – −
 LJ J   J LJ  [GATE-2005]
[GATE-2003]
Q.10 For a system with the transfer
Q.7 The state variable description of a 3(s − 2)
function H ( s ) = 2 , the
linear autonomous system is, X=AX, s + 4s 2 − 2s + 1
where X is the two dimensional matrix A in the state space form
state vector and A is the system =X Ax + Bu is equal to
0 2 1 0 0 0 1 0
matrix given by A =   . The
2 0 
a)  0 1 0   b)  0 0 1 
roots of the characteristic equation  −1 2 −4   −1 2 −4 
are
a) −2 and + 2 b) − j2 and + j2 0 1 0  1 0 0
c) −2 and − 2 d) +2 and + 2 c)  3 −2 1  d)  0 0 1 
[GATE-2004] 1 −2 4   −1 2 −4 
[GATE-2006]
Statement for common data
question Q.8 and Q.9:
Statement for Linked Answer Questions
A state variable system
Q.11 and Q.12:
s 1  1 
X(t) =   X ( t ) +   U(t), The sate space equation of a described by
0 −3 0 x=Ax+Bu, y= Cx where x is where x is state
With initial condition vector, u is input, y is output and
X ( 0 ) = [ −1 3] and the unit step
T
0 1  0 
A= =  B =  C [1 1]
input u(t) has 0 −2  1 

Q.8 The state transition matrix Q.11 The transfer function G(s) of this
  system will be
a)  1
1
(1 − e −3t )  s s +1

3
 a) b)
0 e −3t 
(s + 2) s(s − 2)
s 1
 1 − t −3t  c) d)
b)  1
3
( e − e ) (s − 2) s(s+2)
  [GATE-2008]
0 e− t 
 1 − t −3t 
c)  1
3
( e − e ) Q.12 A unity feedback is provided to the
above system G(s) to make it a
 
0 e −3t  closed loop system as shown in
figure.
1
d) 
(1 − e )−t

0 e− t 
[GATE-2005]
For a unit step input r(t), the steady
Q.9 The state transition equation state error in the input will be
 t − e− t   t − e− t  a)0 b) 1
a) X ( t ) =  − t  b) X ( t ) =  −3t  c)2 d) ∞
 e   3e  [GATE-2008]

© Copyright Reserved by Gateflix.in No part of this material should be copied or reproduced without permission
Statement for common data Question Where y is the output and u is the
Q.13 and Q.14: input. The system is controllable for
a) a1 ≠ 0, a 2 =0, a 3 ≠ 0
A system is described by the following state b) a1 =0, a 2 ≠ 0, a 3 ≠ 0
and output equations
dx1 (t) c) a1 =0, a 2 ≠ 0, a 3 =0
= − 3x1 (t) + x 2 ( t ) + 2u(t)
dt d) a1 ≠ 0, a 2 ≠ 0, a 3 =0
dx 2 (t) [GATE-2012]
=−2x 2 ( t ) + u(t)
dt
Common Data for Questions Q.17 and
y(t) = x1 (t) ,when u(t) is the input and y(t) is
Q.18
the output.
The state variable formulation of a system
is given as
Q.13 The system transfer function is
X   −2 0   x1  1
  x  + 1 u, x1 ( 0 )
s+2 s+3 = 1
a) 2 b) 2   
s +5s-6 s +5s+6  2
X  0 −1 2  
2s+5 2s-5 x 
c) 2 d) 2 = 0,= x2 ( 0 ) 0 And y = [1 0]  1 
s +5s+6 s +5s-6  x2 
[GATE-2009]
Q.17 The system is
Q.14 The state-transition matrix of the
a) Controllable but not observable
above system is
b) Not controllable but observable
 e-3t 0  e-3t e-2t -e-3t  c) Both controllable and observable
a)  -2t -3t  b)  
e +e e-2t  0 e-2t  d) Both not controllable and not
e-3t e-2t +e-3t  e3t e-2t -e-3t  observable
c)   d)   [GATE-2013]
 0 e-2t  0 e-2t 
[GATE-2009] Q.18 The response y(t) to a unit step
input is
Q.15 The system = X& AX + BU and 1 1 1 1
a) - e-2t b) 1- e-2t - e-t
 −1 2  0 2 2 2 2
= A =  , B   is
 0 2 1  c)  e -e
-2t -t
d) 1-e -t

a) stable and controllable [GATE-2013]


b) stable but uncontrollable
c) unstable but controllable Q.19 The state transition matrix for the
d) unstable and uncontrollable system
[GATE-2010]  x 1  1 0   x1  1
=  x  1 1   x  + 1 u is
 2   2  
Q.16 The state variable description of an
e 0 
t
 et 0
LTI system is given by a)  t b)
t  2 t t
X& 0 a
1 1 0   X1   0  e e  t e e 
&        et 0 e t te t 
=  X2   0 0 a 2   X2  +  0  u c)  t d) 
t t 
X&       − te e  0 e 
 3   a 3 0 0   X3   1 
[GATE-2014]
 X1 
 
y = (1 0 0 )  X 2  Q.20 Consider the system described by
X  following state space equations
 3

© Copyright Reserved by Gateflix.in No part of this material should be copied or reproduced without permission
 x 1   0 1   x1  0 
= x   −1 −1  x  + 1  u;
 2   2  
x 
y = [1, 0]  1 
x2 
If μ is unit step input, then the
steady state error of the system is
a) 0 b) 1/2
c) 2/3 d) 1
[GATE-2014]
a) When u1 is the only input and y1
Q.21 In the signal flow diagram given in is the only output
the figure, u1 and u2 are possible b) When u2 is the only input and y1
inputs whereas y 1 and y 2 are is the only output
possible outputs. When would the c) When u1 is the only input and y2
SISO system derived from this
is the only output
diagram be controllable and
d) When u2 is the only input and y2
observable?
is the only output
[GATE-2015]

ANSWER KEY:
1 2 3 4 5 6 7 8 9 10 11 12 13 14
(a) (c) (b) (b) (a) (a) (a) (a) (c) (b) (d) (a) (c) (b)
15 16 17 18 19 20 21
(c) (d) (a) (a) (c) (a) (b)

© Copyright Reserved by Gateflix.in No part of this material should be copied or reproduced without permission
EXPLANATIONS

Q.1 (a) t
−1  e 2( t − τ )   e 2t 
s − A  = ∫= 4( t − τ )  .δ ( t ) dt  4t 
 s + 3 −1 
−1  e  e 
0
=
 0 s + 2 
 1 1  Output y(t) = [ 4 0] .x(t)
s + 3 ( s + 2 )( s + 3)   e 2t 
= = [ 4 0] .  4t 
 1  e 
 0
 ( s + 2 )  Hence, y ( t ) = 4e 2t
−1
e At= −1 s − A 
Q.4 (b)
 e −3t e −2t − e −3t  1 2
  = = 0
 0 e −2t  Qc
0 0
x ( t ) = e At X(O) ⇒ Uncontrollable. Characteristic
 e −3t e −2t − e −3t   10  equation:
   s −A =
 0 e −2t   −10  0
 20e −3t−10e −2t  s−2 −3
∴ x (t) =
  =0
−2t s−5
 −10e
0

lim 0 s 2 − 7s + 13 = 0

= X ss x (t)  
= ⇒ eigen values,
t→∞ 0 =
s 3.5 ± j0.866
i.e. roots lies on right side of s-plane.
Q.2 (c) ⇒ unstable.
−1
e At= −1 s − A 
Q.5 (a)
Q.3 (b) State transition matrix
 e −2t 0 
t φ( t) =  −t 
.
X ( t ) e At X ( O ) + ∫ e A( t − τ ) BU(τ)dτ
=  0 e 
 2
0 Initial conditions, x ( O ) =  
∴ x (t) =
0(given)  3
Zero input response is given by
∴ x (t) =
X ( t ) = φ ( t ) x ( O)
O
t
 e 2(t − τ) 0  1  e −2t 0   2   2e −2t 
+∫ = = −t     −t 
4(t − τ)   
. .δ(t)dt
 0 e  1  0 e   3   3e 
0 State of the system at 𝑡𝑡 = 1 𝑠𝑠
 2e −2   0.271
x=( t ) t =1 = −1   
 3e  1.100 

© Copyright Reserved by Gateflix.in No part of this material should be copied or reproduced without permission
0 1  1
Q.6 (a) = =  &B  
dω  0 − 3 0
=
Let x1 = and x 2 ω  s 0 0 1 
dt s =
− A   − 
x 2 = x1  0 s   0 − 3
d 2 ω B dω K 2 K  s −1 
+ + ω= Va = 
dt 2
J dt LJ LJ  s s + 3
d 2ω B dω K 2 K s + 3 1
= − − ω + Va  0
dt 2
J dt LJ ω −1  s 
s − A  =
s ( s + 3) − 0x(−1)
2
B K K
⇒ x 1 = − x1 − x 2 + Va
J LJ LJ 1 1 
x 2 = x1 s
s(s + 3) 
= 
 x 1   x1   1 
=
 x  P  x  + QVa  0
 2  2 s + 3 
Where 1 1  1 1 
  − 
 B K2  K −1 s 3  s s + 3 
− − s − A  = 
=P = 
LJ  and Q  LJ 
 
 J   1
 1 0  0  0 s+3 
State transition matrix
Q.7 (a) = −1 ( s − A ) 
−1

 0 2  
System a matrix= A=    
 2 0
φ (t) =  1
1
3
(1 − e −3t ) 
 s 0  0 2  
s −=A  −  0 e −3t 
0 s   2 0
 s −2  Q.9 (c)
= 
 −2 s  ZIR (zeroinput response)
Characteristic equation  1 
 1 (1 − e −3t )   −1
= φ (t) × X(O) =
⇒ s −A =   3 
0 3
 −3t
s −2  0 e 
=0  −1 + 1 − e −3t 
−2 s = 
−3t
⇒ s2 − 4 = 0  3e 
Roots of the characteristic equation  −e −3t
=  −3t 
are -2 and +2.  3e 
ZSR (zero state response)
Q.8 (a)
= −1 ( s − A ) BU(s) 
−1
0 1  1  
=x ( t )   x ( t ) +  0  V(t) …(i)
 0 − 3   1 1  
  s s(s + 3)   1  1 
x ( t ) Ax ( t ) + Bu(t)
= …(ii)
= −1     
Comparing eq. (i) and (ii), we get  0 1  0 s 
A=System matrix 
  s + 3  

© Copyright Reserved by Gateflix.in No part of this material should be copied or reproduced without permission
−1 1/ s 
2
t Q.11 (d)
= =     X Ax + Bu
=
 0  0
and y = cx
∴State transition equation
where
 − e −3t   t 
=ZIR + ZSR = −3t  +   0 1  0
 3e   0  = A =  ,B  ;
 0 −2  1
 t − e −3t 
=  −3t  = C [1= 0] &D 0
 3e 
 s 0 0 1 
S =− A   − 
 0 s   0 −2 
Q.10 (b)  s −1 
= 
Y (s) 3(s − 2) 0 s + 2
=
H (s) = 2
U ( s ) s + 4s 2 − 2s + 1 −1 1 s + 2 1
Y ( s ) X1 ( s ) s − A  =s(s + 2)  0 s 
. 
X1 ( s ) U ( s ) Transfer function
−1
 1  = C s − A  B + D
= 3(s − 2)  3 
 s + 4s − 2s + 1 
2
s + 2 1  0 
X (s) 1  0 s   1 
Let 1 = 3
U(s) s + 4s − 2s + 1
2 = [1 0] 
s(s + 2)
S3X1 ( s ) + 4s 2 x1 ( s ) − 2sx1 ( s ) + x1 ( s ) =
u(s) 1
d s 
Replacing s by 0]  
1
dt = [1=
s(s + 2) s(s + 2)
d 3 x1 d 2 x1 dx1
+4 2 −2 + x1 = u(t) …(i)
dt 2 dt dt Q.12 (a)
dx1 1
Let = x= 2 x 1 G (s) =
dt s(s + 2)
H (s) = 1
2
d x1 and
= x= 2 x3
dt r (t) = u (t)
Replacing eq. (i)
1
x 3 + 4x 3 − 2x 2 + x1 = u(t) ⇒ R (s) =
s
x 3 = − x1 + 2x 2 − 4x 3 + u(t)
R(s)
x 1 = x 2 = E=
Error (s)
1 + G ( s ) H(s)
x 2 = x 3 1/ s
x 3 = − x1 + 2x 2 − 4x 3 + u(t) =
1
1+
 X 1   0 1 0   X1   0  s(s + 2)
       s+2
= X 2   0 0 1   X 2  +  0  u(t) ⇒ E (s) =
 X 3   −1 2 −4   X 3   1  s ( s + 2) + 1
 
Steady state error, using final value
0 1 0
theorem
So, A =  0 0 1  Lt
 −1 2 −4  ess = sE(s)
s→0

© Copyright Reserved by Gateflix.in No part of this material should be copied or reproduced without permission
Lt s(s + 2) 2s + 5 2s + 5
= = 0 = = 2
s → 0 s ( s + 2) + 1 ( s + 2 )( s + 3) s + 5s + 6

Q.13 (c) Q.14 (b)


Selecting X1 (t) and X 2 (t) as state s + 2 1 
 0 s + 3
variables. 
dx ( t )
[ sI − A ]−1 =
X 1 ( t ) = 1
( s + 2 )( s + 3)
dt  1 1 
= −3x1 ( t ) + x 2 ( t ) + 2u(t)  s + 3 ( s + ) (s + 3) 
dx 2 (t) = 
X 2 ( t ) = =−2x 2 ( t ) + u(t)  1 
dt  0 s + 2 

 X 1 ( t ) 
   1 1 1 
X2 ( t ) s + 3 s + 2 − s + 3
= 
 − 3 1   x1 ( t )   2   0 1 
=    +   u(t)
 0 −2   x 2 ( t )   1   s+2 
State transition matrix
X AX + BU
=
= −1 ( sI − A ) 
−1
 −3 1   2  
=
So, A =  ,B  
 0 −2  1  1 1 1 

s + 3 s + 2 s + 3
y ( t ) = x1 (t) = −1  
x (t)  0 1 
y ( t ) = [1 0]  1   s+2 
x2 ( t )
 e −3t e −2t − e −3t 
=
y CX + DU = 
 0 e −2t 
=
So, C [1=
0] &D 0
 s 0   −3 1  Q.15 (c)
s =
− A   − 
 0 s   0 −2   −1 2  0
=A =  &B 1
 s + 3 −1   0 2  
=
 0 s + 2   s 0   −1 2 
s + 2 1  [ sI − A=]   −  
0 s   0 2
 0 s + 3
s − A  =  s + 1 −2 
−1

( s + 2 )( s + 3) = 
 0 s − 2
Transfer function s − 2 2 
−1
= C s − A  B + D  0 s + 1

[ sI − A ] =
−1
…(i)
s + 2 1  ( s + 1)( s − 2 )
 0 s + 3  2 
 −1
[ ] +0 Transfer function = C s − A  B
( s + 2 )( s + 3)  1 
1 0
So denominator of eq. (i) gives
Poles of the system
 2 ( s + 2 ) + 1 ( s + 1)( s − 2 ) =
[1 0]  
0
=  s+3  s = −1 & 2
( s + 2 )( s + 3) One pole lies in RHS of s-plane.

© Copyright Reserved by Gateflix.in No part of this material should be copied or reproduced without permission
Hence, the so, system is unstable.  −2 0  1  −2 
For controllability, is Qc defined as =A =    
 0 −1 1  −1
Qc = [ B : AB] For controllability, B : AB ≠ 0
 −1 2   0   2  1 −2
=AB =     or = −1
 0 2 1  2 1 −1
Qc = [ B : AB] − ( −2 ) = 1 ≠ 0
0 2 The system is controllable.
= 
1 2 1
CT =  
Qc ≠ 0 0
Hence the system is controllable.  −2 0   1   −2 
=
A T CT =    
 0 −1  0   0 
Q.16 (d)
For observability, CT : A T CT ≠ 0
0 a1 0
A =  0 0 a 2  , or
1 −2
=0
 a 3 0 0  0 0
The system is not observable.
0
B =  0  Q.18 (a)
 1   s 0   −2 0 
S −=A    −  
C = [1 0 0]  0 s   0 −1
T

For system to be controllable, the s + 2 0 


= 
metric Qc must be nonsingular.  0 s + 1
Qc =  B AB A 2 B  1 0 
s − A  =s + 2 1 

−1
AB = [ 0 a 2 0]
T
 
 0 s + 1
 0 0 a1a 2 
[ B]
−1
A =  a 2 a 3
2
0 0  S − A 
 0 a1a 3 0   1 
 1 0  s + 2
1
A 2 B = [ a1a 2 0 0] = =s+2 1    
2

  1  1 
0 0 a1a 2   0 s + 1
 s + 1 
Qc =  0 a 2 0 
C s − A  [ B]
−1

 1 0 0 
 1 
Qc = − a1a 22 s + 2 1
= [1= 0]  
For Qc to be nonsingular  1  s+2
Qc ≠ 0  s + 1 
∴ a1 ≠ 0 and a 2 ≠ 0 and a 3R G (s) =
1
s+2
Q.17 (a) Y (s) 1
=
 −2 0  1 X (s) s + 2
=A =  ,B  
 0 −1 1

© Copyright Reserved by Gateflix.in No part of this material should be copied or reproduced without permission
1 1 1 1  1
=
Y(s) = − ess =
s(s + 2) 2  s s + 2 
 1+ ∞
ess = 0
Y (=t)
1
2
(1 − e −2t )
1 1 −2t Q.21 (b)
= − e
2 2

Q.19 (c)
1 0  1
Give A =  B = 1
1 1  
−1
 s 0   1 0  
[SI − A ]= 
−1
− 
 0 s   1 1  
 1 
 0 
(s − 1)
[SI − A ] =
−1
 
 1 1 
 (s − 1) 2 (s − 1) 
 
The state transition matrix
e At L−1 [SI − A ]
−1
=
 et 0
e At =  t 
 te et 

Q.20 (a)
Transfer function ⟹ C [SI — A1]−1.B
−1
S −1  0 
= [1 0]    
1 (s + 1)  1 
1
Transfer function = 2
s + s +1
G(s) 1
= 2
1 + G(s) s + s + 1
1
⇒ G (s ) = 2
s +s
Steady state error for unit step
A
ess =
1+ Kp
1
ess =
1 + lim G(s)
s →0

1
ess =
1
1 + lim
s →0 s + s
2

© Copyright Reserved by Gateflix.in No part of this material should be copied or reproduced without permission
GATE QUESTIONS(IN)

Q.1 The state-variable representation of where


a plant is given by 0 1 1 
x =Ax + Bu, y =Cx F=   ;G=   ;H [1 0]
-4 -2  0 
Where the state, u is the input and y Here, x is the state vector, u is the
is the output. Assuming zero initial input, and y is the output. The
conditions, the impulse response of
damping ratio of the system is
the plant is given by a)0.25 b)0.5
a) exp(At) c)1 d)2
b) ∫ exp[A(t − τ)]Bu ( τ ) dτ [GATE-2009]
c) C exp ( At ) B
Q.4 The transfer function of the system
d) C ∫ exp[A(t − τ)]Bu ( τ ) dτ described by the state- space
[GATE-2006] equations.
 x 1  -4 -1  x1  1
Q.2 The state space representation of a  x  =  -3 -1  x  + 1 u,y = [1 0]
 2   2  
0 1 
system is given by x=    X+  x1 
0 -3  x  is
 2
1 
0  u,y = [1 0] x. The transfer a) 2
s
b)
2s
  s +5s+1 2
s +5s+1
Y(s) 3s 4s
function of the system will be c) 2 d) 2
U(s) s +5s+1 s +5s+1
1 1 [GATE-2011]
a) b)
S s(s+3)
1 1 Q.5 A system is represented in state-
c) d) 2 space as=X AX + Bu, , where = A
(s+3) S
[GATE-2008] 1 2 1
α 6  and B = 1 . The value of α
  
Q.3 A linear time invariant single –input for which the system is not
single output system has state space controllable is ______.
middle given by [GATE-2015]
dx
= Fx + Gu; y =Hx
dt

ANSWER KEY:
1 2 3 4 5
(d) (a) (b) (a) -3

© Copyright Reserved by Gateflix.in No part of this material should be copied or reproduced without permission
EXPLANATIONS

1 s + 2 −1  1
Q.1 (d) = [1 0]
x 0 ( t ) L−1 (C(SI − A) −1 B.U(S))
= ( S2 + 5S + 1)  −3 S + 4 1
C ( e At *Bu(t) )
⇒ x0 ( t ) = =
1

x 0 ( t ) =C ∫ e A(t-τ) .Bu ( τ ) dτ
( S + 5S + 1)
2

Q.5 (-3)
Q.2 (a) For a system to be uncontrollable,
G (s) =
C ( SI − A ) B; ( SI − A )
−1
its controllability determinant
S −1  should be equal to zero.
=
0 ( s + 3)  Qc =|BAB|=0
1 2  1   3 
−1 1 S + 3 1  AB=     =  
⇒ (SI − A) = α 6  2×2  +1 2×1 α+6 
S(S + 3)  0 3
1 3
1  S + 3 1   1   Qc = BAB → = 0
G (s) =  [1 0]   1 α+6
S ( S + 3)   0 S  0  
⇒ α + 6 − 3 = 0 ⇒ α = −3
1   S + 3 
=  [1 0]  
S ( S + 3)   0 
1
⇒ G (s) =
S

Q.3 (b)
 3 −1 
( SI − A ) =
 
 4 S + 2
⇒ ( SI − A )
−1

1  (s + 2) 1 
=
( s + 2s + 4 )  −4 S
2

1 ( s + 2 ) 1   0 
G ( s ) = [1 0]
( S + 2S + 4 )  −4 S 1
2

1 1 1
= [1 0]  
( S + 2S + 4 )
2
S ( S + 2S + 4 )
2

1
⇒ζ=
2

Q.4 (a)
T ( s ) =C ( SI − A ) B + D
−1

© Copyright Reserved by Gateflix.in No part of this material should be copied or reproduced without permission
7 CONTROLLERS

7.1 INTRODUCTION

In control theory a controller is a device


which monitors and physically alters the
operating conditions of a given dynamic
system. Typical applications of controllers
are to hold settings for temperature,
pressure, flow or speed. The following six
basic control actions are very common
among industrial analog controllers: 7.2.1 DISADVANTAGE
1) Two-position or on-off
2) Proportional An inadequacy in this way of control is that
3) Proportional-plus integral control signal oscillates which may cause
4) Proportional-plus derivative control variable to oscillate around desired
5) Proportional plus integral plus value. Sometimes there is no remedy for
derivative this problem.
e.g. if level of liquid in tank is controlled
7.2 TWO-POSITION OR ON-OFF using valve with only two possible states
CONTROLLERS (open or closed) the level will always
oscillates around desired value.
An on-off controller is the simplest form of
temperature control device. The output 7.3 PROPORTIONAL CONTROLLER
from the device is either on or off, with no
Proportional action is the simplest and
middle state. An on-off controller will
most commonly encountered of all
switch the output only when the
continuous control modes. In this type of
temperature crosses the set point. For
action, the controller produces an output
heating control, the output is on when the
signal which is proportional to the error.
temperature is below the set point, and off
Hence, the greater the magnitude of the
above set point. Two-position or on-off
error, the larger is the corrective action
control is relatively simple and inexpensive
applied.
and, for this reason, is very widely used in
P controller control algorithm is given as:
both industrial and domestic control
systems.  U max for e ( t ) > eo

On-off controller algorithm is defined as: u ( t ) = u o + K p e ( t ) for − eo < e ( t ) < eo
 U if e ( t ) > 0 ( ON state )  U for e ( t ) < −eo

u ( t ) =  max
min

 U min if e ( t ) < 0 ( OFF state ) Where:


Where: uo – Amplitude of control signal when
e (t) – control error (for unit feedback) control error is equal 0
Kp – P controller gain for P mode nominal
u (t) – control signal (controller output). area e(t)<|eo |

© Copyright Reserved by Gateflix.in No part of this material should be copied or reproduced without permission
the error in the system. An integral control
is sometimes called reset control. The
transfer function of the controller is

Many industrial controllers have defined a


proportional band (PB) instead of gain:
100
PB = % M (s ) Ki
KP =
E (s) s
It should be noted that for K p = 1 a t
proportional band is equal PB = 100%. and m(t) = K i ∫ e ( t ) dt
P controller can eliminate forced −∞

oscillations caused by use of on-off


controller. However, a second problem 7.4.1 EFFECTS
arises. There exists now a steady state
error. A relationship between control 1) It increases type and order by ‘1’
signal and error inside area e(t) <| eo | is 2) Integral controller improves the steady
state response
given as:
3) Steady state error reduces
u ( t=
) u o + K pe ( t ) 4) Makes the system lesser stable
Steady state error is then: 5) Speed of response reduces
u ( t ) − uo
e(t) =
Kp 7.5 DERIVATIVE CONTROLLERS
1
i.e. e ( t ) ∝ In derivative controller the signal driving
Kp the controlled system is derived by
Proportional controller can stabilize only differentiating the error in the system. The
1st order unstable process. In general it can transfer function of the controller is
be said that P controller cannot stabilize
higher order processes. Changing
controller gain K can change closed loop
dynamics. A large controller gain will result
in control system with:
a) smaller steady state error, i.e. better
M (s) de ( t )
reference following = K Ds and m(t) = K D
b) faster dynamics, i.e. broader signal E (s) dt
frequency band of the closed loop The o/p of controller can be predicted on
system and larger sensitivity with the basis of knowledge of the slope of error.
respect to measuring noise Therefore a derivative controller is
c) smaller gain and phase margin anticipatory in nature giving output in
advance.

7.4 INTEGRAL CONTROLLER 7.5.1 EFFECTS


In integral controller the signal driving the
1) It improves transient response
controlled system is derived by integrating
2) Not used in isolation.

© Copyright Reserved by Gateflix.in No part of this material should be copied or reproduced without permission
3) Adding derivative action can restrict the M (s) K
overshoots on controlled responses. = KP + i
E (s) s
t
7.6 PD CONTROLLER m(t) K P e ( t ) + K i ∫ e ( t ) dt
and=
−∞
A controller in the forward path, which
changes the controller output 7.7.1 EFFECTS
corresponding to proportional plus
derivative of the error signal is called PD 1) It increases the order & type of the
controller. system
2) Reduces steady state error
3) Bandwidth is reduced

7.8 PID CONTROLLERS

The combination of proportional control


action, integral control action, and
The transfer function of a PD controller is
derivative control action is termed
M (s)
= K P + K Ds and proportional – plus – integral – plus –
E (s) derivative control action. This combined
de ( t ) action has the advantages of each of the
m ( t ) K Pe ( t ) + K D
= three individual control actions.
dt

7.6.1 EFFECTS

1) Transient response is improved


2) It increases damping ratio
3) Type of the system remains unchanged
The equation of a controller with this
4) It reduces peak overshoot
5) It reduces settling time combined action is given by
6) Bandwidth increases K
t
de ( t )
m(t) = K P e ( t ) + P ∫ e ( t ) dt + K P TD
Ti −∞ dt
7.7 PI CONTROLLER Or the transfer function is
U (s) 1
A controller in the forward path, which = K P (1 + + TDs)
E (s) Tis
changes the controller output
corresponding to the proportional plus Where, KP is the proportional gain
integral of the error signal is called PI Ti =
1
is the integral time
controller. Ki
TD = K D is the derivative time

7.8.1 EFFECTS

It improves both steady state as well as


transient response.
The transfer function of a PI controller is

© Copyright Reserved by Gateflix.in No part of this material should be copied or reproduced without permission
8 COMPENSATORS

8.1 INTRODUCTION

Because of the prolonged use of the system


the parameters of the system can change
and output of the system may start
deviating from the desired value. The
system can be either replaced by a new
system or it can be provided with a unit
called compensators.
The compensators are used to improve the 8.2.1 BODE PLOT FOR LEAD
performance of the system during runtime. COMPENSATOR
In this chapter, we shall discuss electric
network realization of basic compensators
and their frequency characteristics.

8.2 LEAD COMPENSATOR

The circuit shows a typical lead derivative


compensator.

• For drawing Bode plot the corner


The designation lead applied to this
frequencies are ωc1 = 1/ T & ωc2= 1/ αT .
network is based on the steady-state
sinusoidal response. The sinusoidal • The maximum phase lead occurs at a
response E2 with a sinusoidal input E1 frequency ωm which is geometric mean of
the two corner frequencies.
(Initial conditions are considered to be
zero) is 1 1 1
= ωm = .
s + 1/ R1C αT T T α
E2 (s ) = E1 (s)
s + ( R 2 + R 1 ) / R 1R 2 C • The maximum phase lead at frequency ωm
E 2 ( s ) s + 1/ T 1 − α 
is given by ∅ m =sin −1 
∴ =
E1 ( s ) s + 1/ αT 1 + α 
• A phase lead compensator shifts the gain
Where, T = R1C= and α R 2 / ( R 2 + R1 ) < 1 crossover frequency to higher values
Now, a lead compensator has a zero at where the desired phase margin is
s = −1/ T & pole at s = −1/ αT . acceptable therefore it is effective when the
slope of the uncompensated system near
the gain crossover frequency is low.
8.3
8.3

© Copyright Reserved by Gateflix.in No part of this material should be copied or reproduced without permission
8.3 LAG COMPENSATOR • The maximum phase lead at frequency
1 − β 
The circuit shown is a typical lag or integral ωm is given by ∅ m = sin −1  
compensator. 1 + β 
• A phase lag compensator shifts the gain
crossover frequency to lower values
where the desired phase margin is
acceptable therefore it is effective when
the slope of the uncompensated system
near the gain crossover frequency is
high.
The output signal is proportional to the
sum of the input signal and its integral. The
designation lag applied to this network is
based on the steady-state sinusoidal
response. The sinusoidal response 𝐸𝐸2 with
a sinusoidal input 𝐸𝐸1 (Initial conditions are
considered to be zero) is
1 + R 2 Cs
E2 (s) = E1 (s)
1 + ( R 2 + R1 ) Cs
E2 (s)  R 2  s + 1/ R 2 C
∴ = 
E1 ( s )  R1 + R 2  s + 1/ ( R 2 + R1 ) C
1 s + 1/ T
= .
β s + 1/ βT
8.4 LAG-LEAD COMPENSATOR
Where, T = R 2 C and= β ( R 2 + R1 ) / R 2 > 1
Now, the lag compensator has zero at A lag-lead compensator is a combination of
s = −1/ T & pole at s = −1/ βT . a lag compensator and a lead compensator.
The lag section has one real pole and one
real zero with the pole to the right of zero.
The lead section also has one real pole and
one real zero but the zero is to be the right
of the pole. The general form of this
compensator is

8.3.1 BODE PLOT FOR LAG


COMPENSATOR

• For drawing Bode plot the corner


frequencies are ωc1 = 1/ βT & ωc2 = 1/ T .

The maximum phase lag occurs at a The transfer function of the lag-lead
frequency ωm which is geometric mean compensator from the figure is
of the two corner frequencies. E2 (s) (1 + T1s)(1 + T2s)
=
1 1 1 E1 ( s ) (1 + αT1s)(1 + βT2s)
=ωm = .
βT T T β Where, T1 = R1C1 , T2 = R 2 C2
β
= ( R 2 + R1 ) / R 2

© Copyright Reserved by Gateflix.in No part of this material should be copied or reproduced without permission
=α R 2 / ( R 2 + R1 )
The pole zero plot is

8.4.1 BODE PLOT LAG-LEAD


COMPENSATOR

Note:
1) Lag-lead network introduces both
steady state and transient response
improvement of the system.
2) For lead lag compensator the position
of compensating poles and zeros of lag
lead compensator can be interchanged.

© Copyright Reserved by Gateflix.in No part of this material should be copied or reproduced without permission
GATE QUESTIONS(EC)

Q.1 A PD controller is used to


compensate a system. Compared to
the uncompensated system, the
compensated system has
a) a higher type number
b) reduced damping
c) higher noise amplification
d) larger transient overshoot
[GATE-2003]
=
a) K P =
100, K D 0.09
K =
b) K P =
Q.2 A double integrator plant, G(s) = 2 , 100, K D 0.9
s =
c) KP =
10, K D 0.09
H(s) = 1 is to be compensated to
achieve the damping ratio ζ =0.5, =
d) KP =
10, K D 0.9
and an undamped natural frequency, [GATE-2007]
ωn =5rad / s. . Which one of the
Q.5 The open-loop transfer function of a
following compensator G e (s) will be 1
suitable? plant is given as G(s) = 2 . If the
s −1
s+3 s + 9.9 plant is operated in a unity feedback
a) b)
s + 9.9 s+3 configuration, then the lead
s−6 s+6 compensator that can stabilize this
c) d)
s + 8.33 s control system is
[GATE-2005] 10(s − 1) 10(s + 4)
a) b)
s+2 s+2
Q.3 The transfer function of a phase-
10(s + 2) 10(s + 4)
lead compensator is given by c) d)
1 + 3Ts s + 10 (s + 1)
G e (s) = where T > 0 [GATE-2007]
1 + Ts
The maximum phase-shift provided
by such a compensator is Q.6 Group I gives two possible choices
a) π / 2 b) π / 3 for the impedance Z in the diagram.
c) π / 4 d) π / 6 The circuit elements in Z satisfy the
[GATE-2006] condition R 2 C2 > R1C1. The transfer
V
function 0 represents a king of
Q.4 A control system with a PD Vi
controller is shown in the figure. If controller. Match the impedances in
the velocity error constant K V = 1000 Group I with the types of controllers
& the damping ratio ζ =0.5, , then in Group II.
the values of K P and K D are

© Copyright Reserved by Gateflix.in No part of this material should be copied or reproduced without permission
s+2
b) G c (s) =
s +1
(s + 1)(s + 4)
Group I c) G c (s) =
(s + 2)(s + 3)
2
d) G c (s) =1 + + 3s
s
[GATE-2010]

Statement of linked answer Q.9 and Q.10


The transfer function of a compensator is
s+a
Group II given as G c (s) =
1. PID controller s+b
2. Lead compensator Q.9 G c (s) is a lead compensator if
3. Lag compensator a) a=1,b=2 b) a=3,b=2
a) Q-1, R-2 b) Q-1, R-3 c) a = -3,b = -1 d) a = 3,b = 1
c) Q-2, R-3 d) Q-3, R-2 [GATE-2012]
[GATE-2008] Q.10 The phase of the above lead
compensator is maximum at
Q.7 The magnitude plot of a rational a) 2rad / s b) 3rad / s
transfer function G(s) with real
c) 6rad / s d) 1/ 3rad / s
coefficients is shown below. Which
[GATE-2012]
of the following compensators has
such a magnitude plot?
Q.11 A lead compensator network
includes a parallel combination of R
and C in the feed-forward path. If
the transfer function of the
compensator is
s+2
a) Lead compensator G c (s) = , the value of RC is ____
s+4
b) Lag compensator [GATE-2015]
c) PID compensator
d) Lead-lag compensator Q.12 The transfer function of a first order
[GATE-2009] controller is given as
K(s + a )
Q.8 A unity negative feedback closed G c ( s ) 
= where, K, a and b
s+b
loop system has a plant with the
are positive real numbers. The
1
transfer function G(s) = 2 condition for this controller to act as
s + 2s + 2 a phase lead compensator is
and a controller G c (s) in the feed a) a < b b) a > b
forward path. For a unit step input, c) K < ab d) K > ab
the transfer function of the [GATE-2015]
controller that gives minimum
steady state error is Q.13 Which of the following can be the
s +1 pole-zero configuration of a phase-
a) G c (s) = lag controller (lag compensator)?
s+2

© Copyright Reserved by Gateflix.in No part of this material should be copied or reproduced without permission
[GATE-2017-01]

Q.14 Which of the following statements is


incorrect?
a) Lead compensator is used to
reduce the setting time.
b) Lag compensator is used to
reduce the steady state error.
c) Lead compensator may increase
the order of a system.
d) Lag compensator always
stabilizes an unstable system.
[GATE-2017-02]

ANSWER KEY:
1 2 3 4 5 6 7 8 9 10 11 12 13 14
(c) (a) (d) (b) (c) (b) (d) (d) (a) (a) (0.5) (a) (a) (d)

© Copyright Reserved by Gateflix.in No part of this material should be copied or reproduced without permission
EXPLANATIONS

Q.1 (c) 3[1 + (Tω) 2 ] =1 + (3Tω) 2


B.W. increases and SNR decreases. 3 + 3T 2 ω2 = 1 + 9T 2 ω2
System becomes more prone to
2= 6(ωT) 2
noise.
1
(ωT) 2 =
Q.2 (a) 3
1
ωT =
3
1 1
= tan −1 3 ×
φmax − tan −1
3 3
π π π
= − =
3 6 6
ξ =0.5
cos −1 0.5= 60° Q.4 (b)
∴ θ= 60° K v = lim sG(s)H(s)
s →0

∠G(s) =
K
v (K P + K Ds)100
= lim s ×
1000
s2 s=
−2.5 + 4.33 j
s →0 s(s + 10)
4.33 ⇒ K P = 100
= −2 tan −1 ;120°
−2.5 Now characteristics eq.
∴ For compensated system (1 + G(s)H(s)) = 0
=∠ 180 − 120;60° (K + K Ds)100
1+ P =
0
(b) & (d) are lag network & for s(s + 10)
compensating lag network K / s 2 , a Putting K P = 100
lead network is required s 2 + 10s + 104 + 100K Ds =
0
∴ Putting s = −2.5 + 4.33 j in (a)
gives s 2 + (10 + 100K D )s + 104 =
0
K(s + 3) 0.5 + j4.33 Comparing with standard second
= = 53°;60° order eq.
s (s + 9.9) 7.4 + j4.33
2

∴ (a) is the correct answer. i.e. s 2 + 2ξωn s + ω2n = 0


So ωn = 100; 2ξωn = 10 + 100K D
Q.3 (d) =
Given ξ 0.5; 2 × 0.5 ×100
Max phase shift = 10 + 100K D
φm =∠G e (s)
K D = 0.9
=φ tan −1 3ωT − tan −1ωT
For maximum phase shift Q.5 (c)
dφ Lead compensator is required for
=0
dt stability.
3T T
⇒ =
1 + (3Tω) 1 + (Tω) 2
2

© Copyright Reserved by Gateflix.in No part of this material should be copied or reproduced without permission
Q.6 (b)
Vi (R1C1s + 1)1 V Q.9 (a)
= − 0
s+a
R1 Z Gc (s) =
V Z(R1C1s + 1) s+b
⇒ 0= − jω + a
Vi R1 G c ( jω ) =
R C s +1 jω + b
In case of Q, Z = 2 2 ω ω
c ( jω )
C 2s + 1 ∠G= tan −1 − tan −1
R2 a b
In case of R, Z = ω ω
R 2 C 2s + 1  − 
Considering Q, = tan −1  a b2 
(R C s + 1) (R 2 C2s + 1)  1+ ω 
V0
= − 1 1  
Vi R1
.
C 2s  ab 
For G c ( s ) to be a lead compensator
Considering R,
V0 (R C s + 1) R2 ∠G c ( jω ) > 0
= − 1 1 .
Vi R1 (R 2 C2s + 1) ω ω
>
Q Given that R 2 C 2 > R 1C1 a b
∴ Considering R, controller is log ⇒ b>a
compensator. Option (a) satisfies the above
and considering Q, Controller is PID equation.
controller
Q.10 (a)
Q.7 (d) For phase to be maximum

=0
Q.8 (d) dt
Steady state error,  ω2   1 1 
sR(s) ⇒ 1 +  − 
ess = lim  2  a b 
s → 0 1 + G(s)G (s)
c
 ω ω   2ω 
r(t) = u(t) − −  =0
 a b   ab 
1
R(s) =  ω2   1 1   ω ω   2ω 
s 1 +  −  =  −  
 2 1 2   1 2  2 
1
s. 1 ω2 ω2
ess = lim s + =
s → 0 1+G(s)G (s)
c
2 4 2
1 ω 2
1
ess = lim ⇒ =
s → 0 1 + G(s)G (s) 4 2
c

s +1 2 ⇒ ω = 2rad / sec.
Taking,= G c (s) = , ess
s+2 3
Q.11 (0.5)
s+2 1
Taking,= G c (s) = , ess s+2
s +1 3 G C (s) = … (i)
(s + 1)(s + 4) 3 s+4
=
Taking, G c (s) = , ess For lead compensator
(s + 2)(s + 3) 5
2
Taking, G c (s) =1 + + 3s, ess =0
s

© Copyright Reserved by Gateflix.in No part of this material should be copied or reproduced without permission
1 + sτ
Transfer function = … (ii)
1 + α sτ
Where,
τ = Lead time constant = R1C
R2
and α =
R1 + R 2
Comparing equation (i) and (ii), we
get
1 1
τ = and ατ =
2 4
1
or α =
2
∴ RC time constant = 0.5

Q.12 (a)
For phase lead compensator

© Copyright Reserved by Gateflix.in No part of this material should be copied or reproduced without permission
GATE QUESTIONS(EE)

Q.1 A lead compensator used for a a) C1 is lead compensator and C2 is


closed loop controller has the a lag compensator
 s b) C1 is lag compensator and C2 is
K 1 + 
following transfer function 
a a lead compensator
 s c) Both C1 and C2 are lead
1 + 
 b compensator
For such a lead compensator d) Both C1 and C2 are lag
a) a < b b) b < a
compensator
c) a > Kb d) a < Kb
[GATE-2008]
[GATE-2003]

Q.2 The system 900 / s(s + 1)(s + 9) is to


Q.4 The second order dynamic system
be such that its gain-crossover dX
frequency becomes same as its = PX + Qu , Y= RX has the
uncompensated phase crossover dt
matrices P, Q and R as follows:
frequency and provides a 45° phase
 −1 1  0 
margin. To achieve this, one may use P= =
 Q =  R [01] The
a) a lag compensator that provides  0 −3  1 
an attenuation of 20dBand a system has the following
phase lag of 45° at the frequency controllability and observability
of 3 3 rad/s properties:
a) Controllable and observable
b) a lead compensator that
b) Not controllable but observable
provides an amplification of
c) Controllable but not observable
20dBand a phase lead of 45° at d) Not controllable and not
the frequency of 3 rad/s observable
c) a lag-lead compensator that [GATE-2014]
provides an amplification of Q.5 For the network shown in the figure
20dB and a phase lag of 45° at below, the frequency (in rad/s) at
the frequency of 3 rad/s which the maximum phase lag
d) a lag-lead compensator that occurs is. ___.
provides an attenuation of 20dB
and a phase lead of 45° at the
frequency of 3 rad/s
[GATE-2007]

Q.3 The transfer function of two


compensators are given below: [GATE-2016]
10 ( s+1) s+10
C1 = ,C2 = Which one
( s+10 ) 10 ( s+1)
of the following statements is
correct?

© Copyright Reserved by Gateflix.in No part of this material should be copied or reproduced without permission
ANSWER KEY:
1 2 3 4 5
(a) (d) (a) (c) 0.316

© Copyright Reserved by Gateflix.in No part of this material should be copied or reproduced without permission
EXPLANATIONS

Q.1 (a) (ω )
 s
K 1 + 
(ω ) pc 1 +
pc 1

9
= tan −1
Transfer function 
a
(ω )
2

 s 1−
pc 1
1 +  9
 b
Zero of TF = − a
(ω )
2
Pole of TF = − b pc 1
For a lead-compensator, the zero is ⇒ 1− =0
9
nearer to origin as compared to
pole, hence the effect of zero is ⇒ (ω ) pc 1 = 3rad / sec
dominant, therefore, the lead- Gain cross frequency of
compensator when introduced in
series with forward path of the
compensated system, ωpc ( ) 2

transfer function the phase shift is Phase cross frequency of


increased. uncompensated system, ωpc ( ) 1
So, from pole-zero configuration of
the compensator a < b ⇒ = ( ) (=
ωgc ω )
2 gc 1 3rad / sec
Phase-margin
Q.2 (d)
Let uncompensated system = 180° + ∠ T ( jω ) ω= ω
( gc ) 2
900
T (s) = 45° 180° + ∠ T ( jω ) ω=
⇒=
s ( s + 1)( s + 9 ) ( ωgc ) 2
Phase crossover frequency of At (ω ) 3rad
= ,
( )
gc 2
uncompensated system = ωpc , at sec
phase angle of∠ T ( jω ) is −135° and
this frequency phase of T ( jω ) is − 180°
phase of uncompensated system is
Put s = jω inT(s) −180° at 3rad / sec Therefore, the
900 compensator provides phase lead of
T ( jω ) =
jω ( jω + 1)( jω + 9 ) 45° at the frequency of 3 rad/sec.
 ω Let XdB is the gain provided by the
∠ T ( jω ) =
−90° − tan −1 ω − tan −1   compensator, so at gain cross
9
frequency.
at ( ω ) , ∠ T ( jω ) = 180
pc 1
°
T ( jω ) com = 1 or 0 dB.
−180° = −90 − tan ( ω )
° −1
pc 1 Gain of uncompensated system
 (ω ) 
100
T ( jω ) un − com =
− tan −1   − 90
pc 1 °
 ω
2

 9  ω 1+ ω 1+  
2

  9
T ( jω ) un − com in dB

40 − 20log ω − 20 log 1 + ω2
=

© Copyright Reserved by Gateflix.in No part of this material should be copied or reproduced without permission
 ω
2

−20 log 1 +  
9
Gain of compensated system
T ( jω ) com= X + T ( jω ) un − com
T ( jω ) com must be zero at gain cross
frequency (ω ) gc 2
As pole is closer origin, pole
dominates zero.
(
T jωgc ) 2 com
= X + 40 − 20 log ωgc( ) 2
Hence C2 is lag compensator.
For phase to be maximum
( )
2
−20 log 1 + ωgc
2
Q.4 (c)
(ω )
2
gc  −1 1  0
−20 log 1+ =
0 =
Given P =  Q  
 0 −3
2
9 1 
X + 40 − 20 log 3 − 20 log 1 + 32 For controllability:
0 1 
=Qc [Q PQ] ⇒ 
2
 3 
−20 log 1 +   = 0 1 −3
9
Qc ≠ 0 ∴ controllable
X = −20dB
So, the compensator provides an For observability
attenuation of 20 dB. 0 0 
= Q0  R T P T R T  ⇒  
Hence option (d) is correct. 1 −3
Q0 = 0
Q.3 (a)
10 ( s + 1) ∴ Not observable
C1 =
( s + 10 ) Q.5 (0.316)
zero at s = -1 The given circuit is standard lag
pole at s = -10 compensator
Whose Transfer function
1
1+
s +1 1 + τs
G (s) = s =
1 1 + 10s 1 + ατs
a +1+
s
So τ = 1,α = 10 the frequency at
As zero is closer origin, zero which maximum phase lag happen
dominates pole. 1 1
Hence C1 is lead compensator. ω= = = 0.316 rad / sec
τ α
m
10
s + 10
C2 =
10 ( s + 1)
zero at s=-10
pole at s=-1

© Copyright Reserved by Gateflix.in No part of this material should be copied or reproduced without permission
GATE QUESTIONS(IN)

Q.1 The transfer function of a position Q.3 Suppose a disturbance signal


servo system is given as d ( t ) = sin 0.2t units is applied .Then at
1
G (s) = . A first order steady state, the amplitude of the
s(s + 1) output e(t) due to the effect of
compensator is designed in a unity disturbance alone is
feedback configuration so that the a) 0.129 unit b) 0.40 unit
poles of the compensated system c) 0.529 unit d) 2.102 unit
are placed at −1 ± j1 and -4. The [GATE-2006]
transfer function of the Q.4 The control action recommended
compensated system is for reducing the effect of
s+3 2s + 3 disturbance at the output (provided
a) b)
2(s + 5) s+5 that the disturbance signal is
5(s + 1.6) 3(2s + 3) measurable) is
c) d)
s+5 s+4 a) cascade control
[GATE-2006] b) P-D control
c) ratio control
Common Data Questions Q.2, Q.3 and d)feedback –feed forward control
Q.4: [GATE-2006]
The following figure describes the block Q.5 A plant with a transfer function
diagram of a closed loop process control 2
system. The unit of time is given in minute is controlled by a PI controller
s(s+3)
Q.2 The digital implementation of the with K p =1 and K i ≥ 0 in a unity
controller with a sampling time of feedback configuration .The lowest
0.1 minute using velocity algorithm is value of K i that ensures zero steady
state error for a step change in the
reference input is
a) 0 b) 1/3
c) 1/2 d) 1
[GATE-2009]

 k
 Q.6 Consider the control system shown
a) m ( k ) 0.5 e ( k ) + 0.5∑e(k) 
= in figure with feed forward action
 i =1  for rejection of a measurable
 k −1
 disturbance d(t). The value of K for
b) m ( k ) = 2.0 e ( k ) + 2.0∑e(k − 1) 
 i =1  which the disturbance response at
the output y(t) is zero mean, is
c) m ( k ) − m (=
k − 1) 0.5 e ( k ) − 0.85e ( k − 1) 
d) m ( k ) −=
m ( k − 1) 0.5 1.05e ( k ) − e ( k − 1) 
[GATE-2006]

© Copyright Reserved by Gateflix.in No part of this material should be copied or reproduced without permission
a) 1 b) −1
c) 2 d) −2
[GATE-2014]

© Copyright Reserved by Gateflix.in No part of this material should be copied or reproduced without permission
ANSWER KEY:
k

1 2 3 4 5 6
(c) (a) (b) (b) (a) (d)

EXPLANATIONS

Q.1 (c)
Let the T.F of the compensator be
G c (s) As, Laplace transform of
CLTF of the compensated system I(s)
Gc (s) ∫i ( t ) dt = s
G (s) Gc (s) s ( s + 1) G c (s)
= = =
1 + G (s) Gc (s) G ( s ) s(s + 1)G c (s) Q.3 (b)
1+ c
s ( s + 1)
Poles of the compensated system re
given as s =−1 ± j1,s =−4 The given diagram can be reduced
as
K (s + b)
Let G c ( s ) = c
(s + a)
Characteristic equation
s ( s + 1)( s + a ) + K c ( s + b )
= ( s + 1) + 12  ( s + 4 ) s3 + (1 + a ) s 2
2
 
+ ( a + Kc ) s + Kcb = (s 2
+ 2s + 2 ) ( s + 4 )
= s3 + 6s 2 + 10s + 81 + a Transfer function
= 6, ( a + K c ) = 10, K c b = 8 H=
C(s) E(s)
(s) = =
1
D(s) D(s) 0.5e −5
8 1+
a= 5, K c= 5, b= = 1.6 s
5
E(s) s 2s
5(s + 1.6) = =
Gc (s) = D(s) 0.5 + 0.5s (s + 1)
(s + 5)
2s
E (s) = D(s)
Q.2 (a) ( s + 1)
Given ( t ) sin 0.2t ⇒=
d= ω 0.2
  1  For getting multiplying factor A
=m ( s ) e(s)  0.5  1 +  
  2s   2s
A=
 0.5  s + 1 ω= 0.2
= 0.5  e ( s ) + e (s)
 s 

© Copyright Reserved by Gateflix.in No part of this material should be copied or reproduced without permission
2 jω 2 × 0.2
=A = = 0.3922
jω + 1 1 + ( 0.2 )
2

A=0.40
So, amplitude of output e(t)
= A × initial amplitude
= 0.4 × 1= 0.4

Q.4 (b)
(PD controller can be used)

Q.5 (a)
E (s) 1
=
R (s)  ki   2 
1 + 1 +   
 s   s ( s + 3) 
S2 (s + 3)
= 2
S ( s + 3) + 2(s + ki)
1 s 2 ( s + 3)
ess = lim s.E ( s ) lim s. . 2
S→ 0 S→ 0 s s ( s + 3) + 2 ( s + ki )
=0∀ki ≥ 0

Q.6 (d)
 50  s + 2 + K  50
Y ( s ) 1 + =  D (s)   + R(s)
 (s + 2)   s + 2  (s + 2)
 K.d ( s ) + 50 ( R ( s ) − Y ( S) ) 
1
Y (s) D (s) +
(s + 2) 
s + 2+ K  50
 s + 2  + ( s + 52) R ( s )
Y ( s ) D ( s ) 

i.e., s+2+K=0
⇒K+2= 0
⇒K= −2

© Copyright Reserved by Gateflix.in No part of this material should be copied or reproduced without permission
ASSIGNMENT QUESTIONS

Q.1 A closed-loop system is shown in


the given figure. The noise transfer
Cn ( s )
function [Cn ( s ) = output
N (s)
corresponding to noise input N(s) is The system is described as
approximately
a) M d y12( t ) + B d y1 ( t ) =k  y 2 ( t ) − y1 ( t ) 
2

dt dt
b) M d y 22( t ) + B d y 2 ( t ) =k  y 2 ( t ) − y1 ( t ) 
2

dt dt
c) M d y12( t ) + B dy1 ( t ) =k  y1 ( t ) − y 2 ( t ) 
2

dt dt
d 2 y2 ( t ) dy 2 ( t )
d) M +B =k  y1 ( t ) − y 2 ( t ) 
dt 2 dt
1
a) for | G1 (s)H1 (s) H 2 (s) |<< 1
G(s)H1 (s) Q.4 Consider the following block
diagrams:
b) − 1 for | G1 (s)H1 (s) H 2 (s) |>> 1 a)
H1 (s)

c) − 1
for | G1 (s)H1 (s) H 2 (s) |>> 1
H1 (s)H 2 (s)
d) 1
for | G1 (s)H1 (s) H 2 (s) |<< 1
G(s)H1 (s)H 2 (s)
b)
Q.2 A signal flow graph is shown in the
given figure. The number of forward
paths M and the number of
individual loops P for this signal
flow graph would be

c)

d)
a) M =4 and P = 4 b) M=6 and P=4
c) M=4 and P=6 d) M=6 and P=6

Q.3 The mechanical system is shown in


the given figure

© Copyright Reserved by Gateflix.in No part of this material should be copied or reproduced without permission
Which of these block diagrams can G1G 2 + G 2 G 3
a)
be reduced to transfer function 1 + G1G 2 H1 + G 2 G 3 H1 + G 4
C (s) G1
= ? G1G 2 + G 2 G 3
R ( s ) 1 − G1G 2 b)
1 + G1G 3 H1 + G 2 G 3 H1 − G 4
a) 1 and 3 b) 2 and 4
G1G 3 + G 2 G 3
c) 1 and 4 d) 2 and 3 c)
1 + G1G 3 H1 + G 2 G 3 H1 + G 4
Q.5 Select the correct transfer function G1G 3 + G 2 G 3
d)
v0(s)/vi(s) from the following, for 1 + G1G 3 H1 + G 2 G 3 H1 − G 4
the given network.
C(s)
Q.9 The overall gain of the block
R(s)
diagram shown is

2 s
a) b)
s ( s + 1) (s + 2)
G1G 2
c)
s
d)
2s a)
( 2s + 1) ( s + 1) 1 − G1G 2 H1 H 2
G1G 2
b)
Q.6 The number of forward paths and 1 − G 2 H 2 − G1G 2 H1
the number of non-touching loop G1G 2
pairs for the signal flow graph given c)
1 − G 2 H 2 + G1G 2 H1H 2
in the figure are, respectively,
G1G 2
d)
1 − G1G 2 H1 − G1G 2 H 2

Q.10 The signal flow graph for a certain


a) 1, 3 b) 3, 2 feedback control system is shown:
c) 3, 1 d) 2, 4 Now consider the following set of
equations for the nodes:
Q.7 Which one of the following effects in
the system is NOT caused by
negative feedback?
a) Reduction in gain
b) Increase in bandwidth
c) Increase in distortion
d) Reduction in output impedance 1. x2 = a1x1 + a9x3
2. x3 = a2x2 + a8x4
C(s) 3. x4 = a3x3 + a5x2
Q.8 The gain of the signal flow
R(s) 4. x5 = a4x4 + a6x2
graph shown is Which of the above equations are
correct?
a) 1, 2 and 3 b) 1, 3 and 4
c) 2, 3 and 4 d) 1, 2 and 4

© Copyright Reserved by Gateflix.in No part of this material should be copied or reproduced without permission
Q.11 Which is the overall transfer
function of the block diagram given?

G1G 2 + G 2 G 3 G1G 3 + G 2 G 3
a) b)
1 + G 2 H1 1 + G 3 H1
G1G 2
G G + G 2G 3 a)
c) G1G 2 + G 2 G 3 d) 1 3 1 + H 2 G1G 2 − H1G 2
1 + G 2 G 3 H1
G1G 2
b)
Q.12 For the feedback system shown in 1 − H 2 G1G 2 + H1G 2
the figure above, which one of the H1G1G 2
c)
following expresses the input- 1 − H 2 G1G 2 + H1G 2
output relation C/R of the overall H1G1G 2
system? d)
1 + H 2 G1G 2 − H1G 2

Q.15 The transfer function for the


diagram shown above is given by
which one of the following?
G G
a) b)
1 − FG + GH 1 + FG − GH
FG GH
c) d)
1 + FGH 1 − FGH

Q.13 The block diagram for a particular a) 1/(1 + sRC) b) sRC/(1 + sRC)
control system is shown in the c) sRC/(1 – sRC) d) 1 + sRC
figure. What is the transfer function
C(s)/R(s) for this system? Q.16 For what value of K, are the two
block diagrams as shown above
equivalent?

a) 1 b) 2
s+a s+a
a) b) c) (s + 1) d) (s + 2)
s−b s+b
s−b s+b Q.17 Consider the following statements
c) d)
s+a s−a with regards to signal flow graph:

Q.14 Which one of the following is the


Y(s)
transfer function for the block
X(s)
diagram given?

© Copyright Reserved by Gateflix.in No part of this material should be copied or reproduced without permission
1. The number of loops are 3.
2. The number of loops are 2. Q.20 Consider the following statements:
3. The number of forward paths 1. The effect of feedback is to
are 3. reduce the system error.
C 40 2. Feedback increases the gain of
4. ratio is
R 81 the system in one frequency
C 28 range but decreases in another.
5. ratio is 3. Feedback can cause a system
R 81
that is originally stable to
Which of these statements are
become unstable.
correct?
Which of these statements are
a) 1, 3, 4 and 5 b) 1, 3 and 4
correct?
c) 2, 3 and 4 d) 3, 4 and 5
a) 1, 2 and 3 b) 1 and 2
c) 2 and 3 d) 1 and 3
C(s)
Q.18 The transfer function for the
R(s) Q.21 For the signal flow diagram shown
system shown above is in the given figure, the
transmittance between x2 and x1 is

G + H1 G + H2
a) b)
1 + GH 2 1 + GH1 rsu efh rsu efh
a) + b) +
H2 GH 2 1 − st 1 − fg 1 − fg 1 − st
c) d)
1 + GH 2 1 + GH1 c)
efh
+
rsu
d)
rst
+
rsu
1 − ru 1 − eh 1 − eh 1 − st
Q.19 The system transfer function for the
block diagram shown is Q.22 Consider the following amplifier
with negative feedback:
If the closed-loop gain of the
amplifier is +100, the value B will be

G1G 2
a)
1 − G 2 H 2 + G1H1
G1G 2 a) − 9 ×10 −3 b) + 9 ×10 −3
b)
1 − H1G1 + G 2 H1 c) − 11 × 10 −3 d) + 11 ×10 −3
G1G 2 H1
c)
1 + G 2 H1 + G1H1 Q.23 For the given system, how can the
G1G 2 H1 steady state error produced by step
d) disturbance be reduced?
1 + G 2 H 2 + G1H1

© Copyright Reserved by Gateflix.in No part of this material should be copied or reproduced without permission
s 1
a) b)
( s + 1) s ( s + 1)
2 2

1 1
c) d)
( s + 1)
2
s ( s + 1)
a) By increasing dc gain of G1(s) G2
(s) Q.27 In closed loop control system, what
b) By increasing dc gain G2 (s) is the sensitivity of the gain of the
c) By increasing dc gain of G1 (s) overall system, M to the variation in
d) By removing the feedback G?
1 1
a) b)
Q.24 For the system given below, the 1 + G(s)H(s) 1 + G(s)
feedback does not reduce the G(s) G(s)
closed-loop sensitivity due to c) d)
1 + G(s)H(s) 1 + G(s)
variation of which one of the
following?
Q.28 Which one of the following is the
transfer function of a linear system
whose output is t2e-t for a unit step
input?
a) K b) A s 2s
c) K α d) β a) b)
(s + 1) 3
(s + 1)3
1 2
Q.25 Which one of the following c) 2 d)
represents the linear mathematical s (s + 1) s(s + 1) 2
model of the physical system shown
in the above figure? Q.29 Which of the following are the
characteristics of closed-loop system?
1. It does not compensate for
disturbances
2. It reduces the sensitivity of plant
parameter variations
3. It does not involve output
measurements
d 2 x(t) dx(t)
a) m 2
+b + kx(t) = f (t) 4. It has the ability to control the
dt dt system transient response.
d 2 x(t) dx(t) Select the correct answer using the
b) m +b + kx(t) = 0
dt 2
dt codes given below:
d 2 x(t) dx(t) a) 1 and 4 b) 2 and 4
c) m 2
+b + kx(t) + f (t) =0 c) 1 and 3 d) 2 and 3
dt dt
d 2 x(t) dx(t) Q.30 The unit step response of a
d) m 2
+b − kx(t) − f (t)
dt dt particular control system is given by
c(t)=1-10e-t. Then its transfer
Q.26 A linear time invariant system, function is
initially at rest when subjected to a 10 s -9
unit step input gave to a response a) b)
s +1 s +1
c(t)=te-t (t ≥ 0). the transfer function 1 - 9s 1 - 9s
of the system is c) d)
s +1 s(s + 1)

© Copyright Reserved by Gateflix.in No part of this material should be copied or reproduced without permission
Q.31 [ −a ± jb] are the complex conjugate b) 0.50 and 2 rad/s
roots of the characteristics equation c) 0.25 and 4 rad/s
of a second order system. Its d) 0.50 and 4 rad/s
damping coefficient and natural
frequency will be respectively Q.35 The open loop transfer function of a
b unity feedback system is given by
a) and a 2 + b 2 K
a +b
2 2 . if the value of gain K is such
s ( s + 1)
b
b) and a 2 + b 2 that the system is critically damped,
a +b
2 2
the closed loop poles of the system
a will lie at
c) and a 2 + b 2
a +b
2 2
a) -0.5 and -0.5 b) ± j0.5
a c) 0 and -1 d) 0.5 ± j 0.5
d) and a 2 + b 2
a +b
2 2

Q.36 Given the transfer function


Q.32 A unity feedback control system has 121
G(s)= 2 of a system.
a forward path transfer function s +13.2s+121
10 (1 + 4s ) Which of the following
G (s) = 2 characteristics does it have?
s (1 + s )
a) Overdamped and settling time 1.1
If the system is subjected to an input b) Underdamped and settling time
t2 0.6s
r(t)=1+ t + ( t ≥ 0 ) , the steady-
2 c) Critically damped and settling
state error of the system will be time 0.8s
a) zero b) 0.1 d) Underdamped and settling time
c) 10 d) infinity 0.707s

Q.33 In the system shown in the given Q.37 The steady state error resulting
figure, r(t)=1+2t ( t ≥ 0 ). The steady- from an input r(t) = 2 + 3t + 4t2 for
state value of the error e(t) is equal given system is
to

a) 2.4 b) 4.0
c) Zero d) 3.2
a)Zero b)2/10
c)10/2 d)infinity Q.38 The unit impulse response of a
second order system is 1/6e-0.8t sin
Q.34 A second order control system is (0.6t). Then the natural frequency
defined by the following differential and damping ratio of the system are
equation: respectively?
d 2c ( t ) dc ( t ) a) 1 and 0.6 b) 1 and 0.8
4 + 8 + 16c ( t ) =
16u ( t )
dt 2 dt c) 2 and 0.4 d) 2 and 0.3
The damping ratio and natural Q.39 A second order control system has
frequency for this system are 100
respectively. M(jω) = . It Mp(peak
100 − ω2 + 10 2 jω
a) 0.25 and 2 rad/s
magnitude) is

© Copyright Reserved by Gateflix.in No part of this material should be copied or reproduced without permission
a) 0.5 b) 1 parameter K and parameter a with
c) 2 d) 2 ramp inputs are respectively

Q.40 The open-loop transfer function for


unity feedback system is given by
5(1 + 0.1s ) a) 1, -1 b) -1, 1
s (1 + 5s )(1 + 20 s ) c) 1, 0 d) 0, 1
Consider the following statements: Q.45 The unit impulse response of a
1. The steady-state error for a step system having transfer function
input of magnitude 10 is equal to K
zero. is shown. The value of α is
s+α
2. The steady-state error for a
ramp input of magnitude 10 is 2.
3. The steady-state error for an
acceleration input of magnitude
10 is infinite.
Which of the statements given
above are correct?
a) Only 1 and 2 b) Only 1 and 1
c) Only 2 and 3 d) 1, 2 and 3 a) t1 b)
t1
Q.41 For a second order system, natural c) t 2 d)
1
frequency of oscillation is 10 rad/s t2
and damping ratio is 0.1. What is the
2% settling time? Q.46 Match List I (system G(s)) with List
a) 40s b) 10s II (Nature of Response) and select
c) 0.4s d) 4s the correct answer using the codes
given below the lists:
Q.42 For a unity feedback control system List I (System G(s))
with forward path transfer function 400
A. 2
K s + 12s + 400
G(s) = , what is error transfer 900
s+5 B. 2
function we(s) used for s +90s+900
determination of error coefficients? 225
C. 2
K K s + 30s + 225
a) b)
s+5 s+K+5 625
D. 2
s+5 K(s + 5) s + 0 s + 625
c) d)
s+K +5 s+K+5 List II (Nature of Response)
1. Undamped
Q.43 The unit step response of a system 2. Critically damped
is [1 – e-t (1 + t)] u (t). What is the 3. Underdamped
nature of the system in turn of 4. Overdamped
stability? Codes:
a) Unstable b) Stable A B C D
c) Critically stable d) Oscillatory a) 3 1 2 4
Q.44 Consider the unity feedback system b) 2 4 3 1
as shown. The sensitivity of the c) 3 4 2 1
steady state error to change in d) 2 1 3 4

© Copyright Reserved by Gateflix.in No part of this material should be copied or reproduced without permission
Q.47 Which one of the following
expresses the time at which second
peak in step response occurs for a
a) sin(t − 45° / 2) b) sin(t + 45o ) / 2
second order system?
π 2π c) 2e − t sin t d) sin t − cos t
a) b)
ωn 1 − ς 2 ωn 1 − ς 2
Q.52 A unity feedback control system has
3π π a forward loop transfer function as
c) d)
ωn 1 − ς 2
1− ς 2 e − Ts
. Its phase value will be zero
[s(s + 1)]
Q.48 What is the value of k for a unity at frequency ω1. Which one of the
k following equations should be
feedback system with G(s) = satisfied by ω1?
s(1+s)
a) ω1 = cot (Tω1) b) ω1 = tan (Tω1)
to have a peak overshoot of 50%?
c) Tω1 = cot (ω1) d) Tω1 = tan (ω1
a) 0.53 b) 5.3
c) 0.6 d) 0.047
Q.53 The open loop transfer function of a
unity feedback control system is
Q.49 The unit step response of a second
k
order system is 1 – e-5t – 5t e-5t given by G(s) = . If gain k is
Consider the following statements: s(s + 1)
1. The undamped natural frequency increased to infinity, then damping
5 rad/s. ratio will tend to become
2. The damping ratio is 1. a) zero b) 0.707
3. The impulse response is 25 t e-5t c) Unity d) Infinite
Which of the statements given
Q.54 The impulse response of a second
above is/are correct?
order under-damped system starting
a) Only 1 and 2 b) Only 2 and 3 from rest is given by: C(t) =12.5 e-6t
c) Only 1 and 3 d) 1, 2 and 3 sin8t; t ≥ 0.
What are the values of natural
Q.50 The damping ratio and natural frequency and damping factor of the
frequency of a second order system system, respectively?
are 0.6 and 2 rad/s respectively. a) 10 units and 0.6
Which one of the following b) 10 units and 0.8
combinations gives the correct c) 8 units and 0.6
values of peak and settling time, d) 8 units and 0.8
respectively for the unit step
response of the system? Q.55 Which one of the following is the
a) 3.33s and 1.95s most likely reason for large
b) 1.95s and 3.33s overshoot in a control system?
c) 1.95s and 1.5s a) High gain in a system
d) 1.5s and 1.95s b) Presence of dead time delay in a
system
Q.51 Consider the following system c) High positive correcting torque
shown in the diagram: d) High retarding torque
In the system shown in the below
diagram x(t) = sin t. Q.56 In the time domain analysis of
What will be the response y(t) in the feedback control systems which one
steady state? pair of the following is not correctly
matched?

© Copyright Reserved by Gateflix.in No part of this material should be copied or reproduced without permission
a) Under damped: Minimizes the c) 4 d) 2
effect of nonlinearities
b) Dominant: Transients die Q.61 The open-loop transfer function
out more rapidly G(s) of a unity feedback control
c) Far away poles to the left half of 1
system is . The system is
s-plane: Transients die out more s(s + 1)
rapidly subjected to an input r(t) sin t. The
d) A pole near to the left of steady state error will be
dominant: Magnitude of transient a) zero b) 1
is small complex poles and near
 π   π
a zero c) 2 sin  t −  d) 2 sin  t + 
 4  4
Q.57 A second order system has a natural
frequency of oscillations of 3 Q.62 A third-order system is
rad/sec and damping ratio of 0.5. approximated to an equivalent
What are the value of resonant second order system. The rise time
frequency and resonant peak of the of this approximated lower order
system? system will be
a) 1.5 rad/sec and 1.16 a) Same as original system for any
b) 1.16 rad/sec and 1.5 input.
c) 1.16 rad/sec and 2.1 b) Smaller than the original system
d) 2.1 rad/sec and 1.16 for any input.
Q.58 Consider the following: c) Larger than the original system
1. Rise time for any input
2. Setting time d) Large or smaller depending on
3. Delay time the input
4. Peak time
What is the correct sequence of the Q.63 A system has a single pole at origin.
time domain specifications of a Its impulse response will be
second order system in the a) Constant
ascending order of the values? b) Ramp
a) 2 – 4 – 1 – 3 b) 3 – 4 – 1 – 2 c) Decaying exponential
c) 2 – 1 – 4 – 3 d) 3 – 1 – 4 – 2 d) Oscillatory

Q.59 A unity feedback system with open Q.64 Which one of the following
20 statements is correct?
loop transfer function of is
s(s + 5) A second order system is critically
excited by a unity step input. How damped when the roots of its
much time will be required for the characteristic equation are
response to settle within 2% of final a) Negative, real and unequal
desired value? b) Complex conjugates
a) 0.25 sec b) 1.60 sec c) Negative, real and equal
c) 2.40 sec d) 4.00 sec d) Positive, real and equal

Q.60 Given a unity feedback system with Q.65 An underdamped second order
K system with negative damping will
G(s) = , the value of K for have the two roots
s(s + 4)
a) On the negative real axis as real
damping ratio of 0.5 is roots.
a) 1 b) 16

© Copyright Reserved by Gateflix.in No part of this material should be copied or reproduced without permission
b) On the left hand side of complex a) 2 b) Unity
plane as complex roots.
c) On the right hand side of c) 1/ 2 d) Zero
complex plane as complex
conjugates Q.70 Consider a second order all-pole
d) On the positive real axis as real transfer function model, if the
roots desired settling time (5%) is 0.60
sec and the desired damping ratio
Q.66 How can the steady-state error in a 0.707, where should the poles be
system be reduced? located in s-plane?
a) By decreasing the type of system a) – 5 ± j4 2 b) −5 ± j5
b) By increasing system gain c) −4 ± j5 2 d) −4 ± j7
c) By decreasing the static error
constant Q.71 The closed loop system shown
d) By increasing the input above becomes marginally stable if
the constant K is chosen to be
Q.67 A control system whose step
response is –0.5(1 + e-2t) is cascaded
to another control block whose
impulse response is e-t. What is the
transfer function of the cascaded
a) 10 b) 20
combination?
c) 30 d) 40
1 1
a) b)
(s + 1)(s + 2) s(s + 1) Q.72 The characteristic equation of a
c)
1
d)
0.5 system is given by
s(s + 2) (s + 1)(s + 2) 3s + 10s + 5s + 2 =
4 3 2
0 . This system is
a) Stable
Q.68 A diaphragm type pressure sensor b) Marginally stable
has two poles as shown in the figure c) Unstable
above. The zeros are at infinity. d) Neither (a) (b) nor (c)
What is its steady state deformation
for a unit step input pressure? Q.73 For which of the following values of
k, the feedback system shown in the
figure is stable?

a) k > 0 b) k < 0
c) 0 < k < 42 d) 0 < k < 60

a) 0.25 b) 0.5 Q.74 A feedback control system is shown


c) 0.707 d) 1 in the given figure. The system is
stable for all positive values of K, If
Q.69 What is the value of the damping
ratio of a second order system when
the value of the resonant peak is
unity?

© Copyright Reserved by Gateflix.in No part of this material should be copied or reproduced without permission
a) T = 0 b) T <0 a) 5 rad / s b) 6 rad / s
c) T > 1 d) 0<T<1 c) 5 rad / s d) 6 rad / s
Q.75 Consider the following equation Q.81 The characteristic equation of a
2s4 + s3 + 3s2 + 5s +10 = 0 control system is given as:
How many roots does this equation s4 + 8s3 + 24s2 + 32s + K = 0
have in the right half of s-plane? What is the value of K for which the
a) One b) Two system is unstable?
c) Three d) Four a) 10 b) 20
c) 60 d) 100
Q.76 If the poles of a system lie on the
imaginary axis, the system will be: Q.82 The characteristics equation for a
a) Stable third-order system is:
b) Conditionally stable q(s) = a0s3 + a1s2 + a2s + a3 = 0.
c) Marginally stable For the third-order system to be
d) Unstable stable, besides that all the
coefficients have to be positive,
Q.77 The characteristic equation of a which one of the following has to be
feedback control system in given by: satisfied as a necessary and
s3 + 6s2 + 9s + 4 = 0 sufficient condition?
What is the number of roots in the a) a 0 a1 ≥ a 2 a 3 b) a1a 2 ≥ a 0 a 3
left-half of the s-plane?
a) Three b) Two c) a 2 a 3 ≥ a1a 0 d) a 0 a 3 ≥ a1a 2
c) One d) Zero
Q.83 Which one of the following is
Q.78 The system having characteristic correct?
equation: s4 + 2s3 + 3s2 + 2s + k = 0 A unity feedback system with
is to be used as an oscillator. What forward path transfer function G(s)
are the values of k and the K
= is stable provided
frequency of oscillation ω ? s(1 + sT1 )(1 + sT2 )
a) k = 1 and ω = 1 r/s the value of K is given by
b) k = 1 and ω = 2 r/s T +T TT
c) k = 2 and ω = 1r/s a) K < 1 2 b) K< 1 2
T1T2 T1 + T2
d) k =2 and ω = 2 r/s
T + T2 TT
c) K > 1 d) K> 1 2
Q.79 What is the range of K for which the T1T2 T1 + T2
open loop transfer function G(s) = Q.84 The open-loop transfer function of
K
represents an unstable unity feedback control system is
s (s + a)
2
K
G(s) = ,0 < a ≤ b
closed loop system? s(s + a)(s + b)
a) K > 0 only b) K = 0 only The system is stable if
c) K < 0 only d) - ∞ < K < ∞ (a + b) ab
a) 0 < K < b) 0 < K <
Q.80 The closed loop transfer function of ab (a + b)
K a
a control system is . c) 0 < K < ab(a + b) d) 0 < K < (a + b)
s(s + 1)(s + 5) + K b
What is the frequency of the
sustained oscillations for marginally Q.85 The characteristic equation of a
stable condition? system is given as s3 +25s2 +10s +50

© Copyright Reserved by Gateflix.in No part of this material should be copied or reproduced without permission
=0. What is the number of roots in
the right half s-plane and on the j ω Q.91 Which of the following may result in
axis, respectively?
instability problem?
a) 1, 1 b) 0, 0
c) 2, 1 d) 1, 2 a) Large error b) High selectivity
c) High gain d) Noise
Q.86 The given characteristic polynomial
s 4 + s 2 + 2s 2 + 2s + 3 =0 has Q.92 For the block diagram shown in the
a) Zero root in RHS of s-plane given figure, the limiting values of K
b) One root in RHS of s-plane for stability of inner loop is found to
c) Two roots in RHS of s-plane be X<K<Y. the overall system will be
d) Three roots in RHS of s-plane stable if and only if

Q.87 Which one of the following


characteristic equations can result
in the stable operation of the
feedback system?
a) s3 + 4s 2 + s − 6 =0
a) 4X<K<4Y b) 2X<K<2Y
b) s − s + 5s + 6 =
3 2
0
X Y
c) s3 + 4s 2 + 10s + 11 =
0 c) X<K<Y d) < K <
2 2
d) s + s + 2s + 4s + 6 =
4 3 2
0
Q.93 The loop transfer function of a
Q.88 The characteristic equation of
system is given by
a control system is given by
s6 + 2s5 + 8s4 + 12s3 + 20s2 + 16s + 16 K(s + 10) 2 (s + 100)
G(s)H(s) =
= 0. The number of the roots of the s(s + 25)
equation which lie on the imaginary The number of loci terminating at
axis of s-plane is infinity is
a) Zero b) 2 a) 0 b) 1
c) 4 d) 6 c) 2 d) 3

Q.89 Consider the unity feedback system Q.94 A control system has G(s)H(s)=
K K/[s(s + 4) (s2 + 4s + 20)](0< K < ∝ ).
with G(s) = . The
2
(s +2s+2)(s+2) What is the number of breakaway
system is marginally stable. What is points in the root locus diagram?
the radian frequency of oscillation? a) One b) Two
c) Three d) Zero
a) 2 b) 3
c) 5 d) 6 Q.95 The characteristic equation of a
control system is given by s(s + 4)(s2
Q.90 The open – loop transfer function of + 2s + 2) + k(s + 1) = 0. What are the
a unity feedback control system is angles of the asymptotes for the root
given by G(s) = K e-Ts where K and T loci for k ≥ 0 ?
are variables and are greater than a) 60o, 180o, 300o b) 0o, 180o, 300o
zero. The stability of the closed-loop c) 120o, 180o, 240o d) 0o, 120o, 240o
system depends on
a) K only b) Both K and T Q.96 The open loop transfer function of a
c) T only d) Neither K nor T feedback system has m poles and n
zeros (m > n)

© Copyright Reserved by Gateflix.in No part of this material should be copied or reproduced without permission
Consider the following statements d)
1. The number of separate root loci
is m.
2. The number of separate root loci
is n.
3. The number of root loci
approaching infinity is (m – n)
4. The number of root loci Q.99 The open loop transfer function of a
approaching infinity is (m + n) closed loop control system is given
Which of the statements given as:
above are correct? K(s + 2)
a) 1 and 4 b) 1 and 3 G(s)H(s) = .What are
s(s + 1)(s + 4) 2
c) 2 and 3 d) 2 and 4
the number of asymptotes and the
Q.97 The characteristic equation of a centroid of the asymptotes of the
feedback control system is given by root-loci of closed loop system?
s3 + 5s2+(K+6)s+K=0. In the root loci 7 
a) − 3;  , 0  b) − 2; (2, 0)
diagram, the asymptotes of the root 3 
loci for large ‘K’ meet at a point in −7 
the s-plane whose coordinates are c) 3;  , 0 d) 2; (−2, 0)
 3 
a) (2, 0) b) (-1, 0)
c) (-2, 0) d) (-3, 0) Q.100 Loop transfer function of unity
feedback system is G(s) =
Q.98 The characteristic equation of a
linear control system is s2+5Ks+10=0.
K(s 2 + 64)
. The correct root locus
The root –loci of the system for s(s 2 + 16)
0<K<∞ is diagram for the system is
a) a)

b) b)

c) c)

© Copyright Reserved by Gateflix.in No part of this material should be copied or reproduced without permission
d) c) 4 d) 5

Q.105 For a given unity feedback system


k(s + 3)
with G(s) = , what
s(s + 1)(s + 2)(s + 5)
is the real axis intercept for root
locus asymptotes?
Q.101 An open loop transfer functions is a) 2/3 b) 1/4
given by c) –5/3 d) –3/2
k(s + 1)
G(s)H(s) = . It has
s(s + 2)(s 2 + 2s + 2) Q.106 How many number of branches the
a) One zero at infinity root loci of the equation s(s + 2)(s +
b) Two zeros at infinity 3) + K(s + 1) = 0 have?
c) Three zeros at infinity a) Zero b) One
d) Four zeros at infinity c) Two d) Three

Q.102 Which of the following is the open Q.107 The addition of open loop zero pulls
loop transfer function of the root the root-loci towards:
loci shown in the figure? a) The left and therefore system
becomes more stable
b) The right and therefore system
becomes unstable
c) Imaginary axis and therefore
system becomes marginally stable
d) The left and therefore system
becomes unstable
K K
a) b)
s(s + T1 ) 2 (s + T1 )(s + T2 ) 2 Q.108 The characteristic equation of a
K K control system is given as:
c) d) K ( s + 1)
(s + T) 3 s (sT1 + 1)
2
1+ =0
s ( s + 4)( s 2 + 2 s + 2)
Q.103 A control system has For large values of s, the root loci for
K (s + 1) K ≥ 0 are asymptotic to asymptotes,
G (s)H(s) = where do the asymptotes intersect
s(s + 3)(s + 4)
on the real axis?
Root locus of the system can lie on
5 2
the real axis a) b)
a) Between s = -1 and s = -3 3 3
b) Between s = 0 and s = -4 5 4
c) − d)
c) Between s = -3 and s = -4 3 3
d) Towards left of s = -4
Q.109 Consider the equation s2 + 2s + 2 +
K(s+2)=0. Where do the roots of this
Q.104 The characteristic equation of a
equation break on the root loci plot?
control system is
a) – 3.414 b) – 2.414
s5+15s4+85s3+225s2+247s + 120 = 0
c) – 1.414 d) – 0.414
What are the number of roots of the
equation which lie to the left of the
Q.110 A system has fourteen poles and two
line s + 1 = 0?
zeros. The slope of its highest
a) 2 b) 3

© Copyright Reserved by Gateflix.in No part of this material should be copied or reproduced without permission
frequency asymptote in its Q.116 What is the initial slope of Bode
magnitude plot is magnitude plot of a type – 2 system?
a)-40 dB/decade a) – 20 db/decade
b)-240 dB/decade b)+20 db/decade
c)-280 dB/decade c) − 40 db/decade
d)-320 dB/decade d)+40 db/decade

Q.111 The phase angle of the system Q.117 What is the error in magnitude at
s+5 the corner frequency for an
G (s ) = 2 varies between
s + 4s + 9 asymptotic Bode magnitude plot for
a) 00 and 900 b) 00 and -900 the term (1 + s τ) ± n ?
c) 0 and -180
0 0 d) -900 and -1800 a) ± 20 n db b) ± 6 n db
c) ± 3 n db d) ± 1 n db
Q.112 The Bode phase angle plot of a
system is shown in the figure. The Q.118 Consider the following statements
type of the system is regarding the frequency response of
a system as shown:
1. The type of the system is one
2. ω3= static error coefficient
(numerically)
ω + ω3
a) 0 b) 1 3. ω2 = 1
2
c) 2 d) 3

Q.113 A minimum phase unity feedback


system has a Bode plot with a
constant slope of -20 db/ decade for
all frequencies. What is the value of
the maximum phase margin for the Select the correct answer using the
system? codes given below:
a) 0o b) 90o a) 1, 2 and 3 b) 1 and 2
c) -90o d) 180o c) 2 and 3 d) 1 and 3

Q.114 At which frequency does the Bode Q.119 In the Bode plot of a unity feedback
magnitude plot for the function K/s2 control system, the value of phase
have gain crossover frequency? angle of G(jω) is –900 at the gain
cross over frequency of the Bode
a) ω =0 r/s b) ω = K r/s
plot, the phase margin of the system
c) ω =K r/s d) ω = K2 r/s is:
a) - 1800 b) + 1800
Q.115 Which one of the following is c) - 900 d) + 900
correct?
The slope of the asymptotic Bode Q.120 What are the gain and phase angle of
magnitude plot is integer multiple of a system having the transfer
a) ± 40 db/ decade function G(s)=(s+1) at a frequency
b) ± 12 db/ octave of 1 rad/sec?
c) ± 6 db/ octave a) 0.41 and 00 b) 1.41 and 450
d) ± 3 db/ octave c) 1.41 and – 450 d) 2.41 and 900

© Copyright Reserved by Gateflix.in No part of this material should be copied or reproduced without permission
Q.121 Consider the following statements in Q.125 The low frequency and high
connection with frequency domain frequency asymptotes of Bode
specifications of a control system: magnitude plot are respectively
1. Resonant peak and peak − 60 db/decade and − 40 db/decade.
overshoot are both functions of What is the type of the system?
the damping ratio ξ only. a) Type 0 b) Type I
2. The resonant frequency ωr = ωn c) Type II d) Type III
for ξ > 0.707.
Q.126 The poles and zeros of an all-pass
3. Higher the resonant peak, higher network are located in which part of
is the maximum overshoot of the the s-plane?
step response. a) Poles and zeroes are in the right
Which of the statements given half of s-plane
above are correct? b) Poles and zeroes are in the left
a) 1 and 2 only b) 2 and 3 only half of s-plane
c) 1 and 3 only d) 1, 2 and 3 c) Poles in the right half and zeroes
in the left half of s-plane
Q.122 A minimum phase transfer function d) Poles in the left half and zeroes
has in the right half of s-plane
a) poles in the right half of s-plane
b) zeros in the right half of s-plane Q.127 Consider the following statements
c) poles in the left half of s-plane regarding the asympotic Bode plots
and zeros in the right half of s- used for frequency response
half analysis:
d) no poles or zeros in the right half 1. The deviation of the actual
of s-plane or on the j ω -axis magnitude response for a zero on
excluding the origin real axis is 3 dB at the corner
frequency.
Q.123 Which of the following transfer 2. The phase angle for a pair of
functions is/are minimum phases complex conjugate poles at
transfer function(s). undamped frequency depends upon
1 (s − 1) the value of damping ratio
1. 2.
(s − 1) (s + 3)(s + 4) Which of the statements given
(s + 2) above is/ are correct
3. a) Only 1 b) Only 2
(s + 3)(s − 4)
c) Both 1 and 2 d) Neither 1 nor 2
Select the correct answer using the
code given below:
Q.128 For the Bode plot of the system G(s)
a) 1 and 3 b) 1 only
c) 2 and 3 d) None 10
= the corner
0.66 s + 2.33s + 1
2

Q.124 The transfer function of a system is frequencies are:


1− s a) 0.66 and 0.33 b) 0.22 and 2.00
. The system is then which one c) 0.30 and 2.33 d) 0.50 and 3.00
1+ s
of the following?
Q.129 Consider the following Nyquist plots
a) Non-minimum phase system
of different control systems:
b) Minimum phase system
c) Low-pass system
d) second-order system

© Copyright Reserved by Gateflix.in No part of this material should be copied or reproduced without permission
Q.131 The Nyquist plot of a servo system is
shown in the figure-I. the root loci
for the system would be
a)
Which of these plot(s) represents
(s) a stable system?
a) 1 alone b) 2, 3 and 4
c) 1, 3 and 4 d) 1, 2 and 4

Q.130 The transfer function of a certain


s
system is given by G ( s ) = . b)
(1 + s )
The Nyquist plot to the system is
a)

c)

b)

d)

c)

Q.132 If the Nyquist plot cuts the negative


real axis at a distance of 0.4, then
the gain margin of the system is
a) 0.4 b) -0.4
d) c) 4% d) 2.5

Q.133 Consider the Nyquist diagram for


given KG(s) H(s). The transfer
function KG(s)H(s) has no poles and
zeros in the right half of plane. If the
(-1+j0) point is located first in
region I and then in region II, the
change in stability of the system will
be from

© Copyright Reserved by Gateflix.in No part of this material should be copied or reproduced without permission
K
c)
s (1 + sT1 )(1 + sT2 )
3

K
d) 2
s (1 + sT1 )(1 + sT2 )

Q.136 Nyquist plot shown in the given


figure is for a type

a) Unstable to stable
b) Stable to stable
c) Unstable to unstable
d) Stable to unstable
a) Zero system b) One system
Q.134 The Nyquist plot of a unity feedback
c) Two system d)Three system
system having open loop transfer
K(s + 3)(s + 5)
function G(s) = for K Q.137 The open loop transfer function of a
(s − 2)(s − 4) unity feedback control system is
= 1 is as shown below. For the 1
given as G (s) = . The
system to be stable, the range of s(1 + sT1 )(1 + sT2 )
values of K is phase crossover frequency and the
gain margin are, respectively,
1 T +T
a) and 1 2
T1T2 T1T2
T1 + T2
b) T1T2 and
T1T2
1 TT
c) and 1 2
a) 0 < K < 1.33 b) 0 < K <1/1.33 T1T2 T1 + T2
c) K > 1.33 d) K > 1/1.33 T1T2
d) T1T2 and
T1 + T2
Q.135 The Nyquist plot of a control system
is shown below. For this system,
Q.138 The Nyquist plot shown, matches
G(s) H(s) is equal to
with the transfer function

K
a) 1 1
s(1 + sT1 ) a) b)
(s + 1)3 (s + 1) 2
K
b) 2 1 1
s (1 + sT1 ) c) 2 d)
(s + 2s + 2) (s + 1)

© Copyright Reserved by Gateflix.in No part of this material should be copied or reproduced without permission
Q.139 The forward path transfer function
=&  0 1  X + 0  U and
of a unity feedback system is given X  −2 −3 1 
1    
by G (s) =
(1 + s) 2
Y [ 2 0] X
What is the phase margin for this The transfer function of this system
system? is:
a) −πrad b) 0rad Y (s) 1
a) = 2
c) π / 2rad d) π rad U ( s ) 2s + 3s + 1
Y (s) 2
Q.140 What is the gain margin of a system b) = 2
when the magnitude of the polar U ( s ) 2s + 3s + 1
plot at phase cross over is ‘a’? Y (s) 1
a) 1/a b) –a c) = 2
U ( s ) s + 3s + 2
c) Zero d) a
Y (s) 2
d) = 2
Q.141 A system with gain margin close to U ( s ) s + 3s + 2
unity or a phase margin close to
zero is Q.145 The state-space representation in
a) Relatively stable b) Oscillatory phase-variable form for the transfer
c) Stable d) Highly stable 2s + 1
function G ( s ) = 2 is
s + 7s + 9
Q.142 The state diagram of a system is
. 0 1 0 
shown in the given figure: a) =
x   y [1 2] x
x +   u;=
 −9 − 7   
1
. 1 0 0 
b) = x   x +   u;=y [ 0 1] x
 −9 −7  1 
.  −9 0  0
c)=x   x +   u;=y [ 2 0] x
The system is  0 −7  1 
a) Controllable and observable
b) Controllable but not observable . 9 −7  0 
c) Observable but not controllable d) x=   x +   u; y= [1 2] x
1 0  1 
d) Neither controllable nor
observable
=
Q.146 & 1 2  X +  0 
Let X
Q.143 The state-variable description of a
0 1  1 
   
Y = [ b 0] X
linear autonomous system is X & =AX
where X is a state vector and
Where b is an unknown constant.
0 2
A= . This system is
2 0 a) Observable for all values of b
The poles of the system are located b) Unobservable for all values of b
at c) Observable for all non-zero
a) -2 and +2 b) -2j and +2j values of b
c) -2 and -2 d) +2 and +2 d) Unobservable for all non-zero
values of b
Q.144 A particular control system is
described by the following state
equations:

© Copyright Reserved by Gateflix.in No part of this material should be copied or reproduced without permission
Q.147 The state-space representation of a b) Uncontrollable for b = 0
system is given by T c) Uncontrollable for all values of b
X&=  0−1 −20  X +  10  UandY =
 11  X d) Controllable for all values of b
Then the transfer function of the
Q.152 Given the matrix A =
system is
0 1 0
1 1
a) 2
s + 3s + 2
b)
s+2
0
 0 1 
s 1 − 6 − 11 − 6
c) 2 d)
s + 3s + 2 s +1 The eigen values of A are:
a) − 1,−2,−3 b) − 1, 2, 3
Q.148 A linear time-invariant system is c) 0, 0, − 6 d) − 6,−11,−6
described by the following dynamic
equation: Q.153 System transformation function
dx ( t ) H(z) for a discrete time LTI system
= Ax ( t ) + Bu ( t )
dt expressed in state variable form
y ( t ) = Cs ( t ) with zero initial conditions is
a) c(zI – A)-1 b + d b) c(zI – A)-1
where c) (zI – A)-1z d) (zI – A)-1
0 1 0 
=A 
− −
=  , B =  , C [1 1] Q.154 Which one of the following is the
 2 3   
1
state-space model of the circuit
The system is shown below?
a) Both controllable and observable
b) Controllable but unobservable
c) Observable but uncontrollable
d) Both uncontrollable and
unobservable

Q.149 What is represented by state  1


 0 0
transition matrix of a system?  x&1t  L   x1 t   
=  x& t      + 1 u(t)
a) Free response  2  x t  
a)
− 1 0   2  C
b) Impulse response  C 
c) Step response
x t
d) Forced response y(t) = [ 0 1]  1 
x2t 
Q.150 The system matrix of a continuous  0 1 
 x&1t    x1 t   0 
time system is given by A = =   1 1    +   u(t)
0 1  x&2 t   − −  x 2 t  1 
− 3 − 5. Then the characteristic b)  C L
  x t
equation is y(t) = [ 0 1]  1 
a) s2 + 5s + 3 = 0 b) s2 – 3s – 5 = 0 x2t 
c) s + 3s + 5 = 0
2 d) s2 + s + 2 = 0  1
0
 x&1t   L   x1 t   1 
=  x& t   1    +   u(t)
• 1 2 0  2  − 0 2   
x t 0
Q.151 Let x =   x + 1 u where b is an c)
 0 b     C 
unknown constant. This system is x t
a) Uncontrollable for b = 1 y(t) = [1 0]  1 
x2t 

© Copyright Reserved by Gateflix.in No part of this material should be copied or reproduced without permission
 1 c) Rise-time of closed loop system
0
 1  
&
x t L   x1 t   0  is reduced
=  x& t   1    +   u(t) d) Gain margin is reduced
 2  − 0 2   
x t 1
d)
 C  Q.159 Which one of the following is the
x t correct expression for the transfer
y(t) = [1 0]  1  function of an electrical RC phase-
x2t  lag compensating network?
RCS RC
Q.155 A linear time invariant system with a) b)
(1 + RCS) (1 + RCS)
input u(t) and output y(t) is
1 1
described by the state-space c) d)
representation as given below. (1 + RCS) (1 + RCS)
x1(t) = x2(t)
Q.160 The pole-zero plot shown in the
x2(t) = x1(t) + x2(t) + u(t)
figure is that of which one of the
and y(t) = x1(t) + 3x2(t)
following?
The transfer function of the system
is
s+3 s+3
a) 2 b) 2
s − s −1 s + s +1
3s + 1 3s + 1
c) 2 d) 2
s + s +1 s − s −1
a) Integrator
Q.156 The transfer function of a phase lead b) PD controller
1 + aTs c) PID controller
compensator is given by d) Lag-lead compensator
1 + Ts
where a>1 and T>0. The maximum
phase shift provided by such a Q.161 What is the effect of phase lead
compensator is compensator on gain cross – over
frequencies and on the bandwidth
 a +1   a −1 
a) tan −1   b) tan −1   (ωb)?
 a −1   a +1  a) Both are increased
 a +1   a −1  b) ωcg is increased but ωb is
c) sin −1   d) sin −1  
 a −1   a +1  decreases
c) ωcg is decreased but ωb is
Q.157 Indicate which one of the following increased
transfer functions represents phase d) Both are decreased
lead compensator?
s +1 6s + 3 Q.162 The transfer function of a P-1
a) b)
s+2 6s + 2 controller is:
s+5 s+8 a) Kp + Ki s b) Kp + (Ki/s)
c) d) 2 c) (Kp/s) + Ki s d) Kp s + (Ki/s)
3s + 2 s + 5s + 6

Q.158 A property of phase-lead Q.163 The transfer function of a phase-


compensation is that the lead compensator is given by:
a) Overshoot is increased 1 + 3Ts
G(s) = where T > 0. What is
b) Bandwidth of closed loop system 1 + Ts
is reduced the maximum shift provided by such
a compensator?

© Copyright Reserved by Gateflix.in No part of this material should be copied or reproduced without permission
a) 90° b) 60°
c) 45° d) 30°

Q.164 In the control system shown above


a) Proportional type
the controller which can give zero
b) Integral type
steady-state error to a ramp input is
c) Derivative type
of
d) Proportional plus derivative type

ANSWER KEY:
1 2 3 4 5 6 7 8 9 10 11 12 13 14
(b) (a) (d) (b) (d) (c) (c) (b) (c) (d) (a) (a) (b) (a)
15 16 17 18 19 20 21 22 23 24 25 26 27 28
(b) (a) (b) (b) (a) (d) (a) (b) (c) (d) (a) (a) (a) (b)
29 30 31 32 33 34 35 36 37 38 39 40 41 42
(b) (c) (c) (b) (a) (b) (a) (b) (d) (b) (b) (d) (d) (c)
43 44 45 46 47 48 49 50 51 52 53 54 55 56
(c) (a) (d) (c) (c) (b) (d) (b) (b) (a) (a) (a) (d) (b)
57 58 59 60 61 62 63 64 65 66 67 68 69 70
(d) (d) (b) (b) (a) (b) (a) (c) (c) (b) (a) (b) (c) (b)
71 72 73 74 75 76 77 78 79 80 81 82 83 84
(c) (c) (c) (c) (b) (c) (a) (c) (d) (a) (d) (b) (a) (c)
85 86 87 88 89 90 91 92 93 94 95 96 97 98
(b) (c) (c) (c) (d) (c) (c) (d) (a) (a) (a) (b) (c) (a)
99 100 101 102 103 104 105 106 107 108 109 110 111 112
(c) (a) (c) (d) (b) (c) (c) (d) (a) (c) (a) (b) (c) (a)
113 114 115 116 117 118 119 120 121 122 123 124 125 126
(b) (b) (c) (c) (c) (b) (d) (b) (c) (d) (d) (a) (d) (d)
127 128 129 130 131 132 133 134 135 136 137 138 139 140
(a) (d) (d) (b) (d) (d) (d) (d) (d) (b ) (a) (b) (c) (a)
141 142 143 144 145 146 147 148 149 150 151 152 153 154
(b) (a) (b) (d) (a) (c) (d) (b) (a) (a) (d) (a) (a) (a)
155 156 157 158 159 160 161 162 163 164
(d) (d) (a) (c) (c) (d) (a) (b) (d) (b)

© Copyright Reserved by Gateflix.in No part of this material should be copied or reproduced without permission
EXPLANATIONS

Q.1 (b) C(s)


= =
G1G 2
.
1 G1
Cn (s) −G ( s ) H 2 (S) R(s) 1 − G1G 2 G 2 1 − G1G 2
=
N(s) 1 + G ( s ) H1 (S)H 2 (S)
Q.5 (d)
1
= − for G ( s ) H1 ( S) H 2 ( S) ? |
H1 (S)

Q.2 (a)
Forward paths
= =
P1 abcef P3 abn lg f
= =
P2 ajigf P4 ajkmef Vo (s) S 2S + 1
= s ||1 + 1= + 1=
Loop gain V1 (s) 1+ s s +1
= =
L1 cd L3 nmd
Q.6 (c)
=
L 2 hi=
L 4 klh Forward paths Individual Loops
P1 = adfi
Q.3 (d)
= =
L1 bL dc
Free body diagram of M is shown 2

below: P2 = aefi
L3 = ec
P3 = ahi
= =
L 4 fgL 5 hgc
Non touching Loop pairs: L1L 4
By network law
fM + fB + fk =
0 Q.7 (c)
d y2 ( t )
2
dy ( t ) Negative feedback decreases the
M 2
+B 2 + k ( y 2 ( t ) − y1 ( t ) ) =
0 distortion.
dt dt
Q.8 (b)
Q.4 (b) G1G 2 + G 3G 2
C(s)
=
R(s) 1 + G1G 2 H1 + G 2 G 3 H1 − G 4

Q.9 (c)
C(s) G1G 2
=
R(s) 1 − G 2 H 2 + G1G 2 H1H 2
C(s) G1
=
R(s) 1 − G1G 2 Q.10 (d)
1. =
x 2 a 1 x1 + a 9 x 3
2.=
x3 a 2x 2 + a8x 4
3. x 4 = a 3 x 3 + a 5 x 2 + a 7 x 4
4.=
x5 a 4x 4 + a 6x 2

© Copyright Reserved by Gateflix.in No part of this material should be copied or reproduced without permission
Q.11 (a) Q.20 (d)
G1G 2 + G 3G 2 1. Feedback reduce system error
1 + G 2 H1 2. Feedback can make a stable
system as unstable
Q.12 (a)
C G Q.21 (a)
= Forward path gain
R 1 + GH − FG Individual Loop gain
P1 = rsu
Q.13 (b)
C(s) 1 + S−1a s + a L1 = st
= = P2 = efh
R(s) 1 + S−1b s + b
L 2 = fg
Q.14 (a) x 2 rsu (1 − gf ) + efh(1 − st)
Y(s) G1G 2 =
= x1 1 − st − fg + stfg
x(s) 1 − G 2 H1 + G1G 2 H 2
x 2 rsu (1 − fg ) + efh(1 − st)
=
Q.15 (b) x1 (1 − st ) (1 − fg)
rsu efh
= +
1 − st 1 − fg

Q.22 (b)
For negative feedback
Vo (s)
= =
R SRC A 10 ×10 ×10
Gain,= AF =
V1 (s) R + 1 1 + SRC 1 + AB 1 + AB
SC B= 9 ×10−3
Q.16 (a) Q.23 (c)
For block diagram 1 Consider R(s)=0
C(s) k
= +1 C(s)
=
G 2 (s)
R(s) S + 1 D(s) 1 + G1 (s)G 2 (s)
S +1+ k
= For , G1 ( s ) G 2 ( s ) ? 1
S +1
1
For k=1 both are same C (s) = D(s)
G1 ( s )
Q.17 (b) Step disturbance can be reduced by
Number of loops are 3 increasing the gain of G1 (s)
Number of forward paths is also 3
C 8 + 12 + 20 40 Q.24 (d)
= =
R 1 + 24 + 16 + 40 81 Transfer function,
Q.18 (b) y (s) Ak
=
T (s ) =
C G + H2
= (Using SFG) R ( s ) S + a + Akβ
R 1 + GH1 %change in T(s)
Sensitivity =
%change in parameter
Q.19 (a)
%change in T ( s ) ∂T k
C(s)
==
G1G 2
(Using SFG) S=
T
= ×
∂k T
k
R(s) 1 + G1H1 − G 2 H 2 %change in k

© Copyright Reserved by Gateflix.in No part of this material should be copied or reproduced without permission
S+a 1
= R(s) =
s + a + Akβ s
∂T β Akβ C (s)
SβT = × = − Transferfunction =
∂β T (s + a + Akβ) R (s )
∂T A S+a 2s
STA = × = =
∂A T s + a + Akβ ( s + 1)
3

T
Sβ is negative hence it doesn’t
reduce the closed loop sensitivity. Q.29 (b)
Closed loop system is less sensitive
Q.25 (a) to parameter variations.
Free body diagram of mass is Closed loop system has the ability to
control the system transient
response.

Q.30 (c)
c(t) = 1 − 10e − t
1 10
C ( s )= +
s s +1
By Newton’s law s + 1 − 10s 1 − 9s
fm + fk + fb = = =
s ( s + 1) s ( s + 1)
f (t)
d2x dx
m + kx + b =
f (t)
Q.31 (c)
2
dt dt
The characteristics equation is given
Q.26 (a) by
=
Response, c ( t ) te − t for t ≥ o ( s + a ) + b2 =
2
0
1
c (s) = S2 + 2as + a 2 + b 2 =
0
( s + 1)
2
Comparing with standard 2nd order
Input, r ( t ) = tu(t) equations,
We get
1
R (s) = ω2n = a 2 + b 2 ⇒ ωn = a 2 + b 2
S
C(s) s a
= 2ξωn = 2a ⇒ ξ =
R(s) ( s + 1)2 a 2 + b2

Q.32 (b)
Q.27 (a)
t2
=
C(s)
M =
G(s) Given r ( t ) = 1 + t +
R(s) 1 + G ( s ) H(s) 2
1 1 1
∂M G 1 R (s) = + 2 + 3
S =
M
× = s s s
∂G M 1 + G ( s ) H(s)
G
sR(s)
ess = lim
s → 0 1 + G ( s ) H(s)
Q.28 (b)
1
Step response, c(t) = t 2 e − t = = 0.1
10
= 2 / (s + 1)3
C(s)
Input r(t) = u(t) Q.33 (a)

© Copyright Reserved by Gateflix.in No part of this material should be copied or reproduced without permission
10(s + 1) Hence, the system is under damped
G (s) =
s 2 (s + 2) Setting time
4 4
r ( t ) =1 + 2t ⇒ R ( s ) = + 2
1 2 = =
t s 4T = = 0.6s
ξωn 6.6
s s
sR(s)
=ess lim = 0
s → 0 1 + G ( s ) H(s) Q.37 (d)
10
G (s) =
Q.34 (b) s (s + 4)
2

d 2c ( t ) dc ( t ) r ( t ) =2 + 3t + 4t 2
4 2
+8 + 16c ( t )
dt dt lt s R(s)
=ess = 3.2
= 16u(t) s → 0 1 + G ( s ) H(s)
Applying LT both sides we get
4s 2 C ( s ) + 85C ( s ) + 16c ( s ) Q.38 (b)
= 16R(s) Given, Impulse response,
1
(Since u(t) is input) c ( t ) = e −0.8t Sin(0.6t)
c(s) 16 4 6
= = In general the impulse response is
R(s) 4s + 8s + 16 s + 2s + 16
2 2

ke − ξωn t sin(ωd t)
rad
ω2n =4 ⇒ ωn =2 ξωn = 0.8

ξ=
1 ωd= ωn 1 − ξ 2= 0.6
2 Solving above equations we get
ωn = 1rad / sec
Q.35 (a)
k ξ = 0.8
OLTF, G ( s ) =
s(s + 1)
Q.39 (b)
C(s) k
CLTF, = 2 M ( jω ) =
100
R(s) s + s + k
−ω + 10 2 jω + 100
2

ξ = 1 for critically damped system


= ω2n 100,
= 2ξωn 10 2
ω = k ⇒ ωn =
2
n k
1
1 ω
= 10ξ
=
2ξωn = 1 ⇒ ωn =
n
2
2 1
1 = Mp = 1
∴ k =ω2n = 2ξ 1 − ξ 2
4
Characteristic equation is given by Q.40 (d)
5(1 + 0.1s)
1
s 2 + s + =0 ⇒ s =−0.5, −0.5 Given G ( s ) =
4 s (1 + 5s ) (1 + 20s)
The above system is type -1
Q.36 (b) For step input
The CE is given by A
ess =
s 2 + 13.25 + 121 =
0 1+ kp
ω2n = 121 ⇒ ωn = 11 k p = lim G ( s ) = ∞
s →0
13.2
2ξωn= 13.2 ⇒ ξ= <1 ∴ ess =
0
22

© Copyright Reserved by Gateflix.in No part of this material should be copied or reproduced without permission
For ramp input s R(s) a
=ess lim =
A s → 0 1 + G ( s ) H(s) k
ess =
kv ∂e k −a k
= = sG ( s ) 5
s ekss =ss × =× = −1
k v lim
s →0 ∂k ess k 2 a
k
10 ∂e
∴ ess = = 2 s ekss = ss ×
a
= ×
1 a
=1
5 ∂a ess k a
For accelerations input k
A
ess = Q.45 (d)
ka
k
t
Given, C ( s ) =
= ∫ s G (s) 0
= S+ ∝
2
ka
s →0 C ( t ) = ke −∝ t
ess = ∞
= 2 , C ( t2 )
At t t= 0.37k

Q.41 (d) 0.37k = ke −∝ t 2


1
=
Given, ωn 10=
rad
, ξ 0.1 At ∝= above equation will be
sec t2
4 satisfied.
For 2% = =
t s 4T = 4sec
ξωn
Q.46 (c)
400
Q.42 (c) → ωn = 20, ξ = 0.3
= R(s) − C(s)
E(s) s + 12s + 400
2

Under damped
K
= R(s) −
E(s) E(s) 900
s+5 → ωn = 30, ξ = 1.5
s + 90s + 900
2
E (s) s+5
= Over damped
R (s ) s + 5 + K 225
→ ωn = 15, ξ = 1
s + 30s + 225
2

Q.43 (c) Critically damped


Unit step response, 625
→ξ= 0
c ( t ) =−
1 e − t (1 + t )  u(t) s + 625
2

Impulse response Undamped


d
h (t) =
= c(t) te − t u(t) Q.47 (c)
dt
C(s) 1 In general peak time is given by
Transfer function, = nπ nπ
R(s) ( S + 1)2 =tp =
ωd ω n 1 − ξ 2
Since poles lie on –ve real axis are
repeated hence system is critical For 2nd peak, n=3 hence,
stable 3π
tp =
ωn 1 − ξ 2
Q.44 (a)
G (s) =
k Q.48 (b)
s(s + a) k
G (s) =
r ( t ) = tu(t) s(s + 1)

© Copyright Reserved by Gateflix.in No part of this material should be copied or reproduced without permission
C(s) k x ( t ) sin
= = t, ω 1rad / sec
= 2
R(s) s + s + k j 1
H ( jω )=
|ω=1 = 145°
ω =k ⇒ ωn = k
2
n 1+ j 2
1 1
2ξωn =1 ⇒ ξ = y(t)
= sin(t + 45°)
2 k 2
− πξ
1− ξ 2
%M p= e ×100%= 50 Q.52 (a)
− πξ π
e
1 1− ξ 2
= Phase, φ = −ωT −
2 2
Substituting the value of ξ ,we get, − tan −1 ( ω )
k = 5.39 At ω= ω1 , ∅= 0
π
Q.49 (d) −ωT − tan −1 ( ω ) =
2
c(t) =
1 − e −5t − 5te −5t
Taking tan both sides, weget
tan ( ω1T ) + ω1
25
c (s) = (by L.T)
S ( s + 5)
2 = ∞
1 − tan ( ω1T ) ω1
1
R (s) = =ω1
1
= cot ( ω1T )
S tan ( ω1T )
C(s) 25
= 2
R(s) s + 10s + 25 Q.53 (a)
ω2n = 25 ⇒ ωn = 5 k C (s) k
=G (s) = 2
2ξωn = 10 ⇒ ξ = 1 s(s + k) R ( s ) s + s + k
Impulse response, ω2n =k ⇒ ωn = k
d
= (1 − e −5t − 5te −5t ) 1
dt 2ξωn =1 ⇒ ξ =
2 k
= 5e −5t − 5e −5t + 25te −5t
As k → ∞, ξ =0
= 25te−5t
Q.54 (a)
Q.50 (b)
c ( t ) = 12.5e −6t sin 8t
=
Given ξ 0.6,
= ωn 2rad / sec
Peak time, ξωn = 6 ωd= ωn 1 − ξ 2= 8
π π
( ωn 10 ) ωn − 2ωn ξ = 64
2 2 2
=
tp = = 1.95sec 6
ωd ω n 1 − ξ 2
=ξ =
10 ωn = 100
4 4 = 0.6 ωn = 10
Setting time=
ts =
ξωn 0.6 × 2
= 3.33sec Q.55 (d)
1 1 1
Q.51 (b) ess= + +
1 + Kp Kv Ka
y(s) S
Given = = H(s)
x(s) 1 + S
Q.56 (b)

H ( jω ) = For dominant pole the transients die
1 + jω out slowly.

© Copyright Reserved by Gateflix.in No part of this material should be copied or reproduced without permission
C(s) 1
=
Q.57 (d) R(s) S
ωn = 3rad / sec c ( t ) = 1 which is constant
ξ = 0.5
ωr = ωn 1 − 2ξ 2 = 3 1 − 0.5 =
3 Q.64 (c)
2 For a critically damped system the
= 2.12rad / sec poles lies on –ve real axis, are
1 1 2 repeated.
=
M r = = = 1.16
2ξ 1 − ξ 2 1 3
1− Q.65 (c)
4 Since the given system is under
Q.58 (d) damped and damping ratio ξ is
Delay time → Rise time → Peak negative hence the roots will be
time → settling time complex conjugates on the right half
of s-plane.
Q.59 (b)
20 Q.66 (b)
G (s) = We know that
s + 5s + 20
2

A
ωn = 20 ⇒ ωn = 20
2
Steady state error, ess =
k
5
2ξωn = 5 ⇒ ξ = By increasing system gain k, ess can
2 20
be reduced
Setting time for ±2% is
4 8 Q.67 (a)
t s 4τ= = = 1.6sec
5 5 Impulse response of 1st block is
2
C1 ( t ) = [−0.5 (1 + e −2t )] = e −2t
d
Q.60 (b) dt
1
= 2
C(s) k
→ CLTF C1 ( s ) =
R(s) s + 4s + k s+2
Impulse response of 2nd block is
ω2n =k ⇒ ωn = k
C2 ( t ) = e− t
2ξωn = 4
1
For ξ = 0.5 C2 ( s ) =
s +1
ωn = 4 The overall transfer function
Since k = ω2n ⇒ k = 16 1
=C ( s ) C=
1 ( s ) C2 ( s )
( s + 2 ) (s + 1)
Q.61 (a)
Since the input is sinusoidal hence Q.68 (b)
the steady state error is zero. The transfer function of the
diaphragm is
Q.62 (b) C(s)
= 2
1
For 3rd order system the rise time R(s) s + 2s + 2
would be smaller that the equivalent 1
2nd order system For unit step input R ( s ) =
S
1
Q.63 (a) ∴ C (s) =
s(s + 2s + 2)
2

© Copyright Reserved by Gateflix.in No part of this material should be copied or reproduced without permission
The steady state value is given by For k = 30, System is marginally
C ( ∞ ) = lim sC ( s ) =
1
= 0.5 stable
s →0 2
Q.72 (c)
Q.69 (c) s4 3 5 2
1
Mr = s3 10 0 0
2ξ 1 − ξ 2 s2 5 2
Given M r = 1 s −4
1

1 s0 2
∴ =1
2ξ 1 − ξ 2 One sign change in first column so,
4ξ (1 − ξ
2 2
)=
1 unstable system.

4ξ 2 − 4ξ 4 =
1 Q.73 (c)
4ξ − 4ξ + 1 =0
4 2
C(s)
= 3
k
( 2ξ − 1)( 2ξ 2 − 1) = R(s) S + 7S + 6s + k
2
2
0
1 6
1 s3
ξ= 7 k
2 s2
42 − k
s1
Q.70 (b) 7
s0
=
Given t s 6sec for ± 5% k
1 So k > 0 and
= ξ 0.707
= 42 − k
2 > 0 for stability
3 7
ts = =0.6 ⇒ ξωn =5 ∴ 0 < k < 42
ξωn
ωn = 5 2 Q.74 (c)
1 C(s) k(1 + Ts)
ωd= ωn 1 − ξ 2= 5 2. = 5 = 3 2
2 R(s) S + S + kTs + k
In general the poles location is s3 1 kT
S= −ξωn ± jωd =−5 ± j5 s2 1 k
s1 kT − k
Q.71 (c)
C(s) k s0 k
− = So kT − k > 0
R(s) S + 6S + 5s + k
3 2

T −1 > 0
1 5  T >1
s3 
2 6 k
s 
1 30 − k 
Q.75 (b)
6 
s s4 2 3 10
s0  3
k  s 1 5
30 − k s 2 −7 10
So, k > 0 & > 0 system is stable 1
6 s 45 / 7
∴ 0 < k < 30 s 0 10

© Copyright Reserved by Gateflix.in No part of this material should be copied or reproduced without permission
2 sign changes so two roots lie in 1 5
right half s − plane s3
6 k
s2
30 − k
Q.76 (c) s1
If poles of a system lie on the 6
s0
imaginary axis the system will be k
marginally stable. K=30
For sustained oscillation
Q.77 (a) Auxiliary equation is
1 9 6s 2 + k =0
s3
6 4 s = −5
2
s2
50 ω = 5rad / sec
s1
6
s0 Q.81 (d)
4
No sing changes so all three poles lie 1 24 k
s4
in L.H.S of s − plane . 1 3 / 24
s3
20 k
s2
Q.78 (c) 80 − k
s4 1 3 k s1
20
3
s 2 2 s0
k
2
s 2 k For k > 80 there will be sign change
s 2−k
1 in the first column
s0 k
Q.82 (b)
For the system to be oscillatory a0 a2
2−k =0⇒ k =2 s3
a1 a 3
The Auxiliary equation is s2
2s 2 + k =0 a1a 2 − a 0 a 3
s1
s =−1 ⇒ ω =1rad / sec
2 a1
s0
a3
Q.79 (d) a a −a a
So, 1 2 0 3 ≥ 0
k a1
Closed loop eq n →
s + as 2 + k
3
∴ a1a 2 ≥ a 0 a 3
31 0
s
s2
a k Q.83 (a)
s1 −
k G=( s ) T1T2S3 + ( T1T2 ) S2 + S =
+k 0
a T1T2 1
s0 s3
k T1 + T2 k
For −∞ < k < ∞ system is unstable s2
T1 + T2 − T1T2 k
s1
Q.80 (a) T1 + T2
s0
Characteristic k
eq n s3 + 6s 2 + 5s + k =0 T +T −TT k
So, 1 2 1 2 ≥ 0
T1 + T2
∴ T1 + T2 ≥ T1T2 k

© Copyright Reserved by Gateflix.in No part of this material should be copied or reproduced without permission
T1 + T2 Q.87 (c)
k<
T1T2 1 10
s3
4 11
s2
Q.84 (c) 29
Characteristics eqn s1
4
s3 + ( a + b ) s 2 + abs + k =0 s0
11
1 ab So coefficient in first column have
s3
a+b k same sing so, system is stable.
s2
( a + b ) ab − k Q.88 (c)
s1
(a + b)
s0 s6 1 8 20 16
0 5
s 2 12 16
So,
( a + b ) ab − k ≥ 0 s4 2 12 16
(a + b)
s3 0 0 0
(a + b)ab ≥ k
s3 1 3
∴ 0 < k < ( a + b ) ab
s2 3 8
Q.85 (b) s1 1/ 3
s3 + 25s 2 + 10s + 50 =
0 s0 8
s3 1 10 Auxiliary equation is
s 2 25 50 A ( s ) → s 4 + 6s 2 + 8 =0
s1 8 dA(s)
= 4s3 + 125
s 0
2 ds
Solving Auxiliary equation we get
Since there are sign change in the
first column hence no rot will lie on s =2 j, − 2 j, 2 j, −2 j
RHS of s- plane Also there is no
occurrence of Auxiliary equation Q.89 (d)
hence no roots on jωaxis. CE is given by
( s2 + 2s + 2 ) ( s + 2 ) + k =0
Q.86 (c)
s3 + 4s 2 + 6s + 4 + k =0
s + s + 25s + 25 + 3 =
4 3 2
0 1 6
1 2 3 s3
s4 4 4+k
1 2 s2
s3 20 − k
0/ξ 3 s1
s2 4
2ξ − 3 s0
4+k
s1
ξ For the system to be marginally
s0
3 stable
Replace zero by ′ξ′ in s 2 row 20 − k
= 0 ⇒ k = 20
2ξ − 3 4
For ξ = 0 = −∞ Auxiliary equation is given by
ξ
Since two sing changes, so two roots 4s 2 + 4 + k =0
in RHS of s- plane 4s + 24 =0 ⇒ ω = 6rad / sec
2

© Copyright Reserved by Gateflix.in No part of this material should be copied or reproduced without permission
Q.90 (c)
The stability of the given system
depends on T alone because as T
increase the system decay slowly
and stability decreases.

Q.91 (c)
High gain result instability problem Since there is real axis loci between
0 an -4
Q.92 (d) Hence, number of B.A.P is one
CE for the inner loop is given by
Q.95 (a)
( s − a )( s + 2a )( s + 3a ) + k =0
Close loop system eqn can be
 −2a 2 − 3a 2  written as
s + 4as + 
3 2
s
 +6a
2 k(s + 1)
 G (s) H (s) =
−6a + k =
3
0 s ( s + 4 ) (s 2 + 2s + 2)
s + 4as + a 2s − 6a 3 + k =
3 2
0 No of zero z=1
2 Number of poles p = 4
1 a
s3 No of asymptotes N = p − z = 3
4a − 6a 3 + k
s2
Angle of asymptotic =
( 2q + 1)180°
s1 4a + 6a − k
3 3
p−z
s0
4a When = q 0,θ= 60°
−6a 3 + k = = 180°
q 1,θ
For stability = = 300°
q 2,θ
6a 3 < k < 10a 3 x<k<y
CE for overall system is Q.96 (b)
s3 + 4as 2 + a 2s − 6a 3 + 2k = 0 The number of separate root loci is
x m= number of poles.
<k<y The number of zeros at infinity is
2 2
m − n and hence m − n root loci will
Q.93 (a) approach infinity.
Number of zeros =3
Number of poles =2 Q.97 (c)
s3 + 5s 2 + ( k + 6 ) s + k =0
s3 + 5s 2 + 6s ( S + 1) k =
0

G (s) H (s) =
( s + 1) k
s + 5s 2 + 6s
3

So number of loci terminating at ∞ k(s + 1)


=
is 0 s ( s + 3) (s + 2)
Asymptotes meet at centroid
Q.94 (a)
−3 − 2 − (−1)
k Centroid, σ =
G (s) H (s) = 2
s ( s + 4 ) (s 2 + 4s + 20)
4
Poles = 0, −4, −2 + 4 j, −2 − 4 j = − = −2 (−2, 0) is the centroid.
2

© Copyright Reserved by Gateflix.in No part of this material should be copied or reproduced without permission
Q.98 (a) Q.102 (d)
The loop gain is given by As shown in fig, two poles are at
5ks origin and one pole is at –ve real
G (s) H (s) = 2
s + 10 axis so,
Poles s = ± j 10 k
T.F = 2
Zero, s = 0 s (sT1 + 1)
One zero is considered at ∞
Q.103 (b)
k(s + 1)
G (s) H (s) =
s ( s + 3) (s + 4)

Q.99 (c) Root locus of the system can lie on


k(s + 2) the real axis between s=0 and
G (s) H (s) = s = −4
s ( s + 1)( s + 4 ) (s + 4)
No of asymptote N = p − 2 Q.104 (c)
N= 4 − 1 Put s= z − 1 we get
N=3
z5 + 10z 4 + 35z 3 + 50z 2 + 24 z =0
Centroid
Therefore 4 roots lie to the left of
=
∑real part of poles − ∑real part of zero line s + 1 = 0 and rot lies on s + 1 =0
p−z
z 5
1 35 24
=
( −1) + ( −4 ) + ( −4 ) − (−2) = −7 z4 0 50
3 3 z 4
1 5
z 3
30 24
Q.100 (a) z 3
5 4
k(s 2 + 64)
G (s) =
2
z 21
s(s 2 + 16) 5
Zeros s = ±j8 z1
4
Poles, s = 0, ±j4 z 0
0
One zero is considered at infinity There are 4 roots lie to the left of
line s+1=0 and 1 root lies on s+1=0.

Q.105 (c)
k(s + 3)
G (s) =
s ( s + 1)( s + 2 ) (s + 5)
real axis intercept
Angle of departure = o°
=
∑real port of pole − ∑real port of zero
Q.101 (c) p−2
For the given transfer function there
σ=
( −1 − 2 − 5) − (−3)
are 4 poles and 1 zero. So there will
3
be three zeros at infinity.

© Copyright Reserved by Gateflix.in No part of this material should be copied or reproduced without permission
−5 Each zero produces a slope of
σ=
3 20dB/dec
14p → −280dB / dec
Q.106 (d) 2z → +40dB / dec
k(s + 1) Total = −240dB / dec
G (s) =
s ( s + 2 ) (s + 3) ∴ slope of highest frequency
asymptotic is −240dB / dec

Q.111 (c)
s+5
G (s ) =
There are 3 root loci branches s + 4s + 9
2

Phase plot of the above system is


Q.107 (a) shown below:
The addition of open loop zero pulls
the rot loci towards the left and
therefore system becomes more
stable.

Q.108 (c)
k(s + 1) Hence angle varies between
G (s) H (s) = 0° to -180°
s ( s + 4 ) ( s 2 + 2s + 2 )
Poles s = 0, −4, −1 ± j1 Q.112 (a)
Zeros, s = −1 Type of the system is determined by
−4 − 1 − 1 − (−1) 5 the number of poles at origin. Since
Centroid, σ = = − the initial angle is 0° hence there are
3 3
no poles at origin Hence Type = 0
Q.109 (a)
Q.113 (b)
k(s + 2)
G (s) H (s) = 1
s + 2s + 2
2
G (s) =
s
To find break point consider
ωgc = 1rad / sec
G ( s ) H ( s ) = −1
∴180o + ∅ |ω== 180 − 90
= 90
s 2 + 2s + 2 ωgc
∴k =
s+2
Q.114 (b)
dk ( s + 2 )( 2s + 2 ) − (s + 2s + 2)
2

= 0 k
G (s) =
ds (s + 2) 2 s2
⇒ 2 ( s 2 + 3s + 2 ) − s 2 − 2s − 2 =0 At ω ω=
= gc , G( jω 1
⇒ s 2 − 4s + 2 =0 g( =
jω =
k
1
s= −0.58, −3.41 ωgc
2

s = −3.41 lies on root locus hence it ωgc = krad / sec


is valid break point.
Q.115 (c)
Q.110 (b)
The slope of asymptotic bode plot is
Each pole produces a slope of
−20dB / dec
the integer multiple of ±20dB / dec
or ±6dB / octave

© Copyright Reserved by Gateflix.in No part of this material should be copied or reproduced without permission
1
For ξ =
Q.116 (c) 2
Type 2 ⇒ 2 poels at origin.
ωr = 0
One pole gives −20dB / decade hence
Two poles give −40dB / decade of 3. As M r increases, ξ decreases
Initial slope and M p increase hence it is true.

Q.117 (c) Q.122 (d)


In general error at corner frequency A minimum phase transfer function
is given by ±3n dB is one whose all the poles and zeros
are on the left half of s-plane
Q.118 (b)
1. Initial slope −20dB / decade . Q.123 (d)
Hence it is Type 1 system None of the transfer function are
1 minimum phase. Because none of
k (= ω3 )
−1
2.
=
ω3 them have all their poles and zeros
1 on the left half of s-plane.
ess= = ω3
k
ω + ω3 Q.124 (a)
3. ω2 ≠ 1 1− s
2 is non minimum phase system.
1+ s
Q.119 (d)
= 180° + Φ |ω=ωgc
PM Q.125 (d)
Low frequency asymptote decide
Φ |ω=ωgc = −90° (given) the number of poles at origin which
∴ PM = 180° − 90° = 90° is nothing but type of system
−60dB / dec ⇒ 3 poles are origin
Q.120 (b) Hence Type III system,
G ( s )= s + 1
Q.126 (d)
G ( jω =
) ` jω + 1 For an all pass network the poles lie
M G ( jω
= = ) ω2 + 1 on the left half of s-plane and zero
lies symmetrically on the right half
M |=
ω =1 =
2 1.41 of s-plane
∠G ( jω ) =
Φ= tan −1 ( ω )
Q.127 (a)
Φ |ω=1 = tan −1 (1) 45°
1. In general for n zeros the error
at corner frequency = +3ndB
Q.121 (c) For n = 1 error = +3ndB
1 2. Phase angle for complex
1. Resonant peak, M r =
2ξ 1 − ξ 2 conjugate poles doesn’t depend
Peak overshoot, on damping ratio.
− πξ

=
Mp e 1− ξ 2
×100% Q.128 (d)
10
Both are function of ξ only G (s) =
0.66s + 2.33s + 1
2
ωr ωn 1 − ξ 2
2.=

© Copyright Reserved by Gateflix.in No part of this material should be copied or reproduced without permission
The corner frequencies are nothing When –1 + j0 is located in region I,
but the magnitude of poles and the number of encirclements N = 0
zeros. Hence system is stable
The poles are given by the roots of When –1 + j0 is located in region II
0.66s 2 + 2.33s + 1 =0 then, N = −2
0.66s 2 + 0.33s + 2s + 1 =0 Hence system is unstable.
( 2s + 1) + 1( 2s + 1) =
0
Q.134 (d)
( 0.33s + 1)( 2s + 1) =0
k ( s + 3) (s + 5)
1 −1 G (s) =
s= − , ( s − 2 ) (s − 4)
0.33 2
Hence corner frequencies are 3, 0.5 The number of right half poles is 2
For system to be stable the number
Q.129 (d) of rotation around – 1 + j0 should
Gain margin, GM be 2 in anticlockwise direction.
1 At ω = 11
= −20 log log ( 0.64 ) =
= 3.86dB 4
G ( jω ) ω=ω = k= k(1.33)
G( jω)
pc 3
1
Q.130 (b) For k > the number of
1.33
G ( jω ) =
jω encirclements around –1 + j0 would
1 + jω be 2 in anticlockwise direction
For ω = 0, G ( jω ) hence system is stable.
= 0∟0°(M ∟Φ ) N= P= 2
1 1

∞, G ( jω ) =
For ω = 1∟0°(M 2∟Φ 2 ) Q.135 (d)
Φ1 − Φ 2 =
0 hence ending direction =
Type 2,=
Order 4
is not considered
Since finite poles is neat to the Q.136 (b)
imaginary axis hence starting The number of half circle represents
direction is clockwise the type of system .Here number of
half circles =1 Hence Type =1

Q.137 (a)
ωpc , ∠G ( jω ) =
At ω = −180°
∠G ( jω ) =
−90° − tan −1 ( ωT1 )
− tan −1 ( ωT2 )
Q.131 (d)
−90° − tan −1 ( ωpc T1 )
−180° =
For a stable system both gain
margin & phase margin must be − tan −1 ( ωpc T2 )
positive. 1
⇒ ωpc =
Q.132 (d) T1T2
1 1 1 T +T
GM= = = 2.5 =
Gain Margin = 1 2
a 0.4 G ( jω ) ω= ωpc T1T2

Q.133 (d)

© Copyright Reserved by Gateflix.in No part of this material should be copied or reproduced without permission
Q.138 (b) Qc ≠ 0, Q0 ≠ 0, A ≠ 0
= G ( jω ) 1∟0°
At ω 0,= hence controllable and observable
∞, G ( jω ) =
At ω = 1∟ − 180°
1 Q.143 (b)
∴ G (s) = 2 Characteristic eq n ⇒ SI − A =
0
( s + 1)
 s 0 0 2
0 s  −  2 0  = 0
Q.139 (c)    
1  s −2 
G ( jω ) =  −2 s  = 0
ω +1 2
 
= gc , G ( jω )
At ω ω= 1 ⇒ s2 − 4 =0
⇒ ωgc
2
+1 =0 s = ±2 j
ωgc = 1rad / sec So poles are s = ±2 j and −2j
∅ = ∠G ( jω ) |ω
gc =1rad/sec Q.144 (d)
−2 tan (1) =
= −1
−90 o
Adj[SI − A]
=T.F C B+D
Phase margin = 180o − 90o = 90o SI − A
s + 3 1 0 
Q.140 (a) [2
0]   
=  −2 s  1 
1 1
=GM = s −1
G ( jω ) ω= ωpc a
2 s+3
Y(s) 2
Q.141 (b) =
T.F = 2
Gain margin close to unity or phase U(s) s + 3s + 2
margin close to zero is oscillatory.
Q.145 (a)
Q.142 (a) Y(s) 25 + 1s°
(s)
Let G= = 2
U(s) s + 75 + 9s°
Let s° → x1
s1 → x&1 =
x2
State Model is obtained by Diagram s 2 → x&2
X& =
1 −3x1 + x 2 C2 =
x1 ∴ Y (s) =2x 2 + 1x1..........(1)
X&= u − 2x
2 1 2 U ( s ) =x&2 + 7x 2 + 9x1
 −3 1  0  So, x&2 = U ( s ) − 7 x 2 + 9x1............(2)
=x&   x +   u1
 0 −2  1  x&1 = x 2 .............(3)
C2 = [1 0] x From eqn (1) (2) & (3)
 A AB 0 1 0 
=x   x +   u; y = [1, 2] x
Qc = 0 1  Qc =−1 ≠ 0  −9 −7  1 
 1 −2 
Q.146 (c)
C 1 0 
Q0 = Q0 = 1 ≠ 0 1 2 
CA 0 1  We have C = [ b, 0] and A =  
0 1 

© Copyright Reserved by Gateflix.in No part of this material should be copied or reproduced without permission
Observability 1 0  0 1 
S − =
1   −3 −5
0
Q0 = CT CT A T  0
b b  s 0  0 1 
Q0 =   0 − =
s   −3 −5
0
 0 2b  
Q0 = 2b 2 s 2 + 5s + 3 = 0
System is observable when |Q0 | ≠ 0
so given system observable for all Q.151 (d)
non zero values of b Controllability Qc = [ B, AB]
0 2 
Q.147 (d) Qc =  
C  Adj ( SI − A )  B + D 1 b 
T.F = Qc =−2 ≠ 0, A =b
SI − A
So system is uncontrollable for b =
s + 2 0   1 
[1 1]  0, since P(Qc ) ≠ e(A)
 0 s + 1 0  1
=
( s + 2 ) (s + 1) (s + 1) Q.152 (a)
Characteristics eq n ⇒ SI − A =
0
Q.148 (b)
 s 0 0  0 1 0
Controllability Qc = [ B, AB] 0 s 0  −  0
   0 1  =0
0 1  0 0 s   −6 −11 −6 
Qc =  
1 −3
∴ s3 + 6s 2 + 11s + 6 =0
Qc =−1 ≠ 0 Then s =−1, −2, −3
A= 2 ≠ 0 Since, closed loop poles are nothing
Therefore the system is controllable but eigen value.
Since e Q b = e(A)
Q.153 (a)
System observability
c [ ZI − A ] b + d
−1
Q b = CT CT A T 
1 −2  Q.154 (a)
Qb =  
1 −2  = X1 i L=
1
i ( t ) u(t)
x 2 Vc=
Q b = 0but A= 2 ≠ 0 Applying KCL we get
System is not observable −i ( t ) + i L + i c =
0
Since e Q b ≠ e(A) i c u ( t ) − X1
=
dv c
Q.149 (a) c= u ( t ) − X1
dt
State transition matrix is given by
u X1
e At It is also called zero input x&2= − ........(1)
c c
response It represents transient
Applying kVL we get
response or force for response.
VL ( t ) = Vc (t)
Q.150 (a) di L
L = X2
Characteristic dt
eq n ⇒ SI − A =0

© Copyright Reserved by Gateflix.in No part of this material should be copied or reproduced without permission
X2 1
x&1 =.........(2) Poles, S = −
L RC
0 (t) c (t)
=Y V= V= X 2 ..........(3) Zero, s = ∞
From (1), (2) (3) we get
Q.160 (d)
 1
 0 L 0  The pole zero plot of lag lead
=x&  x+1 u compensator is
− 1 0   c 
 c 
Y = [ 0 1] x

Q.155 (d)
Given x&1 = X 2 Q.161 (a)
Both ωgc and BW are increased by
x&2 = X1 + X 2 + u
phase lead compensator.
= X1 + 3X 2
Y
Y(s) 3s + 1 Q.162 (b)
= 2
U(s) s − s − 1 The transfer function of PI
k
Controller is k p + i
Q.156 (d) s
The maximum phase provided is
 a −1  Q.163 (d)
Φ max = sin −1  
 a +1   a −1 
Φ max = sin −1  
 a +1 
Q.157 (a)  3 −1 
For phased lead Compensator pole = sin −1   = 30°
 3 +1
zero diagram is
Q.164 (b)
Without controller
9
G (s ) = Type -1
s(s + 2)
For ramp input type -1 system gives
S +1 constant steady state error
satisfies the above condition
S+ 2 If integral controller is employed the
Zero, i.e. S = −1 is neat to imaginary 9K
axis compate to pole i.e. S = −2 G (s) = 2 i Type -2
s (s + 2)
Q.158 (c)
ess = 0
Phase lead compensator affects the
transient performance of the
system.

Q.159 (c)
For phase lag network pole must be
near to the imaginary axis
1
For
1 + RCS

© Copyright Reserved by Gateflix.in No part of this material should be copied or reproduced without permission

Potrebbero piacerti anche